You are on page 1of 462

See discussions, stats, and author profiles for this publication at: https://www.researchgate.

net/publication/331864010

The Year of Zayed 2018

Book · March 2019

CITATIONS READS
0 25,887

7 authors, including:

Abdul Naser Tamim Ghada M. Al Aqqad

15 PUBLICATIONS 2 CITATIONS 1 PUBLICATION 0 CITATIONS

SEE PROFILE SEE PROFILE

All content following this page was uploaded by Abdul Naser Tamim on 09 October 2019.

The user has requested enhancement of the downloaded file.


2018

Released MCQ NCDB Exams


Answered Copy

Dr Abdul Naser Tamim SEHA - UAE


Dr Ghada Al Aqqad D.D.S - UAE
Dr Kamal Naser, Amrita Medical Centre-
UAE
Dr Emad Wani, Shadi Dental Centre- UAE
Dr Lina Anka SEHA - UAE
Dr Rouba Zgheibi SEHA - UAE
Maria Teresa Yongson Alejandrino SEHA-
UAE
Introduction:

Dear Colleagues,

National Canadian Dental Board exam NCDB is one of the most prestigious and well known licensing
exams worldwide. It is usually consists of 100 multiple choices questions that is based on clinical
scenarios and daily dental practice judgement.

Each year the organizing committee is releasing thousands of unanswered questions to help the dental
professional to prepare for the exam.

https://ndeb-bned.ca/sites/ndeb/files/pdf/Resources/englishwritten_released_2018.pdf)

We surfed the web for more than ten months propping more than ten websites for groups that are
working on the questions and manage to collect about 2500 questions with their answers.

Keeping in mind that 100 % accuracy is not possible due to the change in knowledge and feed back in
this field. Our target is to trigger the initiative of letting others look for the wrong answers if it does exist.

This project is part of paying back our debit to UAE and homage to Zayed's legacy in this country,
besides, it is about sharing knowledge in our field.

We do hope that we have managed to forward a gift that is suitable for this occasion.

These answers CAN NOT BE RELIED ON but it is the only way to get an idea about the subjects, matters
and topics in the exam. These questions were answered by using internet open sources and any
available references. Some of the answers could be modified upon reference change, but these
changes are minor. Please note that we circulated this collection at non-commercial and personal
level. We’re sharing this with our colleagues ONLY for educational purpose.

P.s: Remarks, comments and ideas would be highly appreciated.

On Behalf of the Team

Dr. Abdul Naser Tamim

P.s: 1- The correct answers are marked with a star 2- the highlighted questions means best answer is
debatable.

http://www.cram.com/flashcards

https://www.worlddentalnetwork.com

http://www.quizzn.com/, http://discussionology.com

https://www.scribd.com/doc/296478944/Released-2016-English

https://quizlet.com/

Dr Abdul Naser Tamim SEHA, Dr Ghada Al Aqqad D.D.S, Dr Kamal Naser - Amrita Medical Centre, Dr Emad Wani - Shadi Dental Centre,
Dr Lina Anka SEHA, Dr Rouba Zgheibi SEHA, Maria Teresa Yongson Alejandrino SEHA 1
The flexibility of a retentive clasp arm is Which component of a partial denture
affected by the framework provides the best indirect
retention?
A. location of the reciprocal arm.
B. length of the retentive arm. * A. Rest. *
C. position on the abutment tooth. B. Circumferential clasp.
D. location of the occlusal rest. C. Lingual strap.
D. Proximal plate.
If adjustment of the occlusal plane of
natural teeth opposed by a complete or In an Angle Class I occlusion the
partial denture is required, it should be
completed A. distal inclined plane of the maxillary
canine articulates with the mesial
A. after the teeth have been set on the inclined plane of the mandibular
trial denture. canine.
B. immediately after making the final B. mesial inclined plane of the
casts. maxillary canine articulates with the
C. upon delivery of the denture. distal inclined plane of the mandibular
D. after the diagnosis and treatment canine. *
plan has been established. * C. the primary canines are end-to-end.
D. the permanent canines are end-to-
In a removable partial denture, a end.
palatal strap is used instead of a
narrow bar because it is Serial extraction

A. more rigid with less thickness. * A. involves the 4 permanent first


B. easier to polish. premolars. *
C. more stable. B. is the treatment for Class II skeletal
D. less irritating to the soft tissues. malocclusions with severe space
E. more hygienic. shortage.
C. is commenced with bilateral
A removable partial denture rest should expansion of the arches.
be placed on the lingual surface of a D. is best suited to Class I dental and
canine rather than on the incisal skeletal malocclusions with minimal
surface because space shortage.
E. requires leeway space maintenance.
A. less leverage is exerted against the
tooth by the rest. * Correction of a lingual crossbite of
B. the enamel is thicker on the lingual tooth 1.2 has the best long term
surface. prognosis if 1.2 is
C. visibility and access are better.
D. the cingulum of the canine provides A. lingually inclined, with 50% overbite*
a natural recess that does not need to B. ideally inclined, with 50% overbite.
be prepared. C. lingually inclined, with 5% overbite.
D. ideally inclined, with 5% overbite.

Dr Abdul Naser Tamim SEHA, Dr Ghada Al Aqqad D.D.S, Dr Kamal Naser - Amrita Medical Centre, Dr Emad Wani - Shadi Dental Centre,
Dr Lina Anka SEHA, Dr Rouba Zgheibi SEHA, Maria Teresa Yongson Alejandrino SEHA 2
Orthopedic correction of a mild skeletal In the mandible, the main growth site is
Angle Class III malocclusion should be in the
started
A. gonial angle.
A. just prior to the pre-pubertal growth B. condylar cartilage. *
spurt. * C. posterior border of the ramus.
B. immediately following the pre- D. inferior and lateral aspects of the
pubertal growth spurt. body of the mandible.
C. shortly after eruption of the upper
first permanent molars. The principal growth sites of the
D. shortly after eruption of the upper maxilla in a downward and forward
second permanent molars. direction include which of the following
sutures?
Which of the following represents the
normal relationship of the primary 1. Frontomaxillary.
canines? 2. Zygomaticomaxillary.
3. Pterygopalatine.
A. The distal inclined plane of the 4. Median palatine.
maxillary canine articulates with the
mesial inclined plane of the mandibular A. (1) (2) (3) *
canine. B. (1) and (3)
B. The mesial inclined plane of the C. (2) and (4)
maxillary canine articulates with the D. (4) only.
distal inclined plane of the mandibular E. All of the above.
canine. *
C. Normal articulation of primary The normal growing mandible exhibits
canines is end-to-end. which of the following characteristics?
What is the earliest age to confirm the A. Resorbs along the posterior rami.
diagnosis of a congenitally missing
B. Grows more vertically than
mandibular second premolar?
horizontally.
C. Has completed 100% of its growth
A. 1 year.
by age 13 in females.
B. 3 years.
D. Has latent post-pubertal growth
C. 5 years. potential. *
D. 7 years. *?
Cleft lip is caused by the failure of
An overjet of 8mm is usually which of the following processes to
associated with fuse?
A. Angle Class I cuspid relationship. A. Maxillary.
B. Angle Class II cuspid relationship. * B. Maxillary and lateral nasal. *
C. Angle Class III cuspid relationship. C. Palatine.
D. Angle Class I molar relationship.

Dr Abdul Naser Tamim SEHA, Dr Ghada Al Aqqad D.D.S, Dr Kamal Naser - Amrita Medical Centre, Dr Emad Wani - Shadi Dental Centre,
Dr Lina Anka SEHA, Dr Rouba Zgheibi SEHA, Maria Teresa Yongson Alejandrino SEHA 3
The upper lip is the result of fusion D. assess the severity of midline
between the deviation.

A. maxillary and mandibular processes. Which of the following types of


B. maxillary and lateral nasal publications is the most reliable source
processes. for making evidence-based clinical
C. maxillary and medial nasal decisions?
processes. *
D. medial and lateral nasal processes. A. Dental manufacturer’s product
E. lateral and medial palatine information.
processes. B. Professional association journals.
C. Systematic review articles from
Developing roots will split when the dental research journals. *
D. Case reports in dental journals.
A. epithelial diaphragm forms too many
lateromedial extensions. * The gingival aspect of a pontic which
touches the alveolar ridge should be
B. dental follicle cells migrate into the
epithelial diaphragm.
A. convex only in the mesiodistal
C. cementoblasts fail to develop on and
direction.
stabilize the root’s surface.
B. concave faciolingually and convex
D. periodontal ligament forms too soon.
mesiodistally.
The permanent first molars of a 7 year C. small and convex in all directions. *
old patient have pronounced, deep D. fabricated to produce slight tissue
occlusal fissures that are stained. Bite- compression.
wing radiographs show a normal
dentino-enamel junction. The most The primary stress bearing area of the
appropriate treatment is maxillary complete denture is the

A. conservative amalgam restorations. A. hard palate. *


B. glass ionomer restorations. B. alveolar ridge.
C. application of fissure sealants. * C. median palatal raphe.
D. topical fluoride application. D. zygoma.
E. no recommended treatment.
The base of a distal extension partial
A panoramic radiograph taken at 10 denture should cover the maximum
years of age is indicated to support area because

A. measure the space between the A. the force transmitted per unit area
temporomandibular joint disc and the will be kept to a minimum. *
condyle. B. maximum number of artificial teeth
B. calculate the amount of crowding in can be placed.
the maxilla. C. phonetics is improved.
C. predict the probability of canine D. strength of the base is increased.
impaction. *

Dr Abdul Naser Tamim SEHA, Dr Ghada Al Aqqad D.D.S, Dr Kamal Naser - Amrita Medical Centre, Dr Emad Wani - Shadi Dental Centre,
Dr Lina Anka SEHA, Dr Rouba Zgheibi SEHA, Maria Teresa Yongson Alejandrino SEHA 4
Guiding planes on abutment teeth for a During setting, a gypsum-bonded
removable partial denture should be investment material will undergo an
prepared before the occlusal rests are additional expansion if it occurs
prepared in order to
A. under water. *
A. avoid post-insertion adjustment. B. under vacuum.
B. facilitate surveying of the cast. C. in a cold environment.
C. improve the retention of the direct D. in a dry environment.
retainer assembly.
D. finalize the location of the occlusal The working time of zinc-phosphate
rest. * cement
E. minimize the preparation needed for
the occlusal rests. A. is shortened if moisture condenses
on the mixing slab during the mixing
Before performing periodontal surgery, process. *
it is important to B. is lengthened if the powder is mixed
with the liquid as quickly as possible.
A. prescribe a mouthwash. C. is shortened if the mixing slab is
B. prescribe systemic antibiotics. cooled.
C. control plaque. * D. is shortened by adding a small
quantity of powder to the liquid a
The primary purpose of surgical minute prior to start mixing.
therapy for the treatment of
periodontitis is to The rate of "set" of polysulfide
impression materials is accelerated by
A. apically position the flap.
B. eliminate periodontal pockets. A. increasing the mixing temperature. *
C. remove the ulcerated epithelium of B. decreasing the mixing temperature.
the periodontal pocket. C. adding oleic acid to the mix.
D. improve access for removal of local D. increasing the base/catalyst ratio.
etiologic factors. *
Polysulfide rubber base impressions
The most likely reason for porcelain should be
fracturing off a long and narrow metal-
ceramic fixed partial denture is that the A. poured immediately.
framework alloy had an insufficient B. poured within an hour. *
C. immersed in a fixing solution before
A. elastic modulus. * pouring.
B. proportional limit. D. immersed in water 10 minutes
C. fracture toughness. before pouring.
D. tensile strength. E. coated with a thin film of separating
medium.

Dr Abdul Naser Tamim SEHA, Dr Ghada Al Aqqad D.D.S, Dr Kamal Naser - Amrita Medical Centre, Dr Emad Wani - Shadi Dental Centre,
Dr Lina Anka SEHA, Dr Rouba Zgheibi SEHA, Maria Teresa Yongson Alejandrino SEHA 5
The use of a reservoir on the sprue of occlusal contact points should be
a wax pattern decreases eliminated?

A. volumetric changes in the casting. A. Inclined lingual plane of the


B. casting porosity from inclusion of mesiobuccal cusp. *
gases. B. Mesial marginal ridge.
C. casting porosity from inclusion of C. Mesial lingual cusp tip.
foreign bodies. D. Central fossa.
D. casting porosity during
solidification* A facebow relates the

The principal advantage of polyether A. mandibular arch to the glenoid


impression materials over polysulfide fossa.
polymer impression materials is B. vertical axis of the maxilla to the
mandible.
A. longer shelf life. C. hinge axis to the maxillary arch. *
B. superior accuracy. D. maxilla to the mandible at the
C. superior surface on the cast. vertical dimension of occlusion.
D. less dimensional change with time *
The vibrating line in a completely
The most appropriate management of edentulous maxilla is
an 8 year old patient with cavitated
enamel lesion of the permanent first A. the junction between the hard and
molars is soft palate.
B. a well-defined line that maintains a
A. tooth brushing and oral hygiene distinct shape and location.
instruction. C. the junction between the movable
B. topical fluoride application. and immovable soft palate. *
C. pit and fissure sealants. D. an imaginary line that crosses the
D. preventive resin restorations. * soft palate posterior to the fovea
palatini.
Compared to heat-cured acrylic resins,
cold-cure acrylic resins are Which of the following is the most likely
cause of periodontal pocket formation
A. harder and more colour stable. following crown preparation?
B. less hard and more colour stable.
C. less hard and less colour stable. * A. Inadequate use of water with the
D. harder and less colour stable. high-speed handpiece.
B. Invasion of the biological width. *
A metal-ceramic crown is being placed C. Existing periodontal disease.
on tooth 1.6. The patient has ideal D. The use of chemical retraction
occlusal relationships and the crown agents.
has been adjusted so that it is not in
supraocclusion. Which of the following

Dr Abdul Naser Tamim SEHA, Dr Ghada Al Aqqad D.D.S, Dr Kamal Naser - Amrita Medical Centre, Dr Emad Wani - Shadi Dental Centre,
Dr Lina Anka SEHA, Dr Rouba Zgheibi SEHA, Maria Teresa Yongson Alejandrino SEHA 6
Histological sections of a lesion following is the most appropriate
removed from the apex of a carious appliance for this patient?
tooth show fibrous tissue and chronic
inflammatory cells. The most likely A. Headgear.
diagnosis is a/an B. Hyrax appliance. *
C. Fixed brackets.
A. odontogenic fibroma. D. Functional appliance.
B. periradicular cyst.
C. periradicular granuloma. * Which type of headgear is most
D. central ossifying fibroma. appropriate for the management of a
10 year old patient with an increased
A large Class III preparation on tooth lower third of the face, incompetent
1.1 extends 2mm on the labial, 4mm lips, an Angle Class II malocclusion
on the lingual and onto the root and vertical maxillary excess?
surface. Which of the following
cavosurface margins should be A. High-pull. *
prepared at 90 degrees? B. Cervical.
C. Combination.
A. Incisal. D. Protraction.
B. Gingival. *
C. Labial. Which of the following analyses is
D. Lingual. most appropriate for use in an adult
patient?
Koplik's spots are seen in the oral
cavity of patients with A. Moyers.
A. chickenpox. B. Bolton. *
C. Tanaka-Johnston.
B. mumps.
C. measles. * Which of the following statements is
D. scarlet fever. correct about Quantitative Light or
E. smallpox. Laser Induced Fluorescence used to
aid in the early detection of caries?
The thickness of the labial bony wall of
a maxillary anterior tooth is typically A. Low specificity when used to detect
smooth surface caries.
A. < 1mm. B. Unable to detect caries around
B. 1 - 2mm. * existing restorations.
C. > 2mm. C. High sensitivity when used to detect
interproximal caries.
A 9 year old patient has a right D. Assesses light reflection as
posterior crossbite and a mandibular opposed to light transmission. *
midline shift to the right. Which of the

Dr Abdul Naser Tamim SEHA, Dr Ghada Al Aqqad D.D.S, Dr Kamal Naser - Amrita Medical Centre, Dr Emad Wani - Shadi Dental Centre,
Dr Lina Anka SEHA, Dr Rouba Zgheibi SEHA, Maria Teresa Yongson Alejandrino SEHA 7
An alginate impression is made of the A young adult has generalized areas of
maxillary arch and there is abundant chalky, opaque, cavitated lesions on
unset alginate remaining on the teeth. the vestibular surface of multiple teeth
Excess of which of the following and interproximal gingiva that is red
components may contribute to the slow and swollen. Which of the following is
setting? the most appropriate diagnosis?

A. Zinc oxide. A. Smooth surface caries.


B. Calcium sulfate. B. Rampant caries. *
C. Potassium alginate. C. Interproximal caries.
D. Sodium phosphate. * D. Secondary caries.
E. Arrested caries.
A fixed partial denture is an appropriate
treatment option for replacing a missing A squamous cell carcinoma located
mandibular first permanent molar when near the midline of the anterior floor of
the the mouth normally spreads to which of
the following lymph nodes?
A. adjacent teeth are heavily restored. *
B. incisors and contralateral molars are A. Unilateral neck lymph nodes.
missing. B. Mediastinal lymph nodes.
C. abutment crown to root ratios are 1 C. Bilateral neck lymph nodes. *
to 1. D. Retropharyngeal lymph nodes.
D. Second and third molars are tipped
mesially. Heavy cigarette smoking significantly
increases the incidence of
A 29 year old patient developed
multiple ulcerated lesions on the buccal A. aphthous stomatitis.
attached gingiva one day ago. No B. geographic tongue.
fever, malaise or lymphadenopathy is C. lichen planus.
present. The most likely diagnosis is D. atrophic glossitis.
E. mucosal pigmentation. *
A. primary herpetic gingivostomatitis.
B. recurrent herpes simplex infection. * Clinical examination of a 42 year old
C. aphthous ulcers. heavy smoker reveals a white patch in
D. necrotizing ulcerative gingivitis. the retromolar/tonsillar pillar region.
The patch cannot be wiped off. The
The white appearance of the oral most likely diagnosis is
mucosa following extended local
application of acetylsalicylic acid is A. lichen planus.
B. leukoplakia. *
A. hyperparakeratosis. C. white sponge nevus.
B. acanthosis. D. frictional hyperkeratosis.
C. coagulation necrosis. * E. pseudomembranous candidiasis.
D. edema.

Dr Abdul Naser Tamim SEHA, Dr Ghada Al Aqqad D.D.S, Dr Kamal Naser - Amrita Medical Centre, Dr Emad Wani - Shadi Dental Centre,
Dr Lina Anka SEHA, Dr Rouba Zgheibi SEHA, Maria Teresa Yongson Alejandrino SEHA 8
Primary herpes simplex virus infection E. Nasopalatine cyst.

A. usually occurs after the age of 20. Acute or subacute suppurative


B. has a slow onset. osteomyelitis occurs most frequently in
C. may be subclinical. * the
D. lasts for 1 - 2 days.
E. affects only the gingiva. A. anterior maxilla.
B. posterior mandible. *
Hypochromic anemia is associated C. posterior maxilla.
with D. anterior mandible.

A. iron deficiency. * Which of the following restorations is


B. aminopyrine therapy. the most appropriate for the
C. vitamin B12 deficiency. replacement of a maxillary permanent
D. folic acid deficiency. lateral incisor where there is 4.5mm of
mesial-distal space and an intact
Unilateral numbness of the chin is central incisor?
associated with
A. Implant supported restoration.
A. malignancy. * B. Cantilever pontic FPD using canine
B. Bell's palsy. abutment. *
C. apical abscess. C. Removable partial denture.
D. trigeminal neuralgia. D. Three-unit metal-ceramic full-
coverage fixed dental prosthesis.
When cavitated carious lesions are
present there is The most appropriate gingival contour
of a fixed partial denture connector is
A. exposure of the dentin protein
matrix. A. concave.
B. demineralization by matrix B. convex. *
Vmetalloproteinases. * C. flat.
C. bacterial protease inhibition.
D. a denatured inorganic phase. Which of the following viruses has the
greatest chance of transmission in an
Which of the following conditions is an occupational exposure to a vaccinated
example of a mucous retention dental healthcare worker?
phenomenon?
A. Hepatitis B.
B. Hepatitis C. *
A. Nicotine stomatitis.
C. HIV.
B. Koplik’s spots. D. HPV.
C. Ranula. *
D. Residual cyst.

Dr Abdul Naser Tamim SEHA, Dr Ghada Al Aqqad D.D.S, Dr Kamal Naser - Amrita Medical Centre, Dr Emad Wani - Shadi Dental Centre,
Dr Lina Anka SEHA, Dr Rouba Zgheibi SEHA, Maria Teresa Yongson Alejandrino SEHA 9
Ideally, within how many hours should
one receive medical attention for A. 0.50mm.
percutaneous exposure to blood B. 1.00mm.
borne pathogens? C. 1.50mm.
D. 2.00mm. *
A. 2. *
B. 4. Proportionally, how far below the
C. 6. superior border of the lower 1/3 of the
D. 8. face are the commissures of the
mouth located?
Recurrent unilateral submandibular
swelling and pain just prior to meals is A. At the commissures.
indicative of
B. 1/4.
A. an odontogenic infection. C. 1/3. *
B. sialolithiasis. * D. 2/3.
C. ranula. E. 3/4.
D. sarcoidosis.
E. Sjögren’s syndrome. A 4 year old has generalized bone
loss, mobile teeth and generalized
The manufacturer’s maximum calculus. Which condition should NOT
recommended number of 1.8ml be included in the differential
cartridges of 3% mepivacaine that diagnosis?
may be safely administered to a 65kg
adult is A. Cyclic neutropenia.
B. Papillon-Lefèvre syndrome.
A. 5. C. Chédiak-Higashi syndrome.
B. 6. D. Crouzon syndrome. *
C. 7. * E. Leukocyte adhesion deficiency
D. 8. syndrome.
E. 9.
Which of the following is true about
The maximum recommended number the supraeruption of unopposed
of 1.8ml cartridges of 2% lidocaine molars?
with epinephrine 1:100,000 that may
be safely administered to a 17kg child A. Supraeruption occurs 60% of the
is approximately time.
B. Supraeruption is more prevalent in
A. 0.5. the mandibular arch.
B. 1. C. Unopposed molars have a mean
C. 1.5. * supraeruption of 3.0mm.
D. 2. D. Attachment loss is one of the main
E. 2.5. predictors. *

A3 shades of composite resins should


be light-cured in increments limited to
a maximum of

Dr Abdul Naser Tamim SEHA, Dr Ghada Al Aqqad D.D.S, Dr Kamal Naser - Amrita Medical Centre, Dr Emad Wani - Shadi Dental Centre,
Dr Lina Anka SEHA, Dr Rouba Zgheibi SEHA, Maria Teresa Yongson Alejandrino SEHA 10
A 9 year old presents for treatment
immediately following a facial injury A 7 year old has just lost a permanent
resulting in a fracture of a maxillary maxillary central incisor due to trauma.
central incisor that involves the enamel The tooth cannot be found. The most
only. The tooth tests negative to an appropriate immediate management is
electric pulp tester. This finding to
indicates that the tooth
A. order a soft tissue x-ray. *
A. is nonvital and should be extracted. B. test adjacent teeth for vitality.
B. is nonvital and endodontic therapy is C. maintain space with a removable
indicated. appliance.
C. has a root fracture and should be D. prescribe an antibiotic.
extracted.
D. should be observed and tested What structure is most likely to dislodge
again at a later date. * a complete mandibular denture?

A 1.5mm diameter carious pulp A. Retromolar pad.


exposure occurs on a permanent first B. Buccinator muscle.
molar of a 7 year old child. The tooth is C. Mylohyoid muscle. *
vital and has no periapical involvement. D. Pterygomandibular raphe.
The most appropriate initial treatment
is to perform a/an The instrument most easily broken in a
root canal is a
A. pulp capping.
B. partial pulpotomy * A. barbed broach. *
C. pulpotomy. B. rotary instruments.
D. pulpectomy. C. k-file.
E. extraction. D. Hedstrom file.

An 8 year old patient has a permanent Chelating agents are useful to treat a
central incisor with a necrotic pulp and tooth with a an
a wide open apex. The most
appropriate management is to perform A. curved canal.
a/an B. sclerotic canal. *
C. apical (periradicular) abscess.
A. pulpotomy. D. root perforation.
B. apexification. * E. open apex.
C. regeneration.
D. root canal therapy.

Dr Abdul Naser Tamim SEHA, Dr Ghada Al Aqqad D.D.S, Dr Kamal Naser - Amrita Medical Centre, Dr Emad Wani - Shadi Dental Centre,
Dr Lina Anka SEHA, Dr Rouba Zgheibi SEHA, Maria Teresa Yongson Alejandrino SEHA 11
The radiographs of a 9 year old with
tooth 1.1 completely erupted and tooth What is the most appropriate initial
2.1 unerupted reveal a palatally management of a tooth which is
located mesiodens. The most sensitive to percussion but responds
appropriate management is to normally to electric pulp testing?

A. monitor the eruption of tooth 2.1 for A. Pulpectomy.


another year. B. Occlusal assessment. *
B. uncover the mesiodens, wait for C. Pulpotomy.
eruption and then extract it. D. Restoration replacement.
C. extract the mesiodens and allow
passive eruption of tooth 2.1. * A child has received a successful
D. extract the mesiodens and inferior alveolar nerve block using
orthodontically extrude tooth 2.1. 1.5ml of 2% lidocaine with 1:100,000
epinephrine. However, during
Following the removal of a vital pulp, a placement of a rubber dam clamp on
root canal is medicated and sealed. the first permanent molar, the child
The patient returns with symptomatic complains that the “tooth ring” is
apical periodontitis (acute periradicular hurting. Which of the following is the
periodontitis). The most probable most appropriate management?
cause is
A. Wait 15 minutes until more
A. overinstrumentation. * profound anesthesia is achieved.
B. lateral perforation. B. Anesthetize the lingual nerve with
C. incorrect medication. the remaining lidocaine.
D. infection. C. Anesthetize the long buccal nerve
with the remaining lidocaine. *
Which of the following statements is D. Proceed with treatment without
true regarding the use of nickel rubber dam.
titanium (Ni Ti) rotary endodontic files?
The smear layer present on the root
A. Files which are more tapered canal wall after cleaning and shaping
fracture less often than less tapered is best removed by using
ones.
B. Files are highly resistant to fatigue A. EDTA. *
failure. B. hydrogen peroxide.
C. Fractures tend to happen near the C. chlorhexidine.
tip of files. * D. isopropyl alcohol.
D. Fractures tend to happen in smaller
diameter files than larger ones.

Dr Abdul Naser Tamim SEHA, Dr Ghada Al Aqqad D.D.S, Dr Kamal Naser - Amrita Medical Centre, Dr Emad Wani - Shadi Dental Centre,
Dr Lina Anka SEHA, Dr Rouba Zgheibi SEHA, Maria Teresa Yongson Alejandrino SEHA 12
A patient complains of the In a child with a high caries risk, the
discolouration of an unrestored most appropriate cement for a band
maxillary central incisor. and loop space maintainer is
Radiographically, the pulp chamber
and the root canal space are A. zinc phosphate.
obliterated, there is no evidence of B. glass ionomer. *
caries and the periodontal ligament C. zinc oxide eugenol.
space appears normal. An external D. polycarboxylate.
bleaching procedure has not been
successful. The most appropriate Which of the following is the most
management would be to important preventive therapy for an 8
year old living in a non-fluoridated
A. perform nonsurgical root canal community, with closed contacts in the
therapy and nonvital bleaching. posterior quadrants and disto-occlusal
B. perform nonsurgical root canal restorations on all primary first
therapy and fabricate a post retained molars?
porcelain fused to metal crown.
C. perform nonsurgical root canal A. Daily sodium bicarbonate rinse.
therapy and fabricate a porcelain B. Fluoride varnish at 1 month
veneer. intervals.
D. fabricate a porcelain fused to metal C. Weekly 0.2% NaF rinse.
crown. D. Twice daily brushing with fluoride
E. fabricate a porcelain veneer. * toothpaste. *

The epithelium covering the lesions of Which of the following is the hardest?
hyperplastic pulps is believed to be
derived from the A. Amalgam.
B. Enamel. *
A. reduced enamel epithelium. C. Composite resin.
B. epithelium of the gingiva. * D. Type IV alloy.
C. odontoblastic layer.
D. epithelial rests of Malassez. Following the extraction of a primary
E. remnants of the dental lamina. first molar in a 4 year old child, the
most appropriate management is to
The most important factor in
determining the dosage of systemic A. regularly assess the arch
fluoride supplementation is development.
B. perform space analysis.
A. daily water consumption. C. insert a space maintainer. *
B. climate. D. extract the contra-lateral molar.
C. water fluoride concentration. * E. extract the opposing molar.
D. total daily fluoride intake.

Dr Abdul Naser Tamim SEHA, Dr Ghada Al Aqqad D.D.S, Dr Kamal Naser - Amrita Medical Centre, Dr Emad Wani - Shadi Dental Centre,
Dr Lina Anka SEHA, Dr Rouba Zgheibi SEHA, Maria Teresa Yongson Alejandrino SEHA 13
A 6 year old child has a non-vital The most appropriate management is
primary mandibular second molar to
which has a draining sinus tract from
the bifurcation area. The most A. give local anesthetic, pack and
appropriate management is suture.
B. apply firm pressure and ice for 10
A. extraction. * minutes.
B. observation. C. obtain an international normalized
C. pulpotomy. ratio (INR) and a complete
D. direct pulp capping. blood count *
D. give local anesthetic and
Which of the following is an indication electrocauterize the socket.
for a pulpotomy on a primary tooth?
Marginal leakage of a composite resin
A. Radiographic evidence of internal restoration will
root resorption.
B. Involvement of interradicular bone A. not be detectable.
with no fistula. B. be minimized by use of a bonding
C. Involvement of interradicular bone agent. *
with a fistula. C. decrease with longevity.
D. Signs and symptoms of reversible
pulpitis. * Nickel-chromium alloys designed for
porcelain bonded to metal crowns
The diagnosis for a patient having a should be used with caution because
sinus tract associated with a nonvital
pulp who presents with no symptoms is A. nickel is an allergen. *
B. the modulus of elasticity is low.
A. acute apical abscess (acute C. these alloys cannot be soldered.
periradicular abscess).
B. asymptomatic apical periodontitis The extraction of a primary maxillary
(chronic periradicular periodontitis). central incisor at the age of 6 years will
C. periodontal abscess. cause
D. chronic apical abcess (chronic
periradicular abscess). * A. loss of intercanine space.
B. increased intercanine space.
Following a simple extraction of tooth C. no change in intercanine space. *
4.7, hemostasis was achieved. Forty- D. decreased overjet.
eight hours later, there is renewed
bleeding from the extraction site. Firm
pressure fails to achieve hemostasis.

Dr Abdul Naser Tamim SEHA, Dr Ghada Al Aqqad D.D.S, Dr Kamal Naser - Amrita Medical Centre, Dr Emad Wani - Shadi Dental Centre,
Dr Lina Anka SEHA, Dr Rouba Zgheibi SEHA, Maria Teresa Yongson Alejandrino SEHA 14
In a 4 year old the most appropriate
treatment for a chronically infected, The gingival tissues remain healthier
non-restorable primary first molar is to when margins of crowns are placed

A. extract it and place a space A. about lmm below the gingival crest.
maintainer. * B. above the gingival crest. *
B. observe it until it exfoliates. C. at the gingival crest.
C. extract it only.
D. observe it until it becomes Ankylosed primary second molars
symptomatic. may clinically exhibit

A line angle NOT present on a Class I A. percussion sensitivity.


cavity preparation on tooth 1.5 is B. discolouration.
C. temperature sensitivity.
A. mesiopulpal. D. buccolingual displacement.
B. buccopulpal. E. infra-occlusal position. *
C. linguopulpal.
D. axiopulpal. * After completion of root canal therapy
on a maxillary first premolar with
The principal internal retention for a moderate mesial and distal lesions
Class V amalgam cavity preparation is and intact buccal and lingual surfaces,
established at the the restoration of choice is a/an

A. occluso-axial and gingivo-axial line A. MOD composite resin.


angles. * B. MOD onlay. *
B. mesio-axial and disto-axial line C. MOD pin retained amalgam.
angles. D. MOD bonded amalgam.
C. mesio-gingival and disto-gingival
line angles. Which drug is indicated as part of the
management for a codeine overdose?
To minimize fractures of amalgam
restorations in primary teeth, cavity A. Atropine.
preparations should have B. Diphenhydramine.
C. Epinephrine.
A. concave pulpal floors. D. Flumazenil.
B. rounded axiopulpal line angles. * E. Naloxone. *
C. bevelled cavosurface margins.
D. lateral walls parallel to the external
surface of the tooth.

Dr Abdul Naser Tamim SEHA, Dr Ghada Al Aqqad D.D.S, Dr Kamal Naser - Amrita Medical Centre, Dr Emad Wani - Shadi Dental Centre,
Dr Lina Anka SEHA, Dr Rouba Zgheibi SEHA, Maria Teresa Yongson Alejandrino SEHA 15
The most appropriate management for Which of the following clinical
a child with a primary tooth that caused conditions is the most serious?
a severe, throbbing toothache the
previous night is to A. Acute apical abscess of a
mandibular central incisor.
A. prescribe an analgesics. B. Mid facial cellulitis. *
B. prescribe an antibiotic. C. Chronic apical abscess of a
C. remove carie and place a temporary mandibular third molar.
restoration. D. Infected dentigerous cyst.
D. perform a pulpotomy.
E. perform a pulpectomy. * Following the injection of 1.8ml of 2%
lidocaine with 1:100,000 epinephrine,
Which of the following is common to all a nervous 22 year old male with well
forms of hemorrhagic shock? controlled insulin dependent diabetes
states that he feels dizzy and weak.
A. Sepsis. Beads of sweat have accumulated on
B. Hypovolemia. his forehead and upper lip. He is quite
C. Hypertension. pale. The initial management of this
D. Vasoconstriction. patient is to
E. Impaired tissue perfusion. *
A. administer glucagon 1.0mg.
A diastema between two maxillary B. administer epinephrine 0.5mg.
central permanent incisors could be C. administer diphenhydramine 50mg.
associated with the follwings, EXCEPT D. elevate the patient's legs and
administer 100% oxygen. *
A. a mesiodens. E. call 911 and begin CPR.
B. a congenital absence of permanent
maxillary lateral incisors. In the absence of its permanent
C. a large labial frenum. successor, a primary first molar of a 7
D. adenomatoid odontogenetic tumour* year old

Hemorrhagic shock is a circulatory A. should be treated endodontically to


disturbance characterized by prevent root resorption.
B. may remain for years with no
A. increase in blood pressure. significant resorption. *
B. alteration in circulating blood C. should be extracted.
volume. * D. is more susceptible to dental
C. elevation of temperature. caries.
D. decrease in amount of interstitial
fluid.

Dr Abdul Naser Tamim SEHA, Dr Ghada Al Aqqad D.D.S, Dr Kamal Naser - Amrita Medical Centre, Dr Emad Wani - Shadi Dental Centre,
Dr Lina Anka SEHA, Dr Rouba Zgheibi SEHA, Maria Teresa Yongson Alejandrino SEHA 16
A patient has an acute apical abscess
on tooth 1.3. The tooth must be After partial pulpotomy of a permanent
extracted. In addition to a palatine central incisor in an 8 year old child,
injection, the most appropriate local the most important clinical criterion of
anesthetic technique is a/an success is

A. buccal infiltration. A. completion of root formation. *


B. infraorbital block. * B. retained natural colour of the tooth.
C. middle superior alveolar block. C. pulp regeneration.
D. intraligamentary injection. D. formation of pulp stones.

The most important factor in stainless Localized gingival recession of a


steel crown retention in a primary permanent mandibular incisor in an 8
tooth is the year old can be caused by

A. preservation of the coronal bulge. * A. vitamin C deficiency.


B. maintenance of parallelism. B. ankyloglossia. *
C. placement of accessory grooves. C. localized aggressive periodontitis.
D. insertion of retentive pins. D. traumatic occlusion.
E. necrotizing ulcerative gingivitis.
Which of the following conditions is the
most likely cause of a maxillary Direct pulp capping of permanent
midline diastema? teeth in children under the age of 12
years is most likely to be successful
A. Mesiodens. * for
B. Space shortage.
C. Gemination. A. teeth that are symptomatic.
D. Dens invaginatus. B. teeth that are hyperemic.
C. teeth with open apices. *
What is the correct position of the D. pulp exposures 3-5mm in size.
needle tip for the administration of
local anesthetic for an inferior alveolar Extensive caries located only on the
nerve block? primary maxillary incisors of a 2 year
old is generally indicative of
A. Anterior to the buccinator muscle.
B. Medial to the medial pterygoid A. fluorosis of the primary dentition.
muscle. B. fever related hypoplasia.
C. Lateral to the ramus of the C. amelogenesis imperfecta.
mandible. D. demand breast or bottle feeding. *
D. Superior to the mandibular
foramen*
E. Inferior to the pterygomandibular
raphe.

Dr Abdul Naser Tamim SEHA, Dr Ghada Al Aqqad D.D.S, Dr Kamal Naser - Amrita Medical Centre, Dr Emad Wani - Shadi Dental Centre,
Dr Lina Anka SEHA, Dr Rouba Zgheibi SEHA, Maria Teresa Yongson Alejandrino SEHA 17
The diagnosis of a 1.5cm white patch B. canines.
after incisional biopsy is severe C. premolars.
epithelial dysplasia. The most D. molars. *
appropriate management is
In a sagittal split osteotomy of the
A. complete excision. * mandible, the neurovascular bundle
B. repeat biopsy. should remain in which segment of the
C. radiation therapy. mandible?
D. observation.
A. Proximal.
Periodontitis as a manifestation of B. Distal. *
systemic disease as described in the C. Lateral.
Armitage 1999 classification includes D. Condylar.

A. diabetes. A 30 year old male suffers an orbital


B. leukemia. * injury. How many bones form the
C. atherosclerosis. orbit?
D. osteoporosis.
A. 4.
The most likely cause of localized B. 5.
facial gingival recession in a healthy 17 C. 6.
year old individual is D. 7. *
E. 8.
A. minimal attached gingiva.
B. occlusal trauma. The teeth of a Miller bone file are
C. traumatic toothbrushing. * designed to smooth bone with a
D. factitious injury.
A. pull stroke. *
A significant anatomic difference B. push stroke.
between the peri-implant marginal soft C. sawing motion.
tissue and the gingival marginal tissue D. circular motion.
of a tooth exists in the
In a dental office, what is the most
A. epithelial attachment. common cause of respiratory
B. type of collagen. distress?
C. vascular supply. *
D. sulcular epithelium. A. Anaphylaxis.
B. Bronchospasm.
Most commonly, palatal exostoses C. Hyperventilation. *
occur at the level of D. Myocardial infarction.

A. incisors.

Dr Abdul Naser Tamim SEHA, Dr Ghada Al Aqqad D.D.S, Dr Kamal Naser - Amrita Medical Centre, Dr Emad Wani - Shadi Dental Centre,
Dr Lina Anka SEHA, Dr Rouba Zgheibi SEHA, Maria Teresa Yongson Alejandrino SEHA 18
What is the minimum CD4 count to Which of the following can be
perform elective dental treatment? associated with an impacted tooth?

A. 100 cells/mm3. A. Periapical osseous dysplasia


B. 200 cells/mm3. * (periapical cemento-osseous dysplasia).
C. 300 cells/mm3. B. Peripheral odontogenic fibroma.
D. 400 cells/mm3. C. Cementoblastoma.
E. 1000 cells/mm3. D. Ameloblastoma. *
Which of the following organs is A commom feature of the oral mucosal
typically associated with multiple lesions of lichen planus, leukoplakia,
spider angiomas? hyperkeratosis and discoid lupus
erythematosus is that they are
A. Lungs.
B. Liver. * A. premalignant.
C. Kidneys. B. hyperkeratotic. *
D. Pancreas. C. due to chronic infection.
E. Gallbladder. D. granulomatous proliferations.
E. associated with drug use.
The most common location for
periapical osseus dysplasia What is the most likely diagnosis of an
(periapical cemento-osseus ulcerated gingival lesion whose biopsy
dysplasia) is the report confirms epithelial basal layer
separation from the lamina propria?
A. maxillary anterior region.
B. mandibular anterior region. * A. An aphthous ulcer.
C. mandibular premolar region. B. Erosive lichen planus.
D. maxillary posterior region. C. Pemphigus vulgaris.
D. Mucous membrane pemphigoid
If mucous glands are seen in the (cicatricial pemphigoid). *
epithelial lining of a dentigerous
cyst, this is most appropriately Which of the following is NOT
called characteristic of periodontitis?

A. anaplasia. A. Hypertrophy of the gingiva. *


B. metaplasia. * B. Pocket formation without suppuration.
C. dysplasia. C. Destruction of the periodontal
D. neoplasia. ligament.
E. hyperplasia. D. Alveolar bone resorption.

Dr Abdul Naser Tamim SEHA, Dr Ghada Al Aqqad D.D.S, Dr Kamal Naser - Amrita Medical Centre, Dr Emad Wani - Shadi Dental Centre,
Dr Lina Anka SEHA, Dr Rouba Zgheibi SEHA, Maria Teresa Yongson Alejandrino SEHA 19
A patient successfully treated 8 years When smokers are compared to
ago for moderate chronic periodontitis nonsmokers, probing depths after
now presents with generalized surgical periodontal treatment are
erythematous and edematous gingiva
with bleeding on probing. There is no A. greater. *
clinical or radiographic evidence of B. smaller.
further attachment loss. The most C. the same.
likely diagnosis is
Pathologic migration of teeth is
A. gingivitis. * associated with
B. chronic periodontitis.
C. aggressive periodontitis. A. lip incompetence.
D. recurrent periodontitis. B. tongue thrust.
C. bruxism.
During CPR, if a patient is not D. periodontitis. *
breathing but has a pulse, how often
should a breath be given? Every Loss of attachment in periodontal
disease occurs
A. 3-4 seconds.
B. 5-6 seconds. * A. at a consistent rate.
C. 7-8 seconds. B. in cycles lasting for about 3 months.
D. 9-10 seconds. C. in random cycles. *

Which of the following is NOT a A "'pregnancy tumor"' is


feature of aggressive periodontitis?
A. not associated with local irritants.
A. Localized and generalized forms. B. a benign neoplasm.
B. Familial aggregation. C. clinically identical to a pyogenic
C. Necrotic tissue. * granuloma. *
D. Presence of Aggregatobacter
actinomycetemcomitans. A dental restoration with a marginal
discrepancy (void) located 1mm
The earliest clinical sign of gingivitis is subgingivally can affect gingival health
by
A. increased tooth mobility.
B. bleeding on probing. * A. creating an environment conducive
C. change in color of the attached to an altered microbial flora. *
gingiva. B. impinging on the biologic width.
D. change in consistency of the C. causing furcation involvement.
attached gingiva. D. causing leaching of restorative
materials into the gingival crevice.

Dr Abdul Naser Tamim SEHA, Dr Ghada Al Aqqad D.D.S, Dr Kamal Naser - Amrita Medical Centre, Dr Emad Wani - Shadi Dental Centre,
Dr Lina Anka SEHA, Dr Rouba Zgheibi SEHA, Maria Teresa Yongson Alejandrino SEHA 20
Radiographs of a periodontally related In clinically normal periodontium, the
osseous defect can be used to confirm distance between the bottom of the
the gingival sulcus and the alveolar crest is

A. number of bony walls. A. 0.5-1mm.


B. measurement of the clinical B. 1.5-2mm. *
attachment loss. C. 3-5mm.
C. location of the epithelial attachment.
D. presence of bone loss. * Which syndrome has multiple cysts of
the jaws?
The Periodontal Screening and
Recording System (PSR®) is designed A. Gardner’s.
to B. Gorlin. *
C. Peutz-Jeghers.
A. document the progression of D. Sjögren’s.
periodontitis.
B. assess the periodontal status of a Which of the following is most
patient. * radiopaque?
C. measure loss of attachment for
large patient populations. A. Lamina dura.
D. evaluate oral hygiene. B. Enamel.
C. Cortical bone.
Compared to nonsmokers, patients D. Gold inlay. *
who smoke have E. Composite restoration.

A. increased salivary antibodies A radiopaque area within the alveolar


(immunoglobulin A). process containing several rudimentary
B. increased serum IgG antibody teeth suggests a/an
response to periodontal pathogens.
C. decreased serum T lymphocytes. * A. adenomatoid odontogenic tumour.
D. decreased collagenase in B. compound odontoma. *
periodontal tissue. C. complex odontoma.
D. calcifying epithelial odontogenic
What is the most likely diagnosis for a tumour.
patient who complains of dull, constant
pain in his jaws upon awakening? The radiographic change most
suggestive of multiple myeloma is
A. Acute pulpitis.
B. Sinusitis. A. punched-out radiolucent lesions. *
C. Bruxism. * B. multiple radiopaque lesions.
D. Chronic gingivitis. C. diffuse “ground glass” appearance.
D. generalized demineralization.

Dr Abdul Naser Tamim SEHA, Dr Ghada Al Aqqad D.D.S, Dr Kamal Naser - Amrita Medical Centre, Dr Emad Wani - Shadi Dental Centre,
Dr Lina Anka SEHA, Dr Rouba Zgheibi SEHA, Maria Teresa Yongson Alejandrino SEHA 21
Which of the following lesions is most
radiopaque? A. decreasing focal spot-to-object
distance and decreasing object-to-film
A. Calcifying odontogenic cyst. distance.
B. Fibrous dysplasia of bone. B. decreasing focal spot-to-object
C. Ameloblastoma. distance and increasing object-to-film
D. Complex composite odontoma. * distance.
C. increasing focal spot-to-object
A filter is inserted in the path of the x- distance and decreasing object-to-film
ray beam to distance. *
D. increasing focal spot-to-object
A. limit the area of surface exposure. distance and increasing object-to-film
B. absorb scatter radiation. distance.
C. absorb long wave length radiation in
the primary beam. * Increasing the kVp results in
D. make it possible to use higher decreased
kilovoltage for improving image quality.
A. density of the image.
“Ground glass” is the classical B. contrast of the image. *
description of the radiographic C. energy of the x-ray beam.
appearance found in
In radiography, too high a
A. acute osteomyelitis. temperature of the developer will
B. fibrous dysplasia. * cause increased
C. early periapical osseous dysplasia
(periapical cemento-osseous A. contrast.
dysplasia). B. density. *
D. Paget’s disease of bone. C. grey levels.
D. resolution.
Crown-root ratio and residual bone
support can best be seen Proper collimation of the useful beam
radiographically in a for film size and focal spot-film
distance reduces
A. panoramic radiograph.
B. bite-wing radiograph. A. image definition.
C. periapical radiograph using the B. secondary radiation. *
bisecting angle technique. C. radiographic contrast.
D. periapical radiograph using the D. intensity of the central beam.
paralleling technique. *
The most appropriate radiograph(s) to
Mean x-ray beam energy is a function determine the location of an impacted
of maxillary cuspid is/are
A. exposure time.
B. tube current. A. occlusal.
C. tube voltage. * B. periapical.
D. collimation. C. periapical and occlusal. *
D. panoramic.
Radiographic image distortion can be
minimized by

Dr Abdul Naser Tamim SEHA, Dr Ghada Al Aqqad D.D.S, Dr Kamal Naser - Amrita Medical Centre, Dr Emad Wani - Shadi Dental Centre,
Dr Lina Anka SEHA, Dr Rouba Zgheibi SEHA, Maria Teresa Yongson Alejandrino SEHA 22
Bite-wing radiographs are most valuable for Intraosseous nutrient canals are most
detecting frequently seen on which of the following
radiographs?
A. hyperemia of the pulp.
B. occlusal carious lesions. A. Mandibular posterior periapical.
C. proximal surface caries. * B. Mandibular anterior periapical. *
D. cervical caries. C. Maxillary posterior periapical.
D. Maxillary anterior periapical.
Which of the following exposure factors will
result in the most penetrating x-rays? A periapical radiograph reveals a mesiodens in
a 7 year old. The maxillary right central incisor
A. 10kVp - 65mA. has erupted only partially. The maxillary left
B. 85kVp - 5mA. central incisor has not yet appeared. The most
C. 90kVp - 10mA. * appropriate management is to
D. 65kVp - 15mA.
E. 75kVp - 40mA. A. remove the mesiodens and observe
progress carefully. *
Regardless of the focal spot-to-film distance B. allow the mesiodens to erupt before
employed for intraoral radiographic attempting extraction.
examinations, the diameter of the primary C. remove the mesiodens, immediately band
beam at the patient's skin surface should NOT the unerupted central incisor and initiate
be greater than orthodontic therapy.
D. allow the mesiodens and the right central
A. the longest side of the film. incisor to erupt into the oral cavity to determine
B. the size of the lead diaphragm. their relative positions.
C. 7cm. *
D. the size of the filter. The most appropriate opportunity for
orthodontic treatment involving growth
The highest risk of radiation-induced congenital manipulation is
malformation occurs during the
A. during the late primary dentition.
A. First trimester. * B. after the eruption of third molars.
B. Second trimester. C. after skeletal growth is complete.
C. Third trimester. D. during the period of greatest growth velocity.
D. neonatal period. *

The x-rays which are most apt to be absorbed Interceptive orthodontic treatment has NO
by the skin and cause an injury are effect on the

A. central rays. A. final basal bone width of the mandible. *


B. deep penetrating rays. B. direction of growth of the mandible.
C. aluminum filtered rays. C. space available for the eruption of the
D. rays of long wave-length. * permanent dentition.
D. angulation of the teeth in the permanent
dentition

Dr Abdul Naser Tamim SEHA, Dr Ghada Al Aqqad D.D.S, Dr Kamal Naser - Amrita Medical Centre, Dr Emad Wani - Shadi Dental Centre,
Dr Lina Anka SEHA, Dr Rouba Zgheibi SEHA, Maria Teresa Yongson Alejandrino SEHA 23
Maxillary skeletal arch expansion
A. prior to complete eruption of the
A. is easiest in the preadolescent maxillary central incisors.
patient. * B. prior to eruption of the maxillary
B. requires surgery in skeletally lateral incisors.
immature patients. C. after complete eruption of the
C. decreases arch space. maxillary lateral incisors.
D. increases overbite. D. during intra-oral emergence of the
maxillary canines.
The use of a maxillary anterior bite E. after complete eruption of the
plane may result in maxillary canines. *

A. translation of the maxillary incisors. In a child, correction of a bilateral


B. extrusion of the mandibular posterior constriction of the maxillary
incisors. arch has the WORST long term
C. extrusion of the maxillary posterior prognosis for stability if
teeth. *
D. no occlusal changes. A. the maxillary posterior teeth are
centred on the alveolar process.
Ideal orthodontic tooth movement B. a quadhelix is used for treatment.
involves C. there is a functional shift from initial
contact to maximum intercuspation. *
A. large forces. D. there is a history of prolonged
B. undermining resorption. thumb sucking.
C. frontal resorption. *
D. subcrestal incisions. Which of the following malocclusions is
most appropriately corrected
In orthodontics, how much activation is immediately upon diagnosis?
produced from a single turn of a
palatal expansion screw? A. Angle Class II division 1.
B. Angle Class II division 2.
A. 0.25mm. * C. Cross-bite with a lateral functional
B. 0.5mm. shift. *
C. 0.75mm. D. Midline diastema.
D. 1.0mm.
A posterior crossbite in the deciduous
Appropriate forces for orthodontic dentition will most likely
tooth movement are
A. be present in the permanent
A. intermittent and light. dentition. *
B. continuous and heavy. B. self-correct with the eruption of the
C. intermittent and heavy. permanent teeth.
D. continuous and light. * C. self-correct with the cessation of an
associated habit.
Orthodontic closure of a 2mm D. result in inadequate arch length.
maxillary midline diastema is most
appropriately accomplished

Dr Abdul Naser Tamim SEHA, Dr Ghada Al Aqqad D.D.S, Dr Kamal Naser - Amrita Medical Centre, Dr Emad Wani - Shadi Dental Centre,
Dr Lina Anka SEHA, Dr Rouba Zgheibi SEHA, Maria Teresa Yongson Alejandrino SEHA 24
The most appropriate management of
an avulsed maxillary central incisor in a The most appropriate time for surgical
10 year old is the placement of a treatment of a patient with mandibular
prognathism is
A. removable space maintainer. *
B. bonded resin bridge (Maryland A. just before the beginning of the
bridge). prepubertal growth spurt.
C. conventional fixed partial denture. B. just after the end of the prepubertal
D. single tooth implant. growth spurt.
C. just before eruption of the second
The leeway space is permanent molars.
D. just after eruption of the second
A. the distance between the maxillary permanent molars.
and mandibular occlusal surfaces when E. following the completion of growth. *
the mandible is in rest position.
B. the difference in size between the Which of the following is a possible
combined widths of the primary cuspids cause of an anterior crossbite?
and molars and the permanent cuspids
and bicuspids. * A. An abnormal labial frenum.
C. the space distal to the mandibular B. A skeletal Class II malocclusion.
primary canine and mesial to the C. The early loss of a primary maxillary
maxillary primary canine. canine.
D. the space between primary anterior D. An over-retained primary maxillary
teeth. incisor. *

When the primary maxillary second A Class III malocclusion is normally


molar is lost prematurally, the associated with
permanent first molar usually moves
with the A. sleeping habits.
B. growth discrepancy. *
A. mesiobuccal cusp rotating lingually. * C. tooth size - jaw size discrepancy.
B. mesiobuccal cusp rotating buccally. D. trauma.
C. crown tipping buccally.
D. crown tipping lingually Ankylosis of the primary maxillary
central incisors is commonly related to
In an 8-year old, the optimum time for
treatment of an anterior crossbite of A. trauma. *
dental origin is B. thumbsucking.
C. chronic gingivitis.
A. as soon as possible. * D. delayed eruption of the permanent
B. after the maxillary canines are in teeth.
position.
C. after the maxillary premolars erupt.
D. following completion of jaw growth.

Dr Abdul Naser Tamim SEHA, Dr Ghada Al Aqqad D.D.S, Dr Kamal Naser - Amrita Medical Centre, Dr Emad Wani - Shadi Dental Centre,
Dr Lina Anka SEHA, Dr Rouba Zgheibi SEHA, Maria Teresa Yongson Alejandrino SEHA 25
A tongue thrust is most often found in An Angle Class II, division 1
a child with malocclusion can be differentiated
from an Angle Class II, division 2
A. a deep overbite. malocclusion based upon the
B. an anterior open-bite. *
C. an Angle Class II, division 2 A. molar relationship.
malocclusion. B. severity of the Angle Class II
D. a bimaxillary protrusion. malocclusion.
C. amount of overbite.
A 4 year old with a thumbsucking D. inclination of maxillary incisors. *
habit presents with a 1.5cm anterior E. amount of crowding present.
open bite. If the habit ceases within
six months, the anterior open bite will, The term "subdivision" in malocclusion
over time, most likely classification refers to molar
relationships that are
A. remain the same.
B. increase. A. bilateral.
C. decrease. * B. unilateral. *
D. decrease and the midline will C. mildly abnormal.
correct. D. severely abnormal.

A mesial step relationship most Enamel maturation is completed


commonly results in which of the
following permanent first molar A. when the enamel organ is complete.
relationships? B. prior to eruption but after root
formation has begun. *
A. Angle Class I. * C. at eruption.
B. Angle Class II. D. following eruption.
C. Angle Class III.
The start of a mandibular bone
The area most susceptible to formation coincides with
excessive space loss following
premature loss of a primary tooth is in A. tooth germ development.
the region of the B. completion of fusion of the facial
processes.
A. mandibular first premolar. C. bone formation in the maxilla.
B. maxillary first premolar. D. completion of a mandible cartilage
C. mandibular second premolar. model. *
D. maxillary second premolar. *

Dr Abdul Naser Tamim SEHA, Dr Ghada Al Aqqad D.D.S, Dr Kamal Naser - Amrita Medical Centre, Dr Emad Wani - Shadi Dental Centre,
Dr Lina Anka SEHA, Dr Rouba Zgheibi SEHA, Maria Teresa Yongson Alejandrino SEHA 26
A thumbsucking habit should be Twenty-four hours following the simple
extraction of tooth 4.7, a patient
A. ignored. presents with severe bleeding from the
B. of concern if it persists beyond 5 extraction site. The most likely cause
years of age. * of this bleeding is a/an
C. considered a manifestation of the
natural nutritional needs of the child. A. disturbance of the hemostatic plug*
D. considered a primary cause of Class B. infection of the socket.
II division 1 malocclusion. C. undiagnosed coagulopathy.
D. dry socket.
Between the ages of 6 and 12,
mandibular arch length will normally An alcoholic patient with bleeding
tendencies may require injection of
A. increase with the eruption of the which vitamin prior to an extraction?
permanent mandibular canines.
B. increase with eruption of the A. Vitamin B12.
mandibular premolars. B. Vitamin C.
C. remain the same. C. Vitamin D.
D. decrease with the eruption of the D. Vitamin K. *
permanent mandibular incisors.
E. decrease with the eruption of the When prescribing nonsteroidal anti-
mandibular premolars. * inflammatory drugs (NSAIDs), it is
important to consider that
The permanent maxillary canine most prostaglandins
commonly erupts
A. impair blood coagulation.
A. before the mandibular permanent B. induce vasoconstriction.
canine. C. prevent edema.
B. before the maxillary first premolar. D. protect the gastric mucosa. *
C. before the maxillary second
permanent molar. * A patient complains of acute pain 24
D. after the maxillary second hours after the insertion of a
permanent molar. restoration in a tooth with no pre-
existing periapical pathology. The
A mandibular fracture during normal tooth is vital and tender to percussion.
mastication is most likely to occur in a The radiograph will show
patient with
A. an apical radiolucency.
A. osteoporosis. B. osteosclerosis.
B. a large intraosseous lesion. ?? C. condensing osteitis.
C. an impacted tooth at the inferior D. a normal apex. *
border.
D. advanced alveolar atrophy. *

Dr Abdul Naser Tamim SEHA, Dr Ghada Al Aqqad D.D.S, Dr Kamal Naser - Amrita Medical Centre, Dr Emad Wani - Shadi Dental Centre,
Dr Lina Anka SEHA, Dr Rouba Zgheibi SEHA, Maria Teresa Yongson Alejandrino SEHA 27
Fordyce spots or granules is/are E. 0.40.

A. sweat glands. Upon returning to the operatory, the


B. implanted epithelium. dentist notices the patient is looking at
C. cystic formations. the dental chart. Which of the following
D. sebaceous glands. * statements is correct?
E. hyperkeratosis.
A. The patient has a legal right to
Pulp vitality tests are used to access the chart at any time.
differentiate between B. The dentist can charge an
administrative fee to the patient to view
A. periapical osseous dysplasia the chart.
(periapical cemento-osseous dysplasia) C. A written request is required before
and a periapical granuloma. * access to the chart can be granted. *
B. periapical osseous dysplasia
(periapical cemento-osseous dysplasia) Which of the following variables has
and an incisive canal cyst. the greatest effect on bond strength?
C. periapical granuloma and a periapical
cyst. A. Moisture control. *
D. periapical granuloma and a periapical B. Size of the bracket base.
abscess. C. Type of acid etch.
D. Composition of the resin cement.
A patient with end-stage renal disease E. Patient oral hygiene.
has several multilocular radiolucent
lesions associated with vital mandibular What is the most appropriate
teeth. A histopathologic analysis reveals medication to control the dental pain
the presence of multinucleated giant following the adjustment of orthodontic
cells. The most appropriate appliances in a child?
management for this patient is to
A. Acetaminophen. *
A. initiate endodontic treatments. B. Ibuprofen.
B. perform a mandibular resection. C. Naproxen.
C. extract the involved teeth. D. Aspirin.
D. obtain a serum parathormone (PTH)
level. * Pulpectomy is the most appropriate
E. commence antibiotic therapy. treatment in a primary tooth with

What is the dose of epinephrine in an A. reversible pulpitis.


Epi Pen®? B. advanced internal root resorption.
C. advanced external root resorption.
A. 0.20. D. radicular pulp necrosis. *
B. 0.25.
C. 0.30. *
D. 0.35.

Dr Abdul Naser Tamim SEHA, Dr Ghada Al Aqqad D.D.S, Dr Kamal Naser - Amrita Medical Centre, Dr Emad Wani - Shadi Dental Centre,
Dr Lina Anka SEHA, Dr Rouba Zgheibi SEHA, Maria Teresa Yongson Alejandrino SEHA 28
What is the most likely diagnosis of a 9 Which of the following is NOT an
year old with speech problems, early clinical manifestation of local
macroglossia, unilateral premature anesthetic overdose?
eruption of teeth, and moderate
scoliosis? A. Nystagmus.
B. Slurred speech.
A. Down syndrome. C. Decreased heart rate. *
B. Hemi hypertrophy. * D. Increased respiratory rate.
C. Gigantism.
D. Cleidocranial dysplasia. What is the dose of epinephrine in
an EpiPen®Jr.?
For which of the following medical
emergencies is atropine most A. 0.15. *
commonly used? B. 0.20.
C. 0.25.
A. Anaphylaxis. D. 0.30.
B. Bradycardia. * E. 0.35.
C. Bronchospasm.
D. Hypotension. A 55 year old patient wants to
maintain bone mass following
Which of the following is NOT a implant surgery and plans to take
manifestation of hyperglycemia? extra calcium. Supplemental calcium
is most beneficial when consumed in
A. Moist skin. *
B. Acetone breath. A. divided doses with meals. *
C. Abdominal pain. B. divided doses between meals.
D. Decreased consciousness. C. a single dose with meals.
D. a single dose between meals.
Which of the following is NOT a clinical
finding of hypothyroidism? A patient with bulimia who purges
several times a day should be
A. Anxiety. advised to increase intake of
B. Mild tremor.
C. Puffy eyelids. A. lean chicken.
D. Exophthalmos. * B. bananas. *
C. whole wheat bread.

Dr Abdul Naser Tamim SEHA, Dr Ghada Al Aqqad D.D.S, Dr Kamal Naser - Amrita Medical Centre, Dr Emad Wani - Shadi Dental Centre,
Dr Lina Anka SEHA, Dr Rouba Zgheibi SEHA, Maria Teresa Yongson Alejandrino SEHA 29
A 65 year old, underweight patient D. T2N2M1.
confirms a longstanding problem with
alcohol abuse. The medical history Which of the following is NOT a risk
reveals complaints of muscle factor for primary molar ankylosis?
weakness, low appetite, lack of energy
and forgetfulness, which the patient A. Bruxism. *
attributes to getting old. The patient’s B. Genetics.
diet is most likely deficient in C. Trauma.
D. Missing permanent successor.
A. ascorbic acid.
B. folic acid. Which of the following is/are a risk of
C. thiamin. * orthodontic treatment?
D. retinol.
1. Discomfort.
What is the most probable syndrome 2. Decalcification.
affecting a 9 year old patient with a 3. Root resorption.
history of 3 keratocystic odontogenic 4. Development of a craniofacial
tumours (odontogenic keratocyst)? syndrome.

A. Nevoid basal cell carcinoma A. (1) (2) (3) *


syndrome (Gorlin Syndrome). * B. (1) and (3)
B. Familial colorectal polyposis C. (2) and (4)
(Gardner’s Syndrome). D. (4) only
C. Crouzon Syndrome. E. All of the above.
D. Apert Syndrome.
When using fixed orthodontic brackets,
A 70 year old patient was diagnosed teeth will commonly
with squamous cell carcinoma of the
lateral border of the tongue. The 1. Tip.
tumour measures 3.5 x 3.0 cm. A CT of 2. Translate.
the neck confirms the presence of a 3. Rotate.
2.5 cm ipsilateral submandibular lymph 4. Intrude.
node suggestive of locoregional
disease. Chest X-rays and liver A. (1) (2) (3)
function tests were within normal limits. B. (1) and (3)
The TNM classification of this patient’s C. (2) and (4)
disease is D. (4) only
E. All of the above. *
A. T1N1M0.
B. T1N2M0.
C. T2N1M0. *

Dr Abdul Naser Tamim SEHA, Dr Ghada Al Aqqad D.D.S, Dr Kamal Naser - Amrita Medical Centre, Dr Emad Wani - Shadi Dental Centre,
Dr Lina Anka SEHA, Dr Rouba Zgheibi SEHA, Maria Teresa Yongson Alejandrino SEHA 30
Temporary anchorage devices (TADS)
have the greatest effect on which Which of the following improves the
orthodontic movement? prognosis of pulp vitality after indirect
pulp capping?
A. Extrusion.
B. Torqueing. 1. Use of calcium hydroxide.
C. Intrusion. * 2. Having a well sealed restoration.
D. Tipping. 3. Reducing other traumas to the pulp.
4. Removing all caries in the pulp cap
The most effective appliance to correct area.
mid-face retrusion in the early mixed
dentition is (a) A. (1) (2) (3) *
A. inter-arch elastics. B. (1) and (3)
B. chin cup. C. (2) and (4)
C. reverse-pull headgear. * D. (4) only
D. bionator. E. All of the above.

What is the correct rate of ventilation Which of the following is/are part of
for an unconscious patient with a optimal post preparation procedure?
palpable pulse?
1. Use of non-end cutting rotary
A. 4-6 breaths per minute. instruments.
B. 6-8 breaths per minute. 2. Minimal canal enlargement.
C. 8-10 breaths per minute. 3. Passive post fit.
D. 10-12 breaths per minute. * 4. Diameter of the post that is half the
E. 12-14 breaths per minute. width of the root.

Which of the following CANNOT be A. (1) (2) (3) *


viewed on a lateral cephalometric B. (1) and (3)
radiograph? C. (2) and 4)
D. (4) only
A. Upper airway clearance. E. All of the above.
B. Mandibular dimensions.
C. Maxillary constriction. *
D. Lower face height.

Dr Abdul Naser Tamim SEHA, Dr Ghada Al Aqqad D.D.S, Dr Kamal Naser - Amrita Medical Centre, Dr Emad Wani - Shadi Dental Centre,
Dr Lina Anka SEHA, Dr Rouba Zgheibi SEHA, Maria Teresa Yongson Alejandrino SEHA 31
When preparing a post space in a D. (4) only
figure-8 shaped premolar, which of the E. All of the above. *
following is/are appropriate?
When compared to parallel-sided
1. Use one post in the straightest posts, tapered posts
canal.
2. Achieve post fit between the mesial 1. Require less dentin removal.
and distal walls of the straightest 2. Exert a wedge effect on the
canal. remaining dentin.
3. Extend core material 2mm apically 3. are more consistent with root
in the nonposted section of the figure- anatomy.
8. 4. Provide better retention.
4. Remove tooth structure to make an
oval shaped canal. A. (1) (2) (3) *
B. (1) and (3)
A. (1) (2) (3) * C. (2) and (4)
B. (1) and (3) D. (4) only
C. (2) and (4) E. All of the above.
D. (4) only
E. All of the above. Posts in endodontically treated teeth
should have
Which of the following will improve the
prognosis of a rebonded fractured 1. High strength.
tooth fragment? 2. A low elastic limit to flex with the
tooth.
1. Total etching and dentin bonding. 3. Good radiopacity.
2. Use of a groove in the fragment. 4. Bondability to tooth structure.
3. Having a single larger fragment to
rebond. A. (1) (2) (3) *
4. Reinforcing the fracture line with B. (1) and (3)
composite. C. (2) and (4)
D. (4) only
A. (1) (2) (3) E. All of the above.
B. (1) and (3)
C. (2) and (4)

Dr Abdul Naser Tamim SEHA, Dr Ghada Al Aqqad D.D.S, Dr Kamal Naser - Amrita Medical Centre, Dr Emad Wani - Shadi Dental Centre,
Dr Lina Anka SEHA, Dr Rouba Zgheibi SEHA, Maria Teresa Yongson Alejandrino SEHA 32
If a tooth has an inadequate ferrule, lymphadenopathy and bad breath.
which of the following is/are effective Vesicles are seen on the hard palate,
strategies to increase tooth structure the soft palate, the gingiva, the tongue
available for crown preparation? and the lips. The patient’s temperature
is 38.5ºC. What is the most probable
1. Surgical crown lengthening. diagnosis?
2. Sub-gingival preparation and
prolonged temporization. A. Erythema multiforme.
3. Orthodontic eruption. B. Primary herpetic gingivostomatitis. *
4. Elective endodontic treatment C. Hand-foot-mouth disease.
gaining retention from a post in the D. Herpetiform aphtaus ulcers.
root.
A patient in the late mixed dentition
A. (1) and (3) * stage has an end-to-end first molar
B. (2) and (4) relationship with normal skeletal and
C. (1) (2) (3) incisor relationships. What is the most
D. (4) only probable occlusal relationship at age
E. All of the above. 20?

Which of the following is the most A. Angle Class II without anterior


appropriate solution for controlling overlap.
pulpal bleeding after a pulp exposure B. Angle Class II with anterior overlap.
prior to pulp capping? C. Angle Class I without anterior
overlap.
A. Saline. * D. Angle Class I with anterior overlap. *
B. Chlorhexidine.
C. Sodium hypochlorite. Root resorption of primary teeth can be
D. Ferric sulfate. accelerated by

Dentists may choose NOT to provide A. the absence of a permanent


treatment for a patient when the replacement.
B. the premature loss of the adjacent
1. Treatment cannot be completed in a primary teeth.
timely fashion. C. the presence of premature occlusal
2. Patient has a serious communicable contact.
disease. D. inflammation of the pulp. *
3. Dentist does not have sufficient
training for procedure. Root resorption of primary teeth can be
4. Treatment is for an urgent situation. delayed by

A. (1) and (3) * A. the absence of a permanent


B. (2) and (4) replacement. *
C. (1) (2) (3) B. the premature loss of adjacent
D. (4) only primary teeth.
E. All of the above. C. the presence of premature occlusal
contact.
A 3 year old patient complains of pain D. inflammation of the pulp.
of the mouth and headaches for 24 E. occlusal crossbite.
hours. The clinical exam reveals
bilateral painful regional

Dr Abdul Naser Tamim SEHA, Dr Ghada Al Aqqad D.D.S, Dr Kamal Naser - Amrita Medical Centre, Dr Emad Wani - Shadi Dental Centre,
Dr Lina Anka SEHA, Dr Rouba Zgheibi SEHA, Maria Teresa Yongson Alejandrino SEHA 33
The mandibular intercanine width increases until A. decrease in the curve of Spee.
full eruption of the permanent B. decrease in the overbite and overjet.
C. increase in frequency of eruption irregularities*
A. incisors. * D. increase in vertical dimension.
B. canines.
C. Second molars. The most appropriate management for atypical
D. Third molars. swallowing is

In a Tanaka and Johnson mixed dentition A. tongue trap appliance.


analysis, the mesiodistal width of permanent B. occipital traction.
maxillary canines and premolars is determined by C. myofunctional treatment. *
the width of the D. Hawley appliance.

A. permanent maxillary incisors. The most appropriate management of a


B. permanent mandibular incisors. * mandibular lateral incisor that has erupted
C. primary canines and maxillary molars. lingually to the corresponding primary tooth is to
D. primary canines and mandibular molars.
A. monitor the situation only. *
A 10 year old patient is missing a permanent B. extract the primary tooth and monitor.
maxillary left lateral incisor. There are no other C. extract the primary tooth and immediately
orthodontic problems. A dental implant is planned reposition the permanent tooth.
when the patient is an adult. The most
appropriate management is to The most appropriate time to initiate surgical
treatment of a prognathic mandible immediately
A. extract the primary maxillary left canine as follows the
soon as possible.
B. encourage the permanent left canine to erupt A. primary dentition.
into an Angle’s Class I relationship. B. early mixed dentition.
C. let the permanent canine erupt into a mesial C. late mixed dentition.
position and reposition it into an Angle’s Class I D. cessation of growth. *
as soon as possible.
In orthodontic treatment, extraction of premolars
D. let the permanent canine erupt into a mesial
will help to
position and reposition it into an Angle’s Class I
before placing the implant. E. let the permanent
A. open the vertical dimension.
canine erupt into a mesial position and place the
implant distal to the canine. B. labialize the incisors.
C. correct a midline deviation.
Premature loss of primary molars may result in D. improve stability. *

Dr Abdul Naser Tamim SEHA, Dr Ghada Al Aqqad D.D.S, Dr Kamal Naser - Amrita Medical Centre, Dr Emad Wani - Shadi Dental Centre,
Dr Lina Anka SEHA, Dr Rouba Zgheibi SEHA, Maria Teresa Yongson Alejandrino SEHA 34
A 3 year old complains of a sore left Folate deficiency is associated with an
arm for 10 days. A physician has increased risk of
diagnosed a fracture and a radiograph
reveals an old fracture. The most A. birth defects. *
appropriate diagnosis is B. microcytic anemia.
C. low serum homocysteine.
A. infantile osteoporosis. D. elevated high-density lipoprotein
B. osteogenesis imperfecta. * cholesterol.
C. battered child syndrome.
D. Münchhausen syndrome. Which of the following is the most
important determinant for the
Epinephrine antagonizes the effects of maximum length of a post in an
histamine by endodontically treated tooth?

A. preventing the release of histamine. A. Distance between the crestal bone


B. acting on the central nervous and root apex.
system. B. Crown/root ratio.
C. producing physiologic actions C. Maintenance of an apical gutta-
opposite to that of histamine. * percha seal. *
D. competitive blocking of histamine at
the cellular receptor site. The vibrating line of the palate

The principal action of caffeine is on A. delineates the movement of the soft


the palate. *
B. is always on the hard palate.
A. cerebral cortex. * C. is a well-defined line across the
B. corpus callosum. palate.
C. hypothalamus. D. is not a useful landmark in complete
D. spinal cord. denture fabrication.

The presence of hepatitis B surface Which of the following is the most


antigen (HBsAg) and hepatitis B e effective method of cleaning
antigen (HBeAg) in blood indicates the endodontic instruments prior to
individual sterilization?

A. is infectious for hepatitis B. * A. Manual brushing.


B. has acquired immunity to hepatitis B. High pressure water jet.
B. C. Washing with antimicrobial soap.
C. is not infectious for hepatitis B and D. Washing in an ultrasonic bath for 5
has not acquired immunity to hepatitis minutes. *
B.
D. has never been infected with
hepatitis B.

Dr Abdul Naser Tamim SEHA, Dr Ghada Al Aqqad D.D.S, Dr Kamal Naser - Amrita Medical Centre, Dr Emad Wani - Shadi Dental Centre,
Dr Lina Anka SEHA, Dr Rouba Zgheibi SEHA, Maria Teresa Yongson Alejandrino SEHA 35
A 65 year old patient who has
recovered from a stroke 6 months A nonsurgical endodontic treatment
previously and has a history of was completed on tooth 1.6 six months
endocarditis requires the extraction of ago. The temporary restoration was
a mandibular molar. The vital signs are lost two months ago. The tooth has no
Blood Pressure: 135/85 mmHg caries or fractures. All cusps are
Pulse: 76/min present but are weak and unsupported.
Respiratory Rate: 16/min What is the most appropriate
The most appropriate immediate management prior to placing a full gold
management of this patient is to crown?

A. proceed with the treatment. A. Placing a bonded composite resin


B. use prophylactic antibiotics. * restoration to seal the access.
C. use local anesthetic with no B. Post placement in the palatal canal
epinephrine. followed by a core build-up.
D. delay treatment for 3 months. C. Removal of 4-5mm of coronal gutta-
percha from all canals followed by a
Although a dentist recommends an all- core build-up.
ceramic crown to a patient who D. Endodontic retreatment followed by
requires restoration of a fractured a core build-up. *
maxillary lateral incisor, the patient
requests a full gold crown be placed. Radiographic examination reveals
The most appropriate management is early evidence of internal resorption.
to Which is the most appropriate initial
management?
A. insist on placing a more esthetic
crown. A. Perform an apicoectomy.
B. place a provisional crown on the B. Perform a pulp extirpation. *
tooth indefinitely. C. Perform a pulpotomy.
C. place a full gold crown as the patient D. Observe and re-evaluate in 3-6
requested. * months.
D. restore the tooth with composite
until the patient reconsiders. The most appropriate management for
a patient with bilateral
Overadjustment of a wrought wire temporomandibular joint clicking
denture clasp can lead to fracture without pain is
because of a/an
A. no treatment. *
B. temporomandibular joint surgery.
A. increase in modulus of elasticity. C. an anterior repositioning splint.
B. decrease in ductility due to strain D. cortisone injections.
hardening. *
C. decrease in the yield strength.
D. increase in fracture toughness.

Dr Abdul Naser Tamim SEHA, Dr Ghada Al Aqqad D.D.S, Dr Kamal Naser - Amrita Medical Centre, Dr Emad Wani - Shadi Dental Centre,
Dr Lina Anka SEHA, Dr Rouba Zgheibi SEHA, Maria Teresa Yongson Alejandrino SEHA 36
A 45 year old with insulin-dependent D. reduce the diameter of the primary
diabetes mellitus has a morning dental beam. *
appointment. During the examination, E. reduce exposure time.
the patient complains of being
lightheaded and weak. Sweating is Solitary eosinophilic granuloma is
observed. The patient is most likely associated with
experiencing
A. multiple myeloma.
A. hyperglycemia. B. hyperparathyroidism.
B. hypoglycemia. * C. hypoparathyroidism.
C. syncope. D. Langerhans cell histiocytosis. *
D. hyperventilation.
E. cerebrovascular accident. In which of the following conditions is
there a risk of malignant change after
Maxillary furcation involvements are radiotherapy?
best assessed clinically by probing with
a A. Fibrous dysplasia. *
B. Mucocele.
A. Williams probe. C. Lymphangioma.
B. Nabers probe. * D. Torus palatinus.
C. Michigan “O” probe.
D. Periodontal Screening and A 27 year old woman complains of
Recording (PSR) probe. burning mouth, fatigue, palpitations
and lack of energy. An oral exam
Patients who are positive for the shows angular cheilitis and atrophic
interleukin-1 (IL-1) glossitis. What is the most likely
diagnosis?
A. are at increased risk for severe
periodontal disease. * A. Iron deficiency anemia. *
B. have a decreased inflammatory B. Crohn’s disease.
response in the presence of bacteria.
C. Chronic lymphocytic leukemia.
C. are more likely to respond
favourably to periodontal therapy. D. Vitamin B deficiency.
D. have decreased bacterial pathogens
associated with active periodontal The cell of the dental pulp most
disease. capable of transforming into other cells
is the
The main purpose of collimation of an
x-ray beam is to A. fibroblast.
B. mesenchymal cell. *
A. permit the use of lower kilovoltage C. odontoblast.
during exposure. D. histiocyte.
B. filter out useless short rays.
C. permit use of the long cone
technique.

Dr Abdul Naser Tamim SEHA, Dr Ghada Al Aqqad D.D.S, Dr Kamal Naser - Amrita Medical Centre, Dr Emad Wani - Shadi Dental Centre,
Dr Lina Anka SEHA, Dr Rouba Zgheibi SEHA, Maria Teresa Yongson Alejandrino SEHA 37
Crestal bone loss around implants prior Which of the following conditions is
to occlusal loading is more significantly NOT a sequela of a tooth completely
associated with impacted in bone?

A. smooth collars. A. Development of a


B. roughened collars. dentigerous/follicular cyst around its
C. internal connections. crown.
D. external connections. * B. External resorption of the tooth.
C. Osteonecrosis of the adjacent
A tissue-level implant should be used bone. *
when D. Development of a benign neoplasm
adjacent to its crown.
A. the edentulous site is in the esthetic
zone. A 10 year old girl has poorly formed,
B. platform switching is desired. distorted permanent right maxillary
C. a 2-stage surgical approach is canine, lateral and central incisors.
planned. The remaining teeth and supporting
D. ease for oral hygiene is desired to structures are unremarkable. The
preserve crestal bone. * patient most likely has

Which of the following is NOT a A. ectodermal dysplasia.


malignancy? B. segmental odontomaxillary
dysplasia.
A. Leukemia. C. radicular dentin dysplasia.
B. Lymphoma. D. regional odontodysplasia. *
C. Leiomyoma. * E. coronal dentin dysplasia.
D. Melanoma.
The palatine tonsils are found
A strong association exists between
the presence of fissured tongue and A. surfacing the posterior third of
the presence of tongue dorsum.
B. between the palatoglossal and
A. geographic tongue. * palatopharyngeal arches. *
B. hairy tongue. C. on the posterior wall of the
C. lingual varicosities. nasopharynx.
D. median rhomboid glossitis. D. on the bilateral lateral ventral
aspects of the tongue.
Which of the following lesions may
appear radiolucent on a radiograph? Which of the following is LEAST likely
to share histological features with
A. Dense bone island/idiopathic lichen planus?
osteosclerosis.
B. Periapical osseous dysplasia. * A. White sponge nevus. *
C. Sialolith. B. Oral mucosal cinnamon reaction.
D. Osteopetrosis. C. Oral graft-versus-host disease.
D. Lupus erythematosus.

Dr Abdul Naser Tamim SEHA, Dr Ghada Al Aqqad D.D.S, Dr Kamal Naser - Amrita Medical Centre, Dr Emad Wani - Shadi Dental Centre,
Dr Lina Anka SEHA, Dr Rouba Zgheibi SEHA, Maria Teresa Yongson Alejandrino SEHA 38
A feature of primary syphilis is using local anesthesia. It is
appropriate to defer treatment if the
A. atrophic glossitis. patient’s blood pressure is equal to or
B. a gumma. above
C. a chancre. *
D. a mucous patch. A. 120/80.
B. 140/90.
Mucosal thickening in the maxillary C. 160/110. *
sinus D. 180/110.

1. May fill the entire air space of the A 20 year old has a solitary
sinus. radiolucent lesion in the left maxillary
2. Is often usually an incidental tuberosity. The histologic diagnosis is
radiographic finding. an odontogenic
3. May be caused by a periapical keratocyst/keratogenic odontogenic
infection. tumour. What is the recurrence rate of
4. May be associated with nasal this tumour?
discharge.
A. It does not recur.
A. (1) (2) (3) B. 10-30%. *
B. (1) and (3) C. 40-60%.
C. (2) and (4) D. 70-90%.
D. (4) only
E. All of the above. * What is the most common intraoral
location of squamous cell carcinoma?
Which of the following is LEAST likely
to be associated with fissured tongue? A. Lateral tongue. *
B. Buccal mucosa.
A. Oral lichen planus. * C. Tonsilar bed.
B. Melkersson-Rosenthal syndrome. D. Attached gingiva.
C. Geographic tongue.
Which of the following statements
A dentist who has been diagnosed as regarding physiologic changes in an
a chronic Hepatitis B carrier can elderly patient is correct?
perform exposure prone procedures
A. Creatinine production is reduced. *
A. using Standard precautions. * B. Plasma albumin is increased.
B. in the absence of HBeAg. C. Relative tissue perfusion is
C. following a review by an Expert unaltered.
Panel.

A 74 year old patient being treated for


hypertension requires deep scaling

Dr Abdul Naser Tamim SEHA, Dr Ghada Al Aqqad D.D.S, Dr Kamal Naser - Amrita Medical Centre, Dr Emad Wani - Shadi Dental Centre,
Dr Lina Anka SEHA, Dr Rouba Zgheibi SEHA, Maria Teresa Yongson Alejandrino SEHA 39
Which of the following drugs can be The CNS depressant effects of
administered to manage pain following midazolam are reversed by the
the acute onset of a migraine? administration of

A. Amitriptyline. A. diphenhydramine.
B. Nifedipine. B. flumazenil. *
C. Propranolol. C. naloxone.
D. Sumatriptan. * D. ranitidine.

The metabolic clearance of which of Which of the following drugs is


the following drugs is NOT reduced in CONTRAINDICATED in patients with
an elderly patient? Long QT syndrome?

A. Carbamazepine. A. Azithromycin. *
B. Warfarin. B. Clotrimazole.
C. Morphine. C. Diazepam.
D. Clindamycin. * D. Acyclovir.

Infections transmitted in dental offices An 89 year old patient has rampant


have been documented caries secondary to vestibular food
pocketing. The LEAST likely
A. at a rate comparable to health care predisposing disease is
associated infections in hospitals.
B. occasionally, primarily with hepatitis A. Parkinson’s.
B. * B. Lewy body dementia.
C. with equal frequency for viruses C. Alzheimer’s.
and bacteria. D. Addison’s. *
D. primarily in offices, which do not
control bacterial levels in water from Which of the following statements
dental units. regarding geriatric patients is correct?

After latex gloves have been put on, A. Drugs that primarily depend on
washing the gloved hands with an pulmonary excretion for elimination
antimicrobial soap should be prescribed in smaller doses
or at greater time intervals.
A. adds a further layer of safety to B. Multiple drug ingestion is less likely
barrier protection. to result in toxicity due to inhibition of
B. is unnecessary but acceptable biotransformation.
practice. C. Multiple drug use predisposes the
C. can compromise the integrity of the elderly to multiple side effects that may
glove. * impact on their oral health. *
D. should be done if a surgical
procedure is planned.

Dr Abdul Naser Tamim SEHA, Dr Ghada Al Aqqad D.D.S, Dr Kamal Naser - Amrita Medical Centre, Dr Emad Wani - Shadi Dental Centre,
Dr Lina Anka SEHA, Dr Rouba Zgheibi SEHA, Maria Teresa Yongson Alejandrino SEHA 40
Which of the following analgesics is
CONTRAINDICATED for a 76 year old A. Alendronate.
patient with angina? B. Pilocarpine.
C. Glycopyrrolate. *
A. Acetaminophen. D. Nystatin.
B. Ibuprofen.
C. Codeine. An appropriate reason to delay the
D. Tramadol. * extraction of a mandibular third molar
with acute pericoronitis is
Which of the following statements
regarding physiologic changes in an A. risk of osteomyelitis.
elderly patient is NOT correct? B. risk of trismus.
C. difficulty achieving anesthesia.
A. Reduced gastric emptying can lead D. risk of spread of infection. *
to a decrease in drug absorption.
B. Renal blood flow is decreased. Which of the following impression
C. Lipid soluble drugs remain in the materials will still result in an accurate
body for a longer period of time. cast when poured two weeks after
D. Relative tissue perfusion is making the impression?
unaltered. *
A. Polyether.
The percent of elderly persons known B. Polysulfide.
to colonize asymptomatic C. difficile in C. Addition silicone. *
the gut is typically in a range of up to D. Condensation silicone.

A. 3%. Which of the following statements is


B. 10%. true regarding the radiographic
C. 30%. appearance of furcation involvements?
D. 50%. *
A. A definitive diagnosis can be made
Which of the following drugs inhibits from a radiograph.
salivary flow? B. Bone loss is greater than it appears
on a radiograph. *
A. Penicillin V. C. They are best evaluated with
B. Loratadine. periapical radiographs.
C. Probantheline. * D. Furcation involvements cannot be
D. Nystatin. seen on radiographs.

Which of the following drugs inhibits


salivary flow?

Dr Abdul Naser Tamim SEHA, Dr Ghada Al Aqqad D.D.S, Dr Kamal Naser - Amrita Medical Centre, Dr Emad Wani - Shadi Dental Centre,
Dr Lina Anka SEHA, Dr Rouba Zgheibi SEHA, Maria Teresa Yongson Alejandrino SEHA 41
In the Vita® Classical Shade Guide, B. 30-40.
the hue of the A shade series is C. 50-60. *

A. red. The most appropriate vertical


B. grey. angulation to make a bite-wing
C. yellow. radiograph in the primary dentition is
D. brown. *
A. – 10º.
In the Vita® Classical Shade Guide, B. – 5º.
the hue of the D shade series is C. 0º.
D. + 5º.
A. red. * E. + 10º. *
B. grey.
C. yellow. What is the most appropriate film size
for making an occlusal radiograph in a
D. brown.
3 year old patient?
The most appropriate recall interval for
an 8 year old patient with high caries A. 0.
risk is B. 1.
C. 2. *
A. 3 months. * D. 4.
B. 6 months.
C. 9 months. Which of the following statements
about fats/lipids is correct?
D. 12 months.

What percentage of natal teeth are A. Naturally occurring trans fats have
supernumerary? been confirmed to increase disease
risk.
A. 10. * B. LDL-cholesterol is found in foods of
only animal origin.
B. 50.
C. Most of the cholesterol circulating
C. 90.
in the human body is synthesized by
the body. *
The percentage of documented child
abuse cases estimated to involve D. The essential fatty acids are only
orofacial injuries is the omega-3 fats.

A. 10-20.

Dr Abdul Naser Tamim SEHA, Dr Ghada Al Aqqad D.D.S, Dr Kamal Naser - Amrita Medical Centre, Dr Emad Wani - Shadi Dental Centre,
Dr Lina Anka SEHA, Dr Rouba Zgheibi SEHA, Maria Teresa Yongson Alejandrino SEHA 42
Five years ago, a 55 year old patient most appropriate management is to
was treated for squamous cell perform
carcinoma of the oral cavity with
ablative surgery, chemotherapy and A. minimal surgical debridement
intensity-modulated radiotherapy. The immediately.
posterior mandible received a dose of B. aggressive surgical debridement
70 Gy. The extraction of tooth 3.7 with immediately.
local anesthesia is planned because C. minimal surgical debridement after
the tooth cannot be restored. The most delaying 3 months. *
appropriate management is extraction D. aggressive surgical debridement
with after delaying 3 months.

A. transfusion of packed red blood A patient has been receiving dental


cells prior to the extraction. treatment over a period of 1 year. A
B. hyperbaric oxygen treatment prior to root canal treatment has recently been
the extraction. started but not completed when she
C. hyperbaric oxygen treatment prior to declares that her husband has lost his
and following the extraction. * job and she is not able to pay the
D. local anesthetic containing no balance of the fee assessed for her
vasoconstrictor. root canal treatment. She is presently
free of pain. The dentist can terminate
A 55 year old patient had a squamous treatment
cell carcinoma of the oral cavity 5
years previously. His medical A. only after completion of the root
management involved surgery, canal treatment. *
chemotherapy and intensity-modulated B. for failure to honor financial
radiotherapy (IMRT) of 65 Gy which commitments.
included the mandible. The most C. until the patient is able to pay for it.
appropriate management for the D. only after completion of the root
extraction of tooth 3.7 under local canal treatment and a full crown
anesthesia is restoration.
E. by referring the patient to an
A. transfusion with packed red blood endodontist.
cells.
B. hyperbaric oxygen treatment prior to A rotary endodontic file separates in
tooth extraction. the canal. The most appropriate
C. hyperbaric oxygen treatment prior to management is to
and following tooth extraction. *
D. using local anesthetic without A. inform the patient of the separated
epinephrine. file at the time of the incident. *
B. inform the patient of the separated
A 50 year old female had intravenously file if the tooth becomes symptomatic.
administered bisphosphonates for the C. make a notation in the patient’s
past 3 years. She now has an area of chart without informing the patient.
exposed necrotic bone with sharp D. recommend an apicoectocy.
edges in an edentulous maxilla. The

Dr Abdul Naser Tamim SEHA, Dr Ghada Al Aqqad D.D.S, Dr Kamal Naser - Amrita Medical Centre, Dr Emad Wani - Shadi Dental Centre,
Dr Lina Anka SEHA, Dr Rouba Zgheibi SEHA, Maria Teresa Yongson Alejandrino SEHA 43
Residual root tips will not be visible on recommend an alternate sweetener
panoramic radiographes of a that can be used in baking. Which of
completely edentulous patient if the following should NOT be
recommended?
A. they are not close enough to the
focal trough of the machine. * A. Equal® (aspartame). *
B. the patient is wearing dentures B. Splenda® (sucralose).
when the image is produced. C. Sugar Twin® (sodium cyclamate).
C. a digital panoramic machine is D. Xyla® (xylitol).
used.
D. the patient moves during production Which of the following statements
of the image. about sodium is correct?

A 35 year old female patient has A. Salt-sensitive people should avoid


multiple petechial hemorrhages of the foods like oranges and bananas.
palatal mucosa and ecchymoses of the B. A high salt intake aggravates but
right and left buccal mucosae. Which does not cause hypertension. *
of the following is the most likely C. Renal sodium excretion varies
laboratory finding? indirectly with total sodium intake.
D. The sodium recommendation is
A. Anemia. increased during pregnancy.
B. Prolonged PTT.
C. Elevated INR. The permanent teeth most frequently
D. Thrombocytopenia. * ankylosed are the

Which of the following tests is/are A. canines.


useful in the diagnosis of periradicular B. incisors.
periodontitis C. molars. ?
D. premolars. *
1. Thermal test.
2. Electric pulp test. The permanent teeth most commonly
3. Periapical radiograph. transposed are the
4. Probing.
A. mandibular incisor and mandibular
A. (1) (2) (3) * canine.
B. (1) and (3) B. mandibular canine and mandibular
C. (2) and (4) first premolar.
D. (4) only. C. maxillary upper lateral incisor and
E. All of the above. maxillary canine.
D. maxillary canine and maxillary first
premolar. *
A patient wants to reduce her caries
risk and asks her dentist to

Dr Abdul Naser Tamim SEHA, Dr Ghada Al Aqqad D.D.S, Dr Kamal Naser - Amrita Medical Centre, Dr Emad Wani - Shadi Dental Centre,
Dr Lina Anka SEHA, Dr Rouba Zgheibi SEHA, Maria Teresa Yongson Alejandrino SEHA 44
When percussion on the occlusal Biopsy of a recent solitary painless
surface of a tooth results in a positive ulcer on the lip in a 20 year old patient
response, the most likely etiology of shows the presence of Treponema
inflammation is pallidum. The ulcer is called (a)

A. pulpal. A. lupus vulgaris.


B. periodontal. B. facies leprosa.
C. periapical. * C. chancre. *
D. noma.
Which of the following statements
about chlorhexidine is INCORRECT? A hereditary condition affecting
odontoblasts can result in
A. It disrupts bacterial cell membranes.
B. It binds tenaciously to oral surfaces. A. regional odontodysplasia.
C. It interferes with adherence of B. fusion or gemination.
plaque-forming microorganisms. C. dilaceration.
D. Mutans streptococci are D. hypercementosis.
preferentially destroyed. E. radicular dentin dysplasia. *
E. It is commonly used for long-term
therapy. * The principal cellular sites of the
cytochrome P450 system are
Which of the following is NOT used to
inhibit calcification of plaque? A. cardiomyocytes.
B. pneumocytes.
A. Fluoride. C. splenocytes.
B. Magnesium. * D. hepatocytes. *
C. Pyrophosphate.
D. Zinc. Dietary consumption of which of the
following inhibits the cytochrome P450
The sensation of sour is mediated by system?

A. transducin. A. Meat.
B. protons blocking K+ channels. * B. Broccoli.
C. Decaffeinated tea.
C. gustducin.
D. Grapefruit juice. *
D. activation of T1R3 receptors.

Dr Abdul Naser Tamim SEHA, Dr Ghada Al Aqqad D.D.S, Dr Kamal Naser - Amrita Medical Centre, Dr Emad Wani - Shadi Dental Centre,
Dr Lina Anka SEHA, Dr Rouba Zgheibi SEHA, Maria Teresa Yongson Alejandrino SEHA 45
Which of the following would NOT be Which of the following characteristics
expected to cause hypokalemia? is NOT seen in all patients with
aggressive periodontitis?
A. Metabolic acidosis. *
B. Lactate accumulation. A. Rapid attachment loss and bone
C. Increased plasma insulin. destruction.
D. Excess aldosterone secretion. B. Amount of microbial deposits
E. Beta-adrenergic stimulation of cells. inconsistent with disease severity.
C. Diseased sites infected with
Hypernatremia may be induced by Aggregatibacter (Actinobacillus)
actinomycetemcomitans. *
A. elevated synthesis of atrial D. Familial aggregation of diseased
natriuretic peptide. individuals.
B. excessive vasopressin secretion.
C. impaired aldosterone secretion. Prevotella intermedia increases
D. diabetes insipidus. * significantly in pregnancy gingivitis
because of increased
During rapid head movement, that
occurs on activities such as running, A. gingival blood vessels with
the mandible is held in place due to increased inflammation.
activation of B. colonization of shallow pockets.
C. levels of plaque fluid in deep
A. temporomandibular proprioceptors. periodontal pockets.
B. fast adapting periodontal D. steroid hormones act as growth
pressoreceptors. factors. *
C. slow adapting periodontal
pressoreceptors. A 50 year old edentulous patient is
D. primary muscle spindle fibres. * complaining that spicy foods are no
longer enjoyable since he received his
Which of the following has a direct new complete upper and lower
effect on periodontal tissues in dentures. The most likely reason for
diabetes mellitus-associated gingivitis? this occurrence is that the

A. Lipopolysaccharides. A. dentures cover his taste buds.


B. Matrix metalloproteinases. * B. free nerve endings are now covered
C. Proinflammatory cytokines. by the dentures.
D. Prostaglandins. C. dentures are obstructing air flow to
the olfactory epithelium.
E. Leukotoxin.
D. None of the above*

Dr Abdul Naser Tamim SEHA, Dr Ghada Al Aqqad D.D.S, Dr Kamal Naser - Amrita Medical Centre, Dr Emad Wani - Shadi Dental Centre,
Dr Lina Anka SEHA, Dr Rouba Zgheibi SEHA, Maria Teresa Yongson Alejandrino SEHA 46
Which of the following bacterial A. 500 psig.
components is considered to be of key B. 1000 psig. *
importance in initiating and sustaining C. 1500 psig.
inflammatory responses in gingiva and D. 2000 psig.
other periodontal tissues?
Which of the following is the most
A. Fimbriae. potent analgesic?
B. Capsule.
C. Lipopolysaccharides. * A. Acetylsalicylic acid 650m*g.
D. Flagella. B. Ibuprofen 600mg.
C. Acetaminophen 300mg with codeine
Which of the following statements is 30mg.
INCORRECT? D. Codeine 60mg.

A. Chronic pain has somatosensory Which of the following is NOT a


and psychosocial impacts. contributor to the pathogenesis of
B. Pain persists only as long as the stomatitis?
injury exists.
C. Perception of acute and chronic A. TNF-α.
pain involves the same regions of the B. Bradykinin.
CNS. * C. Substance P.
D. Pain is a protective mechanism. D. Glucocorticoids. *

When a painful muscle is palpated, the Coxibs are more appropriate than
provoked pain is standard NSAIDs for patients with a
history of
A. localized. *
B. referred to another area. A. renal dysfunction.
C. described as a dull ache. B. asthma.
D. described as a burning sensation. C. gastrointestinal bleeding. *
D. cardiac problems.
Botulinum neurotoxins cause which
type of paralysis? In Turner’s syndrome, the patient is
most likely to exhibit
A. Spastic.
B. Flaccid. * A. delayed tooth eruption.
C. Periodic. B. micrognathia. *
D. Brachiofacial. C. a wide maxilla.
D. oligodontia.
When an oxygen tank is half full, what
is the pressure gauge reading?

Dr Abdul Naser Tamim SEHA, Dr Ghada Al Aqqad D.D.S, Dr Kamal Naser - Amrita Medical Centre, Dr Emad Wani - Shadi Dental Centre,
Dr Lina Anka SEHA, Dr Rouba Zgheibi SEHA, Maria Teresa Yongson Alejandrino SEHA 47
What are the dimensions of pain? The most reliable diagnostic sign of
gingival inflammation is
A. Intensive and sensory.
B. Sensory and peripheral. A. retractability.
C. Affective and peripheral. B. edema.
D. Sensory and affective. * C. bleeding. *
D. cratering.
What is the minimum number of E. altered texture.
nucleotides that can be deleted from a
gene that will result in the smallest A radiograph reveals a radiolucency
change in the amino acid sequence of associated with the apex of tooth 1.5.
the encoded protein? There is a large restoration but the
tooth is asymptomatic and the
A. 1. associated soft tissues appear normal.
B. 2. What is the most likely diagnosis?
C. 3. *
D. 4. A. Acute periradicular periodontitis.
E. 5. B. Chronic periradicular periodontitis. *
C. Acute periradicular abscess.
Sympathetic reflexes accompanying D. Chronic suppurative periradicular
hemorrhage periodontitis.

A. allow blood loss up to 60% of the The antithrombotic effects of


total volume. acetylsalicylic acid may be
B. have no effect on the amount of compromised by
blood that can be lost.
C. are part of the long-term A. codeine.
compensatory response mechanism. B. acetaminophen.
D. have a greater effect on arterial C. ibuprofen. *
pressure than on cardiac output. * D. tramadol.

In periodontal ligaments, enzymatic The most appropriate treatment for a


hydroxylation of proline and lysine unicystic ameloblastoma is
residues on nascent procollagen
requires A. root canal therapy.
B. hemi-mandibulectomy.
A. vitamin C. * C. block resection. *
B. lysyl oxidase. D. curettage.
C. calcium ions. E. radiotherapy.
D. decorin.
E. matrix metalloproteinase.

Dr Abdul Naser Tamim SEHA, Dr Ghada Al Aqqad D.D.S, Dr Kamal Naser - Amrita Medical Centre, Dr Emad Wani - Shadi Dental Centre,
Dr Lina Anka SEHA, Dr Rouba Zgheibi SEHA, Maria Teresa Yongson Alejandrino SEHA 48
A clinical finding common to B. unexplainable genetic factors.
alcoholism, poorly-controlled diabetes C. lingually situated supernumerary
mellitus, uremia and liver disease is teeth.
D. prolonged retention of a primary
A. a smooth tongue. incisor. *
B. increased blood pressure. E. premature eruption of a maxillary
C. a coated tongue. incisor.
D. labial fissures.
E. halitosis. * A patient is in intense pain with a left
hemifacial swelling, which is beginning
Areas of isolated gingival recession are to affect the eye. Examination reveals a
most frequently seen on teeth that are fluctuant swelling over tooth 2.2, which
has deep caries, and is tender to
A. nonvital. palpation and percussion. The most
B. moderately mobile. likely diagnosis is
C. ankylosed.
D. labially prominent in the arch. * A. acute periradicular periodontitis.
B. chronic periradicular periodontitis.
Which of the following conditions C. acute periradicular abscess. *
should NOT commonly be treated D. chronic periradicular abscess.
during the mixed dentition stage?
If removal of teeth is indicated in a
A. Anterior crossbite. patient who is to receive radiation
B. Posterior crossbite. therapy for a carcinoma of the tongue,
C. Maxillary incisor rotation. * the teeth should be extracted
D. Class II molar relationship.
A. prior to the radiation therapy. *
Which of the following unilateral fixed B. during the radiation therapy.
partial dentures is most likely to have C. immediately post-radiation therapy.
insufficient periodontal support? A fixed D. six months post-radiation therapy.
partial denture extending from the
Following an inferior alveolar block
A. maxillary first premolar to the first injection of 2% lidocaine with 1:100,000
molar. epinephrine, a patient experiences a
B. maxillary canine to the first molar. transient tachycardia. Which is the most
C. mandibular central incisor to the first likely cause of this reaction?
premolar. *
D. mandibular second premolar to the A. Vasovagal syncope.
second molar. B. An allergic reaction.
C. Myocardial infarction.
An anterior crossbite of a permanent D. Intravascular injection. *
maxillary incisor in a mixed dentition is E. An anaphylactoid reaction.
most often associated with

A. a functional shift.

Dr Abdul Naser Tamim SEHA, Dr Ghada Al Aqqad D.D.S, Dr Kamal Naser - Amrita Medical Centre, Dr Emad Wani - Shadi Dental Centre,
Dr Lina Anka SEHA, Dr Rouba Zgheibi SEHA, Maria Teresa Yongson Alejandrino SEHA 49
The most frequent radiographic
appearance of metastatic carcinoma A. direct the patient to a hospital
in the jaws is a radiolucency emergency room.
B. extract the tooth using a maximum
A. ill-defined in the anterior maxilla. of four cartridges.
B. ill-defined in the posterior C. extract the tooth using a maximum
mandible. * of two cartridges. *
C. well-defined in the anterior maxilla. D. extract the tooth and advise the
D. well-defined in the posterior patient to take additional
mandible. antihypertensive medication.

Which of the following conditions Repeat panoramic radiographs are


produces a radiolucent image? indicated to

A. Sialolithiasis. A. identify early signs of disease.


B. Osteosclerosis. B. follow-up a specific clinical concern
C. Odontoma. or condition. *
D. Ameloblastic fibroma. * C. monitor high caries risk patients.
D. monitor patients with fixed dental
Five years ago, a 35 year old patient prostheses.
received chemotherapy followed by
adjuvant intensity-modulated Which of the following may be
radiotherapy of 3000 cGy to the mistaken as a pathologic finding on
mandible for treatment of Hodgkin’s periapical radiographs of the premolar
disease. The extraction of tooth segments of the maxillary dental arch?
number 3.7 with local anesthesia is
planned. The most appropriate A. Pneumatisation of the alveolar
management is process. *
B. Nasopalatine foramen.
A. hyperbaric oxygen treatment prior C. Nasolacrimal canal.
to tooth extraction. D. Mental foramen.
B. to prescribe antibiotics following
tooth extraction. Which of the following structures may
C. hyperbaric oxygen treatment prior be mistaken as a pathologic finding on
to and following tooth extraction. * periapical radiographs of the premolar
segments of the maxilla?
A patient with a history of
hypertension presents for an A. Bony septa in the maxillary sinus. *
emergency dental extraction. His B. Nasopalatine foramen.
blood pressure is 158/100 mmHg. C. Nasolacrimal canal.
The dentist intends to use lidocaine D. Mental foramen.
2% with 1:100,000 epinephrine for
local anesthesia. The most
appropriate management is to

Dr Abdul Naser Tamim SEHA, Dr Ghada Al Aqqad D.D.S, Dr Kamal Naser - Amrita Medical Centre, Dr Emad Wani - Shadi Dental Centre,
Dr Lina Anka SEHA, Dr Rouba Zgheibi SEHA, Maria Teresa Yongson Alejandrino SEHA 50
A patient with leukoplakia had an B. Stainless-steel.
incisional biopsy. Which of the following C. ß-titanium.
diagnosis does NOT require complete D. Cobalt chromium.
excision?
Which of the following is NOT typically
A. Carcinoma in situ. associated with failed endodontic
B. Severe epithelial dysplasia. treatment?
C. Hyperkeratosis.*
D. Invasive carcinoma. A. Enterococcus faecalis.
B. Actinomyces israelii. *
The stratified squamous epithelial layer C. Streptococcus mutans.
of oral mucosa does NOT contain D. Candida albicans.
E. Prevotella intermedia.
A. melanocytes.
B. Langerhans cells. Flux is added to the casting metal
C. Merkel cells. during melting to
D. Schwann cells. *
A. minimize oxidation.*
The Bass tooth brushing technique is B. enhance melting.
appropriate for oral hygiene instruction C. increase stiffness.
because
D. decrease fluidity.
A. plaque beneath the gingival margin A common filler added to resin to
is removed. * produce dental composites is
B. debris is forced from the
embrasures. A. calcium salt.
C. the horizontal, vertical and circular B. quartz. *
motions.
C. zinc oxide.
D. it increases circulation to the
gingiva. C-factor (configuration factor) is
defined as the ratio of
The tooth with the largest root surface
area is the
A. width to height in the upper anterior
teeth.
A. maxillary canine.
B. mercury to alloy in a dental
B. maxillary first molar. * amalgam.
C. mandibular canine. C. bonded to unbonded surfaces in a
D. mandibular first molar. preparation. *
D. filler to monomer in a composite
Which of the following orthodontic alloy resin.
wires has the greatest effectiveness of
activation?

A. Nickel-titanium. *

Dr Abdul Naser Tamim SEHA, Dr Ghada Al Aqqad D.D.S, Dr Kamal Naser - Amrita Medical Centre, Dr Emad Wani - Shadi Dental Centre,
Dr Lina Anka SEHA, Dr Rouba Zgheibi SEHA, Maria Teresa Yongson Alejandrino SEHA 51
When closing a diastema with direct
composite, care must be taken to The level of streptococcus mutans has
ensure that the been shown to be significantly higher in
the bacterial plaque adjacent to which
A. width to height relationships are type of posterior restoration?
appropriate.
B. entire embrasure is filled. A. Glass-ionomer.
C. restoration does not extend B. Composite resin. *
subgingivally. * C. Amalgam.
D. enamel is reduced 0.3 to 0.5mm D. Gold castings.
prior to etching.
In comparison to visible light, X-rays
When closing a diastema with direct
composite, care must be taken to A. have a longer wave length.
ensure that B. have higher energy.*
C. travel faster.
A. only supragingival enamel is etched. D. can be focused.
B. primer is used to increase bond
strength. A healthy 40 year old female patient
C. glass-ionomer cement is used complains of periodic acute,
subgingivally. excruciating pain in the right infraorbital
D. interproximal gingiva is adequately region. The pain is of short duration
retracted. * and can be elicited by lightly touching
the skin of the area. Which of the
Which of the following is a following drugs is the most appropriate
CONTRAINDICATION to the use of management?
Tylenol 3®?
A. Carbamazepine. *
A. Concurrent use of penicillin V. B. Acetazolamide.
B. Asthmatic reaction to acetylsalicylic C. Succinylcholine.
acid. D. Penicillin.
C. An allergy to acetaminophen. *
D. A gastric ulcer. An infant has asymptomatic small
whitish outgrowths at the junction of
A lower molar requiring a crown has an the soft palate and hard palate. What is
extensive MOD amalgam restoration. the most likely diagnosis?
The crown margin is most appropriately
placed A. Dental lamina cysts.
B. Epstein pearls.
A. on the existing amalgam. C. Bohn nodules. *
B. at the amalgam/tooth junction. D. Fordyce granules.
C. 1mm apical to the amalgam margin.
*

Dr Abdul Naser Tamim SEHA, Dr Ghada Al Aqqad D.D.S, Dr Kamal Naser - Amrita Medical Centre, Dr Emad Wani - Shadi Dental Centre,
Dr Lina Anka SEHA, Dr Rouba Zgheibi SEHA, Maria Teresa Yongson Alejandrino SEHA 52
A 6 year old has circumscribed D. Angulation of the impaction. *
suppurative lesions that look like
cigarette burns on the left ear lobe and Fluoridated toothpaste will be most
the right knuckles. The parents are effective in remineralizing
cooperative and respond to the dentist
in a coherent manner. The most likely A. smooth surface decalcification. *
diagnosis is B. interproximal caries.
C. bruxism-related attrition.
A. battered child syndrome. D. pit and fissure caries.
B. impetigo. *
C. osteogenesis imperfecta. The need to frequently replace intact
D. Münchhausen syndrome. orthodontic elastics is a direct
E. von Willebrand desease. consequence of

An apprehensive 77 year old patient, A. creep.


weighing approximately 60kg, requires B. low modulus.
the removal of several mandibular C. stress relaxation. *
teeth under local anesthesia. The most D. high ductility.
appropriate method to manage this
patient’s anxiety is to administer Which of the following statements is
correct with respect to root caries
A. 20mg of diazapam, intravenously. lesions?
B. 20mg of diazapam, orally.
C. 100mg of secobarbital, orally. A. A specific microoganism causes
D. nitrous oxide and oxygen. * root caries.
B. Early lesions involve adjacent
Following extraction of teeth, root enamel.
fragments can be left in place when C. Colour of the lesion is a relaible
indicator of caries activity.
A. there is no plan to place a dental D. The majority of lesions begin
implant in the site. interpoximally. *
B. the root is small, not infected and
located deep in the bone. * To achieve adequate retention when
C. the maxillary sinus is pneumatized placing a post in a posterior tooth with
and close to the tooth roots. a normal canal configuration, the post
D. the root fragment is mobile and no requires a
more than 7-8mm in length.
A. length of 7 to 8 mm.
Which of the following is most likely to B. length equal to the restored crown
affect the difficulty of extraction of an height. *
impacted mandibular third molar? C. parallel sided design.
D. resin luting cement.
A. Root width.
B. Shape of the crown.
C. Size of the pulp chamber.

Dr Abdul Naser Tamim SEHA, Dr Ghada Al Aqqad D.D.S, Dr Kamal Naser - Amrita Medical Centre, Dr Emad Wani - Shadi Dental Centre,
Dr Lina Anka SEHA, Dr Rouba Zgheibi SEHA, Maria Teresa Yongson Alejandrino SEHA 53
A fixed partial denture with a single
pontic is deflected a certain amount, a All of the following are strategies for
span of two similar pontics will deflect increasing the fracture toughness of
dental ceramics EXCEPT
A. the same amount.
B. twice as much. A. slow cooling after sintering. *
C. four times as much. B. tetragonal zirconia addition.
D. eight times as much. * C. self-glazing.
D. crystalline phase dispersion.
Which of the following has the largest
effect on caries risk? All of the following display visceolastic
properties EXCEPT
A. Type of desserts eaten with meals.
B. Total amount of sugar in the diet. A. irreversible hydrocolloids.
C. Frequency of fermentable B. dental porcelain. *
carbohydrate intake. * C. silver amalgam.
D. Quality and range of nutrients in D. dentin.
meals and snacks.
The failure of a commercially pure
A new patient is a heavy smoker and titanium dental implant to
has severe periodontitis. When asked if osseointegrate can potentially be
interested in quitting smoking in the attributed to each of the following
next month, the patient replies, “I don’t EXCEPT
smoke that much so it’s not a big deal”.
What is the most appropriate strategy A. type III/IV bone at the implant site.
to encourage behaviour change? B. titanium oxide formation on the
implant surface. *
A. Encourage the patient to identify an C. habitual smoking.
alternate date to stop smoking. D. uncontrolled diabetes.
B. Explain the benefits of quitting
smoking as soon as possible. A cement base under an amalgam
C. Personalize the risk of continuing restoration should have
smoking to the patient’s oral health. *
D. Ask the patient what would be the A. high modulus and high thermal
easiest first step to take toward diffusivity.
quitting. B. high modulus and low thermal
diffusivity. *
Decreasing the amount of network C. low modulus and high thermal
modifiers in a dental porcelain will diffusivity.
D. low modulus and low thermal
A. decrease its fusion temperature. diffusivity.
B. increase its thermal expansion .
C. decrease its chemical reactivity.
D. increase its potential for
devitrification upon heating. *

Dr Abdul Naser Tamim SEHA, Dr Ghada Al Aqqad D.D.S, Dr Kamal Naser - Amrita Medical Centre, Dr Emad Wani - Shadi Dental Centre,
Dr Lina Anka SEHA, Dr Rouba Zgheibi SEHA, Maria Teresa Yongson Alejandrino SEHA 54
For a ceramometal crown, the The presence of flaws or cracks in a
thermal expansion coefficient of the material
ceramic (porcelain) should be
A. influences the strength of metals
A. slightly lower than that of the more than ceramics.
underlying metal structure. * B. leads to the development of stress
B. equal to that of the underlying concentrations.
metal structure. C. impacts compressive strength more
C. slightly higher than that of the than tensile strength.
underlying metal stucture. D. decreases its elastic modulus. *
D. significantly higher than that of the
underlying metal structure. A material undergoing plastic
deformation
Increasing the crystalline content of a
dental ceramic will generally A. will typically fail in a sudden,
decrease its Catastrophic manner.
B. is experiencing stress below its
A. fracture toughness. proportional limit.
B. opacity. C. will recover its original dimensions
C. translucency. * upon unloading.
D. proportional limit. D. is experiencing a breaking of bonds
within the structure. *
The mucogingival junction with no
interproximal attachment loss is a All of the following are ways of
Miller Class. What is the Miller’s class characterizing the resistance of a
for gingival recession that extends material to permanent deformation
beyond the mucogingival junction with EXCEPT
no interproximal attachment loss?
A. yield stress.
A. I. B. proportional limit.
B. II. * C. elastic limit.
C. III. D. ultimate stress. *
D. IV.
Which of the following posts are the
Drug-induced gingival overgrowth will most radiopaque?
most likely occur on the interdental
papillae of which area? A. Zirconia. *
B. Titanium.
A. Anterior palate. C. Carbon fibre.
B. Posterior palate. D. Plastic.
C. Anterior labial. *
D. Posterior buccal.

Dr Abdul Naser Tamim SEHA, Dr Ghada Al Aqqad D.D.S, Dr Kamal Naser - Amrita Medical Centre, Dr Emad Wani - Shadi Dental Centre,
Dr Lina Anka SEHA, Dr Rouba Zgheibi SEHA, Maria Teresa Yongson Alejandrino SEHA 55
Which of the following is NOT a sign of
pulpal necrosis in immature teeth? A. cotton rolls.
B. rubber dam. *
A. Loss of thermal sensitivity. C. cheek retractors.
B. Coronal discolouration. D. a matrix system.
C. Periradicular radiolucency.
D. Negative response to electric pulp Over time, the effectiveness of a Light
test. * Emitting Diode curing light will be
E. Abnormal mobility.
A. improved.
A large, deeply furrowed tongue is B. the same. *
commonly found in patients with C. reduced.

A. Pierre Robin Syndrome. Over time, the effectiveness of a quartz


B. geographic tongue. tungsten halogen curing light will be
C. ectodermal dysplasia.
D. Down Syndrome. * A. improved.
B. the same.
When probing a healthy peridontium C. reduced. *
using light forces, the probe tip will
most likely extend to the The primary role of calcium hydroxide
in indirect pulp cap procedures is to
A. attached gingiva.
B. junctional epithelium. * A. reduce bacterial load. *
C. transverse fibres of the periodontal B. occlude the dentinal tubules.
ligament. C. build up the internal form of the
cavity preparation.
Which of the following outcomes may D. provide a hermetic seal.
be the result of probing a healthy
periodontium with excessive force? Which of the following is NOT a
malignant lesion of the gingiva?
A. Underestimation of attachment loss.
B. Bleeding on probing. * A. Neurofibroma. *
C. Angular bony defect. B. Proliferative verrucous leukoplakia.
D. Gingival recession. C. Sarcoma.
D. Squamous cell carcinoma.
Bonding composite to enamel is most
appropriately performed by isolation
with

Dr Abdul Naser Tamim SEHA, Dr Ghada Al Aqqad D.D.S, Dr Kamal Naser - Amrita Medical Centre, Dr Emad Wani - Shadi Dental Centre,
Dr Lina Anka SEHA, Dr Rouba Zgheibi SEHA, Maria Teresa Yongson Alejandrino SEHA 56
The host defense mechanisms of the B. the difference between occlusal
gingival sulcus do NOT include vertical dimension and rest vertical
dimension. *
A. production of agglutinins and C. usually l0mm in the premolar region.
antibodies. D. the distance between maximum
B. flushing action of crevicular fluid. intercuspation and centric relation.
C. local antibody production.
D. low tissue turnover rate. * Generalized redness and atrophy of
the tongue are most often associated
Polymerization reactions which create with
water or alcohol by-products are called
A. lichen planus.
A. addition reactions. B. burning mouth syndrome.
B. ring-opening. C. endocrinopathy.
C. cross-linking. D. nutritional deficiency. *
D. condensation reactions. *
The greatest risk to dental health care
Which of the following will increase personnel for acquiring a blood-borne
resistance to dislodging forces on a pathogen is through
removable partial denture?
A. blood spatter on intact skin.
A. Locating direct and indirect retainers B. salivary contamination of clinic
as close as possible to the distal jackets.
extension base(s). C. manufacturing defects in gloves.
B. Locating direct and indirect retainers D. percutaneous injury. *
as far as possible from the distal
extension base(s). Which of the following is/are locally
C. Locating direct retainers as close as delivered antimicrobial(s) agent(s) used
possible to the distal extension base(s) to treat infected periodontal pockets?
and the indirect retainers as far as
possible from the distal extension 1. Metronidazole.
base(s). * 2. Chlorhexidine.
D. Locating direct retainers as far as 3. Doxycycline.
possible from the distal extension 4. Clindamycin.
base(s) and the indirect retainers as
close as possible to the distal A. (1) (2) (3) *
extension base(s). B. (1) and (3)
C. (2) and (4)
The freeway space is D. (4) only.
E. All of the above.
A. the difference between occlusal
vertical dimension and hinge axis
registration.

Dr Abdul Naser Tamim SEHA, Dr Ghada Al Aqqad D.D.S, Dr Kamal Naser - Amrita Medical Centre, Dr Emad Wani - Shadi Dental Centre,
Dr Lina Anka SEHA, Dr Rouba Zgheibi SEHA, Maria Teresa Yongson Alejandrino SEHA 57
Which of the following muscles is most B. Ibuprofen. *
likely to be affected by a depressed C. Codeine.
fracture of the zygomatic arch? D. Tramadol.

A. Lateral pterygoid. Granulation tissue which extends


B. Temporalis. coronally from the pulp of a carious
C. Masseter. * tooth is known as a/an
D. Medial pterygoid.
A. pyogenic granuloma.
Which of the following is associated B. pulp polyp. *
with Gardner syndrome? C. epulis granulomatosum.
D. fibroma.
A. Osteosarcoma.
B. Oligodontia. When using forceps to extract a
C. Osteomas. * maxillary first molar, the forceps
D. Osteomyelitis. movement should be principally in the
buccal direction because the
Patients who have undergone kidney
transplantation are at an increased A. buccal bone is thinner than the
risk of developing palatal bone. *
B. buccal roots are shorter than palatal
A. brown tumours. root.
B. plasma cell gingivitis. C. risk for sinus perforation is
C. erosive lichen planus. minimized.
D. squamous cell carcinoma. * D. furcation is more accessible from
the buccal.
Patients with anemia have an
increased risk of A daily dose of 81 mg of acetylsalicylic
acid is used for its
A. bacterial infections.
B. prolonged bleeding. A. analgesic properties.
C. exercise intolerance. * B. antipyretic effect.
C. antiplatelet action. *
Patients with thrombocytopenia have D. anti-inflammatory function.
an increased risk of
Which of the following compounds
A. bacterial infections. released by inflammatory cells induces
B. prolonged bleeding. * bone resorption?
C. exercise intolerance.
A. Nitric oxide.
Which of the following analgesics can B. Interleukin-1. *
exacerbate peptic ulcers? C. Bradykinin.
D. Alkaline phosphatase.

A. Acetaminophen.

Dr Abdul Naser Tamim SEHA, Dr Ghada Al Aqqad D.D.S, Dr Kamal Naser - Amrita Medical Centre, Dr Emad Wani - Shadi Dental Centre,
Dr Lina Anka SEHA, Dr Rouba Zgheibi SEHA, Maria Teresa Yongson Alejandrino SEHA 58
The Controlled Drugs and Substances 2. decreased eruption of the anterior
Act controls the distribution and use of teeth.
which of the following? 3. decreased eruption of the posterior
teeth.
A. Carbamazepine. 4. Increased eruption of the posterior
B. Celecoxib. teeth.
C. Codeine. *
D. Gabapentin. A. (1) (2) (3)
B. (1) and (3) *
The Controlled Drugs and Substances C. (2) and (4)
Act controls the distribution and use of D. (4) only
which of the following? E. All of the above.

A. Diazepam. * The etiology of an open bite


B. Carbamazepine. malocclusion of dental origin is most
C. Gabapentin. often the result of
D. Celecoxib.
1. increased eruption of the anterior
Ethics is most concerned with teeth.
2. decreased eruption of the anterior
A. patient rights. teeth.
B. standard of right and wrong.* 3. decreased eruption of the posterior
C. legal liability. teeth.
D. provincial codes of conduct. 4. Increased eruption of the posterior
teeth.
Generalized aggressive periodontitis is
characterized by attachment loss A. (1) (2) (3)
affecting the first molars and incisors B. (1) and (3)
and at least C. (2) and (4) *
D. (4) only
A. 1 other permanent tooth. E. All of the above.
B. 3 other permanent teeth. *
C. 5 other permanent teeth. The correlation between malocclusion
D. 7 other permanent teeth. and temporomandibular dysfunction is

The etiology of a deep bite A. weak. *


malocclusion of dental origin is most B. moderate.
often the result of C. strong.
D. definitive.
1. increased eruption of the anterior
teeth.

Dr Abdul Naser Tamim SEHA, Dr Ghada Al Aqqad D.D.S, Dr Kamal Naser - Amrita Medical Centre, Dr Emad Wani - Shadi Dental Centre,
Dr Lina Anka SEHA, Dr Rouba Zgheibi SEHA, Maria Teresa Yongson Alejandrino SEHA 59
When an orthodontic force is applied to A dentist must be prudent in deciding
a maxillary right canine, blood flow to how far to follow a patient’s informed
the periodontal ligament is altered choice for suboptimal treatment
within because

A. seconds. A. the law protects a patient’s right to


B. minutes. * make poor decisions.
C. hours. B. a patient’s informed choice must
D. days. always be honoured.
C. the principle of do-no-harm
Orthodontic growth modification should overrides the patient’s personal choice.
be *

A. started as early as possible. During an incisal clench, the activity of


B. individualized to maximize effect. * the elevator muscles is
C. delayed until the eruption of the
permanent dentition. A. increased. *
D. used in adult treatment. B. decreased.
C. unaffected.
When performing a periodontal
screening and recording (PSR) for a In the context of informed consent,
patient, the code asterisk (*) is used for choice means the ability
a sextant when
A. to accept recommended beneficial
A. a tooth in the sextant needs to be treatment voluntarily.
extracted. B. to refuse recommended beneficial
B. all the teeth in the sextant are treatment voluntarily.
missing. C. to refuse recommmended beneficial
C. a mucogingival problem is present. * treatment with an understanding of
D. a tooth in the sextant has forseeable consequences. *
supraerupted.
The incidence of a second
Which maxillary central incisor mesiobuccal canal in the maxillary first
characteristic is the most limiting in the molar is
construction of a Class II functional
appliance? A. 40%
B. 60%. *
A. Retroclination. * C. 80%.
B. Proclination. D. 100%.
C. Overeruption.
D. Undereruption.

Dr Abdul Naser Tamim SEHA, Dr Ghada Al Aqqad D.D.S, Dr Kamal Naser - Amrita Medical Centre, Dr Emad Wani - Shadi Dental Centre,
Dr Lina Anka SEHA, Dr Rouba Zgheibi SEHA, Maria Teresa Yongson Alejandrino SEHA 60
An ideal occlusion has C. observation with dark-field
microscopy. *
A. an absence of contact on cingulums D. observation with conventional
during protrusion. microscopy.
B. canine guidance or group function
on the working side. * The most likely complication following
C. contact between posterior teeth surgery for a patient with
during protrusion. thrombocytopenic purpura would be
D. posterior tooth contact on the non-
working side. A. angina.
B. hemorrhage. *
A modified hinge non-adjustable C. alveolar osteitis.
articulator is limited in its accuracy to D. delayed healing.
reproduce dynamic jaw movements E. infection.
because it can only reproduce
Which of the following conditions is
A. large centric relation-maximum most likely to lead to thrombosis?
intercuspation slides.
B. One hinge axis movement. * A. Stasis of blood. *
C. eccentric movements by multiple B. Presence of bacteria in blood.
lateroprotrusive registrations. C. Deficiency of circulating platelets.
D. Increased concentration of plasma.
In a healthy 75 year old, which of the E. Lowered oxygenation of
following would be considered normal hemoglobin.
blood pressure?
Periodontal pockets CANNOT be
A. 175/95 mmHg. reduced by
B. 135/86 mmHg. *
C. 130/100 mmHg. A. occlusal adjustment. *
D. 185/94 mmHg. B. scaling and root planing.
C. open flap curettage.
A periradicular granuloma D. guided tissue regeneration.

A. erodes rapidly through bone. Isolated gingival recession is most


B. is asymptomatic. * frequently seen on teeth that are
C. is intensely painful.
D. occurs only in young adults. A. heavily restored.
E. occurs at the apex of a vital tooth. B. labially prominent. *
C. mobile.
Spirochete activity is seen by D. nonvital.

A. staining with Gram's method.


B. staining with methylene blue.

Dr Abdul Naser Tamim SEHA, Dr Ghada Al Aqqad D.D.S, Dr Kamal Naser - Amrita Medical Centre, Dr Emad Wani - Shadi Dental Centre,
Dr Lina Anka SEHA, Dr Rouba Zgheibi SEHA, Maria Teresa Yongson Alejandrino SEHA 61
During cardiopulmonary resuscitation
(CPR), external chest compression of A. sublingual.
an adult patient should be B. submandibular.
C. parotid. *
A. 1 - 2cm.
B. 4 - 5cm. * A 60 year old patient presents with
C. 7 - 10cm. lingual erosion of the maxillary
D. 10 - 20cm. incisors. The most likely cause is

The use of conscious sedation for a A. alcoholism. *


restorative procedure in an office B. ulcerative colitis.
environment C. diabetes mellitus.
D. mitral valve prolapse.
A. requires the services of an E. rheumatoid arthritis.
anesthesiologist.
B. is contraindicated in children. A patient is not able to close her left
C. precludes the use of local eye, wrinkle her forehead or smile on
anesthesia. the left side. The most likely diagnosis
D. requires effective local anesthesia. is
E. may be used in conjunction with
narcotics. * A. contralateral subarachnoid
hemorrhage.
Radiographic examination of a 9 year B. fracture of the base of the skull.
old patient reveals the pre-eruptive C. Horner's syndrome.
position of a mandibular second D. acute mastoiditis.
premolar is tipped 20 degrees from E. facial nerve paralysis. *
vertical. The primary second molar
exhibits no root resorption. The most What is the most common site for
appropriate management is to intraoral squamous cell carcinoma?

A. surgically upright the premolar. A. Gingiva.


B. remove the primary second molar B. Floor of mouth. *
and place a space maintainer. C. Buccal mucosa.
C. recall the patient for reassessment D. Dorsum of tongue.
in 6 months. *
Which of the following is NOT a sign
Which of the following is the most of occlusal trauma?
common tumor of the parotid gland?
A. Fremitus.
A. Pleomorphic adenoma. * B. Gingival recession. *
B. Oncocytoma. C. Widened periodontal ligament.
C. Warthin’s tumor. D. Tooth migration.
D. Canalicular adenoma.

The most radiosensitive salivary gland


is the
A patient with a pre-existing MOD A. restore with a MOD amalgam.
amalgam restoration has just had B. reduce the occlusal out of occlusion
endodontic therapy completed on tooth and restore with a MOD amalgam.
4.6 but cannot afford a laboratory C. cusp cap the buccal and lingual
fabricated final restoration. Interim cusps and restore with a MOD
restorative management of 4.6 with the amalgam.*
best prognosis is to D. restore with a bonded MOD
composite resin.

Dr Abdul Naser Tamim SEHA, Dr Ghada Al Aqqad D.D.S, Dr Kamal Naser - Amrita Medical Centre, Dr Emad Wani - Shadi Dental Centre,
Dr Lina Anka SEHA, Dr Rouba Zgheibi SEHA, Maria Teresa Yongson Alejandrino SEHA 62
Maxillary incisor protrusion can be Which of the following space
treated by maintainers is most appropriate for a
patient with the bilateral loss of
1. Premolar extraction with orthodontic mandibular first primary molars prior to
retraction of the incisors. the eruption of the permanent molars
2. Premolar extraction with surgical and permanent incisors?
repositioning of the anterior
dentoalveolar segment. A. Lingual holding arch.
3. Extraction of the incisors, B. Bilateral distal shoes.
alveoloplasty and prosthodontic C. Bilateral band and loop. *
replacement. D. Nance appliance.
4. Reduction and genioplasty.
The most likely diagnosis for a patient
A. (1) (2) (3) * with an interincisal opening of 30mm
B. (1) and (3) before feeling pain and a maximum
C. (2) and (4) opening of 44mm with pain is
D. (4) only
E. All of the above. A. internal derangement of the TMJ
with reduction.
Metallic salts are included in root canal B. internal derangement of the TMJ
sealers to make the sealers without reduction.
C. trismus of the masticatory muscles.
A. stronger. D. subluxation of the TMJ.
B. radiopaque. * E. myofascial pain. *
C. set more rapidly.
D. tolerated by periapical tissues. Following the administration of a right
E. bond to dentin. inferior alveolar nerve block, right facial
paralysis is noted. Which of the
Following a pulpotomy in a second following muscles is NOT affected?
primary molar with extensive occlusal
caries, what is the most appropriate A. Levator palpebrae superioris. *
restoration? B. Buccinator.
C. Mentalis.
A. Resin-modified glass ionomer. D. Nasalis.
B. Bonded amalgam. E. Orbicularis oris.
C. Composite resin.
D. Stainless steel crown. *

Dr Abdul Naser Tamim SEHA, Dr Ghada Al Aqqad D.D.S, Dr Kamal Naser - Amrita Medical Centre, Dr Emad Wani - Shadi Dental Centre,
Dr Lina Anka SEHA, Dr Rouba Zgheibi SEHA, Maria Teresa Yongson Alejandrino SEHA 63
The postnatal increase in width of the readings. The most appropriate
maxilla results from management is to

A. sutural growth. A. perform pulpectomies on the teeth.


B. appositional growth. B. remove the teeth, fill the root canals
C. both appositional and sutural and replant.
growth. * C. splint the teeth, check vitality in one
month and if negative, treat
The most appropriate management for endodontically. *
a tooth with a history of previous D. splint the teeth and treat
trauma and apical resorption is endodontically immediately.

A. observation over 6 months for Which of the following should NOT be


further resorption. prescribed for a patient receiving
B. complete instrumentation and warfarin?
medication with intracanal calcium
hydroxide. * A. Acetaminophen.
C. immediate instrumentation and B. Metronidazole. *
obturation followed by apical curettage. C. Penicillin.
D. extraction, apical resection, D. Codeine.
retrofilling and replantation.
E. extraction and replacement with a Which of the following is the most
fixed or removable prosthesis. appropriate for pain management
following an emergency pulpectomy
Radiographs of the mandibular incisor for an adult with a history of severe
teeth of a 45 year old healthy patient asthma and nasal polyps?
reveal periapical radiolucencies. The
teeth are vital and asymptomatic. The A. Naproxen.
most appropriate management is to B. Acetylsalicylic acid.
C. Ketorolac.
A. perform a biopsy of the radiolucent D. Acetaminophen. *
lesion.
B. perform endodontic therapy on the Metronidazole can be used to treat
four incisors.
C. place a drain in the affected area. A. denture stomatitis.
D. observe periodically. * B. recurrent aphthous ulcers.
C. necrotizing ulcerative gingivitis
A patient has suffered a blow resulting (NUG). *
in the loosening of three maxillary D. primary herpetic gingivostomatitis.
incisors. Vitality tests give negative

Dr Abdul Naser Tamim SEHA, Dr Ghada Al Aqqad D.D.S, Dr Kamal Naser - Amrita Medical Centre, Dr Emad Wani - Shadi Dental Centre,
Dr Lina Anka SEHA, Dr Rouba Zgheibi SEHA, Maria Teresa Yongson Alejandrino SEHA 64
Which of the following may be used to Which of the following could NOT be
disinfect gutta-percha points? an immediate postoperative
complication of periapical surgery?
A. Autoclave.
B. Chemical solutions. * A. Haemorrhage.
C. Flame sterilization. B. Edema.
D. Dry heat sterilization. C. Paresthesia.
D. Pain.
During endodontic treatment a file E. Mucocele. *
separates. The fragment is 3mm long
and is lodged tightly in the apical third The mechanical objectives of
of the canal. No radiographic changes preparing the root canal system for
at the apex are evident. In addition to obturation with gutta-percha should
informing the patient, the most include
appropriate management is to
A. development of a continuously
A. extract the tooth. tapering cone in the root canal.
B. perform an apicoectomy and place a B. removal of irregularities.
retrograde filling. C. maintenance of an intact foramen.
C. resect the apical section of the root D. All of the above. *
containing the separated instrument.
D. complete the root canal filling and When root canals are treated topically
monitor at recall examination. * with antibiotics rather than with
disinfectants
Which one of the following is the most
appropriate initial treatment for internal A. a greater success rate results.
resorption? B. the same rules of mechanical
preparation and filling must be
A. Pulpectomy. * observed. *
B. Pulpotomy. C. treatment may be completed in
C. Pulp capping. fewer appointments.
D. Apicoectomy. D. there is greater assurance that all
When performing endodontic treatment micro-organisms are destroyed.
on a vital tooth, the most appropriate
termination point of apical root canal The anterior palatine foramen is most
preparation is likely to be radiographically
misdiagnosed as a
A. 0.5 to 1mm short of the radiographic
apex. * A. rarefying osteitis.
B. 3mm short of the radiographic apex. B. nasolabial cyst.
C. slightly through the apical foramen. C. cyst of the incisive papilla. *
D. to the point where the patient feels
sensation.

Dr Abdul Naser Tamim SEHA, Dr Ghada Al Aqqad D.D.S, Dr Kamal Naser - Amrita Medical Centre, Dr Emad Wani - Shadi Dental Centre,
Dr Lina Anka SEHA, Dr Rouba Zgheibi SEHA, Maria Teresa Yongson Alejandrino SEHA 65
The most appropriate emergency When removal of carious dentin results
management of a mature permanent in an exposure of non-vital pulp, the
tooth with acute irreversible pulpitis is most appropriate management is to

A. pulpotomy. A. perform endodontic treatment. *


B. pulpectomy. * B. cap the exposed pulp horn with
C. incision and drainage. calcium hydroxide.
D. trephination. C. occlude the cavity with a light
E. apical surgery. packing of cotton moistened with
eugenol.
A patient's 4 mandibular incisors were D. place a temporary restoration and
traumatized 3 years ago in an accident. observe.
Radiographs now show apical
radiolucencies associated with all 4 Elective root canal therapy may be
teeth. The most appropriate safely and successfully undertaken for
management is to all of the following EXCEPT

A. test the pulp vitality and perform root A. hemophiliacs.


canal therapy on teeth with no B. patients with a history of rheumatic
response. * fever.
B. perform root canal therapy on all 4 C. pregnant patients during first
teeth and curette the periapical area. trimester. *
C. extract the teeth and place a bonded D. pregnant patients during second
bridge. trimester.
D. postpone treatment and recheck
status periodically. Which of the following conditions would
NOT require antibiotic premedication
The prognosis for an avulsed tooth is before endodontic therapy?
principally affected by
A. Valvular heart disease.
A. length of time the tooth was out of B. Cardiac prosthesis.
the mouth. * C. Persistent odontogenic fistula.*
B. condition of the socket when the D. Total joint replacement.
tooth was replanted.
C. removal of necrotic cementum. During post preparation on a maxillary
D. pulp extirpation. central incisor, the labial surface of the
root is perforated. The most appropriate
An 80 year old patient can be expected management is to
to have
A. extract the tooth.
A. a reduced size of the pulp chamber* B. cement the post using zinc-
B. a reduced incidence of pulp stones. oxyphosphate cement.
C. a reduced tendency to pulpal C. cement the post, then raise a flap
fibrosis. and seal the defect surgically.
D. a reduced tendency for pulp D. re-prepare the canal so the post is
bleeding. now totally within the canal and cement
the post. *

Dr Abdul Naser Tamim SEHA, Dr Ghada Al Aqqad D.D.S, Dr Kamal Naser - Amrita Medical Centre, Dr Emad Wani - Shadi Dental Centre,
Dr Lina Anka SEHA, Dr Rouba Zgheibi SEHA, Maria Teresa Yongson Alejandrino SEHA 66
Retention of an inlay is improved by 1. Steroid therapy.
2. Palliative treatment.
1. Addition of an occlusal dovetail. 3. Application of dilute hydrogen
2. Increasing the parallelism of walls. peroxide.
3. Lengthening the axial walls. 4. Control of secondary infection.
4. Placing a gingival bevel.
A. (1) (2) (3)
A. (1) (2) (3) * B. (1) and (3)
B. (1) and (3) C. (2) and (4) *
C. (2) and (4) D. (4) only
D. (4) only E. All of the above.
E. All of the above.
Initial treatment of necrotizing
The immediate treatment of a ulcerative gingivitis includes
periodontal abscess is to 1. Debridement.
2. Occlusal adjustment.
A. establish drainage. * 3. Oral hygiene instruction.
B. prescribe an analgesic. 4. Gingivoplasty.
C. relieve the occlusion.
D. prescribe an antibiotic. A. (1) (2) (3)
B. (1) and (3) *
The most appropriate treatment of C. (2) and (3)
necrotizing ulcerative gingivitis in a D. (4) only
patient with lymphadenopathy is E. All of the above.
1. Periodontal debridement.
2. Occlusal adjustment. Generalized malaise and elevated body
3. Oral hygiene instruction. temperature may be associated with
4. Antibiotic therapy. A. asymptomatic (chronic) apical
periodontitis.
A. (1) (2) (3) B. acute apical abscess. *
B. (1) and (3) C. symptomatic irreversible pulpitis.
C. (2) and (4) D. asymptomatic irreversible pulpitis.
D. (4) only
E. All of the above. * Which of the following medications is
CONTRAINDICATED in the
The purpose of a periodontal dressing management of a patient who is taking
is to warfarin?
A. enhance wound healing. A. Acetaminophen.
B. protect the wound from injury. B. Acetylsalicylic acid. *
C. increase patient comfort. C. Codeine.
D. All of the above. * D. Clindamycin.

Treatment of primary herpetic


gingivostomatitis should include

Dr Abdul Naser Tamim SEHA, Dr Ghada Al Aqqad D.D.S, Dr Kamal Naser - Amrita Medical Centre, Dr Emad Wani - Shadi Dental Centre,
Dr Lina Anka SEHA, Dr Rouba Zgheibi SEHA, Maria Teresa Yongson Alejandrino SEHA 67
Which of the following require C. acyclovir capsules. *
prophylactic antibiotics prior to dental D. dexamethasone elixir.
procedures causing a bacteremia?
Nystatin is the most appropriate drug
A. Implanted cardiac pacemakers. to treat
B. Prosthetic cardiac valves. *
C. Coronary artery bypass grafts. A. aphthous stomatitis.
D. Cardiac stents one year after B. candidiasis. *
placement. C. periodontal abscess.
D. necrotizing ulcerative gingivitis.
Oral nitrate is used to treat the
symptoms of Which of the following procedures
requires antibiotic prophylaxis for a
A. hypertension. patient with a prosthetic heart valve?
B. angina. *
C. arrhythmia. A. Inferior alveolar nerve block.
D. tachycardia. B. Postoperative suture removal. *
C. Restoration of occlusal caries.
Which of the following drugs will have D. Making an alginate impression.
the most rapid onset of action?
Which is the most appropriate
A. Oral ibuprofen. prophylactic antibiotic for a patient
B. Intramuscular penicillin. with mitral valve prolapse with
C. Subcutaneous epinephrine. regurgitation undergoing a surgical
D. Inhaled nitrous oxide. * dental procedure?

In a patient who is allergic to penicillin, A. Clindamycin.


the most appropriate antibiotic to treat B. Vancomycin.
an infection of endodontic origin is C. Erythromycin.
D. Amoxicillin.
A. amoxicillin. E. None of the above. *
B. azithromycin.
C. clindamycin. * In the treatment of necrotizing
D. tetracycline. ulcerative gingivitis with associated
lymphadenopathy, which of the
Appropriate management for the relief following medications is the most
of symptoms of primary herpetic appropriate?
gingivostomatitis in an
immunocompromised patient may A. An anti-inflammatory.
include B. A topical antibiotic.
C. A systemic antibiotic. *
A. penicillin V. D. An analgesic.
B. triamcinolone acetonide in
Orabase®.

Dr Abdul Naser Tamim SEHA, Dr Ghada Al Aqqad D.D.S, Dr Kamal Naser - Amrita Medical Centre, Dr Emad Wani - Shadi Dental Centre,
Dr Lina Anka SEHA, Dr Rouba Zgheibi SEHA, Maria Teresa Yongson Alejandrino SEHA 68
A very apprehensive patient B. they predispose patients to candidial
experiencing pain may be prescribed infection.
a barbiturate, chloral hydrate or an C. they form stable complex with the
antihistamine to control the anxiety. In developing tooth matrix.
which of the following would you D. they may be substituted for
expect an exaggerated response to amoxicillin in patients that require
the use of these drugs? coverage to prevent subacute bacterial
endocarditis. *
A. A diabetic.
B. An arthritic patient. Shortly after the administration of an
C. A patient with chronic renal inferior alveolar nerve block, a healthy
disease. * adult patient rapidly develops a facial
rash. Which of the following signs and
Flurbiprofen is an symptoms should be watched for
before initiating the planned dental
A. antibiotic. treatment?
B. muscular relaxant.
C. anti-inflammatory. * 1. Pallor and perspiration.
D. antidepressant. 2. Shortness of breath.
3. Hyperventilation.
50- A patient has a history of 4. Edema of the lips.
shortness of breath and ankle edema.
You would suspect A. (1) (2) (3)
B. (1) and (3)
A. asthma. C. (2) and (4) *
B. emphysema. D. (4) only
C. rhinophyma. E. All of the above.
D. cardiac insufficiency. *
A healthy, 23 year old patient
Tetracycline therapy instituted either experiences a warm sensation,
in the second trimester or post-partum diaphoresis, nausea, light headedness
in the infant is responsible for and then loses consciousness
approximately 30 seconds following
A. minor changes in the the injection of 1.8ml of 2% lidocaine
hydroxyapatite of the enamel. with 1:100,000 epinephrine for an
B. discoloration of deciduous teeth. inferior alveolar nerve block. What is
C. discolouration of permanent teeth. the most likely diagnosis for his
D. discolouration of the deciduous or situation?
permanent teeth. *
A. Allergic reaction.
Tetracyclines have all of the following B. Overdose of local anesthetic.
properties EXCEPT C. Syncope. *
D. Intravascular injection.
A. absorption is impaired when taken
with milk.

Dr Abdul Naser Tamim SEHA, Dr Ghada Al Aqqad D.D.S, Dr Kamal Naser - Amrita Medical Centre, Dr Emad Wani - Shadi Dental Centre,
Dr Lina Anka SEHA, Dr Rouba Zgheibi SEHA, Maria Teresa Yongson Alejandrino SEHA 69
Which of the following conditions is an
indication for the removal of impacted A. radiolucent.
mandibular third molars? B. painless.
C. neoplastic. *
A. Recurrent pericoronitis. * D. inflammatory.
B. Prevention of crowding of
mandibular incisors. Which of the following is characteristic
C. Reduction of mandibular fracture of periapical osseous dysplasia
risk. (periapical cemento-osseous
D. Horizontal impaction. dysplasia)?

An acute apical abscess must be A. Pain.


associated with all of the following B. Expansion.
EXCEPT C. Biopsy is not usually necessary. *
D. Requires endodontics or extraction.
A. pain.
B. sensitivity to percussion. Which procedure(s) require(s)
C. non-vital tooth. antibiotic prophylaxis in a patient
D. periapical radiolucency. * susceptible to bacterial endocarditis?

The most common clinical finding in 1. Impressions for partial dentures.


the diagnosis of an acute apical 2. Suture removal.
abscess is 3. Mandibular block anesthetic
injection.
A. mobility of the tooth. 4. Periodontal surgery.
B. pain on percussion. *
C. discoloration of the crown. A. (1) (2) (3)
D. presence of a cellulitis. B. (1) and (3)
E. lymph node enlargement. C. (2) and (4) *
D. (4) only
Radiographic examination of a healthy E. All of the above.
38 year old patient shows a 4mm
diameter, well-defined radiolucency at Which of the following has the greatest
the apex of tooth 4.1. The tooth has a effect on the mechanical properties of
normal response to vitality tests. The composite resin?
most appropriate management is
A. Modulus of elasticity of the filler
A. incision and drainage. particles.
B. endodontic treatment. B. Weight fraction of the filler particles.
C. observation. * C. Volume fraction of filler particles. *
D. apicoectomy. D. Hardness of filler particles.
E. to open the tooth for drainage. E. Size of the filler particles.

A periapical granuloma is all of the


following EXCEPT

Dr Abdul Naser Tamim SEHA, Dr Ghada Al Aqqad D.D.S, Dr Kamal Naser - Amrita Medical Centre, Dr Emad Wani - Shadi Dental Centre,
Dr Lina Anka SEHA, Dr Rouba Zgheibi SEHA, Maria Teresa Yongson Alejandrino SEHA 70
Desquamation of the gingiva usually
occurs as a result of The most appropriate treatment for a
permanent molar with occlusal caries
A. inflammation. confined to the enamel of the central
B. benign neoplasia. pit is a/an
C. normal cell turnover. *
D. a developmental abnormality. A. amalgam.
B. composite resin.
After the crown completion stage, C. fluoride varnish application.
trauma to a developing tooth may be D. preventive resin restoration. *
responsible for
A 45 year old patient has 32
A. enamel hypoplasia. unrestored teeth. The only defects are
B. gemination. deeply stained grooves in the posterior
C. dilaceration. * teeth. Clinical examination reveals no
D. fusion. evidence of caries in the grooves. The
most appropriate management is
When preparing a cavity in a primary
molar, there is a small mechanical A. application of a resin based pit and
exposure of one of the pulp horns. fissure sealants.
There is a slight hemorrhage and the B. application of a glass ionomer pit
dentin surrounding the exposure is and fissure sealants.
sound. The most appropriate C. conservative Class I amalgams.
treatment is D. prophylactic odontotomy.
E. no treatment. *
A. extraction and space maintenance.
B. pulp capping, a base and If a patient in her first trimester of
restoration. * pregnancy requires the replacement of
C. pulpectomy and restoration. a large MOD amalgam restoration with
D. base and restoration. extensive recurrent caries and thermal
sensitivity, the most appropriate
The most significant factor treatment is to
contributing to the long-term success
of the restoration of an endodontically A. delay treatment until after the baby
treated tooth is the is born.
B. restore with reinforced zinc oxide
A. type of post utilized. eugenol. *
B. remaining coronal tooth C. restore with amalgam.
structure.?? D. restore with a composite resin.
C. presence of extracoronal
coverage*
D. type of core material used.

Dr Abdul Naser Tamim SEHA, Dr Ghada Al Aqqad D.D.S, Dr Kamal Naser - Amrita Medical Centre, Dr Emad Wani - Shadi Dental Centre,
Dr Lina Anka SEHA, Dr Rouba Zgheibi SEHA, Maria Teresa Yongson Alejandrino SEHA 71
Which of the following clinical findings
will give the most favourable prognosis A. place amalgam restorations over the
for successful direct pulp capping? next few months.
B. excavate caries and place temporary
1. No hemorrhage at the exposure restorations within the next few weeks. *
site. C. delay any treatment until the hygiene
2. The exposure is small. improves.
3. The tooth is hypersensitive to heat. D. restore all teeth with composite resin
4. The exposure site is over the next few months.
uncontaminated.
A patient complains of tooth pain which
A. (1) (2) (3) is sharp and stabbing when chewing
B. (1) and (3) sweet or cold foods. Pain is relieved by
C. (2) and (4) * warmth and direct pressure. The most
D. (4) only likely diagnosis is
E. All of the above.
A. a carious lesion with pulpal
When performing a functional analysis inflammation.
of occlusion, the mandible may be B. a carious lesion with pulp
observed to exhibit a shift from centric degeneration.
relation to centric occlusion in all but C. traumatic occlusion.
one of the following directions D. a cracked tooth. *

A. forward. Sensitivity related to a noncarious


B. backward. * cervical lesion is most likely explained
C. upward. by the
D. lateral.
A. thermal conductivity of dentin.
During cavity preparation with a rubber B. hydrodynamic theory. *
dam, a small mechanical pulp C. dentinogenesis process.
exposure occurs. The most D. neurogate mechanism.
appropriate management is to E. inorganic component of dentin.

A. swab the exposure with A vertical cross-section of a smooth


chlorhexidine. surface carious lesion in enamel
B. place a pulp capping material. * appears as a triangle with the
C. perform a pulpotomy.
D. perform a pulpectomy. A. base at the dentino-enamel junction.
B. base facing toward the pulp.
A 16 year old patient has multiple C. apex pointing to the enamel surface.
extensive carious lesions. The most D. apex pointing to the dentino-enamel
appropriate management is to place junction. *
the patient on a preventive regime and
to

Dr Abdul Naser Tamim SEHA, Dr Ghada Al Aqqad D.D.S, Dr Kamal Naser - Amrita Medical Centre, Dr Emad Wani - Shadi Dental Centre,
Dr Lina Anka SEHA, Dr Rouba Zgheibi SEHA, Maria Teresa Yongson Alejandrino SEHA 72
Which of the following factors Dentigerous cysts should be
influence(s) the development of root completely enucleated because
caries?
A. the epithelium of the cyst can
1. A diet high in refined carbohydrates. degenerate and form neoplastic cells. *
2. Periodontal disease. B. the epithelial lining of the cyst has a
3. The anatomy of the cemento- high recurrence potential.
enamel junction. C. the connective tissue of the cyst
4. Xerostomia. wall can become osteoblastic.
D. continued growth is likely to result in
A. (1) (2) (3) a supernumerary tooth.
B. (1) and (3)
C. (2) and (4) A healthy 66 year old patient who had
D. (4) only a myocardial infarct eight years
E. All of the above. * previously requires an extraction. The
most appropriate management is to
A 19 year old female with an otherwise
healthy dentition presents with erosion A. admit the patient to hospital for
of the lingual surfaces of all maxillary extraction with local anesthesia.
anterior teeth. This is most likely B. admit the patient to hospital for
caused by extraction with general anesthesia.
C. extract the tooth in the office using
A. xerostomia. preoperative sedation and local
B. occlusal parafunction. anesthesia without a vasoconstrictor.
C. diet high in citrus fruit. D. extract the tooth in the office using
D. bulimia. * local anesthesia with a
vasoconstrictor*
A patient reports pain on mastication
since the placement of a metal- The most appropriate antibiotic for a
ceramic crown 2 weeks earlier. The periapical dental abscess is
most likely cause is
A. pen V. *
A. hyperemia. B. cephalosporin.
B. supraocclusion. * C. erythromycin.
C. dentin hypersensitivity. D. metronidazole.
D. acute pulpitis. E. ampicillin.

An incipient carious lesion is described Management of a “dry socket” should


as being in include

A. dentin without pulpal involvement. A. saline irrigation of socket. *


B. the cementum only. B. vigorous curettage of the socket.
C. the enamel and in the dentin up to C. placement of topical antibiotics in
1mm. the socket.
D. the enamel only. * D. a prescription for systemic
antibiotics.

Dr Abdul Naser Tamim SEHA, Dr Ghada Al Aqqad D.D.S, Dr Kamal Naser - Amrita Medical Centre, Dr Emad Wani - Shadi Dental Centre,
Dr Lina Anka SEHA, Dr Rouba Zgheibi SEHA, Maria Teresa Yongson Alejandrino SEHA 73
Marsupialization is the most Which of the following is the most
appropriate surgical technique in the appropriate for determining the
management of morphology of the temporomandibular
joint disc?
A. hygroma.
B. cystic ameloblastoma. A. Arthrography.
C. ranula. * B. Cone beam CT.
D. osteomyelitis. C. Magnetic resonance imaging. *
D. Corrected tomography.
Which of the following is a basic
design principle for a mucoperiosteal The placement of a post in an
flap? endodontically treated tooth with
minimal coronal tooth structure
A. Narrow based. provides
B. Limited bone exposure.
C. Able to be repositioned over bone. * A. retention for the core. *
D. Mesial release incision. B. a ferrule effect.
C. reinforcement of the remaining root
With respect to extraction of teeth, structure.
forceps beaks should? D. resistance to root fracture.

A. allow for a 180° rotation. An 8 year old patient presents with


B. be applied parallel to the long axis 4mm crowding of the mandibular
of the tooth. * incisors. The patient has a Class I
C. be pulled coronally during malocclusion and the arches are well
extraction. aligned. All cephalometric values are
D. be placed on the crown of the tooth. normal. The most appropriate
management for this patient is
Which impacted mandibular third
molar is usually easiest to remove? A. monitor and recall in 6 months. *
B. perform pulpotomies and
A. Mesio-angular. * interproximal discing on the mandibular
B. Horizontal. primary canines.
C. Disto-angular. C. extract the primary mandibular
D. Inverted. canines only.
D. extract the primary mandibular
Which of the following is a canines and place a lower lingual
CONTRAINDICATION for a single holding arch.
tooth extraction in a general dental
office? A 4 year old has a primary central
incisor that is yellow but asymptomatic.
A. The patient is severely hyperthyroid The most probable diagnosis is
and not under treatment. *
B. The patient is a well-controlled A. pulpal necrosis.
diabetic. B. pulpal calcification. *
C. The tooth is seated in a C. internal resorption.
pneumatized maxillary sinus. D. external resorption.
D. The tooth has internal root
resorption.

Dr Abdul Naser Tamim SEHA, Dr Ghada Al Aqqad D.D.S, Dr Kamal Naser - Amrita Medical Centre, Dr Emad Wani - Shadi Dental Centre,
Dr Lina Anka SEHA, Dr Rouba Zgheibi SEHA, Maria Teresa Yongson Alejandrino SEHA 74
Which of the following space
maintainers is/are most appropriate for A patient dislocates his mandible for
a 4 year old child whose mandibular the first time. After reduction, the most
first primary molars have been appropriate management is to
extracted?
A. inject the joint with hydrocortisone.
A. Bilateral band and loops. * B. inject the joint with a sclerosing
B. Lingual holding arch. solution.
C. A removable appliance. C. have the patient exercise the
D. Distal shoe appliances. mandible to avoid trismus.
D. immobilize for ten days. *
Which is the most appropriate initial
treatment for a 16 year old patient Management of a patient with an acute
presenting with multiple extensive periradicular abscess should include
carious lesions on 20 teeth?
1. Elimination of the cause.
A. Place amalgam restorations as 2. Drainage.
quickly as possible. 3. Supportive therapy.
B. Excavate and place provisional 4. External hot compresses.
restorations. *
C. Place the patient on a preventive A. (1) (2) (3) *
regime and delay any treatment. B. (1) and (3)
D. Restore all teeth with gold inlays to C. (2) and (4)
utilize the strength of the material. D. (4) only
E. All of the above.
A periapical radiograph of a 7 year old
child reveals an inverted mesiodens. The most appropriate treatment of an
The right central incisor is partially ameloblastoma is
erupted and the left central incisor has
not yet erupted. The most appropriate A. chemotherapy.
management is to B. enucleation.
C. resection. *
A. allow the mesiodens to erupt.
D. radiotherapy.
B. remove the mesiodens and
orthodontically erupt the central incisor. In the treatment of necrotizing
C. remove the mesiodens and place ulcerative gingivitis with associated
the patient on observation. * lymphadenopathy, which of the
D. wait for the mesiodens and the following medications is the most
unerupted central incisor to erupt. appropriate?
Displacement of fractures is influenced A. An anti-inflammatory.
by B. A topical antibiotic.
C. A systemic antibiotic. *
A. age.
D. An analgesic.
B. hemorrhage.
C. edentulism.
D. muscle attachments. *

Dr Abdul Naser Tamim SEHA, Dr Ghada Al Aqqad D.D.S, Dr Kamal Naser - Amrita Medical Centre, Dr Emad Wani - Shadi Dental Centre,
Dr Lina Anka SEHA, Dr Rouba Zgheibi SEHA, Maria Teresa Yongson Alejandrino SEHA 75
The most appropriate management of a 3. Radicular cyst.
painless ulcer of 2 months duration 4. Lateral periodontal cyst.
affecting the lateral border of the
tongue is to A. (1) (2) (3)
B. (1) and (3)
A. prescribe systemic antibiotics. C. (2) and (4) *
B. perform an incisional biopsy. * D. (4) only
C. re-evaluate in 6 months. E. All of the above.
D. obtain a complete blood count.
E. obtain a cytologic smear. An 80 year old man develops multiple
painful skin vesicles along the
Which of the following conditions may distribution of the right infraorbital
result from horizontally brushing the nerve. The most likely diagnosis is
teeth?
A. psoriasis.
A. Erosion. B. herpes zoster. *
B. Abrasion. * C. pemphigus vulgaris.
C. Attrition. D. candidiasis.
D. Hypoplasia.
The most common site of intraoral
The characteristic oral lesion(s) of squamous cell carcinoma is the
pemphigus is/are
A. palate.
A. vesicles and bullae. * B. floor of the mouth. *
B. Fordyce granules. C. gingiva.
C. white plaques. D. buccal mucosa.
D. hairy tongue.
E. candidiasis. Which of the following conditions is/are
associated with AIDS?
A pleomorphic adenoma is
characterized by 1. Acute marginal periodontitis.
2. Hairy leukoplakia.
A. metastases. 3. Candidiasis.
B. recurrance. * 4. Geographic tongue.
C. pain.
D. ulceration. A. (1) (2) (3) *
B. (1) and 3)
Which of the following is NOT a true C. (2) and (4)
cyst? D. (4) only
E. All of the above.
1. Keratocystic odontogenic tumour
(odontogenic keratocyst).
2. Traumatic bone cyst.

Dr Abdul Naser Tamim SEHA, Dr Ghada Al Aqqad D.D.S, Dr Kamal Naser - Amrita Medical Centre, Dr Emad Wani - Shadi Dental Centre,
Dr Lina Anka SEHA, Dr Rouba Zgheibi SEHA, Maria Teresa Yongson Alejandrino SEHA 76
Which of the following is/are D. necrotizing ulcerative periodontitis.
associated with xerostomia?
1. Atropine administration. Hemangiomas of the jaws
2. Acute anxiety state.
3. Mikulicz's disease. A. never occur in bone.
4. Sjögren syndrome. B. are malignant.
C. can appear cystic radiographically. *
A. (1) (2) (3) D. are metastatic lesions.
B. (1) and (3)
C. (2) and (4) An odour of acetone on the breath can
D. (4) only be found in patients with
E. All of the above. *
A. bronchiectasis.
The histopathologic changes in B. rhinitis.
chronic gingivitis are characterized by C. salicylate poisoning.
D. diabetes mellitus. *
A. loss of rete pegs and destruction of
the basement membrane. Oral lesions can be found in all of the
B. hyalinization of the principal fibres following conditions EXCEPT for
of the periodontal ligament.
C. an inflammatory infiltrate of plasma A. leukemia.
cells and lymphocytes. * B. pernicious anemia.
D. an inflammatory infiltrate in which C. infectious mononucleosis.
polymorphonuclear cells predominate. D. obstructive jaundice. *

The most likely diagnosis of a Osteomyelitis of the mandible may


proliferative lesion found at a denture follow
periphery is a/an
1. Radiotherapy.
A. epulis granulomatosum. 2. Dentoalveolar abscess.
B. epulis fissuratum. * 3. Fracture.
C. giant cell granuloma. 4. Vincent’s angina.
D. squamous cell carcinoma. A. (1) (2) (3)
B. (1) and (3)
Leukemic gingivitis has a similar C. (2) and (4)
clinical appearance to D. (4) only
E. All of the above. *
A. ascorbic acid deficiency gingivitis. *
B. infectious mononucleosis.
C. thrombocytopenic purpura.

Dr Abdul Naser Tamim SEHA, Dr Ghada Al Aqqad D.D.S, Dr Kamal Naser - Amrita Medical Centre, Dr Emad Wani - Shadi Dental Centre,
Dr Lina Anka SEHA, Dr Rouba Zgheibi SEHA, Maria Teresa Yongson Alejandrino SEHA 77
Which of the following is/are associated The characteristic colour seen in the
with an impacted tooth? crowns of teeth with internal resorption
is due to
1. Odontogenic adenomatoid tumour.
2. Periapical osseous dysplasia A. deposition of pigment in the cells of
(periapical cemento-osseous the odontoblast layer.
dysplasia). B. the presence of hyperplastic
3. Calcifying epithelial odontogenic vascular pulp tissue. *
tumour. C. a change in the consistency of the
4. Cementoblastoma. dentin.
D. the difference in the refractive
A. (1) (2) (3) indices of the normal and affected
B. (1) and (3) * areas.
C. (2) and (4) E. the degeneration and necrosis of
D. (4) only the pulp tissue.
E. All of the above.
Which of the following is a proliferative
Periapical odontogenic cysts are response of the soft tissue to an
primarily associated with irritant?

A. impacted wisdom teeth. A. Cellulitis.


B. congenitally missing teeth. B. Abscess.
C. nonvital teeth. * C. Pyogenic granuloma. *
D. Aphthous ulcer.
Preoperative endodontic radiographs
will show the Dysplastic lesions of squamous
epithelium occur most often on the
A. presence of a pulp exposure.
B. exact location of the apical foramen. A. palate.
C. presence of active infection. B. gingiva.
D. vitality of the pulp. C. buccal mucosa.
E. size of the pulp chamber and root D. dorsum of the tongue.
canal(s). * E. floor of the mouth. *

The aluminum filter in an x-ray machine The shape of the distobuccal border of
prevents which of the following from a mandibular denture is determined
reaching the patient? primarily by the

A. Long wavelength x-rays. * A. buccinator muscle.


B. High frequency x-rays. B. tendon of the temporalis muscle.
C. X-rays at the edge of the x-ray beam. C. masseter muscle. *
D. Gamma radiation.

Dr Abdul Naser Tamim SEHA, Dr Ghada Al Aqqad D.D.S, Dr Kamal Naser - Amrita Medical Centre, Dr Emad Wani - Shadi Dental Centre,
Dr Lina Anka SEHA, Dr Rouba Zgheibi SEHA, Maria Teresa Yongson Alejandrino SEHA 78
In primary molars, the cusp with the B. submit the tissue for histological
largest pulp horn is the analysis. *
C. perform a cytologic smear.
A. distolingual. D. culture the fluid.
B. distobuccal. E. order blood tests.
C. mesiolingual.
D. mesiobuccal. * Exfoliative cytology is indicated in the
diagnosis of
The most appropriate management of
an intruded 5.1 with the apex displaced A. lichen planus.
toward the labial bone plate is to B. aphthous ulceration.
C. herpes simplex. *
A. extract. D. benign mucous membrane
B. leave in place and perform a pemphigoid.
pulpectomy. E. erythema multiforme.
C. reposition and perform a
pulpectomy. Nystatin is the most appropriate drug to
D. allow spontaneous repositioning. * treat

A 58 year old woman complains of A. aphthous stomatitis.


electric shock-like pain on the left side B. candidiasis. *
of the chin and lip when eating or C. periradicular abscess.
taking a hot shower. Which of the D. necrotizing ulcerative gingivitis.
following medications is most
appropriate to confirm the diagnosis? Aggressive periodontitis has all of the
following features EXCEPT
A. Amoxicillin.
B. Carbamazepine. * A. rapid attachment loss.
C. Acetaminophen and codeine. B. specific periodontal microbial
D. Ibuprofen. pathogens.
C. radiographic evidence of bone loss.
Keratocystic odontogenic tumours D. ulcerations of the gingiva. *
(odontogenic keratocysts) have a/an
Disuse atrophy of the periodontium
A. inflammatory origin. causes
B. mixed radiopaque/radiolucent
appearance. A. changes in the arrangement of fibre
C. solid consistency. bundles.
D. tendency to recur. * B. narrowing of the periodontal
ligament.
The most appropriate method to C. osteoporosis of the alveolar process.
diagnose a cystic tumour is to D. decrease in tooth mobility.
E. All of the above. *
A. examine the fluid under a
microscope.

Dr Abdul Naser Tamim SEHA, Dr Ghada Al Aqqad D.D.S, Dr Kamal Naser - Amrita Medical Centre, Dr Emad Wani - Shadi Dental Centre,
Dr Lina Anka SEHA, Dr Rouba Zgheibi SEHA, Maria Teresa Yongson Alejandrino SEHA 79
Pathologic migration of teeth is a D. create new attachment.
clinical feature of
Most bone loss following the
A. disuse atrophy. placement of dental implants occurs at
B. necrotizing ulcerative gingivitis.
C. plaque-induced gingivitis. A. 0 – 1 year. *
D. generalized chronic periodontitis. * B. 2 – 4 years.
C. 5 – 7 years.
Regular use of oral irrigators will D. 10 – 12 years.

A. prevent plaque formation. The primary objective of periodontal


B. remove plaque. flap surgery in the treatment of
periodontitis is to
C. remove calculus.
D. remove debris. *
A. remove granulation tissue.
E. prevent bacteremia.
B. provide access for periodontal
debridement. *
The most reliable measurement of the
effectiveness of root planing at re- C. add bone support.
evaluation 4-6 weeks later is D. correct gingival architecture.

A. root smoothness. Which of the following root surfaces


have concavities that make root
B. absence of plaque.
planing difficult?
C. absence of bleeding upon probing.
*
A. Mesial of maxillary first premolars. *
D. increased sulcular fluid flow.
B. Lingual of mandibular first
premolars.
A patient presents with 5mm of
gingival recession on the labial of C. Mesial of maxillary incisors.
tooth 1.3. The most predictable D. Distal of the palatal roots of
surgical procedure to achieve root maxillary molars.
coverage on this tooth is a
The objective of root planing during
A. free autogenous gingival graft. periodontal therapy is to remove
B. subepithelial connective tissue
graft. * A. plaque, calculus, contaminated
cementum and junctional epithelium.
C. laterally positioned flap.
B. plaque and calculus exclusively.
D. double papilla pedicle graft.
C. plaque, calculus and crevicular
A daily chlorhexidine rinse following epithelium.
periodontal flap surgery is primarily D. plaque, calculus and contaminated
used to cementum. *
E. all cementum associated with
A. enhance regeneration. periodontitis.
B. encourage wound healing.
C. facilitate plaque control. *

Dr Abdul Naser Tamim SEHA, Dr Ghada Al Aqqad D.D.S, Dr Kamal Naser - Amrita Medical Centre, Dr Emad Wani - Shadi Dental Centre,
Dr Lina Anka SEHA, Dr Rouba Zgheibi SEHA, Maria Teresa Yongson Alejandrino SEHA 80
Which of the following periodontal
procedures is indicated on a maxillary A. there is sufficient tooth bulk in the
canine that will receive a full crown abutment teeth for retention.
with subgingival margins when the B. the interocclusal distance will be
abutment has 1mm of attached physiologically tolerated. *
gingiva, no sign of inflammation or loss C. the aesthetic appearance of the
of attachment? patient will be improved.
D. a favorable crown-root ratio is
A. Root planing. established.
B. Coronally positioned flap.
C. Localized gingivectomy. Subgingival calculus
D. Autogenous connective tissue graft.
E. There is no indication that this tooth A. does not have a definite
requires periodontal treatment. * predelection for a specific site or sites.
B. derives from subgingival plaque.
Compared to a set of opposing C. may take longer to form compared
complete dentures, a maxillary denture to supragingival calculus.
opposing a full complement of natural D. All of the above. *
teeth is more often associated with
A surgical template (stent/guide) for an
A. less denture tooth wear. immediate maxillary denture is used to
B. greater incidence of denture
fracture. * A. control hemorrhage while the new
C. improved retention of the denture. denture is being fabricated.
D. improved stability of the denture. B. protect the extraction sites while
E. improved appearance of the fitting the denture.
denture. C. assist in remounting the denture
prior to refining the occlusion.
In periodontal flap surgery, the initial D. indicate areas that require additional
incision is made to hard or soft tissue reduction.*

A. remove the sulcular lining of the Following root planing, reduction in


pocket. * pocket depth is due to
B. aid in healing.
C. sever the attachment of the oblique A. shrinkage of the gingival tissue.
fibres of the periodontal ligament. B. epithelial attachment.
D. excise the keratinized gingiva. C. connective tissue attachment.
D. All of the above. *
In fixed bridge construction, when the
vertical dimension has to be
increased, the most important
consideration is that

Dr Abdul Naser Tamim SEHA, Dr Ghada Al Aqqad D.D.S, Dr Kamal Naser - Amrita Medical Centre, Dr Emad Wani - Shadi Dental Centre,
Dr Lina Anka SEHA, Dr Rouba Zgheibi SEHA, Maria Teresa Yongson Alejandrino SEHA 81
The gingivectomy approach to pocket After periodontal surgery, sensitivity to
elimination results in thermal change is reduced by

A. healing by primary intention. 1. Replaning the roots.


B. adequate access to correct irregular 2. Keeping the roots free of bacterial
osseous contours. plaque.
C. retention of all or most of the 3. Adjusting the occlusion.
attached gingiva. 4. Desensitizing the roots with an
D. None of the above. * appropriate medicament.

A surgical flap approach to periodontal A. (1) (2) (3)


pocket elimination permits B. (1) and (3)
A. healing by primary intention. C. (2) and (4) *
B. retention of gingiva. D. (4) only
C. access to perform osseous E. All of the above.
recontouring.
D. All of the above. * Maintenance care for a patient treated
for periodontal disease includes
Gingivectomy is indicated for periodic assessment of

1. Pseudopockets. 1. Tooth mobility.


2. Suprabony pockets. 2. Probing depth.
3. Fibrotic gingival enlargements. 3. Gingival inflammation.
4. Infrabony pockets. 4. Oral hygiene status.

A. (1) (2) (3) * A. (1) (2) (3)


B. (1) and (3) B. (1) and (3)
C. (2) and (4) C. (2) and (4)
D. (4) only D. (4) only
E. All of the above. E. All of the above. *

Root planing is used in the treatment of The prognosis for a replanted


pockets which are developed permanent tooth is most
influenced by the
1. Edematous.
2. Fibrotic. A. length of time the tooth has been out
3. below the mucogingival junction. of the mouth. *
4. Infrabony. B. use of an antibiotic.
C. effectiveness of the irrigation of the
A. (1) (2) (3) socket.
B. (1) and (3) D. rigidness of the splint applied.
C. (2) and (4) E. thoroughness of the curettage of the
D. (4) only root surface.
E. All of the above. *

Dr Abdul Naser Tamim SEHA, Dr Ghada Al Aqqad D.D.S, Dr Kamal Naser - Amrita Medical Centre, Dr Emad Wani - Shadi Dental Centre,
Dr Lina Anka SEHA, Dr Rouba Zgheibi SEHA, Maria Teresa Yongson Alejandrino SEHA 82
Resective osseous surgery is best
suited for periodontal sites with Which of the following conditions has
the POOREST prognosis?
A. severe attachment loss.
B. severe intrabony defects. A. Gingival tissue with edematous red
C. teeth with short roots. interdental papillae.
D. early to moderate bone loss. * B. Gingival tissue with 1mm pocket
labial to a mandibular incisor.
Which of the following conditions has C. Firm gingival tissue with
the WORST prognosis for a furcation generalized 5 and 6mm pockets.
involved tooth? D. Class II furcation involvement in
maxillary molars. *
A. Wide root separation. *
B. Narrow root separation. A loss of sensation in the lower lip may
C. A bifurcation ridge. be produced by
D. A cemento-enamel projection.
A. Bell’s palsy.
Which type of periodontitis is generally B. trigeminal neuralgia.
treated WITHOUT systemic antibiotics? C. malignancy in the body of the
mandible. *
A. Localized severe aggressive D. fracture in the mandibular canine
periodontitis in a 16 year old patient. region.
B. Localized severe aggressive
periodontitis in a 25 year old patient. Which of the following is an etiologic
factor in the development of
C. Severe refractory chronic
necrotizing ulcerative gingivitis?
periodontitis.
D. Severe generalized chronic
A. Gluten intolerance.
periodontitis. *
B. Acute stress. *
E. Generalized severe aggressive
periodontitis in a 30 year old patient. C. Lack of attached gingiva.
D. Gingival trauma.
The principal reason for a needle
aspiration of an intraosseous Which of the following is contagious?
radiolucent lesion of the mandible is to
A. Pemphigus.
A. obtain a specimen for B. Primary herpetic gingivostomatitis. *
histopathological analysis. C. Recurrent aphthous stomatitis.
B. determine the presence of a D. Necrotizing ulcerative gingivitis.
vascular lesion. *
C. decompress a fluid-filled lesion.
D. drain purulent material.

Dr Abdul Naser Tamim SEHA, Dr Ghada Al Aqqad D.D.S, Dr Kamal Naser - Amrita Medical Centre, Dr Emad Wani - Shadi Dental Centre,
Dr Lina Anka SEHA, Dr Rouba Zgheibi SEHA, Maria Teresa Yongson Alejandrino SEHA 83
The preconditioning of a high glass
content all-ceramic restoration prior to A. Red, swollen, increased stippling.
bonding is achieved by B. Cyanotic, cleft formation, lack of
stippling.
A. sandblasting. C. Red, swollen, lack of stippling. *
B. acid etching with phosphoric acid. D. Pink, swollen, lack of stippling.
C. roughening the surface with a
diamond bur. Which of the following is LEAST likely
D. acid etching with hydrofluoric acid. to influence the development of
E. degreasing with acetone. * plaque induced gingivitis?

The most common complaint of a A. Pregnancy.


patient with gingivitis is B. Diabetes.
C. Traumatic occlusion. *
A. bleeding on brushing. * D. Dental plaque.
B. painful gums. E. Calculus.
C. changes in the texture of the
gingiva. The predominant micro-organisms
D. bad breath. associated with periodontitis are

Severe chronic periodontitis is defined A. Gram-positive aerobes.


by B. Gram-negative aerobes.
C. Gram-positive anaerobes.
A. attachment loss greater than 5mm. D. Gram-negative anaerobes. *
*
B. periodontal pockets greater than Periodontitis
5mm.
C. presence of purulent exudate. 1. Develops from gingivitis.
D. presence of a Class I furcation 2. Goes through stages of tissue
involvement. destruction and quiescence.
3. is associated with bone loss.
Gingival bleeding associated with 4. Is caused by occlusal trauma.
plaque induced gingivitis is due to
A. (1) (2) (3) *
A. a vitamin C deficiency. B. (1) and (3)
B. destruction of vessels of the C. (2) and (4)
periodontal ligament. D. (4) only
C. excessive pocket depth. E. All of the above.
D. microulceration of sulcus
epithelium. *

Which of the following changes in


colour, contour and texture are
indicative of plaque induced gingivitis?

Dr Abdul Naser Tamim SEHA, Dr Ghada Al Aqqad D.D.S, Dr Kamal Naser - Amrita Medical Centre, Dr Emad Wani - Shadi Dental Centre,
Dr Lina Anka SEHA, Dr Rouba Zgheibi SEHA, Maria Teresa Yongson Alejandrino SEHA 84
The principal microorganism in
localized aggressive periodontitis is Subgingival calculus

A. Porphyromonas gingivalis. A. accumulates predominantly on


B. Fusobacterium nucleatum. mandibular incisors and maxillary
C. Aggregatobacter molars.
actinomycetemcomitans. * B. derives its dark colour from food and
D. Prevotella intermedia. drinks.
C. is the cause of periodontitis.
Which of the following is NOT a risk D. is mineralized subgingival plaque. *
factor for periodontitis?
Which of the following is NOT a sign of
A. Smoking tobacco. occlusal trauma?
B. Poorly controlled diabetes.
C. Coronary heart disease. * A. Fremitus.
D. Poor oral hygiene. B. Gingival recession. *
C. Widened periodontal ligament.
Diagnosis of periodontitis is clinically D. Tooth migration.
demonstrated by
During orthodontic treatment, a healthy
A. bleeding from the base of the adolescent will most frequently present
periodontal pocket. with
B. loss of periodontal attachment. *
C. pain upon probing. A. gingivitis. *
B. horizontal bone loss.
The periodontal condition showing C. necrotizing ulcerative gingivitis.
localized advanced vertical bone loss D. vertical bone loss.
involving the first molars and the
incisors is diagnosed as A 16 year old healthy patient has good
oral hygiene with minimal plaque and
A. necrotizing ulcerative periodontitis. calculus, but severe interproximal
B. aggressive localized periodontitis. * attachment loss affecting the first
C. chronic localized periodontitis. molars and incisors. The most likely
D. periodontitis as a manifestation of a diagnosis is
systemic disease.
A. localized aggressive periodontitis. *
Clinical diagnosis of periodontitis B. localized chronic periodontitis.
requires the presence of C. generalized aggressive
periodontitis.
A. bleeding upon probing. D. generalized chronic periodontitis.
B. loss of periodontal attachment. *
C. a periodontal pocket.
D. tooth mobility.

Dr Abdul Naser Tamim SEHA, Dr Ghada Al Aqqad D.D.S, Dr Kamal Naser - Amrita Medical Centre, Dr Emad Wani - Shadi Dental Centre,
Dr Lina Anka SEHA, Dr Rouba Zgheibi SEHA, Maria Teresa Yongson Alejandrino SEHA 85
Which of the following would E. T-cell lymphocyte.
differentiate clinically between an acute
apical abscess (acute periradicular The most appropriate management
abscess) and an acute periodontal for a permanent central incisor with a
abscess on a single rooted tooth? necrotic pulp and a wide open apex is

A. Pain upon palpation. A. pulpotomy.


B. Tooth mobility. B. apexification. *
C. Pain upon percussion. C. revascularization.
D. Pulp vitality testing. * D. root canal therapy using gutta-
percha.
The muscles used when closing the E. root canal therapy followed by a
jaws to maximum intercuspation retrograde filling.
include
In an 8 year old patient the most
A. medial pterygoid, lateral pterygoid appropriate treatment of a vital first
and masseter muscles. permanent molar with closed apices
B. temporalis, medial pterygoid, and a large carious exposure is
masseter and geniohyoid muscles.
C. medial pterygoid, temporalis and A. pulpotomy with MTA.
masseter muscles. * B. pulpectomy. *
D. lateral pterygoid, masseter, C. direct pulp capping with calcium
temporalis and geniohyoid muscles. hydroxide.
D. indirect pulp capping.
A patient experiences pain and some
gingival swelling in the anterior The maxillary central incisors of a 2
segment of the mandible. The year old child have been traumatically
mandibular lateral incisor has a intruded 4mm. The most appropriate
shallow restoration, is tender to immediate management is to
percussion and gives a positive
response to an electric pulp test. There A. carefully remove both incisors.
is some mobility. What is the most B. reposition the intruded teeth.
likely diagnosis? C. make the patient comfortable
without disturbing the teeth. *
A. Acute apical abscess (acute D. reposition and splint the intruded
periradicular abscess). teeth.
B. Symptomatic irreversible pulpitis. E. order an occlusal radiograph.
C. Periodontal abscess. *
D. Reversible pulpitis. Oral hygiene for infants’ teeth should
begin when
Which of the following is the most
active cell in synthesizing and A. the first primary molars erupt.
secreting antibodies? B. all primary teeth erupt.
C. the first tooth erupts. *
A. Mast cell. D. the infant is weaned from the
B. Macrophage. nursing bottle or breast.
C. Eosinophilic granulocyte.
D. Plasma cell. *

Dr Abdul Naser Tamim SEHA, Dr Ghada Al Aqqad D.D.S, Dr Kamal Naser - Amrita Medical Centre, Dr Emad Wani - Shadi Dental Centre,
Dr Lina Anka SEHA, Dr Rouba Zgheibi SEHA, Maria Teresa Yongson Alejandrino SEHA 86
The most appropriate restoration for a A. Use of separate etching and
primary first molar with extensive bonding agents rather than a self-
carious destruction of the crown is a etching bonding agent.
B. Roughening of the enamel with a 2
A. posterior composite resin. round bur.
B. pin retained amalgam. C. Maintaining a dry field until the
C. stainless steel crown. * sealant is set.
D. resin-modified glass ionomer. D. Use of a bonding agent prior to
sealant application. *
The eruption of a maxillary permanent
first molar is prevented by a slight The efficacy of pit and fissure sealants
interference with the crown of the is affected by
primary second molar. Which of the
following is the most appropriate A. occlusal relationship.
management? B. opacity of the sealant.
C. stage of tooth eruption. *
A. Extract the primary molar. D. type of polymerization reaction.
B. Reduce the distal surface of the E. systemic fluoride treatment.
primary molar.
C. Unlock the permanent first molar A 7 year old patient presents with a
with elastic and (or) separation wire. * crown fracture of a permanent
D. Remove the soft tissue overlying the maxillary central incisor that occurred
occlusal surface of the permanent 2 hours ago. The incisal half of the
molar. crown is missing, resulting in a 2mm
E. Wait until more tuberosity growth exposure of vital pulp. What is the
occurs. most appropriate initial management
for this tooth?
A labial frenum causes a diastema
between the permanent maxillary A. Apexification.
central incisors. The lateral incisors B. Pulpotomy. *
and canines have not erupted. The C. Pulp capping.
most appropriate immediate D. Pulpectomy.
management is to
Radiographic examination of an
A. perform a frenectomy. intruded primary maxillary central
B. close the space with a fixed incisor reveals no root fracture and no
orthodontic appliance. displacement in the permanent tooth
C. observe the case until the eruption germ. The most appropriate
of permanent maxillary lateral incisors management would be to
and canines. *
D. close the space with a removable A. extract the tooth.
orthodontic appliance. B. bring the tooth into position and
E. close the space after the eruption of ligate it.
the permanent lateral incisors. C. observe periodically. *
D. remove the tooth and replant it in a
Which of the following procedures will more favorable position.
NOT improve the retention of a fissure
sealant?

Dr Abdul Naser Tamim SEHA, Dr Ghada Al Aqqad D.D.S, Dr Kamal Naser - Amrita Medical Centre, Dr Emad Wani - Shadi Dental Centre,
Dr Lina Anka SEHA, Dr Rouba Zgheibi SEHA, Maria Teresa Yongson Alejandrino SEHA 87
What is the most appropriate
endodontic management for a 9 year At the first post-insertion appointment,
old patient for tooth 1.1, with an open a patient with a new removable partial
apex and a necrotic pulp? denture complains of a tender
abutment tooth. The most likely cause
A. Pulpotomy. is
B. Apexification. *
C. Pulpectomy. A. an overextended border on the
D. Revascularization. partial.
E. Apical surgery. B. inadequate polishing of the
framework.
Fixed partial denture pontics should C. improper path of insertion.
D. the occlusion. *
A. completely replace the missing
supragingival tooth structure. When epinephrine 1:1000 is
B. have a concave surface touching administered intramuscularly for the
the mucosa. management of anaphylaxis in an
C. have minimal soft tissue coverage. adult, the most appropriate volume for
* an initial dose is
D. hide the porcelain-metal junction on
their gingival aspect. A. 0.02 – 0.04ml.
B. 0.3 – 0.5ml. *
The rate of set of alginate impression C. 1.0 – 1.5ml.
materials can be increased by D. 1.6 – 1.8ml.

A. increasing the water/powder ratio. Which of the following factors could


cause a partial denture framework to fit
B. increasing the temperature of the
tighter in the mouth than on the cast?
mixing water. *
C. adding sodium phosphate to the
A. Too much water in the mix of the
mixing water.
stone for the cast. *
D. decreasing the amount of mixing.
B. Not enough water in the mix of the
stone for the cast.
In which of the following will the effects
of polymerization shrinkage be C. Duplication impression slightly
greatest? oversized.
D. Improper wax-up of the partial
A. Class I occlusal restoration. * denture.
B. Preventive resin restoration.
C. Direct veneer restoration.
D. Class IV restoration.

Dr Abdul Naser Tamim SEHA, Dr Ghada Al Aqqad D.D.S, Dr Kamal Naser - Amrita Medical Centre, Dr Emad Wani - Shadi Dental Centre,
Dr Lina Anka SEHA, Dr Rouba Zgheibi SEHA, Maria Teresa Yongson Alejandrino SEHA 88
When making maxillomandibular
records for a complete denture patient, A. Resorption of bone.
the vertical dimension of occlusion is B. Increase in trabeculation.
acceptable when C. Increase in width of cementum.
D. Decrease in width of periodontal
A. interocclusal distance (freeway ligament. *
space) is 8-10mm.
B. occlusal rims contact evenly and The major connector of a removable
bilaterally at the same time as the lips partial denture should be designed to
touch.
C. the maxillary rim shows just below A. rigidly connect the denture
the upper lip and the mandibular rim is components. *
even with the corners of the mouth. B. act as a stress-breaker.
D. it is equal to the rest vertical C. dissipate vertical forces.
dimension. D. distribute forces to the soft tissues.
E. the difference between the occlusal
vertical dimension and the rest vertical When a partial denture framework fits
dimension is 2-4mm. * the master cast but does not fit
properly in the mouth, the error is likely
Rests on terminal abutment teeth for a to be in the
removable partial denture provide
A. impression making. *
A. primary retention. B. casting of the framework.
B. indirect retention. C. design of the framework.
C. occlusal force transmission. * D. preparation of the teeth.
D. lateral force transmission.
A 3 year old presents with subluxated
Reciprocation as applied to removable maxillary central incisors. The
partial dentures design refers to the occlusion is normal. What is the most
appropriate management?
A. function of the occlusal rest to
counteract occlusal forces. A. Reposition the teeth and place the
B. resistance to flexion of the retentive child on a soft diet.
clasp arm. B. Reposition the teeth and splint for 7
C. return to a passive state of the to 10 days.
flexed clasp. C. Place the child on a soft diet and
D. function of the reciprocal clasp arm monitor the teeth. *
to counteract the retentive clasp arm. * D. Extract the traumatized teeth.

Which of the following is NOT a direct


physiological response to additional
forces placed on abutment teeth?

Dr Abdul Naser Tamim SEHA, Dr Ghada Al Aqqad D.D.S, Dr Kamal Naser - Amrita Medical Centre, Dr Emad Wani - Shadi Dental Centre,
Dr Lina Anka SEHA, Dr Rouba Zgheibi SEHA, Maria Teresa Yongson Alejandrino SEHA 89
A 5 year old child has yellow
pigmentation of the deciduous teeth The extension of the lingual anterior
which under ultraviolet light gives a border of a mandibular denture is
bright yellow fluorescence. The most limited by the
likely diagnosis is
A. mylohyoid muscle.
A. tetracycline pigmentation. * B. geniohyoid muscle.
B. pigmentation associated with C. genioglossus muscle. *
chromogenic bacteria. D. fibres of the digastric muscle.
C. amelogenesis imperfecta.
D. enamel hypoplasia. In complete denture construction,
custom trays are recommended for
When compared to a conventional polyvinyl siloxane and polysulfide
complete denture, an overdenture rubber base impression materials to

A. is more fracture resistant. A. obtain a uniform thickness of


B. preserves more alveolar bone. * material. *
C. causes less soft tissue B. facilitate removal of the
inflammation. impression.
D. is less expensive. C. allow for a more uniform setting of
the material.
For a patient allergic to penicillin, what D. eliminate the need for a tray
is the most appropriate drug for adhesive.
antibiotic prophylaxis?
Long term stability in partial dentures
A. Ampicillin. is best ensured by
B. Cephalexin.
C. Clindamycin. * A. use of cast clasps.
D. Erythromycin. B. establishing harmonious
E. Metronidazole. occlusion. *
C. incorporating all undercut areas
Embryologically, fusion of the palatal available.
shelves should be completed by the D. use of indirect retention.

A. Fifth week.
B. Tenth week. *
C. Fifteenth week.
D. Twentieth week.

In a patient with complete dentures,


cheek biting may result from

A. an excessive vertical dimension of


occlusion.
B. the use of steep-cusped posterior
teeth.
C. insufficient coverage of the
retromolar pad areas.
D. insufficient horizontal overlap of the
posterior teeth. *

Dr Abdul Naser Tamim SEHA, Dr Ghada Al Aqqad D.D.S, Dr Kamal Naser - Amrita Medical Centre, Dr Emad Wani - Shadi Dental Centre,
Dr Lina Anka SEHA, Dr Rouba Zgheibi SEHA, Maria Teresa Yongson Alejandrino SEHA 90
A preparation for a Class II amalgam
restoration in primary molars should What is the most likely cause of
include sensitivity related to a noncarious
cervical lesion?
A. occlusally divergent buccal and
lingual walls. A. Loss of dentin bulk.
B. an axial wall that parallels the B. Increased number of open dentinal
dentino-enamel junction. * tubules. *
C. an isthmus that occupies two thirds C. Bacterial contamination.
of the intercuspal distance. D. Hyperocclusion.
D. undercut enamel and dentin.
A differential diagnosis for gingival
An immediate toxic reaction to a local hyperplasia should include which of
anesthetic administration is caused by the following conditions?
a/an
A. Multiple myeloma.
A. deterioration of the anesthetic agent. B. Monocytic leukemia. *
B. hypersensitivity to the C. Erythema multiforme.
vasoconstrictor. D. Pemphigus vulgaris.
C. hypersensitivity to the anesthetic
agent. The most important diagnostic
D. excessive blood level of the element in assessing the periodontal
anesthetic agent. * status of a patient is the

The most frequent cause of death A. results of vitality testing.


occurring under general anesthesia is B. radiographic appearance.
C. depth of periodontal pockets. *
A. overdosage of anesthetic agent. D. mobility of the teeth.
B. cardiac arrest.
C. traction on the viscera. Which of the following antibiotics may
D. mismanagement of the airway. * be cross-allergenic with penicillin?
E. overdosage of premedication.
A. Neomycin.
The principal reason for a cavosurface B. Cephalexin. *
bevel on a gold inlay preparation is to C. Clindamycin.
D. Erythromycin.
A. remove undermined enamel. E. Tetracycline.
B. improve marginal adaptation. *
C. decrease marginal percolation.
D. increase resistance and retention
forms.

Dr Abdul Naser Tamim SEHA, Dr Ghada Al Aqqad D.D.S, Dr Kamal Naser - Amrita Medical Centre, Dr Emad Wani - Shadi Dental Centre,
Dr Lina Anka SEHA, Dr Rouba Zgheibi SEHA, Maria Teresa Yongson Alejandrino SEHA 91
The muscle attached to the labial
surface of the maxilla above the region A patient presents with a non-healing
of the central incisors is lesion on the side of the nose. It has a
rolled border and has been increasing
A. labii inferioris. in size. The most likely diagnosis is
B. labii superioris.
C. orbicularis oris. * A. a sebaceous cyst.
D. risorius. B. a basal cell carcinoma. *
E. caninus. C. lupus erythematosus.
D. verruca vulgaris.
Which of the following is the principal E. an epulis.
muscle of the floor of the mouth?
Granulation tissue is composed of
A. Stylohyoid.
B. Digastric. A. plasma cells and giant cells.
C. Geniohyoid. B. fibroblasts, capillaries and chronic
D. Mylohyoid. * inflammatory cells. *
E. Platysma. C. fibroblasts and eosinophils.
D. epithelial cells and tissue
A 28 year old patient who has severe phagocytes.
anterior crowding, an Angle Class II
malocclusion, and a high DMFS score The genial tubercles are best
wants orthodontic treatment. Which of visualized on which type of
the following is the most appropriate radiograph?
initial step in management of this
case? A. Occlusal. *
B. Periapical.
A. Cephalometric analysis. C. Bitewing.
B. Diagnostic wax-up. D. Panoramic.
C. Caries risk assessment. *
D. Fluoride rinse prescription. Increased radiographic density is
caused by
Exophthalmia may be a sign of
A. decreased mA.
A. hypoadrenalism. B. decreased kVp.
B. hyperadrenalism. C. decreased target-object distance. *
C. hypothyroidism. D. increased object-film distance.
D. hyperthyroidism. *
E. hypoparathyroidism.

Dr Abdul Naser Tamim SEHA, Dr Ghada Al Aqqad D.D.S, Dr Kamal Naser - Amrita Medical Centre, Dr Emad Wani - Shadi Dental Centre,
Dr Lina Anka SEHA, Dr Rouba Zgheibi SEHA, Maria Teresa Yongson Alejandrino SEHA 92
A healthy 78 year old patient presents ligament space appears normal. The
with three new carious lesions on root most appropriate management would
surfaces. This is most likely the result be to
of
A. perform root canal treatment and
A. age related changes in cementum nonvital bleaching.
composition. B. perform root canal treatment and
B. the architecture at the CEJ. fabricate a post retained metal-ceramic
C. changes in dietary pattern. * crown.
D. chronic periodontal disease. C. perform root canal treatment and
fabricate a porcelain veneer.
Which of the following dental materials D. fabricate a metal-ceramic crown.
shows a chemical bond to clean E. fabricate a porcelain veneer. *
dentin?
Spontaneous fracture of the mandible
A. Calcium hydroxide. may occur during mastication if the
B. Zinc-phosphate. patient
C. Zinc-oxide and eugenol.
D. Glass ionomer. * A. has an impacted third molar close to
the inferior border of the mandible.
The selective grinding rule for B. is edentulous and there is advanced
removing premature contacts on the atrophy of the mandible. *
working side of complete dentures is C. is a child with unerupted premolars.
to grind D. is a postmenopausal woman with a
full dentition and generalized
A. only the fossae of teeth. osteoporosis.
B. the buccal cusps of maxillary and
lingual cusps of mandibular teeth. * At the preparation stage for a 4.6 DO
C. only the mandibular teeth. restoration, a brown, non-cavitated
D. lingual and buccal cusps of lesion is observed on the 4.7 mesial
maxillary teeth. proximal surface. There is no evidence
E. the maxillary and mandibular on a bitewing radiograph of a
cuspids. radiolucency extending into the dentin.
What is the most appropriate
Which of the following pontic designs management of the mesial surface of
will make it most difficult to maintain tooth 4.7?
optimal oral hygiene?
A. Perform no operative treatment and
A. Modified ridge lap. apply fluoride. *
B. Hygienic. B. Prepare the lesion with a ball
C. Saddle. * diamond and restore with a flowable
D. Conical. composite resin.
E. Ovoid. C. Prepare and restore with a mesio-
occlusal slot resin modified glass
A patient complains of the ionomer.
discolouration of an upper central D. Prepare and restore with a mesio-
incisor. Radiographically, the pulp occlusal slot composite resin.
chamber and the root canal space are
obliterated and the periodontal

Dr Abdul Naser Tamim SEHA, Dr Ghada Al Aqqad D.D.S, Dr Kamal Naser - Amrita Medical Centre, Dr Emad Wani - Shadi Dental Centre,
Dr Lina Anka SEHA, Dr Rouba Zgheibi SEHA, Maria Teresa Yongson Alejandrino SEHA 93
A patient on broad spectrum antibiotics
for 4 weeks has widespread, sore, red Osteoporosis may result from
and white oral mucosal lesions. The
most likely diagnosis is A. hypothyroidism.
B. acromegaly.
A. candidiasis. * C. diabetes.
B. leukoplakia. D. prolonged steroid therapy. *
C. erythema multiforme.
D. erosive lichen planus. Which of the following are signs of
E. pemphigoid. aggressive periodontitis?

Cleidocranial dysostosis can be 1. Rapid attachment loss.


associated with (a) 2. Specific periodontal microbial
pathogens.
A. premature loss of teeth. 3. Onset before the age of 35.
B. micrognathia. 4. Ulcerations of the gingiva.
C. high incidence of clefts.
D. associated high caries index. A. (1) (2) (3) *
E. multiple supernumerary and B. (1) and (3)
unerupted teeth. * C. (2) and (4)
D. (4) only
Angular cheilitis in older patients who E. All of the above.
wear complete dentures is most likely
associated with Which of the following drugs does NOT
cause gingival enlargement?
A. antibiotic therapy.
B. decreased vertical dimension. * A. Nifedipine.
C. decreased salivary flow. B. Cyclosporine.
D. vitamin B deficiency. C. Phenytoin.
D. Prednisolone.
A 10 year old has diffuse bilateral white *
areas on the buccal mucosa, dorsum A patient who has until recently been on
of the tongue and the floor of the prolonged corticosteroid therapy may
mouth. The mother tells the dentist that have
she has noticed this appearance for at
least 3 years, but she was not A. increased bleeding time.
concerned since she has had a similar B. hyposensitivity to pain.
condition all of her life. The most likely C. decreased tolerance to physiological
diagnosis is stress. *
D. an increased metabolic rate.
A. leukoplakia. E. high level of plasmatic cortisol.
B. white sponge nevus. *
C. frictional keratosis.
D. allergic reaction to cinnamon.

Dr Abdul Naser Tamim SEHA, Dr Ghada Al Aqqad D.D.S, Dr Kamal Naser - Amrita Medical Centre, Dr Emad Wani - Shadi Dental Centre,
Dr Lina Anka SEHA, Dr Rouba Zgheibi SEHA, Maria Teresa Yongson Alejandrino SEHA 94
Which antibiotic is primarily
bactericidal? A. subgingival finish line.
B. supragingival finish line.
A. Penicillin. * C. undercontoured crown.
B. Erythromycin. D. overcontoured crown. *
C. Tetracycline.
D. Chloramphenicol. A rubber dam in which there is
E. Clindamycin. leakage interproximally most likely
has
Administration of succinylcholine to a
patient deficient in serum cholinesterase A. holes punched too far apart.
would most likely result in B. holes punched too close
together. *
A. convulsions. C. too broad a rubber dam arch
B. hypertension. form.
C. prolonged apnea. * D. too much tension on the rubber
D. acute asthmatic attack. dam holder.

The pulpal floor of an occlusal amalgam


preparation on a mandibular first
premolar should

A. be 2mm into the dentin.


B. slope apically from mesial to distal.
C. be parallel to the buccolingual cusp
plane.*
D. be perpendicular to the long axis of
the tooth.
The above diagram demonstrates a
lateral view of a tracing illustrating
The most appropriate treatment for an
the border movements of a
endodontically treated molar with an
mandibular incisor point in an
existing MOD restoration is a
Angle’s Class I occlusion. The
opening pathway from maximum
A. cast gold inlay. intercuspation follows
B. bonded composite resin.
C. bonded silver amalgam. A. pathway A.
D. cast restoration with cuspal B. pathway A and B.
coverage. * C. pathway C.
D. pathway H. *
After the cementation of a crown,
E. none of these pathways.
chronic gingivitis would most likely be
the result of a/an

Dr Abdul Naser Tamim SEHA, Dr Ghada Al Aqqad D.D.S, Dr Kamal Naser - Amrita Medical Centre, Dr Emad Wani - Shadi Dental Centre,
Dr Lina Anka SEHA, Dr Rouba Zgheibi SEHA, Maria Teresa Yongson Alejandrino SEHA 95
For complete dentures, the most surfaces of the maxillary posterior
important factor affecting speech is the teeth. The most appropriate restorative
material for these lesions is
A. reproduction of the palatal rugae.
B. position of the molar teeth. A. microfilled composite resin.
C. position of the anterior teeth. * B. hybrid composite resin.
D. palatal thickness. C. amalgam.
D. Resin-modified glass ionomer. *
Interproximal reduction of the mesial of
the lower primary cuspids is indicated Chlorhexidine is an effective antiplaque
to agent due to its ability to

A. encourage mesial movement of the A. bind to the positively charged


deciduous molars. pellicle.
B. provide space for alignment of the B. damage the microbial cell
permanent incisors when crowding is membrane. *
1-3mm. * C. neutralize the bacterial end-products
C. provide space for alignment of the such as lactic acid.
permanent incisors when crowding is D. non-selectively reduce the oral
3-5mm. microbial flora.
D. decrease inter-cuspid arch width.
During the extraction of an impacted
A healthy 66 year old patient who had tooth 3.8 the lingual nerve is damaged.
a myocardial infarct eight years ago All of the following can occur EXCEPT
requires an extraction. The most a
appropriate management is to
A. loss of taste from the anterior 2/3 on
A. admit the patient to hospital for the left side of the tongue.
extraction with local anesthesia. B. deviation of the tongue to the left on
B. admit the patient to hospital for protrusion. *
extraction with general anesthesia. C. decreased salivary output from the
C. extract the tooth in the office using left sublingual gland.
preoperative sedation and local D. decreased salivary output from the
anesthetic without a vasoconstrictor. left submandibular gland.
D. treat the patient in the office as a E. numbness of the floor of the mouth
normal patient. * on the left side.

Biocompatibility tests conducted in A primary infection of syphilis occurring


vitro on the tongue is referred to as a/an

A. follow screening with an animal A. herpetic ulcer.


model. B. aphthous ulcer.
B. predict human cell growth or death. C. gumma.
C. reproduce the in vivo environment. D. chancre. *
D. are more easily standardized than E. mucous patch.
clinical studies. *

A 78 year old patient presents with


several carious lesions on the root

Dr Abdul Naser Tamim SEHA, Dr Ghada Al Aqqad D.D.S, Dr Kamal Naser - Amrita Medical Centre, Dr Emad Wani - Shadi Dental Centre,
Dr Lina Anka SEHA, Dr Rouba Zgheibi SEHA, Maria Teresa Yongson Alejandrino SEHA 96
One week following extraction of teeth D. constrictive pericarditis.
1.8 and 4.8, an 18 year old male
returns to the dental office complaining Osteomalacia is seen in
of persistent bleeding from the
extraction sites. The medical history is A. Vitamin C deficiency.
unremarkable, except for episodes of B. adult rickets. *
bruising and joint swelling as a child. C. protein deficiency.
Subsequent blood tests show normal D. hyperparathyroidism.
bleeding time and a factor VIII level of
14%. The most likely cause of the The physiopathology of sleep apnea is
bleeding is most likely related to

A. a dry socket. A. excessive oropharyngeal muscular


B. ibuprofen intake. tonus during sleep.
C. hemophilia A. * B. central nervous system respiratory
D. cirrhosis of the liver. depression. *
E. vitamin K deficiency. C. obstruction in lower airways.
D. a severe Angle Class III
In healthy gingiva, the level of the malocclusion.
interproximal alveolar crest is related to
the In a xerostomic patient, which salivary
glands are most likely responsible for
A. thickness of the alveolar process. the lack of lubrication?
B. location of the gingival margin.
C. amount of underlying trabecular A. Accessory.
bone. B. Labial.
D. position of the cemento-enamel C. Parotid.
junction. * D. Sublingual and submandibular. *

Which of the following tests is/are In periodontal surgery, it is important to


specific in the diagnosis of anemia? determine the position of the base of
the pocket to the
A. A complete blood count.
B. Measuring the concentration of A. cemento-enamel junction.
hemoglobin. B. tip of the inter-dental papilla.
C. White cell count and hematocrit C. furcation.
determination. D. mucogingival junction. *
D. Hematocrit determination,
hemoglobin concentration and red cell The primary use of nitrous oxide and
count. * oxygen is as a(n)

Patients with a history of ankle A. substitute agent for local


swelling, shortness of breath and anesthesia.
orthopnea are most likely suffering B. general anesthetic agent.
from C. agent for conscious sedation. *
A. asthma.
B. emphysema.
C. congestive heart failure. *

Dr Abdul Naser Tamim SEHA, Dr Ghada Al Aqqad D.D.S, Dr Kamal Naser - Amrita Medical Centre, Dr Emad Wani - Shadi Dental Centre,
Dr Lina Anka SEHA, Dr Rouba Zgheibi SEHA, Maria Teresa Yongson Alejandrino SEHA 97
Anginal pain is
When the gingival margin of a Class II
A. provoked by exercise or cold composite resin preparation is located
weather. * less than 1mm occlusal to the
B. not relieved by rest. cemento-enamel junction, the risk of
C. relieved by digoxin. marginal leakage is reduced by
D. aggravated by deep inspiration.
A. incremental curing.
One millilitre of a 2 solution of B. placing a glass ionomer liner. *
lidocaine hydrochloride contains how C. placing an initial increment of
many milligrams of the drug? flowable resin.
D. beveling the gingival margin.
A. 2
B. 20 * Which of the following will increase the
C. 200 flexibility of a removable partial
D. 2000 denture clasp arm?

A patient complains of pain in a A. Increasing the length. *


mandibular molar when chewing hard B. Increasing the diameter.
foods and drinking cold liquids. C. Decreasing the taper.
Electric pulp tests and radiographic D. Decreasing the retentive undercut.
appearance are normal. The pain is
most likely caused by Osteoporosis is linked to

A. acute periradicular periodontitis. A. low estrogen levels. *


B. chronic periradicular periodontitis. B. high testosterone levels.
C. a cracked tooth. * C. low androgen levels.
D. chronic pulpitis. D. high progesterones levels.

The etiology of noncarious cervical Which drug is indicated for the


lesions is management of an acute asthmatic
attack
A. multifactorial in nature. *
B. determined by the shape of the A. Fluticasone.
lesion. B. Salbutamol. *
C. determined by an occlusal C. Triamcinolone.
analysis. D. Budesonide.
D. correlated to the dominant hand
used for brushing.

Dr Abdul Naser Tamim SEHA, Dr Ghada Al Aqqad D.D.S, Dr Kamal Naser - Amrita Medical Centre, Dr Emad Wani - Shadi Dental Centre,
Dr Lina Anka SEHA, Dr Rouba Zgheibi SEHA, Maria Teresa Yongson Alejandrino SEHA 98
Moistened dentin is prefrerred over dry D. orthodontic treament.
dentin prior to the application of dentin E. scaling and root planning. *
bonding agents because
A. curing time is reduced. What is the most likely diagnosis of a
B. the collagen matrix is maintained. white lesion on the retromolar pad
C. the smear layer is removed by the opposing a non-functional molar?
application of water. *
D. enamel bonding is improved. A. Alveolar ridge keratosis. *
B. Candidiasis.
In complete denture contruction, the C. Lichen planus.
physiologic rest position D. Squamous cell carcinoma.

A. provides a quide to establish the Which of the following laws is the


vertical dimension of occlusion. * LEAST relevant to the dental patient
B. determines the level of the occlusal record as a legal document?
plane.
C. provides a guide for the selection of A. Provincial.
cusp inclination. B. Criminal.
D. determines the shape of the C. Consumer. *
compensating curve. D. Common.
E. is useful to the determination of
condylar inclination. Which of the following medications
can be taken by a patient who has
Which of the following statements is been prescribed lithium?
true?
A. Acetaminophen.*
A. A major percentage of B. Flurbiprofen.
radiographically detected proximal C. Ibuprofen.
radiolucensies in enamel are cavitated. D. Naproxen.
B. Caries progression through enamel
can take as long as 6 to 8 years. * Patients who have been prescribed
C. Caries progression can only be an acetaminophen/codeine
arrested in enamel. combination must be advised
D. Arrested carious lesions in enamel
are softer than intact enamel. A. to complete the prescribed
medication.
The disease-control phase of treatment B. that it can cause diarrhea.
includes C. that it can cause hypertension.
D. to avoid consuming alcohol. *
A. pit and fissure sealants.
B. fixed prosthodontics.
C. occlusal therapy.

Dr Abdul Naser Tamim SEHA, Dr Ghada Al Aqqad D.D.S, Dr Kamal Naser - Amrita Medical Centre, Dr Emad Wani - Shadi Dental Centre,
Dr Lina Anka SEHA, Dr Rouba Zgheibi SEHA, Maria Teresa Yongson Alejandrino SEHA 99
The following violations provide A. Beneficence.
grounds for a criminal action in battery B. Justice.
EXCEPT C. Non maleficence.
D. Autonomy.
A. intentional touching. E. Scholarship. *
B. defamation causing harm. *
C. treatment without consent. Which of the following is NOT a
D. harmful contact. recognized ethics-based principle?

Which type of legal action can result in A. Citizenship. *


the payment of damage? B. Justice.
C. Non maleficence.
A. Causation. D. Autonomy.
B. Discipline. E. Veracity.
C. Intention.
D. Tort. * A 32 year old patient presents with
several shallow, round ulcers, 3-5mm
The legally set age for authorization of in diameter, on the left and right
health care treatment buccal mucosa. The most likely
diagnosis is
A. is 16 years of age.
B. indicates ‘mature minor’ status. A. recurrent herpes.
B. traumatic ulcers.
C. presumes capacity. *
C. pemphigus vulgaris.
D. applies universally. D. aphthous ulcers. *
To advance a patient’s welfare, Which of the following has been
dentists should refer to a specialist implicated in the development of oral
considering all of the following pigmentation?
EXCEPT
A. Atorvastatin.
A. experience.
B. Chloroquine. *
B. fees. *
C. Fluoxetine.
C. knowledge.
D. Omeprazole.
D. skills.
E. Valsartan.
Which of the following is NOT a
recognized ethics-based principle?

Dr Abdul Naser Tamim SEHA, Dr Ghada Al Aqqad D.D.S, Dr Kamal Naser - Amrita Medical Centre, Dr Emad Wani - Shadi Dental Centre,
Dr Lina Anka SEHA, Dr Rouba Zgheibi SEHA, Maria Teresa Yongson Alejandrino SEHA 100
Which of the following medications is Which of the following is most likely to
most appropriate in the management have its effect reduced in a patient with
of a patient experiencing an acute genetic polymorphism of CYP2D6?
asthmatic attack?
A. Acetaminophen.
A. Salbutamol. * B. Amoxicillin.
B. Sodium cromoglycate. C. Clindamycin.
C. Fluticasone. D. Codeine. *
D. Zileuton. E. Ibuprofen.

Administration of which of the Which of the following is associated


following is most likely to predispose a with drug-induced bruxism?
patient to seizures?
A. Amitriptyline.
A. Acetaminophen. B. Clonazepam.
B. Codeine. C. Fluoxetine. *
C. Ibuprofen. D. Phenelzine.
D. Ketorolac.
E. Meperidine. * The Controlled Drugs and Substances
Act controls the distribution and use of
The most frequent cause of which of the following?
hyposalivation in elderly patients is
A. Acetaminophen.
A. Alzheimer’s disease. B. Diazepam. *
B. multiple medications. * C. Ibuprofen.
C. salivary gland atrophy. D. Penicillin V.
D. Sjögren’s syndrome.
The Controlled Drugs and Substances
Metabolism of ibuprofen is NOT Act controls the distribution and use of
affected by which of the following?

A. decreased cardiac output. * A. Amitriptyline.


B. plasma protein binding. B. Amoxicillin.
C. gastric emptying rate. C. Ibuprofen.
D. hepatic enzyme activity. D. Oxycodone. *

Dr Abdul Naser Tamim SEHA, Dr Ghada Al Aqqad D.D.S, Dr Kamal Naser - Amrita Medical Centre, Dr Emad Wani - Shadi Dental Centre,
Dr Lina Anka SEHA, Dr Rouba Zgheibi SEHA, Maria Teresa Yongson Alejandrino SEHA 101
Renal excretion of drug metabolites is
influenced by all of the following A. Alprazolam.
EXCEPT B. Diazepam. *
C. Lorazepam.
A. plasma protein binding. D. Midazolam.
B. tissue redistribution. * E. Triazolam.
C. volume of distribution.
D. glomerular filtration. In order to prevent gingival recession,
a full gold crown should have
Which of the following drugs is most
likely to induce photosensitivity? A. a slightly narrow food table.
B. a slightly overcontoured tooth form.
A. Tetracycline. * C. normal contour reproduced. *
B. Metronidazole. D. the margins extended 1mm into the
C. Acetaminophen. gingival crevice.
D. Oxycodone.
In a general dental practice setting,
Which of the following is the most patients who are actively infectious
potent corticosteroid? with Mycobacterium tuberculosis
CANNOT be safely treated
A. Cortisone.
B. Dexamethasone. * A. unless there is adherence to
C. Hydrocortisone. standard precautions.
D. Prednisone. B. even if standard precautions are
E. Triamcinolone. used. *
C. unless a preoperative antimicrobial
In a patient with renal failure, which of rinse is used in addition to standard
the following does/do need a dose precautions.
interval adjustment? D. unless preoperative prophylactic
antibiotic coverage and standard
1. Penicillin V. precautions are used.
2. Metronidazole.
3. Erythromycin. Latex gloves should
4. Clindamycin.
A. be washed with plain soap before
A. (1) (2) (3) * initial use.
B. (1) and (3) B. be washed with plain soap when
C. (2) and (4) used between patients.
D. (4) only C. be washed with a disinfectant
E. All of the above. solution only.
D. not be washed. *
Which of the following has clinically
active metabolites?

Dr Abdul Naser Tamim SEHA, Dr Ghada Al Aqqad D.D.S, Dr Kamal Naser - Amrita Medical Centre, Dr Emad Wani - Shadi Dental Centre,
Dr Lina Anka SEHA, Dr Rouba Zgheibi SEHA, Maria Teresa Yongson Alejandrino SEHA 102
When used as a direct pulp capping material, D. hemidesmosomes. *
which of the following produces the best pulp
healing? The most significant modifiable risk factor for
periodontitis is
A. Zinc-oxide eugenol.
B. Glass-ionomer. A. cigarette smoking.
C. Calcium hydroxide. * B. bacterial plaque. *
D. Resin-modified glass ionomer. C. psychological stress.
E. Adhesive resin. D. dental calculus.

The facial growth spurt A 22 year old patient, who is otherwise


healthy, has severe pain and discomfort in the
A. occurs in males before females. mouth. Clinical findings are gingival ulceration
B. starts on average at 7 years for females. in the anterior region of both arches, gingiva
C. starts on average at 13 years for males. covered by a yellow-grey slough, ulcerated
D. parallels body growth. * papillae, gingival bleeding upon slight
provocation, a fetid odor but no radiographic
Regeneration of periodontal tissue is the evidence of bone loss. What is the most
replacement of lost tissue with one that is probable diagnosis?
similar in
A. Localized aggressive periodontitis.
A. function. B. Localized aggressive gingivitis.
B. structure. C. Necrotizing ulcerative periodontitis.
C. structure and function. * D. Necrotizing ulcerative gingivitis. *

Repair of periodontal tissues is the Mesiolingual rotation of permanent first


replacement of lost tissue with one that is maxillary molars
similar in
A. increases arch length.
A. function. * B. results is an Angle *
B. structure. C. causes impaction of the second permanent
C. structure and function. molars.

Calculus attaches to tooth surfaces by all of Which of the following resorbable sutures has
the following EXCEPT the smallest thread size?

A. organic pellicle. A. 3.0 polyglycolic acid.


B. mechanical locking to tooth/root B. 4.0 silk.
irregularities. C. 5.0 chromic gut. *
C. close adaption to cementum. D. 6.0 nylon.

Dr Abdul Naser Tamim SEHA, Dr Ghada Al Aqqad D.D.S, Dr Kamal Naser - Amrita Medical Centre, Dr Emad Wani - Shadi Dental Centre,
Dr Lina Anka SEHA, Dr Rouba Zgheibi SEHA, Maria Teresa Yongson Alejandrino SEHA 103
Which of the following teeth are most
often lost as a result of periodontitis? A. mandibular canines.
B. maxillary premolars.
A. Mandibular molars. C. mandibular premolars.
B. Maxillary molars. * D. maxillary lateral incisors. *
C. Mandibular incisors.
D. Maxillary incisors. Following periodontal surgery, the
curetted root surface is repopulated
In addition to plaque index and probing by cells derived from all of the
depths, which of the following must be following tissues EXCEPT
re-examined following the completion
of initial periodontal therapy? A. periodontal ligament. *
B. cementum.
A. Bleeding on probing. * C. alveolar bone.
B. Bone level. D. epithelium.
C. Root sensitivity. E. gingival connective tissue.

Initial therapy for periodontal pockets When will infiltrating new blood
that are deeper than 5mm must include vessels be histologically detectable
supragingival scaling and following a free gingival graft?

A. polishing. A. 2 to 3 hours.
B. irrigation. B. 2 to 3 days. *
C. root planning. * C. 2 to 3 weeks.
D. antibiotics. D. 2 to 3 months.

The blood vessel which may be injured Histologically, periodontitis is


when harvesting a connective tissue distinguished from gingivitis by
graft from the palatal mucosa is the
A. increased collagen destruction.
A. sphenopalatine. B. increased gingival inflammation.
B. nasopalatine. C. apical migration of dentogingival
C. posterior superior alveolar. epithelium. *
D. greater palatine. *

The labial/buccal attached gingiva on


permanent teeth is normally widest at
the

Dr Abdul Naser Tamim SEHA, Dr Ghada Al Aqqad D.D.S, Dr Kamal Naser - Amrita Medical Centre, Dr Emad Wani - Shadi Dental Centre,
Dr Lina Anka SEHA, Dr Rouba Zgheibi SEHA, Maria Teresa Yongson Alejandrino SEHA 104
Periodontitis:
Which of the following instruments is
A. affects individuals with various CONTRAINDICATED when extracting
susceptibility at different rates. * primary teeth?
B. affects different parts of dentition in
the same individual similarly. A. Rongeur.
C. is generally not progressive in B. Root tip elevator.
nature. C. Cow horn forceps. *
D. Maxillary universal forceps.
When placing an implant in the
posterior mandible, the minimum Over the lifetime of a patient, the
thickness of bone between the implant relative position of the midface in
and the inferior alveolar nerve should comparison to the mandible is
be
A. anterior.
A. 2mm. * B. constant.
B. 4mm. C. posterior. *
C. 6mm.
D. 8mm. A 6 year old patient has an intrusive
injury to tooth 5.2. All of the following
Chlorhexidine is a are possible sequelae to the
permanent successor EXCEPT
A. quaternary ammonium compound.
B. bisbiguanide compound. * A. enamel hypoplasia. *
C. phenol. B. root dilaceration.
D. plant extract. C. delayed eruption.
D. ectopic eruption.
Which of the following is the most
predictable indicator of periodontal The most appropriate suture
stability? combination for a through laceration of
the lower lip would be
A. Patient compliance with
maintenance visit. A. skin: 5.0 nylon, muscle: 3.0 vicryl,
B. Gingival inflammation. mucosa: 3.0 gut. *
C. Plaque index. B. skin: 3.0 silk, muscle: 3.0 silk,
D. Absence of bleeding on probing. * mucosa: 3.0 silk.
C. skin: 3.0 gut, muscle: 3.0 vicryl,
The removal of a partially impacted mucosa: 3.0 silk.
mandibular third molar is appropriate D. skin: 5.0 nylon, muscle: 3.0 silk,
for the prevention of mucosa: 3.0 gut.
E. skin: 3.0 gut, muscle: 3.0 vicryl,
A. anterior crowding. mucosa: 3.0 gut.
B. root caries of the adjacent tooth. *
C. jaw fracture from contact sports.

Dr Abdul Naser Tamim SEHA, Dr Ghada Al Aqqad D.D.S, Dr Kamal Naser - Amrita Medical Centre, Dr Emad Wani - Shadi Dental Centre,
Dr Lina Anka SEHA, Dr Rouba Zgheibi SEHA, Maria Teresa Yongson Alejandrino SEHA 105
Gingival enlargement associated with B. apply firm pressure for ten minutes*
use of cyclosporine is primarily C. obtain an INR and complete blood
characterized by an increase of count.
D. apply dry socket dressing.
A. plasma cells.
B. edema. In severe gingival recession, if the
C. collagen. * marginal tissue extends to the
D. keratinocytes. mucogingival junction, and there is
loss of interdental tissue, the likelihood
Gingival enlargement associated with of complete root coverage after
use of phenytoin is primarily gingival grafting is
characterized by an increase of
A. greater than 75%.
A. plasma cells. B. between 25 and 50%.
B. edema. C. less than 20%.*
C. collagen. *
D. keratinocytes. What is the primary source of elevated
collagenolytic activity in chronic
The minimum time to re-evaluate periodontitis?
tissue response after initial therapy for
a patient with generalized moderate A. P. gingivalis.
chronic periodontitis is B. Macrophages.
C. Neutrophils. *
A. 1-10 days. D. P. intermedia.
B. 11-20 days.
C. 21-30 days. Which of the following is appropriate to
provide postoperative pain control for
D. 31-40 days. *
an 80kg, 20 year old patient with an
allergy to codeine following the
A gingival pocket (pseudopocket) is
removal of 4 erupted third molars?
characterized by
A. Hydromorphone 2 mg, every 4-6
A. loss of attachment.
hours prn.
B. loss of alveolar bone.
B. Ibuprofen 800 mg, 1 hour
C. deepened probing depth. *
preoperatively, followed by 400 mg,
every 4-6 hours prn. *
Following a simple extraction of tooth
C. Acetaminophen 650 mg, with
4.7, hemostasis was achieved. Four
oxycodone 10 mg, every 4-6 hours
hours later, there is renewed bleeding
prn.
from the extraction site. The most
appropriate initial management is to

A. administer local anesthetic with


epinephrine and suture.

Dr Abdul Naser Tamim SEHA, Dr Ghada Al Aqqad D.D.S, Dr Kamal Naser - Amrita Medical Centre, Dr Emad Wani - Shadi Dental Centre,
Dr Lina Anka SEHA, Dr Rouba Zgheibi SEHA, Maria Teresa Yongson Alejandrino SEHA 106
Which of the following should NOT be A patient presents with a chief
prescribed for a patient receiving complaint of “severe pain in my right
warfarin? ear” which began when eating, three
hours ago. An examination reveals
A. Acetylsalicylic acid. * tenderness over the right preauricular
B. Oxycodone. region, maximum interincisal opening
C. Acetaminophen. of 21mm with deflection to the right,
D. Codeine. right lateral excursion of 9mm and left
lateral excursion of 2mm. The most
Which of the following should NOT be likely diagnosis is
prescribed for a patient receiving
warfarin? A. left anterior disc displacement with
reduction.
A. Acetaminophen. B. right anterior disc displacement with
B. Oxycodone. reduction.
C. Ketorolac. * C. left anterior disc displacement
D. Codeine. without reduction.
D. right anterior disc displacement
A 70 year old insulin-dependent patient without reduction. *
has just completed a 7-day course of
ampicillin for a respiratory infection. He After root planing and plaque control
has signs and symptoms consistent instruction, a 22 year old patient still
with oral candidiasis. Which of the shows swollen and edematous gingiva
following drugs is the most appropriate with 3mm pockets and a 4 to 6mm
treatment? band of attached gingiva. The most
appropriate management is
A. Nystatin. *
B. Acyclovir. A. an apically repositioned split
C. Clindamycin. thickness flap.
D. Ampicillin. B. an apically repositioned full
thickness flap.
E. Metronidazole.
C. gingivectomy.
A 43 year old patient with a history of D. further plaque control instruction. *
severe asthma and nasal polyps has E. occlusal adjustment.
an emergency pulpotomy. Assuming
daily maximums are not exceeded, Which of the following is used to verify
which of the following is the most the effectiveness of instrument
appropriate drug management? sterilization?

A. Acetaminophen 100 mg every 4 A. Gram-negative bacteria.


hours. * B. Anerobic bacteria.
B. Naproxen 250 mg every 6-8 hours. C. Spores. *
C. Ketorolac 10 mg every 4 hours. D. Oral microorganisms.
D. Acetylsalicylic acid 650 mg every 4
hours.

Dr Abdul Naser Tamim SEHA, Dr Ghada Al Aqqad D.D.S, Dr Kamal Naser - Amrita Medical Centre, Dr Emad Wani - Shadi Dental Centre,
Dr Lina Anka SEHA, Dr Rouba Zgheibi SEHA, Maria Teresa Yongson Alejandrino SEHA 107
The use of benzodiazepine in the Which of the following teeth is the
elderly to manage anxiety LEAST desirable to use as an abutment
tooth for a fixed partial denture?
A. should be done with caution as it is
metabolized more slowly. * A. Tooth with pulpal involvement.
B. is a safe and reliable practice. B. Tooth with minimal coronal structure.
C. increases the incidence of insomnia. C. Tooth rotated and tipped out of line.
D. requires higher doses for clinical D. Tooth with short, tapered root and a
effects. long clinical crown. *

A 67 year old patient with xerostomia The best method to control the setting
presents with root caries. The most time of an irreversible hydrocolloid
appropriate material to restore these without affecting its physical properties
lesions is is to alter the

A. flowable composite resin. A. water temperature. *


B. hybrid composite resin. B. water:powder ratio.
C. silver amalgam. C. mixing time.
D. glass ionomer cement. * D. composition.

The extraction of a primary maxillary In recording centric relation registration,


central incisor at the age of 6 years will perforation of the recording material
cause must be avoided because

A. loss of intercanine space. A. the material will undergo dimensional


B. increased intercanine space. changes.
C. no change in intercanine space. * B. contact of teeth could deflect the
D. decreased overjet. mandible. *
C. the recording material will be too
Ethically, a dentist can weak and may fracture.

A. compensate a referring dentist. A protrusive relation record should be


B. collect the third party payment made by instructing the patient to
without requesting the patient's portion. protrude the mandible
C. bill an insurance company a higher
than usual fee, for the patient’s benefit. A. 3-6mm. *
D. charge different fees when B. 7-10mm.
warranted by clinical conditions. * C. 11-13mm.

Dr Abdul Naser Tamim SEHA, Dr Ghada Al Aqqad D.D.S, Dr Kamal Naser - Amrita Medical Centre, Dr Emad Wani - Shadi Dental Centre,
Dr Lina Anka SEHA, Dr Rouba Zgheibi SEHA, Maria Teresa Yongson Alejandrino SEHA 108
Assuming a maxillary cast is A. external resorption.
accurately mounted on an articulator, B. pulpal hemorrhage. *
a centric relation record is used to C. discoloured composite restoration.
D. chromogenic bacteria.
A. determine the vertical dimension of
occlusion. When removing bone or sectioning
B. mount the mandibular cast. * roots of teeth with a high-speed
C. establish the occlusal plane. handpiece, the air/water combination
D. record the inclination of the should be set with
condylar guidance.
A. air and water on.
On a semi-adjustable articulator, the B. water only.
incisal guidance is the mechanical C. air only.
analogue of D. neither air nor water. *

A. horizontal guidance. The outline form for a Class I amalgam


B. the curve of Monson. preparation is determined by all of the
C. the curve of Spee. following EXCEPT the
D. horizontal and vertical overlap. *
A. extent to which the enamel has
When compared with that of its been involved by the carious process.
permanent successor, the occlusal B. lateral spread of caries along the
surface of the primary mandibular dentino-enamel junction.
second molar is C. extension that must be made along
the fissures in order to achieve sound
A. larger mesiodistally. * and smooth margins.
B. equal mesiodistally. D. need to terminate the margins on a
C. narrower mesiodistally. cusp ridge or marginal ridge crest. *
D. equal buccolingually and E. extent of undermining of the enamel
mesiodistally. by the carious process.

What is the most serious complication During preparation of a subgingival


of an acute periradicular abscess in Class V on tooth 3.6, a patient
the maxilla? suddenly develops a swelling of the left
face and neck with crepitation. The
A. Pericementitis. most probable diagnosis is a/an
B. Periostitis.
C. Cavernous sinus thrombosis. * A. anaphylactoid reaction.
D. Spontaneous drainage of pus. B. hematoma.
C. cervicofacial emphysema. *
Following trauma, bluish-grey D. type 1 allergic reaction.
discolouration of the crown of an
anterior tooth is due to

Dr Abdul Naser Tamim SEHA, Dr Ghada Al Aqqad D.D.S, Dr Kamal Naser - Amrita Medical Centre, Dr Emad Wani - Shadi Dental Centre,
Dr Lina Anka SEHA, Dr Rouba Zgheibi SEHA, Maria Teresa Yongson Alejandrino SEHA 109
A 3 year old, 16kg child is given 2 The management of syncope
cartridges of 4% prilocaine with following local anesthetic
1:200,000 epinephrine for extraction of administration does NOT include
primary teeth. After a few minutes the
child becomes lethargic, disorientated A. elevating the legs.
and eventually begins to convulse. The B. placing in a supine position.
most probable cause of this reaction is C. administering oxygen.
D. ensuring the airway is open.
A. epinephrine. E. administering epinephrine. *
B. allergic reaction to the local
anesthetic. The tissue which is most sensitive to
C. overdose of local anesthetic. * radiation is
D. epilepsy precipitated by epinephrine.
A. nerve.
What is the goal of dentin conditioning? B. dental pulp.
C. lymphoid. *
A. Remove the smear layer. * D. muscle.
B. Reduce the dentinal fluids in the
tubules. In a peripheral nerve such as the
C. Expose the hydroxyapatite. trigeminal, unmyelinated C type fibres
D. Eliminate the collagen fibrils. convey

To obtain block anesthesia of the A. muscle spindle information.


second division of the trigeminal nerve, B. reflexes such as the jaw jerk reflex.
the solution must be deposited in C. nociceptive pain information. *
proximity to the D. reflexes such as the jaw opening
reflex.
A. foramen ovale.
B. pterygoid plexus. A 17 year old male patient exhibits
C. foramen spinosum. delayed eruption of permanent teeth.
D. infraorbital foramen. Radiographs indicate multiple,
E. pterygopalatine fossa. * impacted permanent teeth. The
patient’s head size is large with
What is the most appropriate location prominent frontal eminences and
for the placement of a retentive pin? slightly constricted facial features.
The most likely diagnosis is
A. At the dentino-enamel junction.
B. In dentin a minimum 0.5mm from the A. osteopetrosis.
dentino-enamel junction. * B. osteitis deformans.
C. In enamel a minimum 0.5mm from C. Gardner's syndrome.
the dentino-enamel junction. D. acromegaly.
D. At least 2.5mm from the cavosurface E. cleidocranial dysplasia. *
margin.

Dr Abdul Naser Tamim SEHA, Dr Ghada Al Aqqad D.D.S, Dr Kamal Naser - Amrita Medical Centre, Dr Emad Wani - Shadi Dental Centre,
Dr Lina Anka SEHA, Dr Rouba Zgheibi SEHA, Maria Teresa Yongson Alejandrino SEHA 110
When tumour cells revert to a more Healthy attached gingiva
primitive, embryonic or undifferentiated
form with an increased capacity for A. has no basal cell layer.
reproduction and a decreased function, B. is closely bound to underlying
this is called muscle.
C. contains elastic fibres.
A. anaplasia. * D. is keratinized. *
B. metaplasia.
C. hypoplasia. A 13 year old complains of red,
D. hyperplasia. bleeding and swollen gums. Clinical
examination reveals this is present
Which periodontal pathogen can use only on the labial gingiva of the
the hormone estrogen as a growth maxillary anterior teeth. What is the
factor? most likely etiologic factor?

A. Porphyromonas gingivalis. A. Blood dyscrasia.


B. Aggregatibacter (Actinobacillus) B. Insulin-dependent diabetes mellitus.
actinomycetemcomitans. C. Mouth breathing habit. *
C. Prevotella intermedia. * D. Pubertal hormones.
D. Tannerella forsythia.
The primary factor for selecting
Predominant organisms in necrotizing periodontal flap surgery rather than
ulcerative gingivitis are gingivectomy is

A. spirochetes and fusiforms. * A. presence of gingival edema.


B. Porphyromonas gingivalis. B. pocket depth.
C. Prevotella intermedia. C. presence of subgingival calculus.
D. Actinomyces viscosus. D. need for access to the bony defect.*

Tooth development begins when the Sucralose


basal layer of cells proliferates to form
a ridge called the A. breaks down at high temperatures
and cannot be used in baking.
A. dental lamina. * B. is a derivative of sucrose. *
B. dental papilla. C. is twice as sweet as table sugar.
C. odontoblastic matrix. D. causes constipation when ingested
in excessive amounts.
The most common type of collagen
found in the gingival connective tissue, A patient who is 4 months pregnant
periodontal ligament, and cementum is requires an extraction. A radiograph
composed primarily of may
A. be taken only if difficulty is
A. type I. * encountered during surgery.
B. type II. B. be taken. *
C. type III. C. not be taken.
D. type IV. D. be taken by panoramic radiography
only.

Dr Abdul Naser Tamim SEHA, Dr Ghada Al Aqqad D.D.S, Dr Kamal Naser - Amrita Medical Centre, Dr Emad Wani - Shadi Dental Centre,
Dr Lina Anka SEHA, Dr Rouba Zgheibi SEHA, Maria Teresa Yongson Alejandrino SEHA 111
Which of the following tissues is the In which salivary gland is a
LEAST sensitive to the effects of x- pleomorphic adenoma most frequently
radiation? found?

A. Hematopoietic. A. Parotid. *
B. Gonadal. B. Submandibular.
C. Bone. * C. Sublingual.
D. Glandular.
Blue sclera is characteristic of
Decreased size or obliteration of pulp
chambers and canals is diagnostic of A. osteopetrosis.
B. osteogenesis imperfecta. *
A. Hand-Schuller-Christian disease. C. osteitis deformans.
B. cleidocranial dysplasia. D. fibrous dysplasia.
C. amelogenesis imperfecta.
D. osteogenesis imperfecta. * A positive Nikolsky's sign is a
diagnostic feature of
In radiology, if target-skin distance is
doubled, the exposure time must be A. lichen planus.
B. erythema multiforme.
A. doubled. C. pemphigus vulgaris. *
B. tripled. D. chronic marginal gingivitis.
C. quadrupled. * E. lupus erythematosus.
D. increased 10 times.
What is the most frequent malignant
The most common cause of bilateral tumour of the tongue?
swelling of the parotid glands in
children is A. Adenoma.
B. Adenocarcinoma.
A. Mikulicz's disease. C. Rhabdomyosarcoma.
B. mumps or acute infectious parotitis* D. Squamous cell carcinoma. *
C. mixed salivary tumours. E. Granular cell myoblastoma.
D. sialolithiasis.
Calculus causes gingival inflammation
Condensing osteitis differs from by
idiopathic osteosclerosis because it is
A. mechanical irritation.
A. expansile. B. stimulating auto immune
B. painful. responses.
C. associated with fever. C. retaining micro-organisms. *
D. associated with pulpal inflammation* D. releasing toxins.

Dr Abdul Naser Tamim SEHA, Dr Ghada Al Aqqad D.D.S, Dr Kamal Naser - Amrita Medical Centre, Dr Emad Wani - Shadi Dental Centre,
Dr Lina Anka SEHA, Dr Rouba Zgheibi SEHA, Maria Teresa Yongson Alejandrino SEHA 112
An early radiographic sign of chronic Which antibiotic is CONTRAINDICATED
periodontitis is for a patient with a history of a Type 1
anaphylactic reaction to penicillin?
A. widening of vascular canals.
B. "notching" of the crestal lamina dura* A. Azithromycin.
C. enlargement of the interdental B. Cephalexin. *
medullary spaces. C. Clindamycin.
D. decreased radiolucency of the D. Erythromycin.
interdental trabeculae.
Which of the following penicillins is most
Space closure is LEAST likely to occur effective against Gram-negative
following the premature loss of the organisms?
primary
A. Nafcillin.
A. maxillary second molar. B. Ampicillin. *
B. mandibular second molar. C. Methicillin.
C. maxillary first molar. D. Penicillin V.
D. mandibular canine. E. Phenethicillin.
E. maxillary central incisor. *
A patient who is jaundiced because of
A circumscribed radiolucent lesion liver disease has an increased risk of
associated with a retained root tip
should be curetted because A. postextraction bleeding. *
B. cardiac arrest.
A. pus in the lesion will infect the blood C. postoperative infection.
clot. D. anaphylactic shock.
B. the fibrous tissue of the lesion will E. pulmonary embolism.
prevent new bone formation.
C. the capillaries of the lesion will cause When given prophylactically to prevent
prolonged bleeding. infective endocarditis, oral amoxicillin
D. the lesion may undergo cystic should be taken
degeneration. *
A. immediately prior to the dental
Which of the following plasma proteins procedure.
has the greatest ability to bind drugs? B. one hour prior to the dental
procedure. *
A. Albumin. * C. four hours prior to the dental
B. Fibrinogen. procedure.
C. Hemoglobin. D. one day prior to the dental
D. Gamma globulin. procedure.
E. B-lipoprotein. E. two days prior to the dental
procedure.

Dr Abdul Naser Tamim SEHA, Dr Ghada Al Aqqad D.D.S, Dr Kamal Naser - Amrita Medical Centre, Dr Emad Wani - Shadi Dental Centre,
Dr Lina Anka SEHA, Dr Rouba Zgheibi SEHA, Maria Teresa Yongson Alejandrino SEHA 113
The most common cause of persistent The extraction of a maxillary primary
post-operative sensitivity following the central incisor at the age of 6 years will
placement of posterior composite resin cause
restorations is
A. loss of intercanine space.
A. hyperocclusion. B. increased intercanine space.
B. microleakage. * C. no change in intercanine space. *
C. acidic primers. D. increased overbite.
D. residual caries. E. decreased overjet.

To maintain pulp health and vitality, the A 10 year old patient complains of
preferred treatment for asymptomatic discomfort in a maxillary primary
teeth with very deep dentinal caries is second molar when eating. The tooth
is mobile with a large mesio-occlusal
A. direct pulp capping using a dentin amalgam restoration. The most likely
bonding agent. diagnosis is
B. to avoid pulp exposure and use
indirect pulp capping. * A. an exfoliating tooth. *
C. direct pulp capping using calcium B. a hyperemic pulp.
hydroxide. C. a hyperplastic pulp.
D. to remove all affected dentin. D. an acute pulpitis.
E. traumatic occlusion.
Polymerization shrinkage in a
composite resin is reduced by The most typical location for the
development of early childhood caries
A. placing a glass ionomer liner on all is the
exposed dentin before placing
composite resin. A. incisal edges of the mandibular
B. doubling the curing time of the resin incisors.
in preparations that are deep. B. incisal edges of the maxillary
C. using a flowable composite on the incisors.
gingival floor of Class II preparations. C. gingival area of the mandibular
D. incremental placement of no more incisors.
than 2mm thicknesses of composite D. gingival area of the maxillary
resin. * incisors. *

"Dens in dente" is most commonly Systemic or topical cortisone therapy


associated with is used in the treatment of

A. supernumerary teeth. A. necrotizing ulcerative gingivitis.


B. dentinogenesis imperfecta. B. erythema multiforme. *
C. osteogenesis imperfecta. C. submaxillary cellulitis.
D. anterior teeth. * D. ptyalism.
E. amelogenesis imperfecta. E. herpes simplex.

Dr Abdul Naser Tamim SEHA, Dr Ghada Al Aqqad D.D.S, Dr Kamal Naser - Amrita Medical Centre, Dr Emad Wani - Shadi Dental Centre,
Dr Lina Anka SEHA, Dr Rouba Zgheibi SEHA, Maria Teresa Yongson Alejandrino SEHA 114
The usual adult dosage of codeine
administered orally is A. stimulated by parasympathetic
activity. *
A. 500-1000mg. B. stimulated by sympathetic activity.
B. 250-500mg. C. inhibited by acetylcholine.
C. 30-60mg. * D. inhibited by cyclic AMP.
D. 2-5mg.
E. None of the above. Tooth 3.6 has a disto-occlusal
amalgam restoration with a gingival
Immediately following a posterior overhang. There is radiographic
superior alveolar block injection, the evidence of bone loss and deep
patient's face becomes quickly and probing depths with bleeding upon
visibly swollen. The immediate probing. Which of the following types of
treatment should be to microorganisms are most likely
associated with the subgingival
A. use pressure followed by cold packs environment in this site?
over the swelling. *
B. use hot packs over the swelling. A. Gram-positive and aerobic.
C. refer the patient to a hospital. B. Gram-positive and anaerobic.
D. administer 100mg hydrocortisone C. Gram-negative and aerobic.
intravenously. D. Gram-negative and anaerobic. *
E. administer diphenhydramine
hydrochloride (Benadryl) 50mg Which element found in radiograph
intravenously. processing solutions is of most concern
environmentally?
Pressure and tension have little effect
on growth of A. Mercury.
B. Lead.
A. the frontomaxillary suture. C. Silver. *
B. the alveolus. D. Copper.
C. the mandible.
D. cartilage. * Side effects of therapeutic doses of
codeine can include
Which of the following is consistent
with reversible pulpitis? 1. Constipation.
2. Drowsiness.
A. Discontinuous lamina dura and a 3. Nausea.
periapical radiolucency. 4. Respiratory depression.
B. Draining sinus tract that traces to
the apex of the tooth. A. (1) (2) (3
C. Pain to cold that ceases with B. (1) and (3)
removal of the stimulus. * C. (2) and (4)
D. Painful response to percussion and D. (4) only
palpation. E. All of the above. *

Saliva production is increased when


blood flow to the salivary glands is

Dr Abdul Naser Tamim SEHA, Dr Ghada Al Aqqad D.D.S, Dr Kamal Naser - Amrita Medical Centre, Dr Emad Wani - Shadi Dental Centre,
Dr Lina Anka SEHA, Dr Rouba Zgheibi SEHA, Maria Teresa Yongson Alejandrino SEHA 115
An anaphylactic reaction to penicillin is An abused woman is most at risk of
most likely to occur being murdered when she
A. Argues with the abuser in front of
1. When the drug is administered their children.
parenterally. B. Separates from her abusive
2. in patients who have already spouse. *
experienced an allergic reaction to the C. Confides in friends and asks for
drug. help.
3. within minutes after drug D. Reports the abuse to law
administration. enforcement.
4. When the drug is administered
orally. When taking radiographs on a 10 year
old child, which of the following should
A. (1) (2) (3) * be used to decrease radiation
B. (1) and (3) exposure?
C. (2) and (4)
D. (4) only A. Prescribe radiographs every 3
E. All of the above. years.
B. A decrease of the Kilovoltage to
The antibiotic of choice for infections of 50kVp.
pulpal origin is C. Use of high speed film. *
D. Application of a radiation protection
A. penicillin V. * badge.
B. metronidazole.
C. erythromycin. The most common primary tooth to
D. tetracycline. become ankylosed is a

In endodontic therapy, the most A. maxillary molar.


effective irrigating solution for B. mandibular molar. *
dissolving organic debris is C. maxillary canine.
D. mandibular canine.
A. ethylenediaminetetraacetic acid
(EDTA). A fracture through the angle of the
B. sodium hypochlorite. * mandible may result in an upward
C. calcium hydroxide. displacement of the proximal fragment.
D. hydrogen peroxide. Which of the following groups of
E. sodium chloride. muscles produce this movement?

Which of the following teeth is LEAST A. Digastric and geniohyoid.


likely to develop a furcation B. Masseter, digastric and lateral
involvement? pterygoid.
C. Masseter, temporal and lateral
A. 1.4. pterygoid.
B. 1.5. * D. Masseter, temporal and medial
C. 1.6. pterygoid. *
D. 1.7.

Dr Abdul Naser Tamim SEHA, Dr Ghada Al Aqqad D.D.S, Dr Kamal Naser - Amrita Medical Centre, Dr Emad Wani - Shadi Dental Centre,
Dr Lina Anka SEHA, Dr Rouba Zgheibi SEHA, Maria Teresa Yongson Alejandrino SEHA 116
What is the most reliable indicator with A. perpendicular to the pulpal floor.
respect to the timing of treatment for B. parallel to the contour of the final
growth modification? restoration.
C. angled 30° away from the pulp
A. Chronologic age. chamber.
B. Dental age. D. parallel to the external root contour*
C. Skeletal age. *
D. Tanner weight chart. Overcontouring in the gingival third of
a provisional restoration contributes to
In a developing crown, ameloblasts the
and odontoblasts
A. retention of the provisional
A. are polarized in opposite directions. restoration.
* B. accumulation of plaque and gingival
B. initially produce an organic inflammation. *
nonmineralized matrix. C. displacement of the gingiva for
C. begin to produce matrix future impression making.
simultaneously. D. marginal integrity of the provisional
D. have rapid cell division. restoration.

The prognosis of guided tissue Which of the following modifications to


regeneration (GTR) is best for the the standard procedure for mixing
treatment of gypsum products will increase the
compressive strength of the set
A. horizontal bone loss. material?
B. one-wall osseous defects.
C. two-wall osseous defects. A. Adding a small amount of salt to the
D. three-wall osseous defects. * water before mixing.
B. Decreasing the water/powder ratio
What is the most likely cause of the by a small amount. *
interdental papilla protruding from C. Using warmer water.
beneath the rubber dam after D. Decreasing the mixing time.
placement?
If a complete occlusal adjustment is
A. The holes were placed too far apart. necessary, interferences should be
B. A light weight dam was used. corrected
C. The holes were placed too close
together. * A. after all restorative procedures are
D. The teeth were not individually completed.
ligated. B. after each restorative procedure.
C. before starting any restorative
When preparing a posterior tooth for an treatment. *
extensive amalgam restoration, a D. during treatment.
retentive pin hole preparation should
be placed

Dr Abdul Naser Tamim SEHA, Dr Ghada Al Aqqad D.D.S, Dr Kamal Naser - Amrita Medical Centre, Dr Emad Wani - Shadi Dental Centre,
Dr Lina Anka SEHA, Dr Rouba Zgheibi SEHA, Maria Teresa Yongson Alejandrino SEHA 117
In composite resin restorations, glass Which of the following medications
ionomer cements can be used as a increases a patient’s risk for intraoral
base because they are candidiasis?

A. sedative to a hyperemic pulp. A. Warfarin.


B. neutral in colour. B. Cyclosporine. *
C. biocompatible. * C. Pentobarbital.
D. compatible with the expansion of D. Ibuprofen.
composite resins. E. Pilocarpine.

In the restoration of an endodontically A patient on anticoagulant drugs who


treated tooth, increasing the post requires an extraction has a
preparation diameter will most likely prothrombin time of 20 seconds. The
increase normal value is 15 seconds. The most
appropriate management is to
A. post stability.
B. cervical resistance. A. administer vitamin K after the
C. stress to the root. * extraction.
D. crown retention. B. administer vitamin K before the
extraction.
Which statement is true? C. extract the tooth and use local
measures to control bleeding. *
A. Radiographs can differentiate D. discontinue anticoagulation drugs
between infected and non-infected one week before extraction.
periapical lesions.
B. A definitive diagnosis of an apical Which of the following drug groups is
lesion cannot be made using LEAST likely to cause xerostomia?
radiography alone. *
C. Periapical radiolucencies are A. Diuretics.
indicative of loss of pulp vitality. B. Antibiotics. *
D. A periapical radiograph can be used C. Antidepressants.
to locate the buccal bone level. D. Anticholenergics.

Opioids would be The problem most likely to result from a


CONTRAINDICATED for the temporary crown with inadequate
management of proximal contacts is

A. pain. A. frequent decementation of the


B. severe cough. temporary crown.
C. diarrhea. B. gingival recession.
D. depression. * C. difficulty in seating the permanent
crown. *
D. an esthetically compromised
restoration.

Dr Abdul Naser Tamim SEHA, Dr Ghada Al Aqqad D.D.S, Dr Kamal Naser - Amrita Medical Centre, Dr Emad Wani - Shadi Dental Centre,
Dr Lina Anka SEHA, Dr Rouba Zgheibi SEHA, Maria Teresa Yongson Alejandrino SEHA 118
An occlusal rest should be prepared so
that Dental plaque

A. occlusal forces are directed A. contains insoluble glucans. *


perpendicular to the long axis of the B. contains food particles.
tooth. C. contains predominantly Gram-
B. occlusal forces are directed parallel negative organisms.
to the long axis of the tooth. * D. becomes less anaerobic as it
C. occlusal forces are concentrated matures.
near the marginal ridge.
D. the tooth is prevented from rotating. Which of the following is associated
with aggressive periodontitis in
The anticariogenic effect of systemic adolescents?
fluoride is related principally to the
A. Treponema denticola.
A. bactericidal action on oral flora. B. Aggregatibacter (Actinobacillus)
B. bacteriostatic action on oral flora. actinomycetemcomitans. *
C. buffering effect on acids produced C. Porphyromonas gingivalis.
by cariogenic bacteria. D. Prevotella intermedia.
D. alteration in the composition of the
enamel. * Normal aging changes in the tooth
tissues include
In a teenage patient with normal
gingiva, interdental plaque removal is A. continuous deposition of
best accomplished with cementum. *
B. augmented dentin sensitivity.
A. a regular toothbrush. C. increased porosity of enamel.
B. a hard nylon bristle brush.
C. an interproximal brush. Which of the following fibre groups are
D. dental floss. * attached to bone?
E. an interdental wood stick.
A. Apical. *
A patient complains of intermittent B. Interradicular.
spontaneous pain in a tooth that was C. Transseptal.
previously treated with a direct pulp D. Circular.
cap. The tooth is not sensitive to
percussion. Both hot and cold produce Cementum
severe pain. The most likely diagnosis
is A. contains minimal amounts of
collagen.
A. reversible pulpitis. B. has areas devoid of matrix-forming
B. symptomatic irreversible pulpitis. * cells. *
C. cracked tooth syndrome. C. vascularized at its periphery.
D. symptomatic apical periodontitis
(acute periradicular periodontitis).

Dr Abdul Naser Tamim SEHA, Dr Ghada Al Aqqad D.D.S, Dr Kamal Naser - Amrita Medical Centre, Dr Emad Wani - Shadi Dental Centre,
Dr Lina Anka SEHA, Dr Rouba Zgheibi SEHA, Maria Teresa Yongson Alejandrino SEHA 119
The physiologic rest position of the disease must possess all of the
mandible is following EXCEPT

A. a position unrelated to tonicity of A. be associated with disease, as


mandibular muscles. evidenced by increases in the number
B. affected by the time of of organisms at diseased sites.
measurement. B. be eliminated or decreased in sites
C. the primary means of determining that demonstrate clinical resolution of
occlusal vertical dimension. * disease with treatment.
D. used when making a centric C. demonstrate a host response, in the
interocclusal record. form of alteration in the host cellular,
not humoral, immune response. *
To initiate caries, bacterial plaque pH D. be capable of causing disease in
must reach experimental animal models.
E. demonstrate virulence factors
A. 5.4. * responsible for enabling the organisms
B. 5.9. to cause periodontal tissue destruction.
C. 6.4.
D. 6.9. In the embryo which of the following
could be missing if pharyngeal pouch
What is the most appropriate action for development is disrupted?
a dentist to take for a female patient
experiencing spousal abuse? A. Thyroid.
B. Maxillary sinus.
A. Speak to the alleged abuser to C. Thymus. *
assess the level of threat to the patient. D. Inner ear.
B. Offer resource materials and
referrals to community agencies to the Defects in the outflow tract of the heart
patient. * in individuals with craniofacial
C. Focus on dental matters and malformations are caused by the
consider the abuse issues in the future. disruption of
D. Counsel the patient to leave the
abuser. A. mesoderm development.
B. neurulation.
On the basis of “Socransky’s C. folding of the embryo.
Modifications of Koch’s Postulates”, the D. neural crest development. *
potential pathogens in periodontal

Dr Abdul Naser Tamim SEHA, Dr Ghada Al Aqqad D.D.S, Dr Kamal Naser - Amrita Medical Centre, Dr Emad Wani - Shadi Dental Centre,
Dr Lina Anka SEHA, Dr Rouba Zgheibi SEHA, Maria Teresa Yongson Alejandrino SEHA 120
Angiotensin II converting enzyme is Rigor mortis is
primarily produced in
A. due to intracellular Ca2+ ion
A. osteoblasts. depletion.
B. epithelial cells of the small B. due to cellular ATP depletion. *
intestine. C. mechanistically identical to a tetanic
C. hepatocytes. contraction.
D. Kupffer cells. D. characterized by spastic paralysis.
E. vascular endothelial cells of the E. an isotonic contraction.
lung. *
Muscle spindles associated with
Daily fluid loss from a healthy stretch reflexes
individual at rest in a temperate
climate is approximately A. are composed of extrafusal muscle
fibres.
A. 250ml. B. are innervated by alpha motor
B. 500ml. neurons.
C. 1.0L. C. mediate autogenic inhibition of
D. 2.5L. * muscle contraction.
E. 5.0L. D. possess fibres that are capable of
contraction. *
Which of the following is increased E. contribute to the main force
after stimulating α1-adrenergic production properties of muscle.
receptors?
Which disease is associated with a
A. Contractility of the heart. higher proportion of Fusobacterium,
Porphyromonas, Prevotella, and
B. SA node activity.
spirochetes?
C. Blood vessel constriction. *
D. Bronchial dilation.
A. Necrotizing ulcerative periodontitis.
B. Chronic periodontitis. *
Heart rate is increased by
C. Aggressive periodontitis.
A. hyperkalemia. D. Root caries.
B. reduced α1–adrenergic receptor E. Early childhood caries.
activity.
Which of the following dental health
C. atropine. *
recommendations is consistent with
D. acetylcholine.
the general nutrition guidelines?
E. elevated vagal tone.
A. Diet soft drinks and potato chips are
an acceptable snack.
B. A piece of fruit and plain yogurt is
an acceptable snack. *
C. Eat a bigger lunch and not snack.
D. Brush your teeth well after snacks.

Dr Abdul Naser Tamim SEHA, Dr Ghada Al Aqqad D.D.S, Dr Kamal Naser - Amrita Medical Centre, Dr Emad Wani - Shadi Dental Centre,
Dr Lina Anka SEHA, Dr Rouba Zgheibi SEHA, Maria Teresa Yongson Alejandrino SEHA 121
Hydantoin treatment causes gingival Which of the following symptoms are
hyperplasia as a result of a/an consistent with an anaphylatic reaction
to penicillin?
A. increased number of epithelial cells.
B. thickening of the basement A. Deafness, dizziness acute anemia
membrane. and bronchial constriction.
C. increased production of B. Crystalluria, nausea, vomiting,
proteoglycans. * diarrhea, and bronchial constriction.
C. Oliguria, hematuria, bronchial
A 20 year old student presents with constriction, and cardiovascular
clinical symptoms of necrotizing collapse.
ulcerative gingivitis (NUG). Food intake D. Urticaria, diarrhea, bronchial
for the last 24 hours indicates a soft constriction and cardiovascular
diet lacking in fruits and vegetables. collapse. *
The patient’s diet is important to
investigate further because Asthmatic patients using corticosteroid
inhalers may develop candidiasis on
A. a deficiency of certain nutrients the dorsal surface of the tongue
causes NUG. because of
B. NUG may be limiting the food
choices the patient is making. A. a systemic antibacterial action.
C. NUG can be cured through B. local destruction of normal oral flora.
modification of diet. C. prolonged local vasoconstriction.
D. patients with NUG lose interest in D. cross-reacting antigens in the
eating. * tongue.
E. local immunosupression. *
A pale 8 year old patient has
generalized gingival enlargement and A 28 year old male patient works long
spontaneous bleeding. The most days that typically include numerous
appropriate initial management of this soft drinks and candy bars, but no
patient is to opportunity to brush his teeth. He has
continued to experience one to two
A. perform an incisional biopsy. carious lesions per year and now wants
B. obtain a cytologic smear. orthodontic treatment to straighten his
C. order a complete blood count. * teeth. Which of the following is the
D. obtain bacterial cultures. most appropriate initial step in
E. obtain a fasting blood glucose level. addressing behavior modification for
this patient?
What is the most likely diagnosis of a
white diffuse lesion with a wrinkled A. Providing a new toothbrush and
appearance on the buccal mucosa sample tube of dentifrice.
which disappears upon stretching? B. Illustrating, with the aid of drawings
the basic caries process.
A. Leukoedema. * C. Explaining how his present habits
B. Lichen planus. increase his caries risk. *
C. Candidiasis. D. Explain that treatment cannot be
D. Linea alba. initiated until he can keep his teeth
E. White sponge nevus. clean.

Dr Abdul Naser Tamim SEHA, Dr Ghada Al Aqqad D.D.S, Dr Kamal Naser - Amrita Medical Centre, Dr Emad Wani - Shadi Dental Centre,
Dr Lina Anka SEHA, Dr Rouba Zgheibi SEHA, Maria Teresa Yongson Alejandrino SEHA 122
A 70 year old female has consumed B. when the correction is
optimal levels of fluorides all her life. accomplished by intrusion of the
She could be expected to have a posterior teeth.
reduced incidence of which of the C. in patients with decreased lower
following? anterior face heights.
D. when serial extractions are
A. Paget’s disease. performed.
B. Dental caries. *
C. Osteoporosis. The most appropriate management of
D. Hyperparathyroidism. a noncavitated, smooth surface
carious lesion is
Which of the following explanations of
the caries process is most appropriate A. placing an amalgam restoration.
for a 10 year old patient who snacks B. placing a composite restoration.
frequently on soft drinks and C. applying topical fluoride. *
doughnuts? D. prescribing a chlorhexidine rinse.
E. observation.
A. When you eat doughnuts and soft
drinks, because of all the sugar in them
A diagnostic test for dental caries
you’ll get cavities.
which has low sensitivity and high
B. Bacteria in your mouth are the main
specificity will result in
cause of caries.
C. Not brushing your teeth means the
A. unnecessary treatment being
sugar from your snack attacks your
provided.
teeth for about twenty minutes.
D. The ‘bugs’ in your mouth eat the B. undetected disease. *
sugar in the food you eat, and change C. patients being alarmed
it into acid which can make holes in unnecessarily.
your teeth. * D. a test whose performance does not
justify its cost.
The most appropriate time to begin
orthodontic correction of an Angle Assuming the daily maximum is not
Class II malocclusion is exceeded, which of the following is/are
appropriate for pain management for
A. following eruption of the maxillary an adult with a history of severe
first permanent molars. asthma and nasal polyps following an
B. following eruption of the maxillary emergency pulpectomy?
permanent central and lateral incisors.
C. several months prior to the pubertal A. Naproxen 250 mg, every 6 to 8
growth spurt. * hours.
D. at the start of the pubertal growth B. Acetylsalicylic acid 650 mg every 4
spurt. hours.
C. Ketorolac 10 mg every 4 hours.
Long term stability of the orthodontic D. Acetaminophen 1000 mg every 6
correction of an anterior open bite is hours. *
better

A. following cessation of a thumb


sucking habit. *

Dr Abdul Naser Tamim SEHA, Dr Ghada Al Aqqad D.D.S, Dr Kamal Naser - Amrita Medical Centre, Dr Emad Wani - Shadi Dental Centre,
Dr Lina Anka SEHA, Dr Rouba Zgheibi SEHA, Maria Teresa Yongson Alejandrino SEHA 123
When compared to quaternary A. obturate the canal.
ammonium oral rinses, 0.12% B. reinforce the root.
chlorhexidine oral rinses have a higher C. reinforce the remaining coronal
antimicrobial activity due to increased tooth structure.
D. retain the core build up. *
A. toxicity.
B. potency. Which of the following analgesics is
C. concentration. CONTRAINDICATED for the long-term
D. substantivity. * management of myofascial pain
syndrome?
Which of the following drugs will NOT
cause gingival hyperplasia? A. Acetylsalicylic acid with codeine.
B. Ibuprofen.
A. cyclosporine. C. Oxycodone. *
B. doxepin. * D. Acetaminophen with codeine.
C. phenytoin.
D. verapamil. A patient with a unilateral left disc
displacement without reduction is most
In a 17 year old, which traumatic dental likely to present with
injury is most likely to result in pulp
necrosis on tooth 1.1? A. normal left excursion and limited
right excursion. *
A. Extrusion. B. limited left excursion and normal
B. Intrusion. * right excursion.
C. Luxation. C. normal bilateral excursions.
D. Subluxation. D. limited bilateral excursions.

An 8 year old patient presents 4 hours A patient with a history of infective


post-trauma with an oblique crown endocarditis is currently taking
fracture of 2.1 exposing 2mm of vital penicillin for an unrelated condition.
pulp. The most appropriate pulpal The most appropriate management for
treatment is periodontal therapy for this patient is to

A. apexogenesis. * A. prescribe a different antibiotic. *


B. apexification. B. increase the dosage of penicillin.
C. extraction. C. maintain penicillin at the present
D. root canal treatment. level.
D. use chlorhexidine solution as a rinse
A post is used in an endodontically prior to therapy.
treated tooth to

Dr Abdul Naser Tamim SEHA, Dr Ghada Al Aqqad D.D.S, Dr Kamal Naser - Amrita Medical Centre, Dr Emad Wani - Shadi Dental Centre,
Dr Lina Anka SEHA, Dr Rouba Zgheibi SEHA, Maria Teresa Yongson Alejandrino SEHA 124
Which ethical principle is involved with been taken in the last year by another
a dentist’s duty to recognize signs of dental office. The most appropriate
abuse and neglect and to know management is to
appropriate interventions?
A. make a panoramic radiograph and
A. Autonomy. bite-wings.
B. Beneficence. * B. make a panoramic radiograph only.
C. Justice. C. make bite-wings radiographs only. *
D. Non maleficence. D. request the existing radiographs.
E. Veracity.
The most appropriate radiographic
Which ethical principle is violated examination for a new patient with an
when a dentist practices while extensively restored dentition and
impaired? generalized periodontal disease is (a)

A. Autonomy. A. full mouth periapicals.


B. Beneficence. B. full mouth periapicals and bitewings.
C. Justice. *
D. Nonmaleficence. * C. panoramic radiograph.
E. Veracity. D. panoramic radiograph and
bitewings.
Which ethical principle is violated E. cone beam computed tomography.
when a dentist increases fees solely
based on the fact that the patient is The most effective method for
covered by dental insurance? protecting dental personnel who
expose radiographs is to have them
A. Autonomy.
B. Beneficence. A. stand behind a barrier during
C. Justice. exposure. *
D. Nonmaleficence. B. stand 2 metres away from the x-ray
E. Veracity. * generator during the exposure.
C. wear a lead apron during exposure.
Which substance found in radiograph D. wear a radiation monitoring device
processing solutions is of most (film badge) during exposure.
concern environmentally?
The most likely condition to mimic
A. Ammonium thiosulfite. periodontal disease is
B. Acetic acid.
C. Silver. * A. lateral periodontal cyst.
D. Sodium sulfite. B. paradental cyst.
C. Langerhans cell histiocytosis. *
A panoramic radiograph and bite- D. disuse atrophy (ridge resorption).
wings are required as part of an
examination for a new patient. The
patient states that radiographs had

Dr Abdul Naser Tamim SEHA, Dr Ghada Al Aqqad D.D.S, Dr Kamal Naser - Amrita Medical Centre, Dr Emad Wani - Shadi Dental Centre,
Dr Lina Anka SEHA, Dr Rouba Zgheibi SEHA, Maria Teresa Yongson Alejandrino SEHA 125
Which permanent tooth is most likely to neck veins and difficulty breathing
have an anomaly? when lying down?

A. Mandibular first premolar. A. Congestive heart failure. *


B. Maxillary lateral incisor. * B. Bronchitis.
C. Mandibular central incisor. C. End stage renal disease.
D. Maxillary first molar. D. Hypertension.

The most common soft tissue Which viral hepatitis does NOT have a
calcification or ossification found on a chronic carrier state?
panoramic radiograph is (a)
A. Hepatitis A. *
A. lymph node. B. Hepatitis B.
B. sialolith. * C. Hepatitis C.
C. phlebolith. D. Hepatitis D.
D. stylohyoid ligament.
A patient had a coronary arterial stent
Antiobiotic prophylaxis prior to tooth placed 1 year ago following a
extraction is required for (a) myocardial infarction and has been
asymptomatic since. The most
A. diabetes mellitus. appropriate management is to
B. mitral valve prolapse with
regurgitation. A. avoid using epinephrine
C. prosthetic heart valve. * impregnated retraction cord. *
D. organic heart murmur. B. use local anesthetics without
E. functional heart murmur. epinephrine.
C. perform only emergency dental
For a patient taking warfarin, which treatment for 1 year.
laboratory test provides the most D. not recline the patient more than 45
accurate information on coagulation degrees.
time? E. prescribe antibiotics to prevent
infective endocarditis.
A. Activated partial thromboplastin
time. The most common medical emergency
B. Ivy bleeding time. in the dental office is a/an
C. Platelet count.
D. INR. * A. allergic reaction.
E. Thrombin time. B. cerebrovascular accident.
C. myocardial infarction.
What is the most likely diagnosis for a D. seizure.
patient who exhibits elevated blood E. syncopal episode. *
pressure, swollen ankles, distended

Dr Abdul Naser Tamim SEHA, Dr Ghada Al Aqqad D.D.S, Dr Kamal Naser - Amrita Medical Centre, Dr Emad Wani - Shadi Dental Centre,
Dr Lina Anka SEHA, Dr Rouba Zgheibi SEHA, Maria Teresa Yongson Alejandrino SEHA 126
A patient on hemodialysis suspected. Which of the following
intravenous antibiotics is most
A. should receive dental treatment only appropriate for the management of the
on the day of hemodialysis. condition?
B. should not be administered lidocaine
for local anesthesia. A. Sulfonamide.
C. is likely to have an increased red B. Aminoglycoside.
blood cell count. C. Cephalosporin.
D. is at greater risk for infection. * D. Erythromycin.
E. Penicillin. *
In domestic violence cases, the highest
risk for victim fatality is associated with Following the IV administration of a
benzodiazepine, a patient becomes
A. an actual or pending separation. * apneic and unresponsive. Which of the
B. severe depression of the abuser. following drugs should be administered
C. prior threats to kill the victim. immediately?
D. a new partner in the victim’s life.
E. a history of abuse of the abuser as a A. Naloxone.
child. B. Propranolol.
C. Amphetamine.
The most common abuser of an elderly D. Flumazenil. *
person is a/an E. Atropine.

A. friend or acquaintance. The most appropriate type of bone


B. adult child. * grafting material for use in the
C. spouse. maxillofacial region is
D. sibling.
A. allogenic.
The statute of limitations states that the B. alloplastic.
period during which an action in C. autogenous. *
negligence against a dentist can be D. xenogenic.
initiated starts when the
Patients with resistance to activated
A. patient first meets the dentist. protein C (factor V Leiden) are at risk
B. treatment is rendered. for
C. treatment is paid for.
D. patient first becomes aware of the A. fibrinolysis.
problem. * B. bleeding diatheses. *
E. patient seeks a second opinion C. thrombocytopenia. ?
about the problem. D. hypercoagulation.
E. erythrocytosis.
A patient presents with a chronic,
purulent draining fistula in the right
submandibular area. Culture results are
inconclusive but actinomycosis is

Dr Abdul Naser Tamim SEHA, Dr Ghada Al Aqqad D.D.S, Dr Kamal Naser - Amrita Medical Centre, Dr Emad Wani - Shadi Dental Centre,
Dr Lina Anka SEHA, Dr Rouba Zgheibi SEHA, Maria Teresa Yongson Alejandrino SEHA 127
Dentist supervised at-home bleaching
technique is NOT indicated for a A patient has a history of controlled
patient who hypertension (BP 123/80 mmHg) and
type 1 diabetes (A1c is 6.3). The
A. is unhappy with the natural colour of patient has had a cardiac
their teeth. catheterization, a coronary artery
B. has existing all-ceramic crowns that stent placement, and a total hip
are lighter than the natural teeth. replacement in the past year. Which
C. has a single dark coloured tooth. * of the conditions in the patient’s
D. has teeth discoloured by medical history requires prophylactic
tetracycline. antibiotic coverage for dental
E. has extrinsic stains. procedures causing bacteremia?

A healthy, 55 year old patient has A. Cardiac catheterization.


alveolar bone loss on 40% of teeth, 3- B. Coronary artery stent.
4mm of clinical attachment loss and C. Diabetes.
probing pocket depths ranging between D. Hip replacement. *
5-6mm with bleeding upon probing.
The most likely diagnosis is The most appropriate analgesic for a
pregnant patient is
A. mild generalized chronic
periodontitis. A. acetaminophen. *
B. mild localized chronic periodontitis B. acetylsalicylic acid.
C. moderate generalized chronic C. hydrocodone and acetaminophen.
periodontitis. * D. ibuprofen.
D. moderate localized chronic
periodontitis. In the absence of “Hanks balanced
salt solution”, what is the most
Overeruption of the posterior teeth appropriate media to transport an
results in increased avulsed tooth?

1. Crowding. A. Saliva.
2. Overbite. B. Milk. *
3. Overjet. C. Saline.
4. Rotation. D. Tap water.

A. (1) (2) (3) What is the most appropriate space


B. (1) and (3) * management for a 4 year old patient
C. (2) and (4) who has lost all their primary
D. (4) only maxillary incisors due to trauma?
E. All of the above.
A. No treatment. *
The most common problem associated B. A removable Hawley appliance.
with two adjacent implants is C. A fixed lingual holding arch.
D. A fixed Nance button appliance.
A. peri-implantitis.
B. lack of interdental papilla. *
C. granulation tissue.

Dr Abdul Naser Tamim SEHA, Dr Ghada Al Aqqad D.D.S, Dr Kamal Naser - Amrita Medical Centre, Dr Emad Wani - Shadi Dental Centre,
Dr Lina Anka SEHA, Dr Rouba Zgheibi SEHA, Maria Teresa Yongson Alejandrino SEHA 128
Splinting of a permanent maxillary
incisor following trauma is required in A. increase its fusion temperature.
the management of a B. decrease its thermal expansion.
C. increase its chemical reactivity. *
A. concussion injury. D. decrease its potential for
B. luxation injury. * devitrification upon heating.
C. Class II fracture.
D. Class IV fracture. During healing following implant
placement, the cells that generate
When producing a metal-ceramic new bone tissue are
crown restoration
A. osteoblasts. *
A. the body-porcelain layer should be B. osteocytes.
rich in opacifying oxides. C. osteoclasts.
B. oversizing the body-porcelain layer D. osteophytes.
prior to sintering should be avoided.
C. the porcelain thermal expansion A 9 year old presents immediately
coefficient should be slightly lower than after the avulsion of a permanent
the metal. * maxillary central incisor. Which of the
D. fusing should be followed by following is the most appropriate
quenching. management prior to re-
implantation?
Voids in a gypsum cast are most likely
the result of A. Rinse the tooth with saline. *
B. Curette the root surface.
A. low surface tension of a silicone C. Amputate the apex.
impression material. D. Extirpate the pulp.
B. high surface tension of an
irreversible hydrocolloid. * A subluxated tooth will have
C. using a hydrophilized addition
silicone. A. mobility and displacement.
D. spraying a surfactant on the B. no mobility and no displacement.
impression. C. mobility and no displacement. *

A new bone grafting material has been Which porcelain stain colour is added
demonstrated to generate bone tissue to give the appearance of
through the recruitment of progenitor translucency?
cells. This potential can best be
described as an example of A. Grey.
B. Blue. *
A. osteoconduction. C. Yellow.
B. osteogenesis. D. White.
C. osteoinduction. * E. Orange.
D. osteopenia.

Increasing the amount of network


modifiers (e.g. CaO, Na2O) in
porcelain will

Dr Abdul Naser Tamim SEHA, Dr Ghada Al Aqqad D.D.S, Dr Kamal Naser - Amrita Medical Centre, Dr Emad Wani - Shadi Dental Centre,
Dr Lina Anka SEHA, Dr Rouba Zgheibi SEHA, Maria Teresa Yongson Alejandrino SEHA 129
For a complete denture patient, which A 10 year old child with no previous
letter or sound is a guide for the caries experience has proximal
position of the incisal edge of the carious lesions in the enamel only of
maxillary incisors? several primary molars. How should
the lesions be managed?
A. M.
B. S. A. No treatment.
C. F. * B. Be treated with topical fluoride,
D. P. proper home care and observation. *
E. J. C. Be smoothed with abrasive strips.
D. Be treated with fissure sealants.
Which sound is associated with the E. Be restored with composite resin.
most common speech defect in
complete denture patients? An inflammatory cellular infiltrate found
in the connective tissue of healthy
A. M. gingiva is
B. S. *
C. F. A. a routine microscopic finding. *
D. J. B. an indication of systemic disease.
E. V. C. composed chiefly of macrophages.
D. a tissue response to food
A zirconia-based ceramic fixed partial decomposition.
denture can be used for a patient with
For acute dental pain, the daily
A. periodontally involved abutment maximum cumulative dose of
teeth. acetaminophen is
B. long clinical crowns. *
C. deep vertical anterior overlap. A. 2400 mg.
D. cantilever pontic. B. 3200 mg.
E. evidence of bruxism. C. 3600 mg.
D. 4000 mg. *
In a restoration following endodontic
therapy, the primary function of the Ludwig’s angina initially involves a
post is to improve massive infection of the

A. distribution of forces along the long A. parapharyngeal and


axis. retropharyngeal spaces.
B. resistance of the tooth to fracture. B. submandibular and sublingual
C. sealing of the root canal. regions. *
D. retention of the definitive C. buccal superficial fascial and canine
restoration. * spaces.
D. maxillary and ethmoidal sinuses.

Dr Abdul Naser Tamim SEHA, Dr Ghada Al Aqqad D.D.S, Dr Kamal Naser - Amrita Medical Centre, Dr Emad Wani - Shadi Dental Centre,
Dr Lina Anka SEHA, Dr Rouba Zgheibi SEHA, Maria Teresa Yongson Alejandrino SEHA 130
Which class of drugs should first be
considered for oral sedation in the Which type of bone grafting material is
adult patient? most likely to have a successful result?

A. Antihistamines. A. Allogenic.
B. Barbiturates. B. Alloplastic.
C. Benzodiazepines. * C. Autogenous. *
D. Opioids. D. Xenogenic.
E. Phenothiazines.
Which of the following is NOT an
Gloves should advantage of using an etchant and
bonding system before insertion of a
A. be washed with mild soap and warm composite resin restoration?
water after a dental procedure before
treating the next patient. A. Improved retention.
B. be removed after use and replaced B. Decreased marginal leakage.
with new gloves before treating C. Greater strength of the restoration. *
another patient. * D. Reduced polymerization shrinking
C. not be used if the patient reports a effect.
latex allergy.
D. not be used in dental procedures Zinc phosphate cement, when used as
requiring fine tactile discrimination. a luting agent, has which of the
E. be placed on the hands before tying following properties?
on a mask.
A. Mechanical retention. *
For sterilization to occur in an B. Insolubility.
autoclave, all of the following must be C. Anticariogenicity.
monitored EXCEPT D. Chemical adhesion.

A. chemical levels. * Which of the following is LEAST likely


B. time. to cause swelling of the gingiva
C. pressure. adjacent to an endodontically treated
D. temperature. tooth?

A drug with anticholinergic side effects A. Internal root resorption. *


may cause B. Vertical root fracture.
C. Periradicular abscess.
A. convulsions. D. Periodontal abscess.
B. diarrhea.
C. tremors.
D. xerostomia. *

Dr Abdul Naser Tamim SEHA, Dr Ghada Al Aqqad D.D.S, Dr Kamal Naser - Amrita Medical Centre, Dr Emad Wani - Shadi Dental Centre,
Dr Lina Anka SEHA, Dr Rouba Zgheibi SEHA, Maria Teresa Yongson Alejandrino SEHA 131
Which of the following statements A. base at the dentino-enamel junction.
about incision and drainage of an acute B. base facing toward the pulp.
periradicular abscess is FALSE? C. apex pointing to the enamel surface.
D. apex pointing to the dentino-enamel
A. The incision should be sutured after junction. *
achieving drainage. *
B. The procedure is only indicated with Shortly after the administration of a
a localized, fluctuant swelling. local anesthetic for the removal of tooth
C. Profound anesthesia of the surgical 2.8, the patient complains of a
site is not always possible. tenseness in the left cheek and left
D. Relief of the pressure and pain is cheek swelling is observed. What is the
immediate after treatment. most likely diagnosis?

A dental laboratory has fabricated a A. Surgical emphysema.


removable partial denture framework. B. Immediate allergic reaction.
The framework fits the master cast well C. Herniation of buccal fat pad.
but when tried in the mouth a stable fit D. Hematoma. *
cannot be achieved. The most likely
cause is What is the most probable complication
of a local anesthetic injection into the
A. improper major connector design. parotid gland?
B. distortion in the final impression. *
C. shrinkage of the alloy during A. Infection.
casting. B. Trismus.
D. improper casting temperature. C. Diplopia.
D. Facial paralysis. *
Treatment of primary herpectic
gingivostomatitis should include For an avulsed, fully developed
permanent tooth that has been
A. topical steroids. reimplanted, a favourable prognosis is
B. application of dilute hydrogen most affected by the
peroxide.
C. cauterization. A. length of time the tooth has been out
D. control of secondary infection. * of the mouth. *
B. use of an appropriate antibiotic.
Which of the following drugs can C. effectiveness of the irrigation of the
interfere with the effectiveness of oral socket.
contraceptives? D. rigidness of the splint.
E. thoroughness of the curettage of the
A. Codeine. root surface.
B. Penicillin V. *
C. Acetaminophen.
D. Magnesium trisilicate.
E. None of the above.

On a bite-wing radiograph, a smooth


surface proximal carious lesion in
enamel appears as a triangle with the

Dr Abdul Naser Tamim SEHA, Dr Ghada Al Aqqad D.D.S, Dr Kamal Naser - Amrita Medical Centre, Dr Emad Wani - Shadi Dental Centre,
Dr Lina Anka SEHA, Dr Rouba Zgheibi SEHA, Maria Teresa Yongson Alejandrino SEHA 132
A patient is hit in the right eye with a
baseball. Which of the following would A. 4½ years.
indicate a right orbital floor fracture? B. 6½ years.
C. 8½ years. *
A. Right periorbital edema. D. 11½ years.
B. Right periorbital ecchymosis.
C. Limited upward movement of the Compared to acetylsalycilic acid, long-
right eye. * term use of celecoxib is associated
D. Premature occlusal contact on the with
maxillary right.
A. reduced gastrointestinal ulcerations.
Which of the following does NOT need *
to be considered when assessing B. increased cardiac protection.
whether exposure of bodily fluids to a C. lower renal toxicity.
dental health care personnel is D. increased safety for asthmatic
significant? patients.
E. increased platelet aggregation
A. Patient’s gender. * inhibition.
B. Type of bodily fluid.
C. Infectious disease status of the Which class of drugs does NOT have
patient. analgesic effects?
D. Amount of bodily fluid involved in
the injury. A. Anticholinergics. *
B. Antidepressants.
The purpose of hand hygiene in
C. Anticonvulsants.
infection control is primarily to reduce
the D. Salicylates.

When comparing ibuprofen and


A. resident microflora in the deep
celecoxib used in therapeutic doses,
tissue layers.
which of the following statements is
B. quantity of transient microorganisms
true?
on the surface of the hands. *
C. dryness of the skin prior to donning
A. Ibuprofen inhibits mostly COX-1,
gloves.
celecoxib inhibits mostly COX-2.
D. risk of patient contamination.
B. Ibuprofen inhibits both COX-1 and
COX-2, celecoxib inhibits mostly
Which of the following indicates a
COX-2. *
failure of a dental implant?
C. Ibuprofen inhibits mostly COX-1,
celecoxib inhibits both COX-1 and
A. Gingival inflammation.
COX-2.
B. Horizontal bone loss of one third of
D. Both ibuprofen and celecoxib are
the implant length.
potent inhibitors of COX-1 and COX-2.
C. Mobility. *
D. Increased probing depths.

At what age is a child expected to have


12 erupted primary teeth and 12
erupted permanent teeth?

Dr Abdul Naser Tamim SEHA, Dr Ghada Al Aqqad D.D.S, Dr Kamal Naser - Amrita Medical Centre, Dr Emad Wani - Shadi Dental Centre,
Dr Lina Anka SEHA, Dr Rouba Zgheibi SEHA, Maria Teresa Yongson Alejandrino SEHA 133
Which of the following is NOT an effect A. Streptococcus viridans.
of acetaminophen? B. Epstein-Barr virus.
C. Candida albicans. *
A. Analgesia. D. Straphylococcus aureus.
B. Anti-inflammatory effect. *
C. Inhibition of CNS cyclooxygenase. Which of the following antibiotics is the
D. Antipyretic effect. LEAST appropriate to be prescribed to
a pregnant patient?
The most appropriate procedure when
a glove has been punctured during a A. Amoxicillin.
restorative appointment is to B. Tetracycline. *
C. Erythromycin.
A. wash the gloved hands using D. Penicillin.
antimicrobial soap and complete the
procedure. Which of the following is NOT a
B. change the glove as soon as property of Lidocaine (Xylocaine®)
possible. *
C. put on an overglove and complete A. local anesthetic.
the procedure. B. topical anesthetic.
D. complete the procedure as C. anticonvulsant. *
expeditiously as possible. D. antiarrhythmic agent.

Which of the following bacterial Which of the following cause


components is a potent inducer of sedation?
proinflammatory cytokines and can
cause fever and shock? 1. Triazolam.
2. Codeine.
A. Lipopolysaccharide. * 3. Meperidine.
B. Peptidoglycan. 4. Ibuprofen.
C. Teichoic acid.
D. Porins. A. (1) (2) (3) *
B. (1) and (3)
An HIV-positive patient has white C. (2) and (4)
lesions on the buccal mucosa. The D. (4) only
lesions can be wiped off to reveal an E. All of the above.
erythematous base. The lesions are
most likely caused by

Dr Abdul Naser Tamim SEHA, Dr Ghada Al Aqqad D.D.S, Dr Kamal Naser - Amrita Medical Centre, Dr Emad Wani - Shadi Dental Centre,
Dr Lina Anka SEHA, Dr Rouba Zgheibi SEHA, Maria Teresa Yongson Alejandrino SEHA 134
Five minutes after injecting a local D. It is time-consuming to place
anesthetic, a patient experiences a compared to composite resin.
generalized warmth of the face, mouth
and upper chest. What is the most In providing maxillary anterior esthetic
likely cause? dentistry, optimal results are obtained
by considering all the following
A. Increased blood pressure. EXCEPT
B. Anaphylactic reaction. *
C. Nervousness. A. midline orientation of the incisors.
D. Vagal shock. B. age and gender of the patient.
C. Curve of Wilson. *
Coxsackie A virus is the etiologic agent D. lower lip line.
in
Which clinical sign indicates palatal
A. papilloma. impaction of the permanent maxillary
B. herpangina. * canines?
C. verruca vulgaris.
D. condyloma accunimatum. A. Buccally positioned first premolars.
B. Early exfoliation of primary canines.
After a tooth surface has been C. Midline central diastema.
completely cleaned, the new D. Lack of canine labial bulges in a 10
glycoprotein coating which forms on year old patient. *
the surface is called
Which of the following is the most
A. pellicle. * appropriate management for a dry
B. plaque. socket?
C. materia alba.
D. biofilm. A. Hydrogen peroxide irrigation of the
socket.
Streptococcus mutans selectively B. Vigorous curettage of the socket.
colonizes C. Placement of a dressing in the
socket. *
A. enamel. * D. A prescription for antibiotics.
B. cementum.
C. tongue. What is the most significant
D. buccal mucosa. radiographic finding in
hyperparathyroidism?
Which statement is true regarding
amalgam? A. Demineralization of teeth.
B. Multiple odontogenic keratocysts.
A. A thin layer can be left to seal the C. Hypercementosis.
cavosurface margins. D. Rampant caries. *
B. It bonds on its own to tooth E. Generalized loss of lamina dura.
structure.
C. It wears at a rate similar to that of
tooth structure. *

Dr Abdul Naser Tamim SEHA, Dr Ghada Al Aqqad D.D.S, Dr Kamal Naser - Amrita Medical Centre, Dr Emad Wani - Shadi Dental Centre,
Dr Lina Anka SEHA, Dr Rouba Zgheibi SEHA, Maria Teresa Yongson Alejandrino SEHA 135
The appropriate management for an C. resistance to corrosion.
avascular white lesion, 5 x 3mm in size, D. strength.
that has been present on the buccal
mucosa for 6 months and has recently Which of the following is NOT a
become ulcerated is function of the wedge in the restoration
of a Class II cavity with amalgam?
A. observation.
B. excisional biopsy. * A. It separates the teeth to allow
C. incisional biopsy. restoration of the contact.
D. aspiration biopsy. B. It assists in the adaptation of the
E. cytologic examination. matrix band to the proximal portion of
the preparation.
What is the most likely cause of voids C. It absorbs moisture from the cavity
in a maxillary diagnostic cast? preparation, allowing the restoration to
be placed in a dry field. *
A. Poor mix of the impression material, D. It provides stability to the matrix
incorporating air into the material. band and retainer assembly.
B. Formation of saliva droplets on the
palate while the impression is setting. Which patient would NOT be
C. Poor mix of the stone. predisposed to liver toxicity following a
D. Too high a water:powder ratio of the dose of 1000mg of acetaminophen?
stone. *
E. Poor flow of the impression material. A. An adult with liver cirrhosis.
B. A chronic alcoholic.
What is the most likely cause of a C. A diabetic. *
maxillary denture dislodging when the D. A 15kg, 4 year old child.
patient opens wide or makes extreme
lateral excursions? Which of the following presents with
high serum calcium levels, thinning of
A. Insufficient posterior palatal seal. cortical bone and giant cell osteoclasts
B. Poor denture base adaptation. in the jaw and drifting teeth?
C. Labial frenum impingement.
D. Coronoid process interference*. A. Hyperthyroidism.
E. Pronounced midpalatal raphe. B. Hyperparathyroidism. *
C. Hypothyroidism.
Cold working a metal increases its D. Hypoparathyroidism.
A. ductility.
B. hardness. *

Dr Abdul Naser Tamim SEHA, Dr Ghada Al Aqqad D.D.S, Dr Kamal Naser - Amrita Medical Centre, Dr Emad Wani - Shadi Dental Centre,
Dr Lina Anka SEHA, Dr Rouba Zgheibi SEHA, Maria Teresa Yongson Alejandrino SEHA 136
Which of the following cells are Which of the following is NOT
characteristic of chronic inflammation associated with the administration of
of the dental pulp? acetylsalicylic acid?

1. Plasma cells. A. Tinnitus.


2. Macrophages. B. Analgesia.
3. Lymphocytes. C. Antipyresis.
4. Neutrophils. D. Constipation. *
E. Inhibition of prostaglandin synthesis.
A. (1) (2) (3) *
B. (1) and (3) The most common form of latex allergy
C. (2) and (4) is a/an
D. (4) only
E. All of the above. A. immediate localized (Type 1)
contact urticaria and erythema.
Which of the following dental B. immediate generalized (Type 1)
procedures could be performed with conjunctivitis and rhinitis.
minimal risk for a 35 year old patient C. immediate generalized (Type 1)
with a severe bleeding disorder? bronchospasm and anaphylaxis.
D. delayed (Type IV) contact
A. Mandibular block anesthesia. dermatitis. *
B. Supragingival calculus removal. *
C. Incisional biopsy. A Vitamin B2 (riboflavin) deficiency
D. Subgingival restoration. usually arises in patients

Which of the following structures may 1. Who are elderly.


be associated with the role of the 2. with acute infection.
central nervous system in sleep 3. Consuming a high protein or fat diet.
(nocturnal) bruxism? 4. Taking systemic antibiotics.

A. Basal ganglia (nigrostriatal). * A. (1) (2) (3)


B. A delta and C nerves. B. (1) and (3)
C. Sphenopalatine ganglion. C. (2) and (4)
D. Petrous nerves. D. (4) only
E. All of the above. *
A "butterfly-rash" of the face can
sometimes be found in Which of the following agents is most
effective in cold testing?
A. erythema multiforme.
B. lupus erythematosus. * A. Ice water.
C. pemphigus vulgaris. B. Air jet.
D. acne rosacea. C. CO2 (dry ice). *
D. Ethyl chloride.

Dr Abdul Naser Tamim SEHA, Dr Ghada Al Aqqad D.D.S, Dr Kamal Naser - Amrita Medical Centre, Dr Emad Wani - Shadi Dental Centre,
Dr Lina Anka SEHA, Dr Rouba Zgheibi SEHA, Maria Teresa Yongson Alejandrino SEHA 137
A patient has a prosthetic heart valve D. Free treatment for subjects is
and is allergic to penicillin. The most justified due to the time and
appropriate antibiotic to be used inconvenience.
prophylactically is
A new patient had an acute pulpitis.
A. erythromycin. Treatment included a pulpotomy and a
B. metronidazole. prescription for 30 tabs of Percocet®.
C. amoxicillin. Known to local police as a drug user,
D. tetracycline. the patient subsequently bragged to a
E. clindamycin. * police officer that it was very easy to
get narcotics from his new dentist.
A mandibular central incisor is to be Several weeks later, a police officer
extracted and added to a partial attends the office explaining that he is
denture. The impression to add a tooth conducting an investigation into health
to the denture should be made professionals complicit in narcotic
trafficking. Which of the following
A. before the extraction with the statements is correct?
denture in the mouth, removing the
denture from the impression then A. There is a legal duty to assist in any
pouring the impression. criminal investigation. *
B. after the extraction without the B. Patient files must be released to
denture in the mouth. police upon request.
C. after the extraction with the denture C. A subpoena is required for the
in the mouth and pouring the dentist to present relevant documents.
impression leaving the denture in the
impression. * Duraflor®
D. after the extraction with the denture
in the mouth, removing the denture A. is only effective on dry, plaque-free
from the impression then pouring the teeth.
impression. B. can remineralize early root carious
lesions. *
Dentists in a prosthetic study club have C. causes unsightly stain on exposed
been asked to recruit subjects for a roots.
clinical trial of a new dental implant D. should only be used on individuals
system. Dentists will receive a $500 in unfluoridated areas.
finder’s fee for each subject recruited.
Subjects will receive the treatment for Gingival response to plaque
free. Which of the following statements microorganisms in elderly patients is
is correct?
A. exaggerated due to an altered host
A. Subjects in this trial are protected by immune response. *
the clinical trial regulations of the Food B. exaggerated due to a change in the
and Drugs Act. type of infecting microorganisms.
B. Ethics review of clinical trials i is C. the same in all ages.
legislated. * D. decreased due to an altered host
C. The finder’s fee is justified because immune response.
of the time required to identify suitable
subjects.

Dr Abdul Naser Tamim SEHA, Dr Ghada Al Aqqad D.D.S, Dr Kamal Naser - Amrita Medical Centre, Dr Emad Wani - Shadi Dental Centre,
Dr Lina Anka SEHA, Dr Rouba Zgheibi SEHA, Maria Teresa Yongson Alejandrino SEHA 138
A chronic alcohol abuser has just been
diagnosed with Wernicke-Korsakoff’s A. C fibres. *
syndrome. He complains of pain and B. A delta fibres.
burning mouth and presents with C. sympathetic fibres.
angular cheilitis and glossitis. He is D. dentinal tubules.
suffering from a severe deficiency of
The mesial furcation of maxillary first
A vitamin A. molars is best probed from the
B. ascorbic acid.
C. vitamin K. A. buccal.
D. thiamin. * B. buccal or lingual.
C. lingual. *
Regarding the Dietary Reference
Intake (DRI) for calcium, Bone loss at menopause accelerates
because estrogens
A. adult men and women require the
same amount. * A. are necessary for calcium
B. more calcium is required during absorption from the small intestine.
pregnancy and lactation. B. inhibit bone resorbing cytokine
C. women over 70 years require more synthesis. *
than men over 70 years. C. inhibit osteoblast activity.
D. under the age of 18, boys require D. promote osteoclast proliferation and
more calcium than girls. differentiation.

Which of the following substances A child has received an inferior alveolar


causes inflammation and pain when nerve block using 1.5ml of lidocaine
released by pulpal fibres? 2% with 1:100,000 epinephrine and
shows signs that the block was
A. Prostaglandin E2. * effective. However, during placement
B. Calcitonin gene related peptide. of a rubber dam clamp on the
C. Neuraminase. permanent first molar, the child
D. Acetylcholine. complains that the “tooth ring” is
hurting. Which of the following is the
Low serum levels of parathyroid most appropriate management?
hormone and vitamin D combined with
low bone mass in the skeleton are A. Wait 15 minutes until more profound
consistent with the diagnosis of anesthesia is achieved.
B. Anesthetize the lingual nerve with
A. hypoparathyroidism. the remaining lidocaine.
B. hypothyroidism. C. Anesthetize the long buccal nerve
C. dietary calcium deficiency. with the remaining lidocaine. *
D. postmenopausal osteoporosis. * D. Proceed with treatment without
rubber dam.
Unmyelinated nerves found in the pulp
that respond to thermal, mechanical,
chemical and inflammatory stimuli are
called

Dr Abdul Naser Tamim SEHA, Dr Ghada Al Aqqad D.D.S, Dr Kamal Naser - Amrita Medical Centre, Dr Emad Wani - Shadi Dental Centre,
Dr Lina Anka SEHA, Dr Rouba Zgheibi SEHA, Maria Teresa Yongson Alejandrino SEHA 139
A 2½ year old child has intruded There is a difference between girls
maxillary central incisors which are and boys with respect to the age at
not in proximity to the permanent which the growth velocity reaches its
successors. What is the most peak.
appropriate management?
A. Boys are six months ahead of girls.
A. Reposition the teeth and splint. B. Girls are six months ahead of boys.
B. Reposition the teeth and do not C. Girls are one year ahead of boys.
splint. D. Girls are two years ahead of boys.
C. Monitor the teeth for re-eruption* *
D. Extract the teeth.
A laboratory remount of processed
In pediatric patients dentures is done in order to correct
occlusal disharmony produced by
A. asthma has a decreasing errors primarily in the
prevalence.
B. asthma is an acute inflammatory A. mounting of the casts on the
disorder. articulator.
C. asthma leads to increased caries. * B. registration of jaw relation records.
D. asthmatic attacks can be triggered C. processing of acrylic. *
by anxiety. D. setting of condylar guidance.

When compared to permanent teeth, Gigantism is caused by


primary teeth have
A. a hyperactive thyroid.
A. a greater thickness of enamel. B. atrophy of the posterior pituitary.
B. a greater thickness of dentin. C. hyperplasia of the anterior pituitary*
C. more prominent cervical D. hyperplasia of the parathyroids.
constriction. *
D. pulps which are smaller in relation In determining the ideal proximal
to crown size. outline form for a Class II amalgam
cavity preparation in a molar the
Which of the following procedures will
increase the retention of a Class III A. axial wall should be 1.5mm deep.
composite resin restoration in primary B. gingival cavosurface margin must
incisor and canine teeth? be placed supragingivally.
C. proximal walls diverge occlusally.
A. Keeping the preparation as small D. facial and lingual proximal
as possible. cavosurface margins must just clear
B. Using labial or lingual dovetails. * contact with the adjacent tooth. *
C. Etching the preparation for an
additional 30 seconds.

Dr Abdul Naser Tamim SEHA, Dr Ghada Al Aqqad D.D.S, Dr Kamal Naser - Amrita Medical Centre, Dr Emad Wani - Shadi Dental Centre,
Dr Lina Anka SEHA, Dr Rouba Zgheibi SEHA, Maria Teresa Yongson Alejandrino SEHA 140
In an Angle Class I occlusion, which contact with the central fossa of the
cusp of which permanent tooth moves mandibular first molar?
between the mesiolingual and
distolingual cusps of the mandibular A. Mesiolingual cusp of the maxillary
second molar in a working side first molar. *
movement? B. Distolingual cusp of the maxillary
first molar.
A. Mesiolingual cusp of the maxillary C. Mesiolingual cusp of the maxillary
first molar. second molar.
B. Distolingual cusp of the maxillary D. Distolingual cusp of the maxillary
first molar. second molar.
C. Mesiolingual cusp of the maxillary
second molar. * For a patient who exhibits bruxism, a
D. Distolingual cusp of the maxillary cast gold inlay is superior to an
second molar. amalgam because the inlay has

What is the name of the area in which A. better adaptation to the cavity walls.
the resin of the adhesive system B. greater thermal conductivity.
micromechanically interlocks with C. better ability to withstand
dentinal collagen? masticatory forces. *
D. easier placement into the cavity
A. Active zone. preparation.
B. Smear layer.
C. Hybrid layer. * Excessive flare of the distobuccal
D. Adhesive zone. cavosurface margin of a Class II
amalgam cavity preparation will result
Which of the following is NOT a in
consequence of vital bleaching with
10% carbamide peroxide? A. unsupported enamel at the margin.
B. weak amalgam at the margin. *
A. Reversible tooth sensitivity. C. poor retention.
B. Soft tissue sloughing. D. poor esthetics.
C. Sore throat.
D. Weakening of the enamel. * Adjustments made to Class II
amalgam preparations to ensure the
A dry and crumbly mix of amalgam tooth structure does not fracture under
can be the result of masticatory forces is a part of the

A. under trituration. * A. convenience form.


B. over trituration. B. retention form.
C. high copper content. C. outline form.
D. lack of zinc content. D. resistance form. *

In an Angle’s Class I occlusion, the


cusp of which permanent tooth is in

Dr Abdul Naser Tamim SEHA, Dr Ghada Al Aqqad D.D.S, Dr Kamal Naser - Amrita Medical Centre, Dr Emad Wani - Shadi Dental Centre,
Dr Lina Anka SEHA, Dr Rouba Zgheibi SEHA, Maria Teresa Yongson Alejandrino SEHA 141
During matrix band removal, the risk A. Adjust the restoration slightly out of
of marginal ridge fracture of an occlusion.
amalgam restoration is reduced by B. Replace the restoration with a
reinforced zinc oxide eugenol
A. completing most of the shaping of restoration.
the marginal ridge before removal. * C. Replace the restoration with a
B. leaving an excess of amalgam in bonded amalgam restoration.
the occlusal area before removal. D. Replace the restoration with a
C. contouring and wedging the band. bonded composite resin restoration. *
D. using universal circumferential
retainers and bands. An anterior endodontically treated
tooth has been restored with a carbon
The volumetric polymerization fibre, a direct restorative core and a
shrinkage of a hybrid composite resin porcelain fused to metal crown. What
is in the order of is the most important factor influencing
the prognosis of this tooth?
A. 0%.
A. Type of core material.
B. 0.1 – 1.0%.
B. Type of luting cement.
C. 2 – 8%. * C. Amount of remaining coronal tooth
D. 10 – 15%. structure. *
D. Alloy composition of the post.
The primary etiological factor for the
development of root caries is Which of the following restorations is
the most appropriate for a primary
A. gingival recession. * molar with 3 or more carious surfaces
B. acquired xerostomia. in a high caries-risk child?
C. poor oral hygiene.
D. cigarette smoking. A. Bonded amalgam.
B. Composite resin.
What is the best predictor of success C. Stainless steel crown. *
for a composite resin restoration? D. Resin modified glass ionomer
cement.
A. Depth of the restoration.
B. Size of the restoration. Recurrent caries
C. Presence of enamel on the entire
periphery. * A. is caused by microleakage at the
D. Presence of flat dentinal walls. restoration-tooth interface. *
B. is initiated primarily on the tooth
Which is the most appropriate surface beyond the restoration.
treatment for a patient who reports C. is caused by voids within the
persistent thermal sensitivity 4 weeks restoration entirety.
after placement of a posterior D. occurs in the pits of tooth structure.
composite resin restoration with
acceptable occlusion?

Dr Abdul Naser Tamim SEHA, Dr Ghada Al Aqqad D.D.S, Dr Kamal Naser - Amrita Medical Centre, Dr Emad Wani - Shadi Dental Centre,
Dr Lina Anka SEHA, Dr Rouba Zgheibi SEHA, Maria Teresa Yongson Alejandrino SEHA 142
What is the most likely cause of long B. cutting into the dentin without
term clinical failure for a metal-ceramic anesthetic. *
crown with porcelain occlusal coverage C. applying ethyl chloride to the crown.
on a molar? D. performing a radiographic
examination of the tooth.
A. Recurrent caries. * E. performing an electric pulp test.
B. Cohesive porcelain fracture.
C. Adhesive porcelain fracture. Which of the following conditions would
D. Loss of retention. NOT require antibiotic premedication
before endodontic therapy?
When using alginate impression
material, which one of the following A. Valvular heart disease.
statements is correct? B. Cardiac prosthesis.
C. Persistent odontogenic fistula. *
A. Store the impression in water at D. Immunosuppressive therapy.
37°C prior to pouring the cast. E. Organ transplant.
B. Remove the impression slowly from
the undercuts. A fracture in an all-ceramic crown may
C. Control the setting time by changing be caused by
the water/powder ratio.
D. Pour the cast immediately following 1. Inadequate ceramic thickness.
disinfection. * 2. Sharp line angles in the tooth
preparation.
The usual adult dosage of codeine 3. Excessive occlusal load.
administered orally is 4. Use of an inappropriate luting
material.
A. 500-1000mg.
B. 250-500mg. A. (1) (2) (3)
C. 30-60mg. * B. (1) and (3)
D. 2-5mg. C. (2) and (4)
D. (4) only
A radiopaque area within the alveolar E. All of the above. *
process containing several rudimentary
teeth suggests a/an Generally, glass ionomer cements
contain
A. periapical cemento-osseous
dysplasia. A. zinc oxide and distilled water.
B. ameloblastoma. B. zinc oxide and polyacrylic acid.
C. compound odontoma. * C. fluoroaluminosilicate powder and
D. complex odontoma. orthophosphoric acid.
E. Pindborg tumor. D. fluoroaluminosilicate powder and
polyacrylic acid. *
Under normal conditions, the most
definitive test to confirm the loss of
pulp vitality is

A. applying warm gutta percha to the


crown.

A known insulin-dependent diabetic patient feels unwell following the

Dr Abdul Naser Tamim SEHA, Dr Ghada Al Aqqad D.D.S, Dr Kamal Naser - Amrita Medical Centre, Dr Emad Wani - Shadi Dental Centre,
Dr Lina Anka SEHA, Dr Rouba Zgheibi SEHA, Maria Teresa Yongson Alejandrino SEHA 143
administration of a local anesthetic A. as soon as the tooth erupts through
and becomes pale and sweaty. This the gingival tissue.
condition does not respond to placing B. after the permanent second molar
the patient in a supine position. The has erupted.
most likely cause is C. immediately after extraction of the
primary second molar *
A. syncope. D. as soon as the extraction site of the
B. adrenal insufficiency. primary second molar has completely
C. hyperglycemia. healed.
D. hypoglycemia. *
E. carotid sinus reflex. An 8 year old patient with all primary
molars still present exhibits a cusp-to-
Epinephrine should NOT be used as cusp relationship of permanent
a vasoconstrictor for patients with maxillary and mandibular first molars.
uncontrolled The management of this patient
should be to
A. hyperthyroidism. *
B. hyperparathyroidism. A. plan serial extractions for more
C. myxedema. normal adjustment of the occlusion.
D. asthma. B. refer the patient to an orthodontist
for consultation.
Immediately following a posterior C. place a cervical headgear to
superior alveolar block injection, the reposition maxillary molars.
patient's face becomes quickly and D. disk the distal surfaces of primary
visibly swollen. The immediate mandibular second molars to allow
treatment should be to normal adjustment of permanent
molars.
E. observe *
A. use pressure followed by cold
packs over the swelling * Which of the following will impede
B. use hot packs over the swelling. healing following the surgical closure
of an oroantral fistula?
C. refer the patient to a hospital.
D. administer 100mg hydrocortisone
1. Poor flap design.
intravenously.
2. Excessive tissue tension.
E. administer diphenhydramine
hydrochloride (Benadryl®) 50mg 3. Blowing the nose.
intravenously. 4. Sinus infection.

To prevent mesial drift of a A. (1) (2) (3)


permanent first molar, the ideal time B. (1) and (3)
to place a distal extension space C. (2) and (4)
maintainer is D. (4) only
E. All of the above. *

Dr Abdul Naser Tamim SEHA, Dr Ghada Al Aqqad D.D.S, Dr Kamal Naser - Amrita Medical Centre, Dr Emad Wani - Shadi Dental Centre,
Dr Lina Anka SEHA, Dr Rouba Zgheibi SEHA, Maria Teresa Yongson Alejandrino SEHA 192
A Le Fort I or Guerin fracture is a B. once the patient has reached the
age of 12.
A. fracture of the zygomatic arch C. only if it develops into a cystic
B. horizontal fracture of the maxilla * lesion.
C. fracture of the malar complex D. as soon as possible *
involving the floor of the orbit
D. pyramidal fracture of the maxilla A 12 year old child presents with
E. craniofacial dysjunction characteristic tetracycline discoloration
of the maxillary and mandibular
The most appropriate treatment incisors and permanent first molars.
following the extraction of a first The probable age at which this child
primary molar in a 4 year old child is received tetracycline therapy was

A. regular assessment of arch A. 6 years.


development B. 4 years.
B. to perform space analysis C. 1 year. *
C. insertion of a space maintainer* D. before birth.
D. extraction of the contra-lateral molar
E. extraction of the opposing molar One week after an amalgam
restoration is placed in the mandibular
A large carious exposure occurs on a first premolar, the patient returns
permanent first molar of a 7 year old. complaining of a sharp pain of short
There is no periapical involvement and duration when eating or drinking
the tooth is vital. The treatment should something cold. Teeth respond
be to normally to electric pulp testing and
heat and the radiographs are normal.
The most likely diagnosis is
A. cap the exposure with calcium
hydroxide and place zinc-oxide and
eugenol. A. hypercementosis
B. perform a pulpotomy and place B. reversible pulpitis *
calcium hydroxide * C. pulpal microabscess
C. perform a pulpectomy. D. acute periradicular periodontitis
D. extract the tooth and place a space
maintainer. The most appropriate radiographic
examination for a 4 year old without
A 6 year old patient has a larger than visible or clinically detectable caries or
average diastema between the anomalies, and with open proximal
maxillary central incisors. The contacts is
radiographic examination shows a
mesiodens. In order to manage the A. maxillary and mandibular anterior
diastema, you should extract the occlusals.
mesiodens B. a pair of posterior bite-wings.
C. maxillary and mandibular posterior
A. after its complete eruption. periapicals.
D. no radiographic examination *

Dr Abdul Naser Tamim SEHA, Dr Ghada Al Aqqad D.D.S, Dr Kamal Naser - Amrita Medical Centre, Dr Emad Wani - Shadi Dental Centre,
Dr Lina Anka SEHA, Dr Rouba Zgheibi SEHA, Maria Teresa Yongson Alejandrino SEHA 193
A cold stimulus applied to a tooth will likely to be misdiagnosed as
produce a hypersensitive response if proximal caries?
the tooth
A. Cemento-enamel junction. *
A. is nonvital. B. Marginal ridge.
B. has a periodontal pocket. C. Carabelli cusp.
C. has a hyperemic pulp. * D. Calculus.
D. has chronic proliferative pulpitis. E. Cemental tear.

Which drug is most adversely affected Radiographically, the opening of the


by ingestion of antacids? incisive canal may be misdiagnosed
as a
A. Cephalexin.
B. Erythromycin. 1. Branchial cyst.
C. Tetracycline.* 2. Nasopalatine cyst.
D. Penicillin V. 3. Nasolabial cyst.
4. Periradicular cyst.
A patient complains of acute pain 24
hours after the insertion of a A. (1) (2) (3)
restoration in a tooth with no B. (1) and (3)
preexisting periapical pathology. The C. (2) and (4) *
tooth is vital and tender to percussion. D. (4) only
The radiograph will show
E. All of the above.
A. an apical radiolucency. An ameloblastoma can develop from
B. acute osteitis. the epithelial lining of which of the
C. root resorption. following cysts?
D. condensing osteitis.
E. normal lamina dura. * A. Periradicular.
B. Dentigerous *
A well circumscribed 3mm radiolucent C. Residual.
lesion is present in the apical region of D. Lateral periodontal.
the mandibular second premolar. The
tooth responds normally to vitality The microscopic appearance of the
tests. The radiolucency is most likely central giant cell granuloma of the
jaws is similar to that of lesions
A. a periradicular periodontitis. which occur in
B. a dentigerous cyst.
C. a rarefying osteitis. A. hyperparathyroidism *
D. the mental foramen * B. Paget's disease.
C. Cleidocranial Dysplasia.
On a bite-wing radiograph of posterior D. Hyperpituitarism.
teeth, which of the following is most

Dr Abdul Naser Tamim SEHA, Dr Ghada Al Aqqad D.D.S, Dr Kamal Naser - Amrita Medical Centre, Dr Emad Wani - Shadi Dental Centre,
Dr Lina Anka SEHA, Dr Rouba Zgheibi SEHA, Maria Teresa Yongson Alejandrino SEHA 194
For which of the following C. Chronic periradicular periodontitis*
pathological conditions would a lower D. Pulp polyp.
central incisor tooth be expected to
respond to heat, cold and electric Root resorption of permanent teeth
pulp test? may be associated with

A. Apical cyst. 1. Excessive orthodontic forces.


B. Acute apical abscess. 2. Chronic periradicular periodontitis.
C. Periapical cemento-osseous 3. Traumatic injury.
dysplasia. * 4. Periapical cemento-osseous
D. Chronic apical periodontitis. dysplasia.

An ankylosed tooth is usually A. (1) (2) (3) *


B. (1) and (3)
A. nonvital. C. (2) and (4)
B. associated with a root fracture. D. (4) only
C. infraerupted * E. All of the above.
D. found in the permanent dentition.
A 4 year old child has a normal
Which of the following is/are complement of deciduous teeth, but in
associated with an unerupted tooth? appearance they are grayish and
exhibit extensive occlusal and incisal
1. Odontogenic adenomatoid tumor. wear. Radiographic examination
2. Periapical cemento-osseous indicates some extensive deposits of
dysplasia. secondary dentin in these teeth. This
3. Calcifying epithelial odontogenic condition is typical of
tumor.
4. Cementoblastoma. A. cleidocranial dysplasia.
B. amelogenesis imperfecta.
A. (1) (2) (3) C. neonatal hypoplasia.
B. (1) and (3) * D. dentinogenesis imperfect *
C. (2) and (4)
D. (4) only Which of the following features would
E. All of the above. be most indicative of a cracked tooth?

Which of the following results from a A. Periapical radiolucency.


necrotic pulp? B. Hypersensitivity to thermal stimuli.
C. Pain upon biting pressure. *
A. Dentigerous cyst. D. Absent vitalometric response.
B. Lateral periodontal cyst.

Dr Abdul Naser Tamim SEHA, Dr Ghada Al Aqqad D.D.S, Dr Kamal Naser - Amrita Medical Centre, Dr Emad Wani - Shadi Dental Centre,
Dr Lina Anka SEHA, Dr Rouba Zgheibi SEHA, Maria Teresa Yongson Alejandrino SEHA 195
A 15 year old presents with C. increased vascularity of the jaws.
hypoplastic enamel on tooth 1.5. All D. increased brittleness of the jaws.
other teeth are normal. This was most
probably caused by a/an If an alginate impression must be
stored for a few minutes before the
A. vitamin D deficiency. cast is poured, it should be placed in
B. generalized calcium deficiency.
C. high fever encountered by the A. water
patient when he had measles at age 3. B. 100% relative humidity. *
D. infection of tooth 5.5 during the C. a 1% aqueous calcium sulfate
development of tooth 1.5 * solution.
E. hereditary factor.
When a radiographic examination is
Condensing osteitis in the periapical warranted for a 10 year old child, the
region is indicative of a/an most effective way to decrease
radiation exposure is to
A. acute inflammation of the pulp.
B. pulpal abscess. A. use a thyroid collar and lead apron*
C. chronic inflammation of the pulp * B. apply a radiation protection badge.
D. early apical abscess formation. C. use high speed film.
D. decrease the kilovoltage to 50kVp.
Myxedema is associated with E. take a panoramic film only.

A. insufficient parathyroid hormone. Which of the following drugs is used in


B. excessive parathyroid hormone. the treatment of mild allergic
C. insufficient thyroid hormone * reactions?
D. excessive thyroid hormone.
A. Isoproterenol.
Which of the following is most often B. Meperidine hydrochloride.
associated with a nonvital tooth? C. Diphenhydramine hydrochloride. *
D. Propoxyphene.
A. Chronic periradicular periodontitis*
B. Internal resorption. A protective mechanism of the dental
C. Periapical cemento-osseous pulp to external irritation or caries is
dysplasia. the formation of
D. Hyperplastic pulpitis.
A. pulp stones.
An end result of ionizing radiation used B. tertiary dentin *
to treat oral malignancies is C. secondary cementum.
D. primary dentin.
A. deformity of the jaws.
B. reduced vascularity of the jaws. *

Dr Abdul Naser Tamim SEHA, Dr Ghada Al Aqqad D.D.S, Dr Kamal Naser - Amrita Medical Centre, Dr Emad Wani - Shadi Dental Centre,
Dr Lina Anka SEHA, Dr Rouba Zgheibi SEHA, Maria Teresa Yongson Alejandrino SEHA 196
Procaine (Novocaine®) is an example
of a local anesthetic which is The most likely diagnosis for a child
chemically classified as an with a painful, fiery-red, diffuse
A. amide. gingivitis is
B. ester. * A. primary herpetic gingivostomatitis*
C. aldehyde. B. aggressive periodontitis.
D. ethamine. C. idiopathic fibromatosis.
E. aminide. D. aphthous stomatitis.

Lidocaine (Xylocaine®) is an example In an infrabony pocket, the epithelial


of a local anesthetic which is attachment is located
chemically classified as an A. within basal bone.
A. amide. * B. coronal to alveolar bone crest.
B. ester. C. apical to alveolar bone crest *
C. aldehyde.
D. ethamine. The location and extent of subgingival
E. aminide. calculus is most accurately determined
clinically by
Regarding dental caries, which of the A. radiopaque solution used in
following is correct? conjunction with radiographs.
A. All carbohydrates are equally B. disclosing solution.
cariogenic. C. probing with a fine instrument *
B. More frequent consumption of D. visual inspection.
carbohydrates increases the risk. *
C. The rate of carbohydrate clearance What is the most likely diagnosis of a
from the oral cavity is not significant. white, diffuse, wrinkled appearing
D. Increased dietary fat increases the lesion of the buccal mucosa which
risk. diminishes in prominence or
disappears upon stretching?
Which of the following is/are clinical A. Leukoedema. *
signs of gingivitis? B. Lichen planus.
1. Loss of stippling. C. Candidiasis.
2. Gingival hyperplasia. D. Linea alba.
3. Decreased pocket depth. E. White sponge nevus.
4. Bleeding on probing.
Aging pulps show a relative increase
A. (1) (2) (3) in
B. (1) and (3) A. sensitivity.
C. (2) and (4) * B. cell numbers.
D. (4) only C. calcification *
E. All of the above. D. vascularity.

Dr Abdul Naser Tamim SEHA, Dr Ghada Al Aqqad D.D.S, Dr Kamal Naser - Amrita Medical Centre, Dr Emad Wani - Shadi Dental Centre,
Dr Lina Anka SEHA, Dr Rouba Zgheibi SEHA, Maria Teresa Yongson Alejandrino SEHA 197
A patient complains of sensitivity E. Exposure to air or light shortens
following placement of a conservative the shelf-life of the tablets.
posterior composite resin restoration.
The most probable cause is The local anesthetic technique
A. acid etching. requiring the needle to contact the
B. polymerization shrinkage * neck of the condyle is the
C. unpolymerized resin. A. posterior superior alveolar nerve
D. prolonged application of the curing block.
light. B. Gow-Gates block. *
E. inadequate base thickness. C. Vazirani-Akinosi block.
D. inferior alveolar nerve block.
The antibiotic of choice for a
periradicular dental abscess is What is the most effective local
A. penicillin V. * anesthetic technique for a patient with
B. cephalosporin. trismus who requires a pulpectomy on
C. erythromycin. a mandibular molar?
D. metronidazole. A. Mental nerve block.
E. ampicillin. B. Gow-Gates block.
C. Vazirani-Akinosi block. *
Which of the following does NOT block D. Inferior alveolar nerve block.
cyclooxygenase-2? E. Buccal nerve block.
A. Acetaminophen. *
B. Acetylsalicylic acid. Hypoglycemia in the conscious
C. Celecoxib. patient is most appropriately managed
D. Diflunisal. with
A. oxygen.
E. Ibuprofen.
B. epinephrine.
Which of the following is NOT correct C. carbohydrates *
regarding nitroglycerin used for D. glucagon.
medical emergencies? E. insulin.
A. It is indicated for the definitive
management of angina pectoris. In addition to oxygen, the drug of
B. It is indicated for the early choice for the management of angina
management of a myocardial is
infarction. A. epinephrine.
C. It should be administered B. acetylsalicylic acid.
sublingually. C. diphenhydramine.
D. Its administration will increase blood D. nitroglycerin *
pressure. * E. atropine.

Dr Abdul Naser Tamim SEHA, Dr Ghada Al Aqqad D.D.S, Dr Kamal Naser - Amrita Medical Centre, Dr Emad Wani - Shadi Dental Centre,
Dr Lina Anka SEHA, Dr Rouba Zgheibi SEHA, Maria Teresa Yongson Alejandrino SEHA 198
Which of the following tissues is A. premalignant adenomatous
LEAST sensitive to ionizing radiation? polyposis coli.
A. Enamel. * B. hamartomatous polyps of the small
B. Oral mucosa. intestine.
C. Salivary gland. C. congenitally missing first premolars.
D. Bone. D. multiple palmar or facial basal cell
carcinomas.
The most effective way of minimizing a E. neoplasms of endocrine organs *
patient’s radiation dose is
A. a lead apron and thyroid collar. Which of the following lesions of the
B. prescription radiography * oral or maxillofacial region is
C. fast emulsion film. premalignant?
D. intensifying screens. A. Ectodermal dysplasia.
B. Focal fibrous hyperplasia.
Which of the following is NOT C. Smoker’s melanosis.
commonly used by general dentists to D. Fibrous dysplasia.
treat temporomandibular disorders and E. Epithelial dysplasia.*
bruxism?
A. Hard acrylic splints. The risk of contracting hepatitis B
B. Nonsteroidal anti-inflammatory following a percutaneous injury from
drugs (NSAIDs). an occupational exposure to the
C. Corticosteroids. * contaminated blood of a patient has
D. Muscle relaxants. been shown to be in the range of
A. 6-30%. *
The presence of more than 104 B. 35-66%.
copies/mL of hepatitis B DNA in blood C. 72-90%.
is indicative of
A. a past exposure to hepatitis B with A major difference between stable
immunity. angina pectoris and myocardial
B. recovery from hepatitis B with liver infarction is that stable angina pectoris
damage. does NOT involve
C. a highly infectious individual * A. crushing substernal pain.
D. an inconclusive immune status. B. ischemic myocardial necrosis. *
C. occlusive coronary artery disease.
A patient diagnosed with multiple D. atherosclerosis.
mucosal neuromas should be further E. females under the age of 80 years.
evaluated for the possibility of

Dr Abdul Naser Tamim SEHA, Dr Ghada Al Aqqad D.D.S, Dr Kamal Naser - Amrita Medical Centre, Dr Emad Wani - Shadi Dental Centre,
Dr Lina Anka SEHA, Dr Rouba Zgheibi SEHA, Maria Teresa Yongson Alejandrino SEHA 199
Which part of the brain is NOT primarily
involved in motor control? Most of the somatosensory information from
A. Cerebellum. the oral cavity reaches the brain through which
B. Basal ganglia. nerve?
C. Occipital lobe. * A. Trochlear.
D. Frontal lobe. B. Hypoglossal.
C. Trigeminal. *
Veillonella species in supragingival plaque D. Glossopharyngeal.
A. act symbiotically with S. mutans to decrease
pH and promote caries activity. Which microorganism does NOT contribute
B. convert lactate to acetic and propionic acid. * significantly to the progression of dentinal
C. enhance the progression of caries by caries?
metabolizing sucrose. A. Actinomyces naeslundii.
D. act as pioneer microorganisms in the B. Lactobacillus casei.
development of plaque. C. Actinomyces viscosus.
D. Streptococcus salivarius. *
A functional parathyroid adenoma would most
likely result in The earliest colonizers of dental plaque are
A. generalized loss of lamina dura * A. Gram-positive rods.
B. nervousness, exophthalmos and heat B. Gram-positive cocci. *
intolerance. C. Gram-negative rods.
C. an increased caries rate. D. Gram-negative cocci.
D. multiple recurrent periodontal abscesses.
Changes in direction of groups of enamel
The speed of propagation of action potentials prisms
along axons is A. have no functional importance.
A. faster for myelinated fibres than for B. result from incremental growth.
unmyelinated fibres. * C. make enamel resistant to fracture. *
B. slower for myelinated fibres than for D. extend to the enamel surface.
unmyelinated fibres.
C. independent of myelination.
D. independent of the axon diameter.

Astrocytes are cells found in the


A. brain *
B. peripheral nerves.
C. tongue.
D. blood.

Which of the following conditions


CONTRAINDICATES treatment with dental
implants?
A. Uncontrolled diabetes mellitus.*

Dr Abdul Naser Tamim SEHA, Dr Ghada Al Aqqad D.D.S, Dr Kamal Naser - Amrita Medical Centre, Dr Emad Wani - Shadi Dental Centre,
Dr Lina Anka SEHA, Dr Rouba Zgheibi SEHA, Maria Teresa Yongson Alejandrino SEHA 200
B. Osteoporosis.
C. Anticoagulant therapy.
D. Age greater than 80 years.

Most of the problems associated with direct


Postoperative sensitivity associated with the posterior composite resin restorations are
placement of direct posterior composite resin related to
restorations is most often the result of
A. high thermal conductivity.
A. polymerization shrinkage. * B. galvanic conduction.
B. large particle size of macrofilled composite C. polymerization shrinkage. *
resins. D. poor shade selection.
C. inadequate etching of the dentin.
D. excessive thickness of a glass ionomer
base.

In the formulation of composite resins:

A. decreasing filler content increases the


modulus of elasticity.
B. increasing filler content decreases
polymerization shrinkage. *
C. increasing filler content increases the degree
of conversion.
D. decreasing filler content increases
radiopacity of the restoration.

Which of the following statements is correct


with respect to zinc containing amalgams? Conventional glass ionomer cements

A. High copper amalgam restorations A. elicit less pulp response than zinc-oxide and
containing zinc demonstrate better overall eugenol cements.
survival rates. * B. do not require a protective liner, such as
B. Amalgams containing zinc should be used calcium hydroxide in a deep preparation.
when contamination with moisture is C. have a lower modulus of elasticity than zinc
unavoidable during condensation. phosphate cements. *
C. Zinc is added during the manufacturing D. bond mechanically to calcium in enamel and
operation to increase the solubility of tin in dentin.
silver. E. are superior to zinc phosphate cement for
D. Amalgams containing zinc produce a luting porcelain (all ceramic) crowns.
significantly better seal than zinc-free
amalgams.

An amalgam coronal-radicular core build-up for endodontically treated

Dr Abdul Naser Tamim SEHA, Dr Ghada Al Aqqad D.D.S, Dr Kamal Naser - Amrita Medical Centre, Dr Emad Wani - Shadi Dental Centre,
Dr Lina Anka SEHA, Dr Rouba Zgheibi SEHA, Maria Teresa Yongson Alejandrino SEHA 201
molar teeth requires A. occlusal extension is wide
faciolingually relative to the proximal
A. an adequate pulp chamber and extension.
ferrule. * B. restoration is a pin-retained cusp
B. a pulp chamber, ferrule and replacement.
amalgam bonding. C. occlusal extension is narrow
C. the presence of a post. faciolingually relative to the proximal
D. the use of retentive threaded pins. extension. *
D. bonded amalgam technique is not
The technique of amalgam bonding being used.
can be
What is the most likely cause of food
A. used as the primary means of impaction at the site of a recently
retaining an amalgam restoration. placed Class II composite resin
B. relied upon to reinforce severely restoration?
weakened cusps.
C. effective in providing an improved A. Inadequate proximal contact. *
initial seal. * B. Gingival overhang.
D. used in conjunction with copal C. Inadequate marginal ridge
varnish. morphology.
D. Poor oral hygiene.
The occlusal cavosurface margin for a
Class I amalgam restoration should be Isthmus fracture during function in a
recently placed proximal-occlusal silver
A. beveled. amalgam restoration (with occlusal
B. 90°. * extension through the occlusal fissure
C. chamfered. system), is most likely due to a
D. acute. preparation with

The placement of a reverse curve in a A. inadequate isthmus depth.*


Class II amalgam preparation aids in B. inadequate isthmus width.
C. a stepped buccal or lingual wall.
A. retention form. D. subgingival proximal extension.
B. resistance form. *
C. convenience form. The most important advantage of using
D. outline form. reinforced zinc-oxide eugenol cement
as a temporary restoration is that it

Proximal retention grooves are most A. stimulates dentin repair.


necessary to provide resistance for B. occludes dentinal tubules.
proximal-occlusal silver amalgam C. desensitizes the pulp. *
restorations when the D. chelates to tooth structure.

Dr Abdul Naser Tamim SEHA, Dr Ghada Al Aqqad D.D.S, Dr Kamal Naser - Amrita Medical Centre, Dr Emad Wani - Shadi Dental Centre,
Dr Lina Anka SEHA, Dr Rouba Zgheibi SEHA, Maria Teresa Yongson Alejandrino SEHA 202
Which of the following is consistent D. 12 months.
with the features of an arrested carious
lesion in an occlusal fissure? Clinical management of gingival
enlargement caused by Phenytoin
A. A small cavitated lesion with (Dilantin®) therapy includes
exposed dentin.
B. A white spot lesion with a frosty A. the use of analgesics to control pain.
surface. B. withdrawal of the medication.
C. A lesion extending into dentin on a C. extraction of the teeth.
bite-wing radiograph. D. gingivectomy and maintenance of
D. A brown spot lesion with a hard good oral hygiene. *
surface. * E. a mouth guard to control mouth
breathing.
Composite resin is a satisfactory core
material for endodontically treated Papillary hyperplasia under a denture is
teeth provided usually due to

A. the resin has a high contrast colour A. a candida infection.


with tooth structure. B. an ill-fitting denture. *
B. there is an adequate ferrule.* C. failure to remove the denture at night.
C. the resin is auto polymerizing. D. an allergy to the denture material.
D. subsequent crown margins are not
located on cementum. Periapical infection from a mandibular
second molar may spread by direct
Which of the following drugs has the extension to the
strongest antisialagogue properties?
1. Buccal space.
A. Codeine. 2. Buccal vestibule.
B. Atropine. * 3. Sublingual space.
C. Acetylsalicylic acid. 4. Submandibular space.
D. Ibuprofen.
E. Penicillin. A. (1) (2) (3)
B. (1) and (3)
Calcification of the permanent first C. (2) and (4)
molars normally begins at D. (4) only
E. All of the above. *
A. 6 months in utero.
B. birth. *
C. 6 months.

Dr Abdul Naser Tamim SEHA, Dr Ghada Al Aqqad D.D.S, Dr Kamal Naser - Amrita Medical Centre, Dr Emad Wani - Shadi Dental Centre,
Dr Lina Anka SEHA, Dr Rouba Zgheibi SEHA, Maria Teresa Yongson Alejandrino SEHA 203
Which of the following is/are true Using a high speed dental handpiece
statements about incision and WITHOUT water coolant will
drainage of an acute apical abscess?
A. produce a smoother surface.
1. A rubber dam drain may be placed B. decrease pulpal damage if used
and sutured to assist drainage. with light pressure.
2. The procedure is only indicated with C. reduce clogging of dental bur.
a localized, fluctuant swelling. D. reduce debris accumulation.
3. Profound anesthesia of the surgical E. increase frictional heat. *
site is not always possible.
4. Relief of the pressure and pain is A sign of gingivitis is
immediate after treatment.
A. bone loss.
A. (1) (2) (3) B. progressive attachment loss.
B. (1) and (3) C. the presence of minimal attached
C. (2) and (4) gingiva.
D. (4) only D. bleeding on probing. *
E. All of the above. *
The re-evaluation of periodontal
Adrenal corticosteroids debridement effectiveness after 4 to 6
weeks is best assessed by examining
A. increase heart rate. clinical attachment levels and
B. cause vasodilation.
C. increase protein synthesis. A. radiographic bone density.
D. reduce inflammation. * B. plaque index.
C. bleeding index. *
In a tooth with complete pulpal D. mobility.
necrosis, the periradicular region is
affected if Following periodontal debridement,
reduction in pocket depth is primarily
A. there is pain to thermal stimuli. due to
B. the tooth throbs when the patient is
lying down. A. decreased inflammation. *
C. there is pain on percussion. * B. reattachment of gingival fibers.
D. there is no response to an electric C. epithelial "adhesion" to the tooth.
pulp test. D. connective tissue regeneration.

Dr Abdul Naser Tamim SEHA, Dr Ghada Al Aqqad D.D.S, Dr Kamal Naser - Amrita Medical Centre, Dr Emad Wani - Shadi Dental Centre,
Dr Lina Anka SEHA, Dr Rouba Zgheibi SEHA, Maria Teresa Yongson Alejandrino SEHA 204
In a patient with an adequate band of B. T cells. *
keratinized tissue, gingivectomy is C. B cells.
indicated for all EXCEPT D. plasma cells.

A. gingival pockets. In gingivitis, the initial cellular immune


B. suprabony pockets. response predominantly involves
C. gingival overgrowths.
D. infrabony pockets. * A. T lymphocytes.
B. B lymphocytes.
During the administration of local C. neutrophils. *
anesthesia, an intravascular injection D. plasma cells.
will occur most often in a/an
Generalized enlarged fibrotic
A. incisive block. interdental papillae of 2 months
B. posterior superior alveolar block. duration are interfering with orthodontic
C. inferior alveolar block. * treatment. There is no evidence of
D. anterior superior alveolar block. attachment loss. Scaling and root
E. long buccal block. planning have not resolved the
condition. Which of the following is the
Compared to nonsmokers, cigarette most appropriate intervention?
smokers are more likely to have
A. Gingival curettage.
A. less bone loss. B. Gingivoplasty. *
B. less gingival recession. C. Osseous resective surgery.
C. more bleeding on probing. D. Guided tissue regeneration.
D. more attachment loss. *
Tooth 3.3 has a 9mm probing depth
Fremitus is with a 6mm three-wall infrabony mesial
defect. It tests vital and is not mobile.
A. tooth mobility of grade II. Which of the following is the most
B. mobility during occlusion. * appropriate treatment?
C. vertical tooth mobility.
D. tooth pain upon percussion. A. Gingival curettage.
B. Modified Widman flap.
The cell-mediated immune response C. Osseous resective surgery.
to bacterial plaque in chronic D. Guided tissue regeneration. *
periodontitis is regulated by

A. neutrophils.

Dr Abdul Naser Tamim SEHA, Dr Ghada Al Aqqad D.D.S, Dr Kamal Naser - Amrita Medical Centre, Dr Emad Wani - Shadi Dental Centre,
Dr Lina Anka SEHA, Dr Rouba Zgheibi SEHA, Maria Teresa Yongson Alejandrino SEHA 205
The predominant immunoglobulin 9:00 a.m. for the extraction of two teeth
isolated from saliva is under local anesthesia?

A. IgG. A. Eat breakfast before the


B. IgM. appointment and delay insulin injection
C. IgA. * until after the appointment.
D. IgD. B. Take insulin before the appointment
and delay breakfast until after the
In the normal periodontium, the extractions.
alveolar crest lies C. Eat breakfast before the
appointment and take insulin according
A. 1 to 2mm coronal to the CEJ. to the regime prescribed by the
B. at the CEJ. physician. *
C. 1 to 2mm apical to the CEJ.* D. Delay breakfast and insulin until
after the dental appointment.
Subgingival plaque in deep periodontal
pockets consists primarily of When placed into sound dentin, a self-
threading pin will
A. gram-positive microorganisms.
B. aerobic microorganisms. A. increase tensile strength of an
C. gram-negative microorganisms. * amalgam restoration.
D. viruses. B. strengthen the amalgam.
C. cause pulpal inflammation.
The microorganisms that initially D. increase the retention of an
colonize the tooth surface are amalgam restoration. *
associated with the genus
The pulpal floor of an occlusal
A. Porphyromonas. amalgam preparation on a mandibular
B. Actinobacillus. first premolar should slope apically from
C. Streptococcus. *
D. Prevotella. A. mesial to distal.
B. buccal to lingual. *
Which of the following preoperative C. distal to mesial.
instructions are most appropriate for a D. lingual to buccal.
well-controlled insulin-dependent
diabetic patient who is scheduled at

Dr Abdul Naser Tamim SEHA, Dr Ghada Al Aqqad D.D.S, Dr Kamal Naser - Amrita Medical Centre, Dr Emad Wani - Shadi Dental Centre,
Dr Lina Anka SEHA, Dr Rouba Zgheibi SEHA, Maria Teresa Yongson Alejandrino SEHA 206
Which of the following requires antibiotic
prophylaxis for a patient with a A. maxillary central incisor.
prosthetic heart valve? B. maxillary lateral incisor. *
C. mandibular central incisor.
A. Inferior alveolar nerve block. D. mandibular lateral incisor.
B. Endodontic instrumentation beyond
apex. * Accessory canals in permanent teeth
C. Restoration of occlusal caries. are most commonly found in the
D. Making an alginate impression.
A. cervical third of the root.
Aspiration prior to a local anesthetic B. middle third of the root.
injection reduces the C. apical third of the root. *

A. toxicity of local anesthetic. When a second canal is located in


B. toxicity of vasoconstrictor. mandibular incisors, it is most
C. possibility of intravascular frequently found
administration. *
D. possibility of paresthesia. A. labial to the main canal.
B. lingual to the main canal. *
An advantage of a metal-ceramic crown C. mesial to the main canal.
over an all-ceramic crown is D. distal to the main canal.

A. superior esthetics. Which statement is true regarding


B. more conservative vestibular electric pulp testing?
preparation.
C. increased restoration strength. * A. It does not confirm health or
D. superior translucency. integrity of the pulp. *
B. Vascularity can be determined.
There is an acute periradiuclar abscess C. True vitality can be determined.
on tooth 1.3. The tooth must be D. Thicker enamel will lead to a
extracted. In addition to a palatal quicker response.
injection, the most appropriate local
anesthetic technique would be The last bone in the craniofacial
complex to stop growing is the
A. vestibular infiltration.
B. infraorbital. * A. frontal bone.
C. middle superior alveolar. B. nasal bone.
D. intraligamentary. C. maxilla.
D. mandible. *
The permanent anterior tooth that
exhibits the greatest variation in size
and shape is the

Dr Abdul Naser Tamim SEHA, Dr Ghada Al Aqqad D.D.S, Dr Kamal Naser - Amrita Medical Centre, Dr Emad Wani - Shadi Dental Centre,
Dr Lina Anka SEHA, Dr Rouba Zgheibi SEHA, Maria Teresa Yongson Alejandrino SEHA 207
A patient, when in full intercuspation,
shows a right side posterior crossbite A. Class II, division 2 dental
and a lower midline that is deviated to malocclusion.
the right. At initial contact there are B. Class III skeletal malocclusion.
bilateral posterior crossbites and C. accentuated maxillary curve of
coincident midlines. The most likely Wilson. *
cause of this finding is D. reverse maxillary curve of Wilson.

A. severe temporomandibular Which statement is FALSE regarding


dysfunction. sodium hypochlorite when used as an
B. Two ideal occlusions. irrigant?
C. true unilateral crossbite.
D. occlusal interference and functional A. It is used in 0.5% to 5.25%
shift. * concentration.
B. Its combination with hydrogen
A clinical diagnostic indication of peroxide does not improve its clinical
palatal impaction of maxillary effectiveness.
permanent canines does NOT include C. It is well tolerated if expressed
beyond the tooth apex. *
A. proclined and laterally flared D. Warming the solution increases its
permanent lateral incisors. clinical effectiveness.
B. delayed exfoliation of primary
canines. The smear layer created by root canal
C. midline central diastema. * instrumentation can be removed by
D. lack of canine buccal bulges in a 10
year old patient. A. hydrogen peroxide and ethyl
chloride.
Which tooth movement is NOT B. sodium hypochlorite and EDTA. *
possible with a removable appliance? C. chlorhexidine and chloroform.
D. calcium hydroxide and phenol.
A. Crown tipping.
B. Root uprighting. * The orifice to the fourth canal in a
C. Crown rotation. permanent maxillary first molar is most
often found
Orthodontic headgear can be used for
all of the following EXCEPT A. under the distobuccal cusp.
B. lingual to the orifice of the
A. anchorage augmentation. mesiobuccal canal. *
B. molar distalization. C. between the distobuccal and the
C. maxillary skeletal expansion. * mesiobuccal orifices.
D. growth modification. D. between the palatal and the
distobuccal orifices.
The occlusal parameter most likely
associated with maxillary anterior
spacing is a/an

Dr Abdul Naser Tamim SEHA, Dr Ghada Al Aqqad D.D.S, Dr Kamal Naser - Amrita Medical Centre, Dr Emad Wani - Shadi Dental Centre,
Dr Lina Anka SEHA, Dr Rouba Zgheibi SEHA, Maria Teresa Yongson Alejandrino SEHA 208
The primary function of root canal
sealer is to A 52 year old patient presents with
restricted or limited mouth opening.
A. prevent discoloration of the tooth. The patient has loss of attached
B. stimulate healing in the apical gingiva and multiple areas of gingival
region. recession and tight skin. A panoramic
C. medicate the canal to eliminate radiograph shows diffuse widening of
remaining bacteria. the periodontal ligament. What is the
D. fill space between gutta-percha and most likely diagnosis?
pulp canal wall. *
A. Progressive systemic sclerosis. *
When odontoblasts are destroyed, B. Hyperparathyroidism.
new odontoblasts are derived from C. Cicatricial pemphigoid.
D. Erythema multiforme.
A. existing odontoblasts. E. Advanced adult periodontitis.
B. macrophages.
C. neural crest cells. Which muscle is primarily responsible
D. undifferentiated cells. * for moving the mandible to a lateral
position?
The washing of hands must be
performed before putting on and after A. Masseter.
removing gloves because it B. Lateral pterygoid. *
C. Medial pterygoid.
1. Reduces the number of skin D. Buccinator.
bacteria which multiply and cause E. Temporalis.
irritation.
2. Completely eliminates skin bacteria. When gypsum is mixed with water and
3. Minimizes the transient bacteria sets to form a dental cast, the powder
which could contaminate hands particles are replaced by crystals of
through small pinholes.
4. Allows gloves to slide on easier A. potassium sulfate.
when the hands are moist. B. potassium phosphate.
C. calcium sulfate. *
A. (1) (2) (3) D. trisodium phosphate.
B. (1) and (3) *
C. (2) and (4) The chemical that is used to retard the
D. (4) only setting reaction in alginate impression
E. All of the above. materials is

The most common clinical A. calcium sulfate.


characteristic of a functional crossbite B. sodium sulfate. *
is C. calcium phosphate.
D. sodium phosphate.
A. midline deviation. *
B. arch asymmetry.
C. pain on closure.
D. severe crowding.

Dr Abdul Naser Tamim SEHA, Dr Ghada Al Aqqad D.D.S, Dr Kamal Naser - Amrita Medical Centre, Dr Emad Wani - Shadi Dental Centre,
Dr Lina Anka SEHA, Dr Rouba Zgheibi SEHA, Maria Teresa Yongson Alejandrino SEHA 209
Hardening of Type IV cast gold dental
alloys by heat treatment increases A high neutrophil count is indicative of
a/an
A. ductility.
B. yield strength. * A. allergic reaction.
C. coring. B. bacterial infection. *
D. elastic modulus. C. blood clotting defect.
E. malleability. D. decrease in antibody production.

The yield strength of an orthodontic A patient with multiple small bruises


wire is (purpura) most likely has a low count
of
A. the same as the proportional limit.
B. decreased by work hardening. A. lymphocytes.
C. the same as the stress at fracture. B. T-cells.
D. higher than the proportional limit.* C. platelets. *
D. erythrocytes.
Strain hardening a metal will reduce its E. eosinophils.

A. modulus of elasticity. Local anesthetics are less effective in


B. ductility. * inflamed tissue because they are
C. proportional limit.
D. yield strength. A. diluted by the edematous fluid.
B. rapidly redistributed by the
Acquired immunodeficiency syndrome increased blood flow.
(AIDS) is caused by a/an C. ionized by the acidic pH. *
D. rapidly degraded by released
A. adenovirus. enzymes.
B. retrovirus. *
C. picovirus. Lymphatic spread from a carcinoma
D. coronavirus. on the tip of the tongue would initially
involve which nodes?
Type I hypersensitivity results from
cross-linking of which immunoglobulin A. Submandibular.
on mast cells? B. Submental. *
C. Deep parotid.
A. IgA. D. Mastoid.
B. IgD. E. Jugulo-omohyoid.
C. IgE. *
D. IgG.
E. IgM.

Dr Abdul Naser Tamim SEHA, Dr Ghada Al Aqqad D.D.S, Dr Kamal Naser - Amrita Medical Centre, Dr Emad Wani - Shadi Dental Centre,
Dr Lina Anka SEHA, Dr Rouba Zgheibi SEHA, Maria Teresa Yongson Alejandrino SEHA 210
Adjustment of the occlusal plane of has penetrated through which
natural teeth opposed by a complete or ligament?
partial denture should be completed
A. Sphenomandibular. *
A. after the teeth have been set on the B. Stylomandibular.
trial denture. C. Stylohyoid.
B. immediately after making the final D. Pterygomandibular.
casts.
C. upon delivery of the denture. During the act of swallowing, the
D. after the diagnosis and treatment auditory (pharyngotympanic) tube is
plan has been established. *
A. opened by the tensor tympani
The dentino-enamel junction is the muscle.
most sensitive portion of a tooth B. closed by the tensor tympani
because muscle.
C. opened by the tensor veli palatine
A. free nerve endings terminate on muscle. *
odontoblasts at this region. D. closed by the levator veli palatine
B. odontoblastic processes branch muscle.
considerably at this region. * E. closed by the superior constrictor
C. ameloblasts make synaptic muscle.
connections with odontoblasts at this
junction. Whooping cough is caused by
D. odontoblastic tubules help convey
hydrostatic forces to the pulp cells. A. bacteria. *
B. a virus.
Local anesthetics block nerve C. mycoplasma.
conduction by interfering with ionic D. yeast.
movement of
Reduced thyroid hormone level in a
A. calcium. child is associated with
B. potassium.
C. sodium. * A. lack of tooth eruption.
D. chloride. B. early tooth eruption.
C. delayed tooth eruption. *
Pain caused by trauma to the oral D. supernumerary teeth.
mucosa would activate
Which of the following may result in
A. muscle spindles. acetone breath?
B. low-threshold mechanoreceptors.
C. nociceptors. * A. Prolonged fasting. *
D. pacinian corpuscles. B. High carbohydrate diet.
E. merkel disks. C. High protein diet.
D. Poor oral hygiene.
Immediately following an inferior
alveolar nerve block, the patient
exhibits facial paralysis. The needle

Dr Abdul Naser Tamim SEHA, Dr Ghada Al Aqqad D.D.S, Dr Kamal Naser - Amrita Medical Centre, Dr Emad Wani - Shadi Dental Centre,
Dr Lina Anka SEHA, Dr Rouba Zgheibi SEHA, Maria Teresa Yongson Alejandrino SEHA 211
Which line angle is NOT present in a The line drawn through the occlusal
Class V amalgam cavity preparation? rests of two principal abutments is

A. Mesioaxial. A. survey line.


B. Axiopulpal. * B. terminal line.
C. Gingivoaxial. C. axis of rotation/fulcrum line. *
D. Distoaxial. D. line of greatest torque.
E. Occlusoaxial.
The characteristics of "Group function
When compared to zinc phosphate occlusion" are:
cement, glass ionomer cement has
a/an A. The teeth on the non-working side
make contact in lateral excursion.
A. lower solubility in oral fluids. B. The teeth on the working side
B. ability to release fluoride. * make contact in lateral excursion. *
C. higher compressive strength. C. Only canine and lateral incisors
D. lower film thickness. make contact in lateral excursion.
D. The posterior teeth on both sides
The leeway space in an 8 year old make contact in lateral excursion.
child
The full palatal strap major connector
A. will provide space for eruption of the is indicated where
permanent incisors.
B. is greater in the maxillary arch than A. there is a high, narrow palatal
in the mandibular arch. vault.
C. occurs with premature loss of B. a well-defined, undercut palatal
primary molars. torus is present.
D. is approximately 3.5mm in the C. very few teeth remain in a flat or
mandibular arch. * U-shaped arch. *
E. allows accommodation of premolars D. palatal tissue is soft and
that are larger than the primary compressible.
molars.
The primary function of gingival
When a simple tipping force is applied retraction cord is to
to the crown of a single-rooted tooth,
the centre of rotation is located A. ensure adequate impression
material in the sulcus. *
A. at the apex. B. displace the epithelial attachment.
B. at the cervical line. C. control the salivary flow.
C. within the apical half of the root. * D. eliminate the intrasulcular debris.
D. within the cervical one third of the
root.

Dr Abdul Naser Tamim SEHA, Dr Ghada Al Aqqad D.D.S, Dr Kamal Naser - Amrita Medical Centre, Dr Emad Wani - Shadi Dental Centre,
Dr Lina Anka SEHA, Dr Rouba Zgheibi SEHA, Maria Teresa Yongson Alejandrino SEHA 212
The most common risk associated with
vital bleaching using 10% Carbamide Which of the following is NOT
peroxide in a custom tray is considered a risk factor for
periodontal disease?
A. superficial enamel demineralization.
B. soft tissue reaction. A. Diabetes.
C. tooth sensitivity. * B. Genetics.
D. cytotoxicity. C. Heart disease. *
D. Tobacco use.
A 10-15 second application of 37%
phosphoric acid on prepared dentin will Which mucogingival surgical
result in all of the following EXCEPT procedure does NOT increase the
zone of attached gingiva?
A. elimination of the smear layer.
B. increased diameter of the dentinal A. Free autogenous gingival graft. *
tubules. B. Laterally positioned flap.
C. demineralization of the superficial C. Coronally positioned flap.
dentin. D. Subepithelial connective tissue
D. elimination of the collagen fibres. * graft for root coverage.

On a semi-adjustable articulator, the Which of the following types of bone


incisal guide table represents contain the insertions of the
periodontal ligament fibres?
A. a reference point for the
establishment of occlusal vertical A. Woven.
dimension. B. Bundle. *
B. the anterior equivalent of condylar C. Lamellar.
guidance. D. Cortical.
C. a mechanical equivalent of the
horizontal and vertical overlap of the The most effective agent used to
anterior teeth. * etch enamel and dentin is
D. the mechanical equivalent of the
Curve of Wilson. A. 5-10% tartaric acid.
B. 30-40% phosphoric acid. *
Which statement best describes hand C. 15-20% maleic acid.
washing for the prevention of disease D. 40-50% citric acid.
transmission?
Which of the following conditions is
A. Hands should be washed for at least NOT associated with the fully
15 seconds, always with an edentulous state?
antimicrobial soap.
B. Hands should be washed thoroughly A. Residual ridge reduction.
and vigorously prior to placement and B. Decrease in masticatory function.
upon removal of gloves. * C. Altered taste perception.
C. Transient flora is more difficult to D. Rheumatoid arthritis. *
remove during routine hand washing
and needs vigorous scrubbing action.
D. An alcohol hand rub is an
acceptable alternative to hand
washing.

Dr Abdul Naser Tamim SEHA, Dr Ghada Al Aqqad D.D.S, Dr Kamal Naser - Amrita Medical Centre, Dr Emad Wani - Shadi Dental Centre,
Dr Lina Anka SEHA, Dr Rouba Zgheibi SEHA, Maria Teresa Yongson Alejandrino SEHA 213
The maximum recommended Which of the following does NOT affect
thickness of an incremental composite probing depth measurement?
resin to be light cured is
A. Probing force.
A. 2mm. * B. Diameter of the probe tip.
B. 3mm. C. Angulation of the probe.
C. 4mm. D. Subgingival calculus.
D. 5mm. E. Position of the mucogingival
junction. *
Which of the following root surfaces
are most likely to have Which of the following medications
flutings/concavities that will make does NOT cause gingival hyperplasia?
thorough root debridement difficult?
A. Cyclosporine.
A. Mesial of teeth 1.1/2.1. B. Nifedipine.
B. Mesial of teeth 1.2/2.2. C. Phenytoin.
C. Mesial of teeth 1.3/2.3. D. Carbamazepine. *
D. Mesial of teeth 1.4/2.4. *
Which of the following treatments is
Which of the following is the most CONTRAINDICATED for a patient with
appropriate treatment for an necrotizing ulcerative gingivitis?
endodontically treated mandibular first
molar with a previously placed MOD A. Antibiotic therapy.
amalgam restoration? B. Local debridement.
C. Topical steroid therapy. *
A. Bonded amalgam restoration. D. Warm saline solution rinses.
B. Composite resin restoration.
C. Ceramic inlay. Gingivectomy is recommended
D. Crown. *
A. when the bottom of the pocket is
At a six month recall visit, which of the apical to the mucogingival junction.
following indicates that gingivitis has B. to eliminate the suprabony pockets
progressed to periodontitis? when the pocket wall is fibrous and
firm. *
A. Attachment loss. * C. to treat moderately deep pockets
B. Bleeding upon probing. with mild intrabony defects.
C. Change in gingival colour.
D. Increased tooth mobility.

Dr Abdul Naser Tamim SEHA, Dr Ghada Al Aqqad D.D.S, Dr Kamal Naser - Amrita Medical Centre, Dr Emad Wani - Shadi Dental Centre,
Dr Lina Anka SEHA, Dr Rouba Zgheibi SEHA, Maria Teresa Yongson Alejandrino SEHA 214
The most practical method to examination shows generalized loss of
significantly reduce the setting time of the lamina dura and a “ground glass”
stone and plaster is to use appearance of the bone. What is the
most likely diagnosis?
A. warm mixing water. * A. Hyperthyroidism.
B. a calcium sulfate dihydrate B. Addison’s disease.
nucleating agent. C. Hypothyroidism.
C. a sodium sulfate nucleating agent. D. Hyperparathyroidism. *
D. a longer mixing time. E. Hypoparathyroidism.

A crown margin can be extended Which anatomical structures form the


subgingivally when required inverted Y (Y line) in maxillary
periapical radiographs?
1. for esthetics.
2. To increase retention. A. Nasopalatine/incisive canal and
3. To reach sound tooth structure. floor of the nasal fossa.
4. for caries prevention. B. Anterior nasal spine and
nasopalatine/incisive canal.
A. (1) (2) (3) * C. Floor of the nasal fossa and
B. (1) and (3) maxillary sinus border. *
C. (2) and (4) D. Zygomatic process of the maxilla
D. (4) only and maxillary sinus border.
E. All of the above.
All of the following appear as midline
The most appropriate management of structures on periapical radiographs
a 4mm diameter carious exposure on a EXCEPT
vital permanent first molar in a 7 year
old is A. nasopalatine/incisive canal.
B. anterior nasal spine.
A. direct pulp capping. C. nasal septum.
B. partial pulpotomy. * D. zygomatic process of the maxilla. *
C. pulpectomy.
D. extraction. Which condition is associated with
elevated serum alkaline phosphatase
The use of an intra-coronal attachment and elevated urinary hydroxyproline
is CONTRAINDICATED for a tooth levels?

A. that is nonvital. A. fibrous dysplasia.


B. requiring a core procedure build up B. Paget’s disease. *
C. Sjögren’s syndrome.
C. supporting a partial denture.
D. Gardner’s syndrome.
D. with short crown length. *

A 65 year old patient has a tendency


for urinary tract stones. Radiographic

Dr Abdul Naser Tamim SEHA, Dr Ghada Al Aqqad D.D.S, Dr Kamal Naser - Amrita Medical Centre, Dr Emad Wani - Shadi Dental Centre,
Dr Lina Anka SEHA, Dr Rouba Zgheibi SEHA, Maria Teresa Yongson Alejandrino SEHA 215
The mean annual radiation dose
equivalent for human populations on A. before age 10. *
earth is approximately B. between l1 and 20 years of age.
C. between 21 and 30 years of age.
A. 3 to 4 micro- (μ) Sv. D. after age 31.
B. 3 to 4 milli- (m) Sv. *
C. 3 to 4 Sv. The predominant cells in the
D. 3 to 4 k Sv. inflammatory exudate of an acute
periodontal abscess are
The most radiosensitive cell type is
a/an A. neutrophils. *
B. eosinophils.
A. neuron. C. basophils.
B. chondrocyte. D. lymphocytes.
C. myocyte. E. monocytes.
D. epithelial basal cell. *
During the setting phase, a dental
Increasing the kVp results in stone mixture will exhibit

A. increased long scale image A. expansion. *


contrast. * B. contraction.
B. increased short scale image C. loss in compressive strength.
contrast. D. gain in moisture content.
C. decreased long scale image
contrast. A 75 year old male patient whose wife
D. decreased short scale image died 10 months ago presents for his
contrast. recall appointment. Looking wasted
and fatigued, he confirms he has lost
Which foramen presents as an apical about 6kg in the last 8 months but is
radiolucency in the mandibular otherwise in good health. The most
premolar region? appropriate management for this
patient is to
A. Lingual.
B. Mandibular. A. recommend that he drink 3 cans of a
C. Mental. * nutritional supplement each day.
D. Incisive. B. refer him to a qualified
dietician/nutritionist and follow up after
Which syndrome presents with his appointment. *
multiple cysts of the jaws? C. refer him back to his physician
requesting a more thorough
A. Gardner’s. assessment. *
B. Gorlin-Goltz. * D. provide him with a copy of Food
C. Peutz-Jeghers. Guide to Healthy Eating.
D. Sjögren’s.

Primary herpetic gingivostomatitis


most frequently occurs

Dr Abdul Naser Tamim SEHA, Dr Ghada Al Aqqad D.D.S, Dr Kamal Naser - Amrita Medical Centre, Dr Emad Wani - Shadi Dental Centre,
Dr Lina Anka SEHA, Dr Rouba Zgheibi SEHA, Maria Teresa Yongson Alejandrino SEHA 216
Hypercementosis may be associated B. reactor.
with C. catalyst.
D. activator. *
A. Paget’s disease.* E. terminator.
B. ameloblastoma.
C. hypophosphatasia. Zinc oxide eugenol cement is a/an
D. multiple myeloma.
A. phosphate cement.
In alginate impression materials, B. phenolic cement. *
sodium phosphate (Na3PO4) is the C. resin modified glass ionomer
cement.
A. reactor. D. polyalkenoic acid cement.
B. catalyst. E. adhesive resin cement.
C. retarder. *
D. disinfectant. An advantage of glass ionomer cement
E. cross linking agent. is

To ensure a clinically acceptable A. low solubility.


setting time, polyalkenoic cements B. wear resistance.
contain C. adhesion to hard tooth tissues. *
D. low incidence of sensitivity.
A. salicylic acid.
B. phosphoric acid. The most toxic form of mercury is
C. maleic acid.
D. tartaric acid. * A. organo mercurial compounds. *
E. itaconic acid. B. inorganic mercuric compounds.
C. inorganic mercurous compounds.
Using more water when mixing dental D. elemental mercury.
stone will result in a cast that exhibits
Polymerization shrinkage associated
A. increased expansion and with the setting of composite resins is a
decreased strength. * result of
B. decreased expansion and
increased strength. A. primary bonds replacing secondary
C. decreased expansion and bonds. *
decreased strength. B. reaction by-products evaporating
D. increased expansion and from the set material.
increased strength. C. unreacted monomer evaporating
from the set material.
Light-cured dental composites set D. temperature changes occurring
when exposed to light. Light is the during the polymerization reaction.
A. initiator.

Dr Abdul Naser Tamim SEHA, Dr Ghada Al Aqqad D.D.S, Dr Kamal Naser - Amrita Medical Centre, Dr Emad Wani - Shadi Dental Centre,
Dr Lina Anka SEHA, Dr Rouba Zgheibi SEHA, Maria Teresa Yongson Alejandrino SEHA 217
Shortly after the administration of a D. failure of development of both the
local anesthetic for the removal of lateral nasal and maxillary processes. *
tooth 2.8, the patient complains of a
tense sensation in the left cheek and When exposing radiographic film, the
left cheek swelling is observed. The amount of radiation received by the
most likely diagnosis is patient is best reduced by

A. surgical emphysema. A. collimation.*


B. immediate allergic reaction. B. decreased object-film distance.
C. herniation of buccal fat pad. C. low kVp correlated with high
D. hematoma. * milliamperage.
D. decreased target-object distance.
During tooth development, epithelial
mesenchymal interactions function In an edentulous patient, the coronoid
process may
A. only at the initial stages when tooth
positions are being laid down. A. limit the distal extension of the
B. through the exchange of small mandibular denture.
molecules. B. affect the position and arrangement
C. when preameloblasts signal of the posterior teeth.
preodontoblasts to start producing C. determine the location of the
dentin. posterior palatal seal.
D. only with epithelium and D. limit the distal extensions of the
mesenchyme from tooth-forming maxillary denture. *
regions. *
Which of the following impression
Enamel pearls form when materials is NOT recommended for
making a final impression for
A. ameloblasts migrate apically down fabrication of dies for a porcelain fused
the root. to metal crown?
B. cells of the epithelial root sheath do
not migrate away from the dentin. * A. Addition silicone.
C. cells of the dental follicle fail to B. Condensation silicone.
develop. C. Irreversible hydrocolloid. *
D. epithelial rests transform into D. Polyether.
ameloblast vesicles.
The inorganic ion which is the chief
Cleft lip and palate usually result from offender in hypertension is

A. failure of proper union of the median A. sodium. *


and lateral nasal processes. B. ammonium.
B. failure of the union of the median C. magnesium.
nasal process with the lateral nasal D. potassium.
and maxillary processes.
C. anhidrotic ectodermal dysplasia.

Dr Abdul Naser Tamim SEHA, Dr Ghada Al Aqqad D.D.S, Dr Kamal Naser - Amrita Medical Centre, Dr Emad Wani - Shadi Dental Centre,
Dr Lina Anka SEHA, Dr Rouba Zgheibi SEHA, Maria Teresa Yongson Alejandrino SEHA 218
After initiating preventive management E. None of the above.
for a 16 year old patient with multiple
extensive carious lesions, which of the After setting, alginate impressions
following restorative treatments is most
appropriate? A. absorb water. *
B. remain dimensionally stable for 12
A. Place amalgam restorations over hours.
the next few months. C. have higher tear strength than
B. Excavate caries and place polyvinylsiloxane impressions.
temporary restorations within the next D. can be poured twice with little effect
few weeks. * on accuracy of the resulting cast.
C. Delay any treatment until the
hygiene improves. For an acid-etched Class III composite
D. Restore all teeth with composite resin, the cavosurface margin of the
resin over the next few months. cavity can be bevelled to

Alteration of the intestinal flora by A. eliminate the need for internal


some chemotherapeutic agents can retention.
interfere with reabsorption of a B. improve convenience form.
contraceptive steroid thus preventing C. aid in finishing.
the recirculation of the drug through D. increase the surface area for
the enterohepatic circulation. Which of etching. *
the following can interfere with this
mechanism? In composite resin restorations, glass
ionomer cements can be used as a
1. Codeine. base because they are
2. Penicillin V.
3. Acetaminophen. A. sedative to a hyperemic pulp.
4. Tetracycline. B. neutral in colour.
C. biocompatible. *
A. (1) (2) (3) D. compatible with the expansion of
B. (1) and (3) composite resins.
C. (2) and (4) *
D. (4) only What is the best imaging modality to
E. All of the above. assess the TMJ disc?

Condensing osteitis in the periapical A. Arthrography.


region is indicative of a/an B. Computed tomography.
C. Magnetic resonance imaging. *
A. acute inflammation of the pulp. D. Corrected conventional tomography.
B. pulpal abscess.
C. chronic inflammation of the pulp. *
D. early apical abscess formation.

Dr Abdul Naser Tamim SEHA, Dr Ghada Al Aqqad D.D.S, Dr Kamal Naser - Amrita Medical Centre, Dr Emad Wani - Shadi Dental Centre,
Dr Lina Anka SEHA, Dr Rouba Zgheibi SEHA, Maria Teresa Yongson Alejandrino SEHA 219
Kilovoltage controls the
Which of the following is NOT a feature
1. Contrast. of bruxism?
2. Speed of the electrons.
3. Penetrating power of radiation. A. Radiographic evidence of the
4. Penumbra. widening of the periodontal ligament.
B. Increased mobility of teeth.
A. (1) (2) (3) C. Premature wear of occlusal
B. (1) and (3) * surfaces.
C. (1) and (4) D. Erosion. *
D. (4) only
E. All of the above. Which of the following is NOT a risk
factor for periodontitis?
Patients who have gingival
enlargements surgically removed A. Smoking.
should be forewarned that there is a B. Poorly controlled diabetes.
high incidence of C. Coronary heart disease. *
D. Poor oral hygiene.
A. altered taste sensation.
B. dentinal hypersensitivity. Soft tissue pockets CANNOT be
C. reoccurrence of gingival reduced by
enlargement. *
D. post-operative swelling. A. occlusal adjustment. *
B. scaling and root planing
Differential diagnosis for short term (debridement).
pain and swelling of the gingiva C. open flap curettage.
associated with an endodontically D. guided tissue regeneration.
treated tooth should include
Crusted hemorrhagic and ulcerative
1. Periodontal abscess. lesions of the lips in a patient with
2. Periapical abscess. target-like skin lesions are typical of
3. Vertical root fracture.
4. Internal root resorption. A. lupus erythematosus.
B. Reiter’s syndrome.
A. (1) (2) (3) * C. Behçet’s syndrome.
B. (1) and (3) D. erythema multiforme. *
C. (2) and (4) E. pemphigus vulgaris.
D. (4) only
E. All of the above.

Dr Abdul Naser Tamim SEHA, Dr Ghada Al Aqqad D.D.S, Dr Kamal Naser - Amrita Medical Centre, Dr Emad Wani - Shadi Dental Centre,
Dr Lina Anka SEHA, Dr Rouba Zgheibi SEHA, Maria Teresa Yongson Alejandrino SEHA 220
Multiple “punched-out” radiolucencies maxillary “6” excess of 3.0mm
of the skull and jaws are most
commonly seen with What effect(s) could this Bolton
relationship have on an Angle Class I
A. metastatic carcinoma. malocclusion?
B. plasmacytoma.
C. multiple myeloma. * 1. Deeper overbite.
D. chondrosarcoma. 2. Maxillary crowding.
E. osteosarcoma. 3. Increased overjet.
4. Maxillary spacing.
Which of the following diseases may
cause an enlargement of the jaws, A. (1) (2) (3) *
development of diastemas and/or a B. (1) and (3)
poorly fitting denture? C. (2) and (4)
D. (4) only
A. Phantom bone disease. E. All of the above.
B. Rickets.
C. Paget’s disease. * What is the earliest age that a
D. Osteoporosis. congenitally missing mandibular
E. Hypophosphatasia. second bicuspid can be confirmed?

Hutchinson’s incisors and mulberry A. 2 years.


molars are associated with B. 4 years. *
C. 6 years.
A. congenital porphyria. D. 8 years.
B. fluorosis.
C. rickets. A survey of the master cast shows that
D. congenital syphilis. * the 3.5 and 3.7 abutments for a fixed
partial denture have different paths of
E. cleidocranial dysplasia.
insertion with respect to 3.7. A semi-
precision attachment is chosen rather
A patient with pain, fever and unilateral
than preparing the teeth again. Where
parotid swelling following a general
should the male part of the attachment
anesthetic most likely has
ideally be located?
A. Mumps.
A. Distal of the 3.5 retainer.
B. sialolithiasis.
B. Distal of the 3.6 pontic.
C. acute bacterial sialadenitis. *
C. Mesial of the 3.7 retainer.
D. Sjögren’s syndrome.
D. Mesial of the 3.6 pontic. *
E. sarcoidosis.

A Bolton relationship has determined a


maxillary “12” excess of 3.5mm

Dr Abdul Naser Tamim SEHA, Dr Ghada Al Aqqad D.D.S, Dr Kamal Naser - Amrita Medical Centre, Dr Emad Wani - Shadi Dental Centre,
Dr Lina Anka SEHA, Dr Rouba Zgheibi SEHA, Maria Teresa Yongson Alejandrino SEHA 221
The angle SNA can be used to
evaluate the The custom tray used in making a
final complete denture impression
A. maxillary protrusion. * must
B. overbite.
C. upper incisor inclination. A. extend to the bottom of the
D. facial height. vestibule.
E. mandibular angle. B. create adequate space for the
impression material. *
When cementing a gold inlay with a C. have a horizontal handle.
zinc phosphate cement, the best way D. be stored in water until ready for
to ensure accurate seating is to use.

A. apply a firm pressure on the inlay The best way to protect the abutments
until the cement is set. * of a Class I removable partial denture
B. mix the cement rapidly to allow from the negative effects of the
ample time for insertion. additional load applied to them is by
C. ask the patient to lightly tap on the
restoration until the occlusion is A. splinting abutments with adjacent
comfortable. teeth.
D. force the restoration in place with an B. keeping a light occlusion on the
orange wood stick and mallet. distal extensions.
E. relieve the internal angles of the C. placing distal rests on distal
inlay before insertion. abutments.
D. using cast clasps on distal
For which of the following reasons abutments.
would a dentist administer an aqueous E. regular relining of the distal
solution of epinephrine hydrochloride? extensions. *

A. Hemostasis. * A recommended method for


B. Vasocontriction. disinfecting alginate impressions is to
C. Acute adrenal insufficiency. immerse the impression for 10
D. Anaphylaxis. minutes in

Healthy attached gingiva A. a complex phenolic.


B. 2% glutaraldehyde.
A. has no basal cell layer. C. 10% ethyl alcohol.
B. is closely bound to underlying D. a 1:10 dilution of sodium
periosteum. * hypochlorite. *
C. contains elastic fibers.
D. has no rete pegs.

Dr Abdul Naser Tamim SEHA, Dr Ghada Al Aqqad D.D.S, Dr Kamal Naser - Amrita Medical Centre, Dr Emad Wani - Shadi Dental Centre,
Dr Lina Anka SEHA, Dr Rouba Zgheibi SEHA, Maria Teresa Yongson Alejandrino SEHA 222
The washing of hands must be E. Vitamin K.
performed before putting on and
after removing gloves because it Which of the following is a possible
cause for a low density radiograph
1. Reduces the number of skin (light film)?
bacteria which multiply and cause
irritation. A. Cold developer. *
2. Completely eliminates skin B. Over exposure.
bacteria. C. Improper safety light.
3. Minimizes the transient bacteria D. Excessive developing time.
which could contaminate hands
through small pinholes. A patient presents with hypodontia,
4. Allows gloves to slide on easier conical teeth, fine, scanty, fair hair, and
when the hands are moist. an intolerance to hot weather. The
most likely diagnosis is
A. (1) (2) (3)
B. (1) and (3) * A. achondroplasia.
C. (2) and (4) B. malignant hyperthermia.
D. (4) only C. ectodermal dysplasia. *
E. All of the above. D. cystic fibrosis.
Lacrimation and flushing of the face The best way for a dentist to ensure
are autonomic phenomena that are efficacy in a disinfection solution is to
occasionally associated with
trigeminal neuralgia involving the A. make a fresh solution every day.
maxillary nerve. This can be B. follow the manufacturer’s
explained by the nerve’s association instructions. *
with which ganglion? C. increase the concentration of the
product.
A. Pterygopalatine. * D. increase the time of contact with the
B. Submandibular. product.
C. Trigeminal.
D. Otic. Which of the following sweeteners
E. Nasociliary. used in sugarless gum is most effective
in preventing caries?
Which of the following is necessary
for collagen formation? A. Xylitol. *
B. Sorbitol.
A. Vitamin A. C. Mannitol.
B. Vitamin C. * D. Glycerol.
C. Vitamin D.
D. Vitamin E.

Dr Abdul Naser Tamim SEHA, Dr Ghada Al Aqqad D.D.S, Dr Kamal Naser - Amrita Medical Centre, Dr Emad Wani - Shadi Dental Centre,
Dr Lina Anka SEHA, Dr Rouba Zgheibi SEHA, Maria Teresa Yongson Alejandrino SEHA 223
The most appropriate treatment for a A smooth, elevated, red patch devoid
vital primary molar with carious pulp of filiform papillae, located in the
exposure and cusp fracture is a/an midline of the dorsum of the tongue
immediately anterior to the
A. pulp capping and amalgam circumvallate papillae is indicative of
restoration.
B. pulpotomy and stainless steel A. benign migratory glossitis.
crown. * B. median rhomboid glossitis. *
C. pulp capping and composite resin C. a granular cell tumor.
restoration. D. iron deficiency anemia.
D. extraction and placement of a E. a fibroma.
space maintainer.
An 8 year old patient with all primary
Upon stimulation of salivary flow, molars still present exhibits a cusp-to-
which gland is the main source of cusp relationship of permanent
salivary volume? maxillary and mandibular first molars
and good alignment of the lower
A. Submandibular. incisors. The management of this
B. Submaxillary. patient should be to
C. Sublingual.
D. Parotid. * A. refer for orthodontic consultation.
B. use a cervical headgear to
A 20 year old female patient is reposition maxillary molars.
suspected of having bulimia. Which of C. disk the distal surfaces of primary
the following signs will help confirm the mandibular second molars.
diagnosis? D. place patient on appropriate recall
schedule. *
1. Enamel erosion of maxillary anterior
teeth. Which of the following could be a
2. Enlargement of the thyroid gland. complication when performing a dental
3. Calluses on the dorsum of the extraction on an insulin-dependent
fingers. diabetic patient?
4. Bulky clothing to disguise weight
loss. A. Diabetic acidosis.
B. Increased bleeding.
A. (1) (2) (3) C. Hypoglycemic shock. *
B. (1) and (3) D. Incomplete anesthesia.
C. (2) and (4) E. Acute adrenocortical insufficiency.
D. (4) only
E. All of the above. *

Dr Abdul Naser Tamim SEHA, Dr Ghada Al Aqqad D.D.S, Dr Kamal Naser - Amrita Medical Centre, Dr Emad Wani - Shadi Dental Centre,
Dr Lina Anka SEHA, Dr Rouba Zgheibi SEHA, Maria Teresa Yongson Alejandrino SEHA 224
Epstein-Barr virus is associated with
which of the following? Root caries risk in adults is

1. Shingles. 1. Associated with previous enamel


2. Oral hairy leukoplakia. caries experience.
3. Chickenpox. 2. Reduced in communities with
4. Infectious mononucleosis. fluoridated water.
3. greater in adults who have high
A. (1) (2) (3) streptococcus mutans counts.
B. (1) and (3) 4. similar in institutionalized and non-
C. (2) and (4) * institutionalized patients.
D. (4) only
E. All of the above. A. (1) (2) (3) *
B. (1) and (3)
A 52 year old patient presents with a C. (2) and (4)
limitation of mouth opening. The D. (4) only
patient has loss of attached gingiva E. All of the above.
and multiple areas of gingival
recession. A panoramic radiograph For a 20 year old, 80kg, patient with a
shows diffuse widening of the confirmed allergy to codeine, which of
periodontal ligament. The most likely the following is/are appropriate for
diagnosis is pain control following the removal of
an impacted third molar?
A. scleroderma. *
B. hyperparathyroidism. 1. Hydromorphone, 2mg every 4 - 6
C. cicatricial pemphigoid. hours p.r.n.
D. erythema multiforme. 2. Ibuprofen, 800mg 1 hour pre-
E. advanced adult periodontitis. operatively followed by 400mg every 4
- 6 hours p.r.n.
Normal aging changes in the hard 3. Acetaminophen, 650mg with
tooth tissues include oxycodone, 10mg every 4 - 6 hours
p.r.n.
1. Continuous deposition of 4. Ketorolac, 10mg every 4 - 6 hours
cementum. p.r.n.
2. Continuous deposition of dentin.
3. Decreased blood supply to the pulp. A. (1) (2) (3)
4. Increased porosity of enamel. B. (1) and (3)
C. (2) and (4) *
A. (1) (2) (3) * D. (4) only
B. (1) and (3) E. All of the above.
C. (2) and (4)
D. (4) only
E. All of the above.

Dr Abdul Naser Tamim SEHA, Dr Ghada Al Aqqad D.D.S, Dr Kamal Naser - Amrita Medical Centre, Dr Emad Wani - Shadi Dental Centre,
Dr Lina Anka SEHA, Dr Rouba Zgheibi SEHA, Maria Teresa Yongson Alejandrino SEHA 225
A patient presents with a 3 week C. 1mm apical to the amalgam margin. *
history of prolonged tooth pain to hot D. 2mm apical to the amalgam margin.
and cold. Three days ago the
symptoms changed to moderate pain The most appropriate treatment of a true
on biting combined with a dull, combined endodontic-periodontal lesion
spontaneous ache relieved by cold. is
The most likely diagnosis is
A. periodontal surgical therapy only.
A. chronic apical abscess. B. nonsurgical root canal therapy only.
B. a cracked tooth. C. periodontal surgical therapy before
C. pulpal necrosis. * non-surgical endodontic treatment.
D. reversible pulpitis. D. nonsurgical root canal therapy before
E. a vertical root fracture. periodontal therapy. *

The incidence of tooth loss due to Hypothyroidism in adults is associated


periodontal disease is highest for with

A. maxillary molars. * A. exophthalmos.


B. maxillary premolars. B. weight loss.
C. mandibular incisors. C. generalized edema. *
D. mandibular premolars. D. tachycardia.
E. mandibular molars. E. mental defects.

Which of the following Class II Division A bacterial infection causes the most
1 malocclusion(s) is/are most likely to significant increase in
be corrected with a cervical headgear?
A. basophils.
A. Retrognathic mandible, retrognathic B. neutrophils. *
maxilla, open bite. C. lymphocytes.
B. Prognathic maxilla, decreased lower D. monocytes.
face height, increased over bite. * E. eosinophils.
C. Increased lower anterior face height,
prognathic mandible, retrognathic While the teeth are set in wax, dentures
maxilla. are tried in to
D. Open bite, prognathic maxilla,
prognathic mandible. A. verify the maxillomandibular records.
B. verify the vertical dimension of
A lower molar requiring a crown has an occlusion.
amalgam restoration extending 1.0 mm C. evaluate esthetics.
sub-gingivally. The crown margin D. All of the above. *
should be placed
A. on the existing amalgam.
B. at the amalgam/tooth junction.

Dr Abdul Naser Tamim SEHA, Dr Ghada Al Aqqad D.D.S, Dr Kamal Naser - Amrita Medical Centre, Dr Emad Wani - Shadi Dental Centre,
Dr Lina Anka SEHA, Dr Rouba Zgheibi SEHA, Maria Teresa Yongson Alejandrino SEHA 226
An infected pulp may cause a(n)
A 45 year old with insulin dependent
A. odontogenic keratocyst. diabetes mellitus comes for a morning
B. dentigerous cyst. dental appointment. During the
C. periapical cemento-osseous examination, the patient complains of
dysplasia. being lightheaded and weak. Sweating
D. simple bone cyst (traumatic bone is observed. The patient is most likely
cyst). experiencing
E. periradicular cyst. *
A. hyperglycemia.
Following root canal therapy, the B. hypoglycemia. *
most desirable form of tissue C. syncope.
response at the apical foramen is D. hyperventilation.
E. cerebrovascular accident.
A. cementum deposition. *
B. connective tissue capsule In a dental office, all of the following
formation. should be used to reduce the risk of
C. epithelium proliferation from the Hepatitis B infection for staff and
periodontal ligament. patients EXCEPT
D. dentin deposition.
A. steam sterilization.
Which of the following is the most B. chemical sterilization. *
powerful jaw-closing muscle? C. standard precautions.
D. medical history.
A. Temporalis. E. staff education.
B. Lateral pterygoid.
C. Masseter. * If a patient loses a permanent maxillary
D. Medial pterygoid. first molar before the age of 11, the

Idiopathic osteosclerosis is 1. Premolar drifts distally.


2. Maxillary second molar erupts and
A. painful. moves mesially.
B. found mostly in the maxilla. 3. Opposing tooth erupts into the space
C. radiolucent. created.
D. nonexpansile. * 4. Overbite increases.

Which of the following impression A. (1) (2) (3) *


materials has the best dimensional B. (1) and (3)
stability? C. (2) and (4)
D. (4) only
A. Polysulfide rubber. E. All of the above.
B. Condensation silicone.
C. Polyvinylsiloxane. *
D. Irreversible hydrocolloid.

Dr Abdul Naser Tamim SEHA, Dr Ghada Al Aqqad D.D.S, Dr Kamal Naser - Amrita Medical Centre, Dr Emad Wani - Shadi Dental Centre,
Dr Lina Anka SEHA, Dr Rouba Zgheibi SEHA, Maria Teresa Yongson Alejandrino SEHA 227
If a person falls from a bicycle, striking greatest potential to cause speech
the chin, the most likely region(s) of the problems is
mandible to fracture is/are:
A. a thick narrow major connector. *
1. Symphysis. B. an anterior and a posterior bar.
2. Condylar necks. C. a thin broad palatal strap.
3. Mid-body. D. narrow horseshoe shaped.
4. Angles of the mandible.
In its classic form, serial extraction is
A. (1) (2) (3) best applied to patients with Class I
B. (1) and (3) occlusions with crowding of
C. (2) and (4) *
D. (4) only A. less than 10mm in each of the upper
E. All of the above. and lower arches and 35% overbite.
B. 10mm or more in each of the upper
Immediately following a left posterior and lower arches and 35% overbite. *
superior alveolar nerve block injection, C. less than 10mm in each of the
the patient’s face becomes quickly and upper and lower arches and 70%
visibly swollen on the left side. The overbite.
immediate treatment should be to D. 10mm or more in each of the upper
and lower arches and 70% overbite.
1. Apply a cold compress.
2. Administer 0.3mg epinephrine Which type of malocclusion should be
(sublingually). corrected as early as possible?
3. Apply pressure.
4. Refer for immediate medical A. Class II Division 1 associated with
treatment. an anterior open bite.
B. Class II Division 2 associated with
A. (1) (2) (3) an increased anterior overbite.
B. (1) and (3) * C. Class III associated with an anterior
C. (2) and (4) open bite.
D. (4) only D. Cross-bite associated with a
E. All of the above. functional shift of the mandible from
initial contact to maximum
The disappearance of a calcium intercuspation. *
hydroxide liner under a restoration after E. Anterior open bite associated with a
a length of time is most likely a result of lip or digit sucking habit.

A. microleakage. * Which of the following would maximize


B. gradual dissolution of the liner into vitamin E intake following osseous
the restorative material. surgery?
C. residual bacterial action on the
hydroxyl ions. A. Lettuce.
D. incorporation into the adhesive B. Wheat germ. *
agent. C. Eggs.
D. Fish.
The maxillary cast partial denture
major connector design with the

Dr Abdul Naser Tamim SEHA, Dr Ghada Al Aqqad D.D.S, Dr Kamal Naser - Amrita Medical Centre, Dr Emad Wani - Shadi Dental Centre,
Dr Lina Anka SEHA, Dr Rouba Zgheibi SEHA, Maria Teresa Yongson Alejandrino SEHA 228
The most likely cause of a cavernous central nervous system in sleep
sinus thrombosis is a periradicular (nocturnal) bruxism?
abscess of a maxillary
A. Basal ganglia (nigrostriatal). *
A. First molar. B. A delta and C nerves.
B. central incisor. C. Sphenopalatine ganglion.
C. Second premolar. D. Petrous nerves.
D. Third molar. *
On bite-wing radiographs of adults
A 23 year old female complains of under the age of 30, the normal
bilateral stiffness and soreness in the alveolar crest is
preauricular region. Her symptoms
have been present for the past week A. at the cementoenamel junction.
and are most pronounced in the B. 1-2mm apical to the
morning. The most likely cause is cementoenamel junction. *
C. 3-4mm apical to the
A. fibrous ankylosis of the cementoenamel junction.
temporomandibular joints. D. not clearly distinguishable.
B. nocturnal bruxism. *
C. early osteoarthritis. For an otherwise healthy patient, with
D. mandibular subluxation. an acute localized periodontal
abscess, initial treatment must include
A line angle NOT present on a Class I
cavity preparation on tooth 1.6 is A. scaling and root planing. *
B. occlusal adjustment.
A. mesiopulpal. C. prescription of an antibiotic.
B. buccopulpal. D. prescription of an analgesic.
C. linguopulpal.
D. axiopulpal. * In a healthy patient whose chief
E. None of the above. complaint is bleeding gums after tooth
brushing, what is the most appropriate
A patient complains of lip and tongue initial management?
hypersensitivity (allodynea) following
intake of hot, spicy food. The sensory A. Elimination of local plaque retention
nerve fibers that are associated with sites.
this form of pain are B. Dietary analysis.
C. Periodontal examination and
A. A-alpha. recording. *
B. A-beta. D. Occlusal examination and
C. A-gamma. recording.
D. A-delta and C. *

Which of the following structures may


be associated with the role of the

Dr Abdul Naser Tamim SEHA, Dr Ghada Al Aqqad D.D.S, Dr Kamal Naser - Amrita Medical Centre, Dr Emad Wani - Shadi Dental Centre,
Dr Lina Anka SEHA, Dr Rouba Zgheibi SEHA, Maria Teresa Yongson Alejandrino SEHA 229
The most likely cause of tooth loss A 45 year old, overweight man reports
following a tunneling procedure to that his wife complains that he snores.
provide complete access for a The initial management of the patient’s
mandibular Class III furcation snoring problem is to
involvement is A. fabricate an appliance to reduce
snoring.
A. root caries. * B. fabricate restorations to increase the
B. root sensitivity. patient’s vertical dimension of
C. pulpal involvement. occlusion.
D. recurrent pocketing. C. refer for an orthognathic surgery
consultation.
In periodontal therapy, “guided tissue D. refer for a sleep assessment. *
regeneration” is most successful in
treating Following root planing, a patient
experiences thermal sensitivity. This
1. Horizontal bone loss. pain is associated with which of the
2. A 3-walled infrabony defect. following?
3. A mandibular Class III furcation
involvement. A. Golgi receptor.
4. A mandibular Class II furcation B. Free nerve endings.
involvement. C. Odontoblastic processes. *
D. Cementoblasts.
A. (1) (2) (3)
B. (1) and (3) Which two muscles are involved in
C. (2) and (4) * sucking?
D. (4) only
A. Caninus and depressor angularis.
E. All of the above.
B. Risorius and buccinator.
The most appropriate treatment of C. Buccinator and orbicularis oris. *
necrotizing ulcerative periodontitis D. Levator labii superioris and
(NUP) in a patient with no fever and no zygomaticus major.
lymphadenopathy is
A patient with a tumor in the right
1. Periodontal debridement. infratemporal fossa shows a significant
2. Antibiotic therapy. shift of the mandible to the right when
opening. Which nerve is involved?
3. Oral hygiene instruction.
4. Topical steroid therapy.
A. Facial nerve VII.
A. (1) (2) (3) B. Glossopharyngeal nerve IX.
B. (1) and (3) * C. Trigeminal nerve V. *
C. (2) and (4) D. Hypoglossal nerve XII.
D. (4) only
E. All of the above.

Dr Abdul Naser Tamim SEHA, Dr Ghada Al Aqqad D.D.S, Dr Kamal Naser - Amrita Medical Centre, Dr Emad Wani - Shadi Dental Centre,
Dr Lina Anka SEHA, Dr Rouba Zgheibi SEHA, Maria Teresa Yongson Alejandrino SEHA 230
Cervical caries on the maxillary malocclusion with a SNA = 82 and a
primary incisors in a 12-month old SNB = 80 . The most likely etiology is
child is most likely caused by
A. dental.
A. lack of systemic fluoride. B. skeletal.
B. poorly formed enamel. C. neuromuscular.
C. lack of calcium during pregnancy. D. dental and neuromuscular. *
D. excessive bottle use. * E. skeletal and neuromuscular.

A patient must push up on his A lateral cephalometric radiograph for


mandible to close his mouth. The a patient with a 3mm anterior functional
most likely cause is shift should be taken with the patient in

A. Bell’s palsy. A. maximum intercuspation.


B. muscular dystrophy. B. initial contact. *
C. multiple sclerosis. C. normal rest position.
D. necrotizing fasciitis. D. maximum opening.
E. myasthenia gravis. * E. protrusive position.
Which articular disease most often Which of the following systemic
accompanies Sjögren’s syndrome? diseases does/do NOT predispose a
patient to periodontitis?
A. Suppurative arthritis.
B. Rheumatoid arthritis. * 1. Cyclic neutropenia.
C. Degenerative arthrosis. 2. Diabetes mellitus.
D. Psoriatic arthritis. 3. Acquired immunodeficiency
E. Lupus arthritis. syndrome.
4. Hereditary hypohydrotic ectodermal
Physiologic (racial) pigmentation dysplasia.
differs from melanoma because
melanoma A. (1) (2) (3)
B. (1) and (3)
A. is macular. C. (2) and (4)
B. contains melanin. D. (4) only
C. affects the gingiva. E. All of the above. *
D. undergoes clinical changes. *

A 12 year old male with a history of


thumbsucking has an Angle Class II

Dr Abdul Naser Tamim SEHA, Dr Ghada Al Aqqad D.D.S, Dr Kamal Naser - Amrita Medical Centre, Dr Emad Wani - Shadi Dental Centre,
Dr Lina Anka SEHA, Dr Rouba Zgheibi SEHA, Maria Teresa Yongson Alejandrino SEHA 231
For which of the following teeth is the Zinc phosphate cement, when used as
risk of root fracture increased if a a luting agent for cast restorations, has
rotational force is used during which of the following properties?
extraction?
1. Insolubility.
A. Upper canine. 2. Anticariogenicity.
B. Lower canine. 3. Chemical adhesion.
C. Upper first bicuspid. * 4. Mechanical retention.
D. Lower first bicuspid.
E. Upper lateral incisor. A. (1) (2) (3)
B. (1) and (3)
As a dentist, it is ethical to refuse to C. (2) and (4)
treat a patient on the basis of D. (4) only *
E. All of the above.
1. Religious beliefs.
2. Physical handicap. A 22 year old presents with a fracture
3. Infectious disease. of the incisal third of tooth 2.1 exposing
4. Recognition of lack of skill or a small amount of dentin. The fracture
knowledge. occurred one hour previously. There is
no mobility of the tooth but the patient
A. (1) (2) (3) complains that it is rough and sensitive
B. (1) and (3) to cold. The most appropriate
emergency treatment is to
C. (2) and (4)
D. (4) only *
A. open the pulp chamber, clean the
E. All of the above.
canal and temporarily close with zinc
oxide and eugenol.
A 30 year old HIV positive patient
B. smooth the surrounding enamel and
presents for the removal of an
apply glass ionomer cement. *
abscessed second molar. The most
appropriate management is to C. smooth the surrounding enamel and
apply a calcium hydroxide cement.
A. refer to another dentist because D. place a provisional (temporary)
universal infection control procedures crown.
are insufficient.
The "smear layer" is an important
B. schedule appointments at the end of
consideration in
the day.
A. plaque accumulation.
C. treat the patient in the same way as
B. caries removal.
all other patients. *
C. pulp regeneration.
D. double glove before starting any
surgical procedures. D. dentin bonding. *

Dr Abdul Naser Tamim SEHA, Dr Ghada Al Aqqad D.D.S, Dr Kamal Naser - Amrita Medical Centre, Dr Emad Wani - Shadi Dental Centre,
Dr Lina Anka SEHA, Dr Rouba Zgheibi SEHA, Maria Teresa Yongson Alejandrino SEHA 232
Which of the following cements can
chemically bond to enamel? 1. Removes the smear layer.
2. increases dentinal permeability.
1. Zinc phosphate cement. 3. Opens the dentinal tubules.
2. Polycarboxylate cement. 4. decalcifies the intertubular and
3. Ethoxy benzoic acid cement. peritubular dentin.
4. Glass ionomer cement.
A. (1) (2) (3)
A. (1) (2) (3) B. (1) and (3)
B. (1) and (3) C. (2) and (4)
C. (2) and (4) * D. (4) only
D. (4) only E. All of the above. *?
E. All of the above.
During dental treatment, a 62 year old
In order to achieve a proper insulin-dependent diabetic, suddenly
interproximal contact when using a complains of severe, crushing,
spherical alloy, which of the following retrosternal pain. The appropriate initial
is/are essential? management would be to stop treatment
and
1. A larger sized condenser.
2. A thinner matrix band. 1. Administer sublingual nitroglycerin.
3. An anatomical wedge. 2. Administer 100 oxygen.
4. Use of mechanical condensation. 3. Monitor the patient.
4. Administer 50 dextrose
A. (1) (2) (3) * intravenously.
B. (1) and (3)
C. (2) and (4) A. (1) (2) (3) *
D. (4) only B. (1) and (3)
E. All of the above. C. (2) and (4)
D. (4) only
Which of the following affect(s) E. All of the above.
polymerization of visible light cured
composite resins? Which one of the following describes the
position of the needle tip during
1. Intensity of the light source. administration of local anesthetic for the
2. Thickness of composite resin. inferior alveolar nerve block?
3. Proximity of light source.
A. Anterior to the pterygomandibular
4. Shade of composite resin.
raphe.
A. (1) (2) (3) B. Medial to the medial pterygoid
muscle.
B. (1) and (3)
C. Superior to the lateral pterygoid
C. (2) and (4)
muscle.
D. (4) only
D. Lateral to the sphenomandibular
E. All of the above. *
ligament. *
Acid etching of dentin with 10-15%
phosphoric acid for 15-20 seconds

Dr Abdul Naser Tamim SEHA, Dr Ghada Al Aqqad D.D.S, Dr Kamal Naser - Amrita Medical Centre, Dr Emad Wani - Shadi Dental Centre,
Dr Lina Anka SEHA, Dr Rouba Zgheibi SEHA, Maria Teresa Yongson Alejandrino SEHA 233
Which of the following constituents of 4. be semilunar in shape.
a local anesthetic cartridge is most
likely to be allergenic? A. (1) (2) (3) *
B. (1) and (3)
A. Lidocaine. C. (2) and (4)
B. Epinephrine. D. (4) only
C. Metabisulfite. * E. All of the above.
D. Hydrochloric acid.
Recurring tooth rotations occur most
Cultures made from a dental abscess frequently after orthodontic correction
indicate the infection is caused by beta due to
hemolytic streptococcus. Which of the
following is the drug of choice? A. density of the cortical bone.
B. persistence of tongue and finger
A. Penicillin. * habits.
B. Erythromycin. C. free gingival and transseptal fibres. *
C. Tetracycline. D. oblique fibres of the periodontal
D. Cloxacillin. ligament.

Titanium implants in the oral cavity are The Frankel functional regulator
CONTRAINDICATED for patients who appliance performs all of the following
EXCEPT
A. are over age 75.
B. are on thyroid replacement therapy. A. increasing vertical dimension.
C. have a terminal disease. * B. repositioning the mandible forward.
D. have diabetes mellitus (controlled). C. retraction of the maxillary molars. *
D. expansion of the dental arches.
In the surgical removal of an impacted
mandibular third molar, which of the A maxillary central incisor that is
following would be considered to be erupting in a lingually directed path
the most difficult? should be

A. Mesio-angular. A. corrected before it reaches the


B. Horizontal. occlusal plane. *
C. Vertical. B. allowed to erupt until all incisors can
D. Disto-angular. * be banded.
C. allowed to erupt into cross-bite and
The design of a mucoperiosteal flap then corrected.
should D. ignored because pressures of the
tongue will correct it as it erupts.
1. Provide for visual access. E. ignored because pressures of the lip
2. Provide for instrument access. will cause the problem to recur.
3. Permit repositioning over a solid
bone base.

Dr Abdul Naser Tamim SEHA, Dr Ghada Al Aqqad D.D.S, Dr Kamal Naser - Amrita Medical Centre, Dr Emad Wani - Shadi Dental Centre,
Dr Lina Anka SEHA, Dr Rouba Zgheibi SEHA, Maria Teresa Yongson Alejandrino SEHA 234
The best space maintainer to prevent D. translation.
the lingual collapse that often occurs
following the early loss of a mandibular Excessive orthodontic force used to
primary canine is a move a tooth may

A. Nance expansion arch. 1. Cause hyalinization.


B. lingual arch. * 2. Cause root resorption.
C. band and loop space maintainer. 3. Crush the periodontal ligament.
D. distal shoe space maintainer. 4. Impair tooth movement.

Which of the following conditions is A. (1) (2) (3)


most appropriately treated upon B. (1) and (3)
diagnosis? C. (2) and (4)
D. (4) only
A. A maxillary midline diastema. E. All of the above. *
B. Posterior cross-bite with midline
discrepancies. * An anterior cross-bite of a permanent
C. Improper axial inclination. maxillary incisor in a mixed dentition is
D. End-to-end molar relationships. often associated with

Following loss of a permanent A. a functional shift.


mandibular first molar at age 8, which B. unexplainable genetic factors.
of the following changes are likely to C. lingually situated supernumerary
occur? teeth.
D. prolonged retention of a primary
1. Distal drift of second premolar. incisor. *
2. No movement of second premolar. E. premature eruption of a maxillary
3. Mesial drift of second permanent incisor.
molar.
4. No movement of second permanent Premature loss of a primary maxillary
molar. second molar usually produces a
A. (1) (2) (3) malocclusion in the permanent dentition
B. (1) and (3) * that is characterized by
C. (2) and (4)
D. (4) only A. anterior crowding.
E. All of the above. B. labially displaced maxillary canines.
C. delayed eruption of the permanent
The predominant type of movement first molar.
produced by a finger spring on a D. a Class II molar relationship on the
removable appliance is affected side. *
E. a Class III molar relationship on the
A. torque. affected side.
B. tipping. *
C. rotation.

Dr Abdul Naser Tamim SEHA, Dr Ghada Al Aqqad D.D.S, Dr Kamal Naser - Amrita Medical Centre, Dr Emad Wani - Shadi Dental Centre,
Dr Lina Anka SEHA, Dr Rouba Zgheibi SEHA, Maria Teresa Yongson Alejandrino SEHA 235
The most damaging characteristic of an C. improve periodontal health of
Angle Class II, division 2 malocclusion abutment teeth.
is the D. decrease costs.

A. deep overbite. * For teeth prepared as abutments for


B. crowding of the mandibular incisors. fixed bridges, unsatisfactory temporary
C. relationship of the molars. crown restorations may result in
D. impaction of the maxillary canines.
E. malposition of the maxillary lateral 1. Tooth sensitivity.
incisors. 2. Gingival recession.
3. Tooth migration.
The junctional epithelium, once it has 4. Occlusal prematurities.
migrated apically, attaches to the
cementum by means of A. (1) (2) (3)
B. (1) and (3)
A. collagen fibres. C. (2) and (4)
B. oxytalan fibres. D. (4) only
C. desmosomes. E. All of the above. *
D. hemidesmosomes. *
A hinge axis facebow records
Compared to unfilled resins, composite
resins have A. Bennett angle.
B. centric relation.
1. Reduced thermal dimensional C. lateral condylar inclination.
changes. D. horizontal condylar inclination.
2. increased strength. E. opening and closing axis of the
3. Reduced polymerization shrinkage. mandible. *
4. Better polishability.
Initial scaling and oral hygiene
A. (1) (2) (3) * instruction in the treatment of
B. (1) and (3) periodontitis results in all of the
C. (2) and (4) following EXCEPT
D. (4) only
E. All of the above. A. pocket shrinkage.
B. decreased hemorrhage during
Mandibular overdentures are used to surgery.
C. evaluation of the patient's motivation.
A. increase the strength of the denture. D. improved healing after surgery.
B. maintain the alveolar ridge E. correction of pathological migration
morphology. * of teeth. *

Dr Abdul Naser Tamim SEHA, Dr Ghada Al Aqqad D.D.S, Dr Kamal Naser - Amrita Medical Centre, Dr Emad Wani - Shadi Dental Centre,
Dr Lina Anka SEHA, Dr Rouba Zgheibi SEHA, Maria Teresa Yongson Alejandrino SEHA 236
When using the periodontal probe to E. All of the above.
measure pocket depth, the
measurement is taken from the The accuracy of alginate impression
materials will be improved if
A. base of the pocket to the
cementoenamel junction. A. the space between the tray and the
B. free gingival margin to the teeth is 1-2mm.
cementoenamel junction. B. the space between the tray and the
C. base of the pocket to the crest of teeth allows 4-5mm of alginate. *
the free gingiva. * C. the impression is removed slowly
D. base of the pocket to the from the undercuts around the teeth.
mucogingival junction. D. the impression is soaked in water
for 1 hour.
Which of the following methods
decrease radiation exposure to The choice and number of abutments
patients? for a fixed partial denture is influenced
by the
1. Thyroid collar and lead apron.
2. Rectangular collimation. 1. Length of the span of the fixed
3. High speed films. partial denture.
4. High kilovoltage. 2. crown-root ratio of the abutments.
3. Amount of periodontal support of the
A. (1) (2) (3) abutments.
B. (1) and (3) 4. Position of the abutments in the
C. (2) and (4) arch.
D. (4) only
E. All of the above. * A. (1) (2) (3)
B. (1) and (3)
Which of the following C. (2) and (4)
pharmacokinetic change(s) occur(s) D. (4) only
with aging? E. All of the above. *

1. Absorption is altered by a decrease A hardened gold alloy will exhibit


in the gastric pH.
2. Metabolism is decreased by a A. less plastic deformation per unit of
reduced liver mass. stress than the same alloy in a
3. Distribution is altered by a softened condition. *
decrease in total body fat. B. greater plastic deformation per unit
4. Excretion is reduced because of of stress than the same alloy in a
lessened renal blood flow. softened condition.
C. no difference in the plastic
A. (1) (2) (3) deformation per unit of stress of the
B. (1) and (3) alloy in hard or soft condition.
C. (2) and (4) *
D. (4) only

Dr Abdul Naser Tamim SEHA, Dr Ghada Al Aqqad D.D.S, Dr Kamal Naser - Amrita Medical Centre, Dr Emad Wani - Shadi Dental Centre,
Dr Lina Anka SEHA, Dr Rouba Zgheibi SEHA, Maria Teresa Yongson Alejandrino SEHA 237
In clinical dentistry, stiffness of wire is a 1. Grow slowly.
function of 2. are generally painless.
3. Can be managed conservatively.
A. length of the wire segment. 4. Can metastasize.
B. diameter of the wire segment.
C. alloy composition. A. (1) (2) (3) *
D. All of the above. * B. (1) and (3)
E. None of the above. C. (2) and (4)
D. (4) only
In acutely inflamed gingival tissue, there E. All of the above.
is an increase in the
A surgical flap not repositioned over
A. number of mast cells. a bony base will result in
B. number of plasma cells.
C. level of histamine. 1. Slower healing.
D. A. and B. 2. Foreign body inflammatory
E. A. and C. * reaction.
3. Wound dehiscence.
A bacterial enzyme capable of altering 4. Necrosis of bone.
the ground substance of the periodontal
ligament is A. (1) (2) (3)
B. (1) and (3) *
A. amylase. C. (2) and (4)
B. hyaluronidase. * D. (4) only
C. dextranase. E. All of the above.
D. streptokinase.
In the management of a patient with
A removable orthodontic appliance, an acute odontogenic infection, the
producing a light force on the labial of a treatment should include:
proclined maxillary central incisor will
cause 1. Elimination of the cause.
2. Drainage.
A. lingual movement of the crown and 3. Supportive therapy.
lingual movement of the root apex. * 4. Tetanus immunization.
B. intrusion of the central incisor and
lingual movement of the crown. A. (1) (2) (3) *
C. lingual movement of the crown and B. (1) and (3)
labial movement of the root apex. C. (2) and (4)
D. intrusion of the central incisor. D. (4) only
E. All of the above.
Benign neoplasms

Dr Abdul Naser Tamim SEHA, Dr Ghada Al Aqqad D.D.S, Dr Kamal Naser - Amrita Medical Centre, Dr Emad Wani - Shadi Dental Centre,
Dr Lina Anka SEHA, Dr Rouba Zgheibi SEHA, Maria Teresa Yongson Alejandrino SEHA 238
Which of the following is the LEAST
likely primary site for the development A. amount of overbite.
of oral squamous cell carcinoma in the B. age of the patient. *
elderly? C. cooperation of the patient.
D. practitioner preference.
A. Dorsum of the tongue. *
B. Floor of the mouth. The most appropriate time to correct a
C. Lateral border of the tongue. cross-bite of the permanent maxillary
D. Tonsillar fossa. central incisor is

The most frequent cause of tooth loss in A. following eruption of the canines.
the elderly is B. following eruption of the central
incisors.
A. bruxism. C. following eruption of the lateral
B. caries. incisors.
C. periodontal disease. * D. during eruption of the central
D. use of a removable partial denture. incisors. *
E. extra oral trauma.
Excessive force in orthodontic tooth
For which of the following malocclusions movement is positively correlated with
is serial extraction most appropriate?
1. Faster movement.
A. Angle Class I. * 2. Slower movement.
B. Angle Class II. 3. Root resorption.
C. Angle Class III. 4. Periodontal damage.

Serial extraction may result in A. (1) (2) (3)


B. (1) and (3)
1. Deepening of the overbite. C. (2) and (4)
2. Lingual tipping of the mandibular D. (4) only
incisors. E. All of the above. *
3. Regional extraction spacing.
4. Uncontrolled tipping of the permanent Angle’s "subdivision" refers to an
teeth. abnormal molar relationship that is

A. (1) (2) (3) A. bilateral.


B. (1) and (3) B. unilateral. *
C. (2) and (4) C. functional.
D. (4) only D. transitional.
E. All of the above. * E. traumatic.

The appliance of choice to correct an


anterior crossbite is determined by the

Dr Abdul Naser Tamim SEHA, Dr Ghada Al Aqqad D.D.S, Dr Kamal Naser - Amrita Medical Centre, Dr Emad Wani - Shadi Dental Centre,
Dr Lina Anka SEHA, Dr Rouba Zgheibi SEHA, Maria Teresa Yongson Alejandrino SEHA 239
Which of the following conditions B. an interference free closure to
should NOT commonly be treated maximal intercuspidation. *
during the mixed dentition stage? C. a deviated closure to maximal
intercuspidation.
A. Anterior cross-bite. D. a large mandible.
B. Posterior cross-bite.
C. Maxillary incisor rotation. * The ANB angle in severe Class II
D. Class II molar relationship. malocclusions is most often

Which Angle’s malocclusion is most A. large. *


commonly associated with mouth B. small.
breathing? C. within normal limits.

A. Class I. Cephalometric standards


B. Class II, division 1. *
C. Class II, division 2. 1. Are racially biased.
D. Class III. 2. May vary with patient development.
3. Include a range of application.
Which of the following congenital 4. Are the basis for diagnosis.
problems often results in a
malocclusion? A. (1) (2) (3) *
B. (1) and (3)
1. Cleft palate. C. (2) and (4)
2. Ectodermal dysplasia. D. (4) only
3. Pierre Robin syndrome. E. All of the above.
4. Cleidocranial dysostosis.
The most common cause of
A. (1) (2) (3) malocclusion with a Class I molar
B. (1) and (3) relationship is
C. (2) and (4)
D. (4) only A. a thumbsucking habit.
E. All of the above. * B. crossbite in the posterior segments.
C. tooth size and jaw size
The defining future of a skeletal discrepancy. *
crossbite is D. improper eruption of permanent
first molars.
A. coincident midlines.

Dr Abdul Naser Tamim SEHA, Dr Ghada Al Aqqad D.D.S, Dr Kamal Naser - Amrita Medical Centre, Dr Emad Wani - Shadi Dental Centre,
Dr Lina Anka SEHA, Dr Rouba Zgheibi SEHA, Maria Teresa Yongson Alejandrino SEHA 240
A 9 year old boy sustains a fracture of 2. Cardiac pacemaker.
the crown of his central incisor with 3. Prosthetic heart valves.
minimal pulp exposure. He is brought 4. All heart murmurs.
to your office within one hour of the
injury. A periapical radiograph reveals A. (1) (2) (3)
that the root of the tooth is complete B. (1) and (3) *
but the apex is open. The most C. (2) and (4)
appropriate management is to D. (4) only
E. All of the above.
A. pulp cap with calcium hydroxide and
restore. * Intravenous administration of
B. perform a pulpotomy and seal epinephrine results in
temporarily.
C. perform a pulpectomy and seal 1. increased systolic pressure.
temporarily. 2. increased heart rate.
D. perform a pulpectomy and fill 3. Palpitations.
immediately with gutta-percha and 4. Respiratory depression.
restore.
A. (1) (2) (3) *
When cementing an inlay, the best
B. (1) and (3)
procedure to ensure accurate seating is
C. (2) and (4)
a
D. (4) only
A. continuous firm pressure on the inlay E. All of the above.
until the cement is set. *
In an infection caused by non-
B. thick mix of cement.
penicillinase producing staphylococcus,
C. rapid and heavy application of
the drug of choice is
pressure until the inlay is seated.
D. very thin mix of cement.
A. penicillin V. *
B. cephalexin.
Water irrigation devices have been
shown to C. tetracycline.
D. vancomycin.
A. eliminate plaque.
Which of the following drugs is used in
B. dislodge food particles from between
the treatment of candida albicans
teeth. *
infections?
C. disinfect pockets for up to 18 hours.
D. prevent calculus formation.
A. Penicillin.
B. Nystatin. *
Antibiotic prophylaxis is recommended
for patients with which of the following? C. Chlorhexidine.
D. Tetracycline.
1. Mitral valve prolapse with
regurgitation.

Dr Abdul Naser Tamim SEHA, Dr Ghada Al Aqqad D.D.S, Dr Kamal Naser - Amrita Medical Centre, Dr Emad Wani - Shadi Dental Centre,
Dr Lina Anka SEHA, Dr Rouba Zgheibi SEHA, Maria Teresa Yongson Alejandrino SEHA 241
Sodium salicylate is administered to The best means of extending the
a patient with rheumatoid arthritis working time of an irreversible
because it is a/an hydrocolloid impression material is to

A. anti-inflammatory agent. * A. extend spatulation time.


B. sedative. B. add additional water.
C. antiseptic. C. use cold water. *
D. antiplatelet agent. D. add a small amount of borax.
E. add potassium sulfate.
Which of the following anesthetic
agents are hydrolyzed by plasma In patients wearing complete dentures,
cholinesterase? the most frequent cause of tooth
contact (clicking) during speaking is
1. Prilocaine.
2. Lidocaine. A. nervous tension.
3. Mepivacaine. B. incorrect centric relation position.
4. Procaine. C. excessive occlusal vertical
dimension. *
A. (1) (2) (3) D. lack of vertical overlap.
B. (1) and (3) E. unbalanced occlusion.
C. (2) and (4)
D. (4) only * A patient with complete dentures
E. All of the above. complains of clicking. The most
common causes are
Which permanent teeth will
commonly be present in an 8 year A. reduced vertical dimension and
old child? improperly balanced occlusion.
B. excessive vertical dimension and
A. All central and lateral incisors. poor retention. *
B. All central and lateral incisors and C. use of too large a posterior tooth
first molars. * and too little horizontal overlap.
C. All central and lateral incisors, first D. improper relation of teeth to the
molars and first premolars. ridge and excessive anterior vertical
D. Only mandibular central and overlap.
lateral incisors and first molars.
For a mandibular denture impression,
Longer trituration of alloy and the muscle determining the form of the
mercury will result in an amalgam lingual flange in the molar region is
restoration which will have
A. mylohyoid. *
A. greater expansion during setting. B. geniohyoid.
B. reduced strength. C. medial pterygoid.
C. increased flow. * D. lateral pterygoid.
E. superior pharyngeal constrictor.

Dr Abdul Naser Tamim SEHA, Dr Ghada Al Aqqad D.D.S, Dr Kamal Naser - Amrita Medical Centre, Dr Emad Wani - Shadi Dental Centre,
Dr Lina Anka SEHA, Dr Rouba Zgheibi SEHA, Maria Teresa Yongson Alejandrino SEHA 242
In complete denture construction, a 3. Traumatize the underlying
custom tray should be fabricated to supporting tissues.
ensure that 4. Induce generalized soreness over
the ridges.
1. The flanges are not overextended.
2. The distal extension is adequate. A. (1) (2) (3)
3. There is relief for muscle B. (1) and (3)
attachments. C. (2) and (4)
4. The tray has adequate space for the D. (4) only
impression material. E. All of the above. *

A. (1) (2) (3) In complete denture fabrication, the


B. (1) and (3) angulation of the horizontal condylar
C. (2) and (4) guidance plane of the articulator is
D. (4) only determined by the
E. All of the above. *
A. cuspal inclination of the teeth
In partial denture design, the major selected.
connector should B. orientation of the occlusal plane.
C. incisal guidance.
A. rigidly connect the bilateral D. centric relation interocclusal record.
components. * E. protrusive interocclusal record. *
B. act as a stress-breaker.
C. not interfere with lateral forces. In designing a removable partial
D. dissipate vertical forces. denture, an effort is made to secure
parallel tooth surfaces to act as
The retention form of a full crown
preparation can be improved by A. guiding planes. *
B. occlusal rest areas.
1. Reducing the taper. C. bracing areas.
2. Increasing axial height. D. indirect retention.
3. Utilizing grooves or boxes.
4. Wider preparation margins. A Kennedy Class II denture with no
provision for indirect retention causes
A. (1) (2) (3) *
B. (1) and (3) A. resorption of the supporting
C. (2) and (4) edentulous area.
D. (4) only B. dislodgement of the saddle area
E. All of the above. during mastication. *
C. temporomandibular joint
Reduction of free way space in dysfunction.
complete denture construction will D. gingivitis.

1. Impair aesthetics.
2. Cause the dentures to click.

Dr Abdul Naser Tamim SEHA, Dr Ghada Al Aqqad D.D.S, Dr Kamal Naser - Amrita Medical Centre, Dr Emad Wani - Shadi Dental Centre,
Dr Lina Anka SEHA, Dr Rouba Zgheibi SEHA, Maria Teresa Yongson Alejandrino SEHA 243
In taking an interocclusal wax record in
a protrusive position, the dentist should A. should result in a long bevel.
examine the wax record to insure that B. is contraindicated because of the
low edge strength of amalgam.
A. the incisal edges of the anterior teeth C. is unnecessary since the tooth
have made contact. structure in this area is strong.
B. the patient has not closed in a lateral D. should remove unsupported enamel
position. which may fracture. *
C. all cusps have penetrated the wax E. should result in a sharp
record and are in contact with the gingivoproximal line angle.
opposing teeth.
D. there is no perforation of the wax It is advisable to polish any restorative
record. * material as smoothly as possible in
order to prevent
Which of the following is/are (a) useful
guide(s) in determining a patient’s A. microleakage.
occlusal vertical dimension? B. accumulation of plaque. *
C. overhanging margins.
1. Appearance. D. electro-chemical action.
2. Phonetics.
3. Observation of the rest position. To ensure maximum marginal strength
4. Pre-extraction profile records. for an amalgam restoration the
cavosurface angle should
A. (1) (2) (3)
B. (1) and (3) A. approach 45 degrees.
C. (2) and (4) B. approach 90 degrees. *
D. (4) only. C. be bevelled.
E. All of the above. * D. be chamfered.

The area of the tooth that is most A characteristic sign of aggressive


sensitive during cavity preparation is periodontitis in an adolescent (juvenile
periodontitis) is
A. dentin.
B. cementum. A. marginal gingivitis.
C. cementoenamel junction. B. painful, burning gingivae.
D. dentinoenamel junction. * C. hyperplastic gingivitis.
D. drifting of the teeth. *
Planing the enamel at the gingival
cavosurface of a Class II amalgam
preparation on a permanent tooth

Dr Abdul Naser Tamim SEHA, Dr Ghada Al Aqqad D.D.S, Dr Kamal Naser - Amrita Medical Centre, Dr Emad Wani - Shadi Dental Centre,
Dr Lina Anka SEHA, Dr Rouba Zgheibi SEHA, Maria Teresa Yongson Alejandrino SEHA 244
Characteristics of Peutz-Jeghers 2. Non-irritating to pulpal tissue at
syndrome include moderate depth.
3. Anticariogenic because of fluoride
A. melanin pigmentation of the lips. * release.
B. yellowish spots on the oral mucosa.
C. small, papillary lesions on the A. (1) only
palate. B. (1) and (2)
D. a rhomboid-shaped red patch on C. (1) and (3)
the dorsum of the tongue. D. All of the above. *

Particulate hydroxyapatite, when Which of the following modifications to


placed subperiostially, the standard procedure for mixing
gypsum products will increase the
1. is highly biocompatible. compressive strength of the set
2. Has a low incidence of secondary material?
infection following surgery.
3. Has a tendency to migrate following A. Adding a small amount of salt to the
insertion. water before mixing.
4. Induces bone formation throughout B. Decreasing the water/powder ratio by
the implanted material. a small amount. *
C. Using warmer water.
A. (1) (2) (3) D. Decreasing the mixing time.
B. (1) and (3)
C. (2) and (4) Filters are placed in the path of the x-
D. (4) only ray beam to
E. All of the above. *
A. increase contrast.
When gold or a gold alloy changes B. reduce film density.
from a liquid to a solid state it C. reduce exposure time.
D. reduce patient radiation dose. *
A. expands.
B. contracts. * Which of the following increases the
C. corrodes. sharpness of a radiograph?
D. becomes brittle.
E. work hardens. A. Small focal spot. *
B. Short focal spot-to-film distance.
Which of the following properties apply C. Long object-to-film distance.
to glass ionomer cements?

1. Chemical bonding to etched dentin.

Dr Abdul Naser Tamim SEHA, Dr Ghada Al Aqqad D.D.S, Dr Kamal Naser - Amrita Medical Centre, Dr Emad Wani - Shadi Dental Centre,
Dr Lina Anka SEHA, Dr Rouba Zgheibi SEHA, Maria Teresa Yongson Alejandrino SEHA 245
Propylthiouracil is a drug used in the 4. Nonpermeable.
treatment of
A. (1) (2) (3) *
A. leukemia. B. (1) and (3)
B. hyperthyroidism. * C. (2) and (4)
C. sodium retention. D. (4) only
D. intestinal parasites. E. All of the above.

The severity of the course of Before performing surgery on a patient


periodontal disease depends upon the who is taking warfarin, which of the
following should be evaluated?
A. resistance of the host.
B. virulence of the organism. A. Bleeding time.
C. number of organisms present. B. Clotting time.
D. A. and B. C. Prothrombin time. *
E. A., B. and C. * D. Coagulation time.

Juvenile periodontitis In inhalation analgesia, what is the


safe maximal nitrous oxide
A. is associated with gram-negative concentration that can be delivered?
anaerobic flora. *
B. is associated with gram-positive A. 50%.
anaerobic flora. B. 60%.
C. is associated with root caries. C. 70%. *
D. has a definite predilection toward D. 80%.
males.
When uprighting a molar to be used as
Tachycardia is a term which describes a bridge abutment, consideration must
a pulse rate of be given to the

A. less than 50. A. local periodontium.


B. less than 60. B. vertical changes due to tipping of
C. less than 70. the molar.
D. more than 70. * C. residual space for a pontic.
D. use of fixed appliances for optimum
Normal sulcular epithelium in man is control.
E. All of the above. *
1. Nonkeratinized.
2. Squamous.
3. Stratified.

Dr Abdul Naser Tamim SEHA, Dr Ghada Al Aqqad D.D.S, Dr Kamal Naser - Amrita Medical Centre, Dr Emad Wani - Shadi Dental Centre,
Dr Lina Anka SEHA, Dr Rouba Zgheibi SEHA, Maria Teresa Yongson Alejandrino SEHA 246
Cleft palate may result in a higher For a patient with complete dentures,
incidence of insufficient space between the
maxillary tuberosity and the
1. Supernumerary teeth. retromolar pad will require
2. Congenitally missing teeth.
3. Altered crown morphology. A. avoiding covering the pad with the
4. Orthognathic surgery. mandibular base.
B. not covering the tuberosity with the
A. (1) (2) (3) maxillary base.
B. (1) and (3) C. surgically reducing the retromolar
C. (2) and (4) * pad.
D. (4) only D. surgically reducing the maxillary
E. All of the above. tuberosity. *

Immediately after tooth extraction, a Extreme resorption of an edentulous


patient with a known cardiac problem mandible can bring the alveolar ridge
experiences angina. This is most likely to the level of the attachment of the
precipitated by
A. buccinator, styloglossus and
A. the patient's upright position in the geniohyoid muscles.
chair. B. mylohyoid, buccinator and
B. an allergy to the anesthetic agent. styloglossus muscles.
C. anxiety and anticipation of pain. * C. superior constrictor, mylohyoid and
D. the epinephrine in the anesthetic buccinator muscles.
agent. D. mylohyoid, buccinator and
genioglossus muscles. *
Following orthodontic alignment,
relapse of the mandibular incisors Prevention of gingival irritation by a
CANNOT major connector of a removable
partial denture is accomplished by
A. be predicted from characteristics of
the original malocclusion. * A. using split palatal bars.
B. occur if the second or third molars B. reducing the size of the connector.
are removed. C. ensuring maximum distribution of
C. occur if retainers are worn until the occlusal forces.
mandibular growth is complete. D. providing relief between the
D. occur if a supracrestal fiberotomy is connector and the gingiva. *
performed.
Upon setting, a mixture of plaster of
Habitual thumbsucking that continues Paris and water will exhibit
after the age of six commonly results in
A. loss in compressive strength.
A. anterior open bite. * B. expansion. *
B. skeletal malocclusion. C. gain in moisture content.
C. mouth breathing. D. contraction.
D. sinusitis.

Dr Abdul Naser Tamim SEHA, Dr Ghada Al Aqqad D.D.S, Dr Kamal Naser - Amrita Medical Centre, Dr Emad Wani - Shadi Dental Centre,
Dr Lina Anka SEHA, Dr Rouba Zgheibi SEHA, Maria Teresa Yongson Alejandrino SEHA 247
To improve denture stability,
mandibular molar teeth should A. pterygomaxillary notches and the
normally be placed fovea palatinae. *
B. pterygomaxillary notches and the
A. over the crest of the mandibular posterior nasal spine.
ridge. * C. posterior border of the tuberosities
B. buccal to the crest of the mandibular and the posterior border of the palatine
ridge. bone.
C. over the buccal shelf area. D. anterior border of the tuberosities,
D. lingual to the crest of the mandibular the palatine raphe and the posterior
ridge. border of the palatine bone.

Irreversible hydrocolloid materials are Radiographically, the lamina dura is a


best removed from the mouth by
A. thick layer of bone forming the inner
A. a quick snap. * surface of the alveolus.
B. a slow teasing motion. B. thin radiolucent line around the roots
C. twisting and rocking. of the teeth.
D. having the patient create a positive C. thick layer of cortical bone.
pressure. D. thin radiopaque line around the
roots of the teeth. *
If an impression were taken with a
polysulfide impression material of teeth In the bisecting angle principle of
exhibiting severe external undercuts a intraoral radiography, the radiopacity
stone model should be poured that can obliterate the apices of
maxillary molars is the
A. immediately.
B. after 30 minutes. * A. maxillary sinus.
C. after 24 hours. B. palatine bone and the zygoma.
C. orbital process of the zygomatic
In treatment planning for a complete bone.
denture, severe undercuts on the D. zygoma and the zygomatic process
edentulous ridge should be of the maxilla. *

A. considered as retentive aids. Hyperplastic lingual tonsils may


B. removed surgically. * resemble which of the following?
C. retained but avoided by the denture
base extension. A. Epulis fissuratum.
D. removed if no cortical bone is B. Lingual varicosities.
involved. C. Squamous cell carcinoma. *
D. Median rhomboid glossitis.
The anatomical landmarks used to help E. Prominent fungiform papillae.
establish the location of the posterior
palatal seal of a maxillary complete
denture include the

Dr Abdul Naser Tamim SEHA, Dr Ghada Al Aqqad D.D.S, Dr Kamal Naser - Amrita Medical Centre, Dr Emad Wani - Shadi Dental Centre,
Dr Lina Anka SEHA, Dr Rouba Zgheibi SEHA, Maria Teresa Yongson Alejandrino SEHA 248
Erratic and inconsistent electric pulp C. insufficient iodine. *
test results can be explained by D. excess calcium.
E. excess sodium.
A. the presence of multiple canals in
various stages of pulp pathosis. In pin-retained restorations, the pin
B. failure to isolate and dry the tooth. holes should be parallel to the
C. pulpal calcification.
D. poor contact between the electrode A. long axis of the tooth.
and the tooth. B. nearest external surface. *
E. All of the above. * C. pulp chamber.
D. axial wall.
Which of the following has the highest
rate of recurrence? Using pins to retain amalgam
restorations increases the risk of
A. Odontogenic keratocyst. *
B. Nasoalveolar cyst. 1. Cracks in the teeth.
C. Median palatal cyst. 2. Pulp exposures.
D. Incisive canal cyst. 3. Thermal sensitivity.
4. Periodontal ligament invasion.
Which of the following conditions is
characterized by abnormally large pulp A. (1) (2) (3)
chambers? B. (1) and (3)
C. (2) and (4)
A. Amelogenesis imperfecta. D. (4) only
B. Regional odontodysplasia. * E. All of the above. *
C. Dentinogenesis imperfecta.
D. Dentinal dysplasia Type I. Kilovoltage controls the

Cements used as cavity liners 1. Contrast of the image.


2. Quantity of X-rays produced.
A. provide a barrier to the passage of 3. Energy of the X-rays.
irritants from restorative materials. 4. Temperature of the cathode
B. reduce sensitivity of freshly cut filament.
dentin.
C. provide thermal insulation to the A. (1) (2) (3)
pulp. B. (1) and (3) *
D. All of the above. * C. (2) and (4)
D. (4) only
Enlargement of the thyroid gland can E. All of the above.
be caused by

A. insufficient fluoride.
B. excess iodine.

Dr Abdul Naser Tamim SEHA, Dr Ghada Al Aqqad D.D.S, Dr Kamal Naser - Amrita Medical Centre, Dr Emad Wani - Shadi Dental Centre,
Dr Lina Anka SEHA, Dr Rouba Zgheibi SEHA, Maria Teresa Yongson Alejandrino SEHA 249
Which of the following, if left untreated, D. None of the above.
is most likely to result in a periapical
lesion? Which of the following bacterial
products have been implicated in the
A. Internal resorption. initiation of inflammatory periodontal
B. Reversible pulpitis. disease?
C. Acute suppurative pulpitis. *
D. Chronic hyperplastic pulpitis. 1. Protease.
E. Diffuse calcification of the pulp. 2. Hyaluronidase.
3. Neuraminidase.
Which of the following is the greatest 4. Endotoxin.
risk factor for rampant caries in 5. Desxyribonuclease.
children?
A. (1) and (2)
A. Frequent ingestion of B. (1) (2) (4) *
polysaccharides. C. (2) (3) (4)
B. Frequent ingestion of high sucrose- D. (2) (3) (5)
containing foods. * E. (3) (4) (5)
C. Severe enamel hypoplasia.
D. Deficiency of vitamin D. The microorganism most commonly
associated with root surface caries is
Which of the following condition(s)
increase(s) susceptibility to dental A. Actinomyces viscosus. *
caries? B. Streptococcus mutans.
C. Streptococcus salivarius.
1. Vitamin K deficiency during tooth D. Lactobacillus acidophilus.
development.
2. Vitamin D deficiency during tooth Streptococcus mutans colonizes
development.
3. Hereditary fructose intolerance. A. cusp surfaces of teeth. *
4. Hyposalivation. B. pits and fissures of teeth.
C. on the tongue.
A. (1) (2) (3) D. at pH of 2.5.
B. (1) and (3)
C. (2) and (4) * Dental plaque developing on tooth
D. (4) only surfaces will result in
E. All of the above.
A. demineralization of enamel.
In long-standing gingivitis, the B. periodontal disease.
subgingival microflora shifts toward C. amelogenesis imperfecta.
D. A. and B. *
A. aerobic bacteria.
B. Gram-positive bacteria.
C. Gram-negative anaerobic bacteria. *

Dr Abdul Naser Tamim SEHA, Dr Ghada Al Aqqad D.D.S, Dr Kamal Naser - Amrita Medical Centre, Dr Emad Wani - Shadi Dental Centre,
Dr Lina Anka SEHA, Dr Rouba Zgheibi SEHA, Maria Teresa Yongson Alejandrino SEHA 250
A lowering of serum calcium is the 4. Extends more than 180° around the
stimulus for the endogenous release of tooth.

A. thyroid hormone. A. (1) (2) (3)


B. adrenocortical hormone. B. (1) and (3)
C. insulin. C. (2) and (4)
D. parathyroid hormone. * D. (4) only
E. adrenalin. E. All of the above. *

Which of the following muscles has two Repeated clasp adjustment can result
separate functions in mandibular in fracture due to
movement?
A. strain hardening. *
A. Masseter. B. increased ductility.
B. Geniohyoid. C. increasing of the modulus of
C. External (lateral) pterygoid. * elasticity.
D. Buccinator. D. lowering of the yield strength.

The location of a crown margin is Which of the following


determined by materials/techniques is
CONTRAINDICATED when using
1. Esthetic requirements. polyvinyl siloxane impression
2. Clinical crown length. materials?
3. Presence of caries.
4. Presence of an existing restoration. A. Displacement cords.
A. (1) (2) (3) B. Electrosurgery.
B. (1) and (3) C. Aluminum sulfate saturated cord. *
C. (2) and (4) D. Aluminum chloride impregnated
D. (4) only cord.
E. All of the above. *
To decrease abutment tooth
A circumferential clasp arm on a tooth sensitivity, a fixed bridge may be
is one which temporarily seated using

1. Originates above the height of A. polycarboxylate cement.


contour. B. acrylic resin cement.
2. Approaches the undercut from an C. zinc oxide eugenol cement. *
occlusal direction. D. glass ionomer cement.
3. Traverses a portion of the
suprabulge of the tooth.

Dr Abdul Naser Tamim SEHA, Dr Ghada Al Aqqad D.D.S, Dr Kamal Naser - Amrita Medical Centre, Dr Emad Wani - Shadi Dental Centre,
Dr Lina Anka SEHA, Dr Rouba Zgheibi SEHA, Maria Teresa Yongson Alejandrino SEHA 251
Heat treatment alters a gold alloy's
A. (1) (2) (3) *
A. hardness. B. (1) and (3)
B. proportional limit. C. (2) and (4)
C. percentage elongation. D. (4) only
D. All of the above. * E. All of the above.

After initial setting, a chemically cured A maxillary complete denture exhibits


glass ionomer cement restoration more retention and stability than a
should have a coating agent applied to mandibular one because it

A. hasten the final set. 1. Covers a greater area.


B. protect the cement from moisture. * 2. Incorporates a posterior palatal seal.
C. retard the final set. 3. Is not subject to as much muscular
D. protect the cement from ultraviolet displacement.
light. 4. Is completely surrounded by soft
E. create a smooth finish. tissue.

The prime advantage of vacuum firing A. (1) (2) (3) *


of porcelain is B. (1) and (3)
C. (2) and (4)
A. better colour. D. (4) only
B. less shrinkage. E. All of the above.
C. more translucency.
D. increased strength. * Cementing a full gold crown with zinc
phosphate includes
A measure of the stiffness of a dental
gold alloy is expressed as its 1. Applying a thick coat of cement
internally.
A. proportional limit. 2. Application of continuous loading.
B. modulus of elasticity. * 3. Cleaning excess cement off while
C. ultimate tensile strength. setting.
D. flow. 4. Having excess cement covering the
margins.
The purpose of relining a distal saddle
of a removable partial denture is to A. (1) (2) (3)
improve B. (1) and (3)
C. (2) and (4) *
1. Tissue adaptation. D. (4) only
2. Occlusion. E. All of the above.
3. Function.
4. Fit of the framework.

Dr Abdul Naser Tamim SEHA, Dr Ghada Al Aqqad D.D.S, Dr Kamal Naser - Amrita Medical Centre, Dr Emad Wani - Shadi Dental Centre,
Dr Lina Anka SEHA, Dr Rouba Zgheibi SEHA, Maria Teresa Yongson Alejandrino SEHA 252
In a removable partial denture, the
major connector should begin 3-6mm Gold casting alloys are classified as
from the free gingival margin in order to Type I-IV according to which of the
following physical properties?
A. improve phonetics.
B. allow sufficient length for the minor A. Percentage of gold present in the
connectors. alloy.
C. prevent inflammation of the gingival B. Surface hardness. *
tissues. * C. Melting point.
D. prevent rotation of the major D. Elastic strength.
connector in an antero-posterior E. Ductility.
direction.
Gold contributes which of the following
The presence of tin in an amalgam properties to a gold-copper alloy?
alloy modifies the reaction and physical
properties of the amalgam in that it A. Corrosion resistance. *
B. Increased strength.
A. speeds the amalgamation rate. * C. Lowered specific gravity.
B. enhances strength. D. Increased hardness.
C. enhances tarnish resistance.
D. reduces flow. The binder in casting investments not
only strengthens the investment, but
Prior to cementing an onlay in a vital also
tooth using a resin cement, the
application of cavity varnish will A. creates a reducing atmosphere in the
mold.
A. protect the pulp. B. tends to reduce both hygroscopic
B. improve seal. and thermal expansions.
C. reduce prevent postoperative C. increases the thermal expansion of
sensitivity. the mold.
D. reduce bond strength. * D. contributes to the overall expansion
E. reduce microleakage. of the mold. *

The interocclusal distance (freeway The use of a retraction cord


space) impregnated with 8% racemic
epinephrine may be hazardous for
A. is the difference between occlusal some patients because of its
vertical dimension and hinge axis
registration. A. local caustic action on the gingival
B. is the difference between occlusal tissue.
vertical dimension and rest vertical B. potential for systemic reaction. *
dimension. * C. local astringent action.
C. is usually l0mm in the premolar
region.
D. is the distance between centric
occlusion and centric relation.

Dr Abdul Naser Tamim SEHA, Dr Ghada Al Aqqad D.D.S, Dr Kamal Naser - Amrita Medical Centre, Dr Emad Wani - Shadi Dental Centre,
Dr Lina Anka SEHA, Dr Rouba Zgheibi SEHA, Maria Teresa Yongson Alejandrino SEHA 253
Which of the following problems of a In processing (polymerizing) an acrylic
permanent fixed bridge, detected at the denture in a water bath, a correct
delivery appointment, can be caused temperature control is desired because
by an inadequate temporary of the possibility of
restoration?
A. shrinkage of the denture.
1. Hypersensitivity of the abutments B. volumetric expansion of the denture
that decreases after permanent luting. causing an open bite.
2. Exposed gingival margins in an area. C. warpage.
3. Contacts with adjacent teeth that D. porosity due to boiling of the
prevent complete seating of the bridge. monomer. *
4. Need for significant occlusal E. crazing of the denture base around
adjustment. the necks of the teeth.

A. (1) (2) (3) In the processing of methyl


B. (1) and (3) methacrylate, denture porosity is most
C. (2) and (4) likely to appear in the
D. (4) only
E. All of the above. * A. thickest portion. *
B. thinnest portion.
Upon examination of an edentulous C. buccal surface.
patient, it is observed that the D. palatal area.
tuberosities contact the retromolar
pads at the correct occlusal vertical Dental porcelain has
dimension. The treatment of choice is
to 1. Low compressive strength.
2. High hardness.
A. reduce the retromolar pads 3. High tensile strength.
surgically to provide the necessary 4. Low impact strength.
clearance.
B. reduce the tuberosities surgically to A. (1) (2) (3)
provide the necessary clearance. * B. (1) and (3)
C. construct new dentures at an C. (2) and (4) *
increased occlusal vertical dimension D. (4) only
to gain the necessary clearance. E. All of the above.
D. proceed with construction of the
denture and reduce the posterior A metal in the wrought condition differs
extension of the mandibular denture to from the same metal in the cast
eliminate interferences. condition in that
Which of the following structures A. the grains are deformed and
affects the thickness of the flange of a elongated.
maxillary complete denture?
B. the yield strength and hardness are
increased.
A. Malar process.
C. if heated sufficiently, recrystallization
B. Coronoid process. * can occur.
C. Mylohyoid ridge. D. All of the above. *
D. Zygomatic process.
E. Genial tubercle.

Dr Abdul Naser Tamim SEHA, Dr Ghada Al Aqqad D.D.S, Dr Kamal Naser - Amrita Medical Centre, Dr Emad Wani - Shadi Dental Centre,
Dr Lina Anka SEHA, Dr Rouba Zgheibi SEHA, Maria Teresa Yongson Alejandrino SEHA 254
The major connector of a removable that fits on its die cannot be seated on
partial denture should be designed to its abutment?

A. connect rigidly the component parts A. The occlusal contacts.


of the partial denture. * B. The taper of the preparation.
B. act as a stress-breaker. C. The proximal contacts. *
C. dissipate vertical forces. D. The impression used to pour the
cast.
Speech defects associated with a
maxillary partial denture can be caused In a fixed bridge, the most favorable
by ratio for an abutment tooth is when the
root
1. Replacing the teeth too soon after
extraction. A. and clinical crown are equal in
2. Providing excessive bulk of denture length.
base. B. is twice the length of the clinical
3. Posterior palatal extension too far crown. *
posteriorly. C. is half the length of the clinical
4. Positioning anterior teeth incorrectly. crown.

A. (1) (2) (3) The polymerization of methyl


B. (1) and (3) methacrylate is
C. (2) and (4) *
D. (4) only A. endothermic.
E. All of the above. B. hydrophilic.
C. hydrolytic.
Following the insertion of complete D. exothermic. *
dentures, a generalized soreness over
the entire mandibular alveolar ridge The purpose of a temporary
can be caused by restoration in an anterior tooth is to

A. Inadequate interocclusal distance. * 1. Maintain aesthetics.


B. impingement on the buccal frenum. 2. Protect dentin and pulp.
C. high muscle attachments. 3. Prevent gingival inflammation and
D. excess border thickness. recession.
4. Prevent tooth movement.
Alginate hydrocolloids
A. (1) (2) (3)
A. shrink when stored in air. B. (1) and (3)
B. show syneresis. C. (2) and (4)
C. expand when stored in water. D. (4) only
D. All of the above. * E. All of the above. *

Which of the following should be


checked first when a cast gold crown

Dr Abdul Naser Tamim SEHA, Dr Ghada Al Aqqad D.D.S, Dr Kamal Naser - Amrita Medical Centre, Dr Emad Wani - Shadi Dental Centre,
Dr Lina Anka SEHA, Dr Rouba Zgheibi SEHA, Maria Teresa Yongson Alejandrino SEHA 255
In the design of a removable partial Papillary hyperplasia on the palate of
denture, guiding planes are made a patient wearing a maxillary complete
denture is most likely to be associated
A. parallel to the long axis of the tooth. with
B. parallel to the path of insertion. *
C. at a right angle to the occlusal A. heavy smoking.
plane. B. an allergy to the acrylic resin.
D. at a right angle to the major C. an ill-fitting denture and poor oral
connector. hygiene. *
D. occlusion with posterior natural
An occlusal rest preparation should be teeth.

A. angular and box shaped with Improper temporary coverage of


parallel vertical walls. bridge abutments can cause
B. rounded and spoon shaped. *
C. flat and with an obtuse angle to the A. increased tooth sensitivity.
proximal surface of the tooth. B. gingival recession.
C. tooth migration.
At his first post insertion appointment, D. occlusal prematurities.
a patient with a new removable partial E. All of the above. *
denture complains of a tender
abutment tooth. The most likely cause The gingival margin of the preparation
is for a full crown on a posterior tooth,
with a clinical crown that satisfies the
A. overextended borders of the partial. requirements for retention and
B. inadequate polishing of the resistance, should be placed
framework.
C. improper path of insertion. A. 0.5mm subgingivally.
D. the occlusion. * B. on the enamel.
C. at least 1mm supragingivally. *
During the fabrication of new complete D. at the cemento-enamel junction.
dentures, which of the following can be E. at the gingival margin.
modified to achieve the desired
occlusion? A cast post and core is used to

1. The compensating curve. 1. Provide intraradicular venting.


2. The orientation of the occlusal plane. 2. Strengthen a weakened tooth.
3. The cusp inclination. 3. Redirect the forces of occlusion.
4. The condylar inclination. 4. Provide retention for a cast crown.

A. (1) (2) (3) * A. (1) (2) (3)


B. (1) and (3) B. (1) and (3)
C. (2) and (4) C. (2) and (4)
D. (4) only D. (4) only *
E. All of the above. E. All of the above.

Dr Abdul Naser Tamim SEHA, Dr Ghada Al Aqqad D.D.S, Dr Kamal Naser - Amrita Medical Centre, Dr Emad Wani - Shadi Dental Centre,
Dr Lina Anka SEHA, Dr Rouba Zgheibi SEHA, Maria Teresa Yongson Alejandrino SEHA 256
An epinephrine-containing retraction B. abutment teeth have large
cord has the potential of undercuts.
C. abutment teeth are rotated.
A. interfering with the setting of the D. abutment teeth are tipped.
impression material. E. residual ridges are severely
B. causing tissue necrosis. resorbed.
C. producing a systemic reaction. *
D. discolouring gingival tissue. Zinc-oxide-eugenol cements are less
soluble than
To ensure the greatest accuracy,
polysulfide base impressions should be A. zinc phosphate cements.
poured B. Bis-GMA resin cements.
C. glass ionomer cements.
A. within 10 minutes. D. calcium hydroxide cements. *
B. within 1 hour. *
C. within 8 hours. Impression trays should be
D. after 24 hours.
A. rigid. *
A metal ceramic posterior fixed partial B. flexible enough to permit easy
denture pontic should insertion.
C. carefully polished on the interior.
A. be constructed to have an occlusal D. held in place by the patient.
surface one quarter the width of the
tooth it replaces. During the setting phase, a dental
B. be constructed to have an occlusal stone mixture will exhibit
surface wider than the width of the
tooth it replaces. A. expansion. *
C. cover as much mucosa as possible. B. contraction.
D. provide adequate embrasure C. loss in compressive strength.
spaces. * D. gain in moisture content.

In treatment planning for a removable The main purpose of flux in soldering is


partial denture, a knife-edge bony ridge to
will
A. dissolve surface oxides and prevent
A. make impression-making difficult. further oxidation. *
B. necessitate relief to the partial B. prevent recrystallization and grain
denture. * growth.
C. cause difficulty in tooth selection. C. prevent oxidation and lower the
melting range of the solder.
A removable partial denture is D. dissolve surface oxides and lower
preferable to a fixed bridge when the the melting range.

A. edentulous areas are large. *

Dr Abdul Naser Tamim SEHA, Dr Ghada Al Aqqad D.D.S, Dr Kamal Naser - Amrita Medical Centre, Dr Emad Wani - Shadi Dental Centre,
Dr Lina Anka SEHA, Dr Rouba Zgheibi SEHA, Maria Teresa Yongson Alejandrino SEHA 257
The addition of platinum to a dental Which of the following materials are
gold alloy results in increased LEAST suitable for impressions for
cast gold restorations?
1. Strength.
2. Hardness. A. Polysulfides.
3. Melting point. B. Polyvinyl siloxanes.
4. Resistance to corrosion. C. Polyethers.
D. Irreversible hydrocolloids. *
A. (1) (2) (3)
B. (1) and (3) The higher modulus of elasticity of a
C. (2) and (4) chromium-cobalt-nickel alloy,
D. (4) only compared to a Type IV gold alloy,
E. All of the above. * means that chromium-cobalt-nickel
partial denture clasp will require
For application of porcelain to a
ceramo-metal alloy, the correct A. a heavier cross section for a clasp
viscosity is achieved by mixing the arm.
porcelain powder with B. a shorter retentive arm.
C. more taper.
A. a porcelain modifier. D. a shallower undercut. *
B. pure methyl alcohol.
C. a mild detergent. In minimizing the firing shrinkage of
D. distilled water. * porcelain, the principal factor is the

In metal-ceramic crowns, the bond A. fusion temperature.


between the materials is B. ratio of flux to feldspar.
C. uniformity of particle size.
A. mainly mechanical. D. thoroughness of condensation. *
B. mainly chemical. *
C. decreased by oxides on the metal In comparing polysulfide, polyether and
surface. addition cured silicone impression
D. decreased when the metal has a materials, which of the following
high yield point. statements is true?

Which of the following physical A. All three of the materials contract


properties would be_least_important for slightly during curing. *
an impression material for partially B. All three of the materials expand
edentulous patients? slightly upon cooling from mouth
temperature (37°C) to room
A. Biocompatibility. temperature (20°C).
B. Dimensional stability. C. After one week, addition cured
C. Ease of manipulation. silicones will undergo more distortion
D. Adhesion to calcium. * than polysulfides.
E. Elasticity. D. Lead oxide is used as an activator
in silicones.

Dr Abdul Naser Tamim SEHA, Dr Ghada Al Aqqad D.D.S, Dr Kamal Naser - Amrita Medical Centre, Dr Emad Wani - Shadi Dental Centre,
Dr Lina Anka SEHA, Dr Rouba Zgheibi SEHA, Maria Teresa Yongson Alejandrino SEHA 258
Correction of an inadequate zone of
attached gingiva on several adjacent A. a panoramic film.
teeth is best accomplished with a/an B. a bite-wing film.
C. a periapical film - bisecting angle
A. apically repositioned flap. technique.
B. laterally positioned sliding flap. D. a periapical film - paralleling
C. double-papilla pedicle graft. technique. *
D. coronally positioned flap.
E. free gingival graft. * The most important diagnostic element
in assessing the periodontal status of a
The instrument best suited for root patient is the
planing is a/an
A. results of vitality testing.
A. hoe. B. depth of periodontal pockets. *
B. file. C. mobility of the teeth.
C. curette. *
D. sickle scaler. An increase of immunoglobulins is
E. ultrasonic scaler. consistent with increased numbers of

Necrotizing ulcerative gingivitis A. fibroblasts.


(NUG) and acute herpetic B. neutrophils.
gingivostomatitis can be C. lymphocytes.
differentiated clinically by (the) D. plasma cells. *

A. location of the lesions. * The most important objective of


B. temperature of the patient. occlusal adjustment of a natural
C. pain. dentition is to
D. lymphadenopathy.
A. prevent temporomandibular joint
The absence of adequate drainage syndrome.
in a periodontal pocket may result in B. increase the shearing action in
mastication.
A. cyst formation. C. improve oral hygiene by preventing
B. abscess formation. * food impaction.
C. epithelial hyperplasia. D. achieve a more favorable direction
D. increased calculus formation. and distribution of forces of occlusion*

Crown-root ratio and residual bone


support can best be seen
radiographically in

Dr Abdul Naser Tamim SEHA, Dr Ghada Al Aqqad D.D.S, Dr Kamal Naser - Amrita Medical Centre, Dr Emad Wani - Shadi Dental Centre,
Dr Lina Anka SEHA, Dr Rouba Zgheibi SEHA, Maria Teresa Yongson Alejandrino SEHA 259
Ultrasonic scalers are most effective in DMF-S is an index for expressing

A. removal of supragingival calculus. * A. dental needs.


B. removal of subgingival calculus. B. tooth mortality.
C. removal of toxins from cementum. C. extent of dental neglect.
D. planing root surfaces. D. dental caries. *

Trauma from occlusion may be Fluorides taken systemically are


diagnosed radiographically by the
presence of 1. Excreted in the urine.
2. Deposited in teeth.
A. cemental tears. 3. Deposited in bone.
B. horizontal bone loss. 4. Deposited in the nails.
C. widening of the periodontal ligament
space. * A. (1) (2) (3)
D. narrowing of the periodontal B. (1) and (3)
ligament space. C. (2) and (4)
D. (4) only
Carious lesions are most likely to E. All of the above. *
develop if a patient has
In a young patient living in an area
A. a high lactobacillus count. with communal water fluoridation, the
B. saliva with low buffering capacity.* fluoride concentration of an erupted
C. plaque on his teeth. tooth is greatest
D. lactic acid in his mouth.
A. at the dentino-enamel junction.
Abrasion is most commonly seen on B. on the surface of the clinical
the crown.
C. at the layer of dentin nearest the
A. lingual surface of posterior teeth. pulp chamber. *
B. occlusal surface of posterior teeth. D. evenly throughout the enamel.
C. incisal edges.
D. facial surfaces of teeth. * Which of the following foods is
LEAST cariogenic?
A clenching habit may be a factor in
A. Canned fruit.
A. suprabony periodontal pocket B. Potatoes.
formation. C. Fruit jello.
B. marginal gingivitis. D. Cheese. *
C. increased tooth mobility. ?* E. White bread.
D. generalized recession.

Dr Abdul Naser Tamim SEHA, Dr Ghada Al Aqqad D.D.S, Dr Kamal Naser - Amrita Medical Centre, Dr Emad Wani - Shadi Dental Centre,
Dr Lina Anka SEHA, Dr Rouba Zgheibi SEHA, Maria Teresa Yongson Alejandrino SEHA 260
Irregularly distributed shallow to
moderate craters in the interseptal Overhangs on restorations initiate
bone are best eliminated by chronic inflammatory periodontal
disease by
A. osteoplasty. *
B. gingivoplasty. A. increasing plaque retention. *
C. deep scaling. B. increasing food retention.
D. bone grafting. C. causing traumatic occlusion.
D. causing pressure atrophy.
In patients with advanced periodontitis,
mobile teeth should be splinted in The periodontium is best able to
order to tolerate forces directed to a tooth

A. reduce gingival inflammation. A. horizontally.


B. accelerate epithelialization after B. laterally.
periodontal surgery. C. obliquely.
C. enhance formation of a new D. vertically. *
connective tissue attachment after
surgery. Fluorides are effective in the
D. None of the above. * prevention of dental caries by

Which treatment procedure is A. increasing the resistance of dentin


indicated for a patient with to bacterial penetration.
asymptomatic age related gingival B. causing tooth enamel to be more
recession? resistant to demineralization. *
C. providing a more favorable pulpal
A. Connective tissue graft. blood supply.
B. Gingivoplasty. D. All of the above.
C. Lateral sliding flap.
D. Gingival graft. Dietary deficiency of vitamin D can
E. No treatment. * result in

Caries in older persons is most A. abnormal formation of osteoid.


frequently found on which of the B. osteitis fibrosa cystica. *
following locations? C. Paget's disease.
D. myositis ossificans.
A. Pits and fissures. E. osteogenesis imperfecta.
B. Proximal enamel.
C. Root surfaces. *
D. Incisal dentin.

Dr Abdul Naser Tamim SEHA, Dr Ghada Al Aqqad D.D.S, Dr Kamal Naser - Amrita Medical Centre, Dr Emad Wani - Shadi Dental Centre,
Dr Lina Anka SEHA, Dr Rouba Zgheibi SEHA, Maria Teresa Yongson Alejandrino SEHA 261
During tooth development, vitamin A response to the accumulation of
deficiency may result in plaque?

A. peg-shaped teeth. A. Plasma cells and monocytes.


B. partial anodontia (hypodontia). B. Polymorphonuclear leukocytes. *
C. Hutchinson's incisors. C. Macrophages.
D. enamel hypoplasia.* D. Lymphocytes.
E. dentinogenesis imperfecta. E. Mast cells.

Maximum shrinkage after gingival In necrotizing ulcerative gingivitis the


curettage can be expected from deepest penetrating microorganisms
tissue that is are

A. fibroedematous. A. cocci.
B. edematous. * B. spirochetes. *
C. fibrotic. C. diplococci.
D. formed within an infrabony D. filamentous rods.
pocket.
E. associated with exudate The colour of normal gingiva is affected
formation. by the

After a tooth surface has been 1. Vascularity of the gingiva.


completely cleaned, the new 2. Epithelial keratinization.
mucoprotein coating which forms on 3. Thickness of the epithelium.
the surface is called 4. Melanin pigmentation.

A. pellicle. * A. (1) (2) (3)


B. plaque. B. (1) and (3)
C. materia alba. C. (2) and (4)
D. primary cuticle. D. (4) only
E. Nasmyth's membrane. E. All of the above. *

Gingival crevicular fluid With the development of gingivitis, the


sulcus becomes predominantly
A. never varies in volume. populated by
B. is a transudate.
C. is derived from mast cells. A. gram-positive organisms.
D. is an exudate. B. gram-negative organisms. *
E. B. and D. * C. diplococcal organisms.
D. spirochetes.
Which cells migrate into the gingival
sulcus in the largest numbers in

Dr Abdul Naser Tamim SEHA, Dr Ghada Al Aqqad D.D.S, Dr Kamal Naser - Amrita Medical Centre, Dr Emad Wani - Shadi Dental Centre,
Dr Lina Anka SEHA, Dr Rouba Zgheibi SEHA, Maria Teresa Yongson Alejandrino SEHA 262
Epidemiology of disease is best The oral mucosa covering the base of
described as the the alveolar bone

A. data obtained from sickness surveys. A. is normally non-keratinized but can


B. usual low level of disease normally become keratinized in response to
found within a population. physiological stimulation.
C. control of disease. B. is closely bound to underlying
D. study of disease patterns in a muscle and bone.
population. * C. does not contain elastic fibres.
D. merges with the keratinized gingiva
Which of the following foods is the most at the mucogingival junction. *
cariogenic? E. has a tightly woven dense
collagenous corium.
A. Cheese.
B. Dark chocolate. Which oral mucosa changes are
C. Jam. possible side effects of chemotherapy?
D. Toffee. *
1. Atrophic thinning.
A magenta-coloured tongue and angular 2. Ulceration.
cheilitis may be associated with a 3. Necrosis.
deficiency of 4. Spontaneous bleeding.

A. ascorbic acid. A. (1) (2) (3)


B. niacin. B. (1) and (3)
C. riboflavin. *B2 C. (2) and (4)
D. thiamine. D. (4) only
E. All of the above. *
In the initiation of dental caries,
lactobacilli are A facebow is used to record the

A. capable of surviving without nutrients. 1. Vertical dimension of occlusion.


B. abundant in calculus. 2. Inter-occlusal relationship.
C. both acidogenic and aciduric. * 3. Horizontal condylar inclination.
D. the only acid-forming bacteria in the 4. Relationship of the maxilla to the
mouth. hinge axis.

The enamel structures most resistant to A. (1) (2) (3)


the action of acids are B. (1) and (3)
C. (2) and (4)
A. cuticles. D. (4) only. *
B. lamellae. E. All of the above.
C. rods. *
D. interprismatic substances.

Dr Abdul Naser Tamim SEHA, Dr Ghada Al Aqqad D.D.S, Dr Kamal Naser - Amrita Medical Centre, Dr Emad Wani - Shadi Dental Centre,
Dr Lina Anka SEHA, Dr Rouba Zgheibi SEHA, Maria Teresa Yongson Alejandrino SEHA 263
Which condition produces a radiopaque 1. Ameloblastoma.
image? 2. Odontogenic myxoma.
3. Keratocystic odontogenic tumor.
A. Osteomalacia. 4. Central giant cell granuloma.
B. Multiple myeloma.
C. Osteopetrosis. * A. (1) (2) (3)
D. Letterer-Siwe disease. B. (1) and (3)
E. Central giant cell granuloma. C. (2) and (4)
D. (4) only
A mixed lesion with a radiolucent rim E. All of the above. *
and corticated border causing impaction
of a permanent tooth is most likely a/an A well circumscribed 3mm
radiolucency in the apical region of the
A. adenomatoid odontogenic tumour. mandibular second premolar may be
B. calcifying epithelial odontogenic
tumour. 1. A periapical granuloma.
C. odontoma. * 2. A periapical cyst.
D. ameloblastic fibro-odontoma. 3. A chronic periapical abscess.
4. The mental foramen.
What is the most significant radiographic
finding in hyperparathyroidism? A. (1) (2) (3)
B. (1) and (3)
A. Demineralization of teeth. C. (2) and (4)*
B. Multiple keratocystic odontogenic D. (4) only
tumors. E. All of the above.
C. Hypercementosis.
D. Rampant caries. Which of the following landmarks may
E. Generalized loss of lamina dura. * be identified on periapical radiographs
of the mandible?
A radiograph displaying a “cotton-wool”'
appearance and generalized 1. External oblique ridge.
hypercementosis is suggestive of 2. Mylohyoid ridge.
3. Mental foramen.
A. fibrous dysplasia. 4. Coronoid process.
B. osteopetrosis.
C. osteogenesis imperfecta. A. (1) (2) (3) *
D. Paget’s disease. * B. (1) and (3)
E. cleidocranial dysplasia. C. (2) and (4)
D. (4) only
Which lesion/s may appear E. All of the above.
radiographically as multilocular
radiolucencies?

Dr Abdul Naser Tamim SEHA, Dr Ghada Al Aqqad D.D.S, Dr Kamal Naser - Amrita Medical Centre, Dr Emad Wani - Shadi Dental Centre,
Dr Lina Anka SEHA, Dr Rouba Zgheibi SEHA, Maria Teresa Yongson Alejandrino SEHA 264
A periapical radiolucency associated
with a vital maxillary central incisor A. regular radiolucent line around the
can represent roots.
B. regular radiopaque line around the
1. A nasopalatine cyst. roots. *
2. A dentigerous cyst. C. irregular radiolucent line around the
3. The foramen of the incisive canal. roots.
4. A periapical granuloma.
The radiographic images most
A. (1) (2) (3) suggestive of multiple myeloma is
B. (1) and (3) *
C. (2) and (4) A. multiple radiopaque lesions.
D. (4) only B. punched out radiolucent lesions. *
E. All of the above. C. ground glass appearance.
D. generalized hypercementosis.
Which of the following entities can
present as periapical radiolucencies? A 25 year old female in her third
trimester of pregnancy presents with an
1. A periapical granuloma. acute dental infection. Which of the
2. A radicular cyst. following is CONTRAINDICATED for
3. A metastatic carcinoma. this patient?
4. The mental foramen.
A. Prescription of clindamycin.
A. (1) (2) (3) B. Prescription of penicillin V.
B. (1) and (3) C. Extraction using 2% lidocaine with
C. (2) and (4) 1:100,000 epinephrine.
D. (4) only D. prescription of ibuprofen. *
E. All of the above. *
Therapeutic doses of morphine
A discontinuous lamina dura may be administered intramuscularly may
a consequence of produce

1. Pulpitis. 1. Constipation.
2. Metastatic carcinoma. 2. Euphoria.
3. Parathyroid hyperplasia. 3. Drowsiness.
4. berated bone. 4. Dysphoria.

A. (1) (2) (3) * A. (1) (2) (3)


B. (1) and (3) B. (1) and (3)
C. (2) and (4) C. (2) and (4)
D. (4) only D. (4) only
E. All of the above. E. All of the above. *

The lamina dura is a/an

Dr Abdul Naser Tamim SEHA, Dr Ghada Al Aqqad D.D.S, Dr Kamal Naser - Amrita Medical Centre, Dr Emad Wani - Shadi Dental Centre,
Dr Lina Anka SEHA, Dr Rouba Zgheibi SEHA, Maria Teresa Yongson Alejandrino SEHA 265
On a periapical radiograph, the A provisional crown must restore the
nasopalatine foramen is located
between the roots of the maxillary 1. Proximal contacts.
2. Axial contours of the tooth.
A. incisors above their apices. 3. Occlusal function.
B. central and lateral incisors below 4. Esthetics.
their apices.
C. central incisors below their apices. * A. (1) (2) (3)
D. central and lateral incisors above B. (1) and (3)
their apices. C. (2) and (4)
D. (4) only
In the mandibular third molar region, a E. All of the above. *
circumscribed radiolucent area 3cm in
diameter contains the crown of the Which of the following pontic designs
developing third molar. The radiolucent is most likely to cause soft tissue
area suggests irritation?

A. a simple bone cyst/traumatic bone A. Polished gold.


cyst. B. Polished acrylic. *
B. a dentigerous cyst. * C. Polished porcelain.
C. Stafne's idiopathic bone cavity. D. Glazed porcelain.
D. an eruption cyst.
Radiographically, the nasopalatine
Traumatic occlusion may cause foramen may be misdiagnosed as a

1. Widening of the periodontal ligament 1. Branchial cyst.


space. 2. Nasopalatine cyst.
2. Vertical destruction of the interdental 3. Nasolabial cyst.
septum. 4. Radicular cyst.
3. decreased definition of the lamina
dura. A. (1) (2) (3)
4. Narrowing of the periodontal B. (1) and (3)
ligament space. C. (2) and (4) *
D. (4) only
A. (1) (2) (3) *
E. All of the above.
B. (1) and (3)
C. (2) and (4) The highest incidence of congenitally
D. (4) only missing lateral incisors is most likely
E. All of the above. seen in a patient with
Radiographically, an osteoma appears A. unilateral cleft lip and palate. *
B. congenital heart disease.
A. radiopaque. * C. Down syndrome.
B. radiolucent. D. hyperthyroidism.
C. either radiopaque or radiolucent.
D. radiolucent, surrounded by a
radiopaque line.

Dr Abdul Naser Tamim SEHA, Dr Ghada Al Aqqad D.D.S, Dr Kamal Naser - Amrita Medical Centre, Dr Emad Wani - Shadi Dental Centre,
Dr Lina Anka SEHA, Dr Rouba Zgheibi SEHA, Maria Teresa Yongson Alejandrino SEHA 266
The highest incidence of congenitally
missing lateral incisors is most likely A. One-wall bony defect.
seen in a patient with B. Two-wall bony defect.
C. Three-wall bony defect. *
A. unilateral cleft lip and palate. *
B. congenital heart disease. When probing the sulcus of healthy
C. Down's syndrome. gingiva, the tip of the periodontal probe
D. hyperthyroidism. extends

Loss of sensation over the distribution A. to the coronal aspect of the


of the inferior dental nerve is a possible epithelial attachment. *
complication of B. to the alveolar bone crest.
C. apical to the epithelial attachment.
A. removal of an impacted mandibular
third molar tooth. * Which of the following statements is
B. removal of a torus mandibularis. true about diazepam? It
C. a forceps removal of a mandibular
second molar. A. improves performance of fine motor
D. distal wedge periodontal surgery. skills.
B. causes amnesia. *
Tooth 4.5 was treated endodontically 2 C. is available without prescription.
years ago and is now sensitive to D. does not produce a dependence
percussion. There is an isolated, syndrome.
narrow 8mm periodontal pocket on the
buccal surface of the tooth. Which of the following drugs may
Radiographic findings are within cause generalized enlargement of
normal limits. The most likely diagnosis interdental papillae?
is a/an
A. Digitalis.
A. endodontic lesion. B. Nifedipine. *
B. vertical root fracture. * C. Captopril.
C. chronic periodontal lesion. D. Propranolol.

Presence of bleeding on probing Frankfort plane extends from

A. is indicative of current inflammation* A. Porion to Orbitale. *


B. is indicative of past inflammation. B. Porion to Gnathion.
C. has a high positive predictive value C. Condylion to Gnathion.
(PPV) for further attachment loss. D. Condylion to Menton.

In which of the following defects is


bone regeneration most likely to occur?

Dr Abdul Naser Tamim SEHA, Dr Ghada Al Aqqad D.D.S, Dr Kamal Naser - Amrita Medical Centre, Dr Emad Wani - Shadi Dental Centre,
Dr Lina Anka SEHA, Dr Rouba Zgheibi SEHA, Maria Teresa Yongson Alejandrino SEHA 267
The central action of caffeine is C. decreases 0-1mm.
principally on the D. decreases 3-4mm.

A. cerebral cortex. * Which of the following is NOT a


B. corpus callosum. component of a dental cartridge
C. hypothalamus. containing 2% lidocaine with 1:100,000
D. spinal cord. epinephrine?

When designing a removable partial A. Methylparaben. *


denture, changing the tilt of the cast on B. Water.
the surveyor alters the C. Sodium metabisulphite.
D. Sodium chloride.
A. path of insertion of the planned
removable partial denture. * For a patient with cardiovascular
B. the position of the survey line on the disease, local anesthesia
cast.
C. the undercut and non-undercut A. affects blood pressure more than
areas. general anesthesia. *
D. the direction of forces applied to the B. affects blood pressure less than
partial denture. general anesthesia.
C. is responsible for bacteremia.
All of the following are possible effects
of acetylsalicylic acid EXCEPT The use of aspirating syringes for the
administration of local anesthetics is
A. reduction of fever. recommended because
B. shortening of bleeding time. *
C. suppression of inflammatory 1. The effectiveness of local
response. anesthesia is increased.
D. bleeding from the gastrointestinal 2. Aspiration of blood is proof that the
tract. needle is in an intravascular location.
3. Their use removes the hazard of
Lidocaine is an rapid injection and provides a distinct
saving of time.
A. amide. * 4. Their use reduces the frequency of
B. ester. accidental intravenous injection.
C. aldehyde.
D. acid. A. (1) (2) (3)
B. (1) and (3)
Between the ages of 5 and 18 years, C. (2) and (4) *
mandibular arch length D. (4) only
E. All of the above.
A. increases 0-1mm.
B. increases 3-4mm. *

Dr Abdul Naser Tamim SEHA, Dr Ghada Al Aqqad D.D.S, Dr Kamal Naser - Amrita Medical Centre, Dr Emad Wani - Shadi Dental Centre,
Dr Lina Anka SEHA, Dr Rouba Zgheibi SEHA, Maria Teresa Yongson Alejandrino SEHA 268
The penetration of a local anesthetic B. only organic matter in the root canal
into nerve tissue is a function of the system. *
C. only inorganic matter in the root
A. length of the central alkyl chain. canal system.
B. lipid solubility of the un-ionized
form* A 3 year old presents 30 minutes after
C. ester linkage between the aromatic facial trauma. Tooth 5.1 is avulsed.
nucleus and the alkyl chain. The father has recovered the tooth
D. amide linkage between the aromatic and has kept it in a wet napkin. Which
nucleus and the alkyl chain. of the following is the most appropriate
management?
Facial nerve paresthesia is most likely
to occur from which of the following A. Replant the tooth followed by
injections? endodontic treatments in 2 weeks.
B. Perform a pulpectomy, then replant
A. Posterior superior alveolar block. the tooth.
B. Inferior alveolar block. * C. Replant the tooth and monitor for
C. Posterior palatine block. possible endodontic treatment.
D. Extraoral maxillary division block. D. Do not replant the tooth. *

Which of the following diseases Which of the following types of


predisposes a patient to exaggerated permanent teeth is more likely to
cardiovascular effects of epinephrine? present with a C-shape canal
anatomy?
A. Hyperparathyroidism.
B. Hyperthyroidism. * A Mandibular first premolars.
C. Hypogonadism. B. Maxillary first premolars.
D. Acromegaly. C. Mandibular second molars. *
D. Maxillary second molars.
Needle deflection increases as
Antibiotics are indicated for
A. depth of injection increases. management of which of the following
B. needle length increases. conditions?
C. needle gauge decreases.
D. All of the above. * A. Chronic apical abscess.
B. Previously treated tooth with acute
Sodium hypochlorite in endodontic periapical periodontitis.
therapy dissolves C. Acute apical abscess with cellulitis*
D. Irreversible pulpitis.
A. both organic and inorganic matter in
the root canal system.

Dr Abdul Naser Tamim SEHA, Dr Ghada Al Aqqad D.D.S, Dr Kamal Naser - Amrita Medical Centre, Dr Emad Wani - Shadi Dental Centre,
Dr Lina Anka SEHA, Dr Rouba Zgheibi SEHA, Maria Teresa Yongson Alejandrino SEHA 269
Pulp reactions to caries include all of The purpose of calibrating examiners
the following, EXCEPT in a randomized controlled clinical trial
is to ensure
A. decrease in dentin permeability.
B. tertiary dentin formation. A. consistent measurement of clinical
C. evaporation of the intratubular fluid* outcomes. *
D. inflammatory and immune B. similarity of experimental and control
reactions. groups.
C. high ethical standards in conducting
Which of the following clinical the trial.
protocols is the most appropriate next D. measurement of all relevant clinical
step in the management of an avulsed outcomes.
permanent tooth, which has just been
replanted? In addition to the advancement of
knowledge, health research is intended
A. Splinting with composite resin only. to produce value for research subjects,
B. Splinting with flexible wire. * for other individuals, and/or for society
C. Splinting with rigid wire. as a whole. What ethical principle does
D. Pulpectomy. this refer to?

Meta-analyses are important to the A. Autonomy.


practice of dentistry because they B. Non-maleficence.
C. Beneficence. *
A. minimize publication bias. D. Justice.
B. define who is at risk for a disease.
C. guide statistical testing for clinical A survey of a random sample of 5,586
procedures. Canadians found that 63% of older
D. provide a quantitative summary adults had visited a dentist in the last
measure from many studies. * year. The 95% confidence interval was
56% to 70%. This means that
Which of the following studies provides
the best evidence for the possible A. between 56% and 70% of older
association between community water adults account for 95% of the dental
fluoridation and hip fracture? visits.
B. there is 95% certainty that 63% of
A. Ecological. older adults had dental visits.
B. Case-series. C. no more than 70% of dental visits
C. Cross-sectional. occurred in 95% of older adults.
D. Retrospective cohort. * D. there is 95% certainty that 56% to
70% of older adults had dental visits. *

Dr Abdul Naser Tamim SEHA, Dr Ghada Al Aqqad D.D.S, Dr Kamal Naser - Amrita Medical Centre, Dr Emad Wani - Shadi Dental Centre,
Dr Lina Anka SEHA, Dr Rouba Zgheibi SEHA, Maria Teresa Yongson Alejandrino SEHA 270
The most effective drug for relief of
angina pectoris is A. maxillary and mandibular tooth
material.
A. codeine. B. primary and the permanent incisors.
B. digitalis. C. primary molars and the premolars. *
C. quinidine. D. maxillary and the mandibular
D. nitroglycerin. * primate space.
E. pentobarbital sodium.
The most appropriate management for
Which of the following drugs is a a patient with mild anterior crowding in
natural alkaloid obtained from opium? an early mixed dentition is to

A. Hydromorphone. A. strip all proximal contacts.


B. Meperidine. B. extract the deciduous canines.
C. Methadone. C. perform a space analysis. *
D. Codeine. * D. regain space in the arch.
E. Morphine.
Thumbsucking after the age of six
The most commonly impacted most often results in
permanent tooth is the
A. a dental malocclusion. *
A. mandibular second premolar. B. a skeletal malocclusion.
B. maxillary lateral incisor. C. mouth breathing.
C. mandibular canine. D. temporomandibular disorder.
D. maxillary canine. *
A furcation involvement in which the
Two millimeters of maxillary incisor probe extends completely through the
spacing in a 3 year old is indicative of a furcation is classified as

A. tooth size-arch size discrepancy. A. incipient.


B. maxillary Bolton deficiency. B. Class I.
C. persistent tongue thrusting. C. Class II.
D. normal pattern of development. * D. Class III. *

Leeway space is the difference in size


between the

Dr Abdul Naser Tamim SEHA, Dr Ghada Al Aqqad D.D.S, Dr Kamal Naser - Amrita Medical Centre, Dr Emad Wani - Shadi Dental Centre,
Dr Lina Anka SEHA, Dr Rouba Zgheibi SEHA, Maria Teresa Yongson Alejandrino SEHA 271
An acute lateral periodontal abscess E. All of the above.
associated with a lateral periodontal
cyst can be differentiated from an acute An endomorph is typically
abscess of pulpal origin by the
A. short and fat. *
A. type of exudate. B. tall and thin.
B. intensity of pain. C. an early maturer.
C. nature of swelling. D. a late maturer.
D. degree of tooth mobility.
E. response to a vitality test. * Tranquilizers and hypnotics

The primary etiologic factor for 1. Are depressants of the respiratory


gingivitis is centre.
2. Are depressants of the cerebral
A. a faulty restoration. cortex.
B. ascorbic acid deficiency. 3. May cause physical dependency.
C. dental calculus. 4. May have convulsant effects.
D. occlusal trauma.
E. dental plaque. * A. (1) (2) (3) *
B. (1) and (3)
Trauma from occlusion C. (2) and (4)
D. (4) only
A. initiates gingivitis. E. All of the above.
B. affects the blood supply to gingivae.
* A midfacial probing depth
C. initiates periodontitis. measurement where the base of the
pocket extends beyond the
Anaphylactic reaction to penicillin is mucogingival junction indicates that
most likely to occur
A. gingival hyperplasia is present.
1. When the drug is administered B. there is no attached gingiva. *
parentally. C. occlusal trauma is present.
2. within minutes after drug D. gingival recession is present.
administration.
3. in patients who have already The width of the attached gingiva can
experienced an allergic reaction to the be accurately measured if
drug. A. gingival inflammation in the area
4. in patients with a negative skin test has been eliminated. *
to penicillin allergy. B. there is an increase in crevicular
fluid.
A. (1) (2) (3) * C. the probing depth is 3mm or less.
B. (1) and (3) D. scaling has not been done.
C. (2) and (4)
D. (4) only

Dr Abdul Naser Tamim SEHA, Dr Ghada Al Aqqad D.D.S, Dr Kamal Naser - Amrita Medical Centre, Dr Emad Wani - Shadi Dental Centre,
Dr Lina Anka SEHA, Dr Rouba Zgheibi SEHA, Maria Teresa Yongson Alejandrino SEHA 272
A pontic replacing a mandibular first an acute dental infection. Which of the
molar should be designed so that following is CONTRAINDICATED for
it/its this patient?

1. Gingival surface is concave and A. Take a radiograph.


adapts closely to the ridge. B. Prescription of penicillin V.
2. Has open gingival embrasures. C. Administration of 2% xylocaine with
3. Conceals the porcelain to metal 1:100,000 epinephrine.
junction on its gingival surface. D. Acetylsalicylic acid for pain control*
4. Gingival surface is convex in all
directions. Which of the following anesthetic
agents is/are metabolized by plasma
A. (1) (2) (3) cholinesterase?
B. (1) and (3)
C. (2) and (4) * 1. Prilocaine.
D. (4) only 2. Lidocaine.
E. All of the above. 3. Mepivacaine.
4. Procaine.
Which of the following is a
CONTRAINDICATION for the use of A. (1) (2) (3)
a resin bonded fixed partial denture B. (1) and (3)
(acid etched bridge or “Maryland C. (2) and (4)
Bridge”)? D. (4) only *
E. All of the above.
A. Angle Class II malocclusion.
B. An opposing free end saddle Stiffness of wire is a function of
removable partial.
C. Previous orthodontic treatment. 1. Length of the wire segment.
D. Heavily restored abutment. * 2. Diameter of the wire segment.
3. Alloy composition.
A physical sign of impending 4. Microstructure.
syncope is
A. (1) (2) (3)
A. pallor. * B. (1) and (3)
B. elevation of blood pressure. C. (2) and (4)
C. tachycardia. D. (4) only
E. All of the above. *
A 25 year old female in her first
trimester of pregnancy presents with

Dr Abdul Naser Tamim SEHA, Dr Ghada Al Aqqad D.D.S, Dr Kamal Naser - Amrita Medical Centre, Dr Emad Wani - Shadi Dental Centre,
Dr Lina Anka SEHA, Dr Rouba Zgheibi SEHA, Maria Teresa Yongson Alejandrino SEHA 273
A Le Fort I fracture is a A. slightly less.
B. the same.
A. fracture of the zygomatic arch. C. slightly more. *
B. horizontal fracture of the maxilla. * D. significantly less.
C. fracture involving the orbital floor.
D. pyramidal fracture of the maxilla. Fractures of the maxilla can best be
E. cranio-facial dysjunction. diagnosed by

Which of the following may be 1. Lateral jaw radiographs.


associated with a fracture of the 2. Clinical examination.
mandible? 3. Evidence of periorbital edema.
4. Anteroposterior radiograph of the
1. Diplopia. skull.
2. Malocclusion.
3. Swelling of the orbit. A. (1) (2) (3)
4. Pain. B. (1) and (3)
C. (2) and (4) *
A. (1) (2) (3) D. (4) only
B. (1) and (3) E. All of the above.
C. (2) and (4) *
D. (4) only Periapical surgery is
E. All of the above. CONTRAINDICATED for a tooth that
has a
Indirect retention is required for a
removable partial denture which has A. large periapical lesion.
B. fistula related to a periapical lesion.
1. A unilateral distal extension base. C. vertical root fracture. *
2. Bilateral distal extension bases. D. a post and core retained crown.
3. An anterior extension base.
4. Complete tooth support. What is the most appropriate time to
extract a mesiodens?
A. (1) (2) (3) *
B. (1) and (3) A. As soon as diagnosed.
C. (2) and (4) B. Just prior to the eruption of the first
D. (4) only permanent molars.
E. All of the above. C. Just prior to the eruption of the
maxillary central incisors. *
The coefficient of thermal expansion of D. Just prior to the eruption of the
the metal relative to the porcelain for maxillary canines.
constructing a metal-ceramic crown
should be

Dr Abdul Naser Tamim SEHA, Dr Ghada Al Aqqad D.D.S, Dr Kamal Naser - Amrita Medical Centre, Dr Emad Wani - Shadi Dental Centre,
Dr Lina Anka SEHA, Dr Rouba Zgheibi SEHA, Maria Teresa Yongson Alejandrino SEHA 274
The main reason for adding copper to Antibiotic coverage should be provided
a dental amalgam alloy is to when performing periodontal surgery
for patients with
A. increase expansion.
B. reduce tarnish resistance. A. myocardial infarction.
C. make amalgamation easier. B. dental implants.
D. reduce the tin-mercury phase. * C. prosthetic heart valve. *
D. coronary artery disease.
Which of the following are
characteristics of restorative glass A silane coupling agent is used to
ionomer cements?
A. control polymerization shrinkage in
1. Release of fluoride. composite resins.
2. Bonding to enamel. B. enhance the bond between a
3. Setting is affected by moisture. porcelain veneer and the resin cement.
4. Irritating to pulpal tissues. *
C. reduce the surface tension when
A. (1) (2) (3) * investing a wax pattern.
B. (1) and (3) D. facilitate the soldering of gold
C. (2) and (4) castings.
D. (4) only
E. All of the above. Acetylsalicylic acid is indicated for the
management of which of the following?
The setting of a zinc-phosphate
cement can best be retarded by A. Myocardial infarction. *
B. Asthma.
A. decreasing the particle size. C. Gastric ulcer.
B. increasing the concentration of D. Hemophilia.
water in the liquid.
C. increasing the rate of addition of the An overdose of acetaminophen can
powder to the liquid. result in
D. cooling the glass mixing slab. *
A. nephrotoxicity.
The higher modulus of elasticity of a B. cardiotoxicity.
chromium-cobalt-nickel alloy, C. neurotoxicity.
compared to a Type IV gold alloy, D. hepatotoxicity. *
means that chromium-cobalt-nickel
partial denture clasp will require Adrenal corticosteroids

A. a thicker cross section. A. increase heart rate.


B. a shorter retentive arm. B. cause vasodilation.
C. more taper. C. increase gastric motility.
D. a shallower undercut. * D. reduce inflammation. *

Dr Abdul Naser Tamim SEHA, Dr Ghada Al Aqqad D.D.S, Dr Kamal Naser - Amrita Medical Centre, Dr Emad Wani - Shadi Dental Centre,
Dr Lina Anka SEHA, Dr Rouba Zgheibi SEHA, Maria Teresa Yongson Alejandrino SEHA 275
A patient who is four months pregnant teeth. The restorative material of choice
requires an extraction. A radiograph is a
may
A. microfilled composite resin.
A. be taken only for surgical purposes. B. hybrid composite resin.
B. be taken for diagnostic or treatment C. silver amalgam.
purposes. * D. glass ionomer cement. *
C. not be taken until end of second E. reinforced zinc oxide and eugenol
trimester. cement.
D. be taken by panoramic radiography
only. Glass ionomer cements contain

The most frequent cause for composite A. zinc oxide and distilled water.
resin restoration failure is B. zinc oxide and polyacrylic acid.
C. fluoro aluminosilica powder and
A. brittleness of the composite. orthophosphoric acid.
B. excessive polymerization shrinkage. D. fluoro aluminosilica powder and
C. presence of voids within the polyacrylic acid. *
material.
D. inadequate moisture control during The short action of thiopental is due to
placement. *
A. rapid redistribution. *
Methyl methacrylate resins will perform B. rapid renal excretion.
better than composite resins for long C. rapid metabolism.
span, temporary bridges because of D. build up of tolerance.
superior E. conjugation with serum proteins.
A. hardness. Epinephrine in a local anesthetic
B. fracture toughness. * solution will
C. wear resistance.
D. dimensional stability. 1. Decrease absorption of the
anesthetic.
The most likely cause of postoperative 2. Assist hemostasis at the site of
sensitivity with Class II composite resin injection.
restorations is 3. Prolong the action of the anesthetic
agent.
A. acid etching of dentin. 4. Assist in post-operative healing.
B. microleakage at the interface. *
C. toxicity of the restorative material. A. (1) (2) (3) *
D. overheating during the finishing B. (1) and (3)
process. C. (2) and (4)
D. (4) only
A 78 year old patient presents with E. All of the above.
several carious lesions on the root
surfaces of the maxillary posterior

Dr Abdul Naser Tamim SEHA, Dr Ghada Al Aqqad D.D.S, Dr Kamal Naser - Amrita Medical Centre, Dr Emad Wani - Shadi Dental Centre,
Dr Lina Anka SEHA, Dr Rouba Zgheibi SEHA, Maria Teresa Yongson Alejandrino SEHA 276
Acute overdose of acetaminophen is B. Gold-palladium.
most likely to lead to C. Gold-silver.
D. Gold-platinum-palladium.
A. depression of the respiratory centre.
B. severe gastric bleeding. For a removable partial denture, a
C. hepatic toxicity. * metal base is preferred to an acrylic
D. anaphylactic shock. base because metal is

Which of the following is a sign of a 1. More hygienic.


severe toxic reaction to ketoconazole? 2. Stronger.
3. Less irritating.
A. Jaundice. * 4. A better thermal conductor.
B. Hypertension.
C. Xerostomia. A. (1) (2) (3)
D. Salivary gland swelling. B. (1) and (3)
C. (2) and (4)
An end result of ionizing radiation used D. (4) only
to treat oral malignancies is E. All of the above. *

A. deformity of the jaws. The main purpose of X-ray collimation


B. reduced vascularity of the jaws. * is to
C. increased vascularity of the jaws.
D. increased brittleness of the jaws.** A. lower the kilovoltage.
B. filter out low energy X-rays.
(end result) C. reduce the diameter of the primary
beam. *
A 60 year old patient requests the D. reduce the exposure time.
replacement of tooth 4.6, which was
extracted many years ago. Tooth 1.6 Overlapping contacts on a bitewing
has extruded 1.8mm into the space of radiograph result from the
the missing tooth. The three unit fixed
bridge replacing the mandibular first 1. Malalignment of teeth.
molar should be fabricated 2. Increased vertical angulation of the
x-ray beam.
A. to the existing occlusion. 3. Incorrect horizontal angulation of the
B. after extracting tooth 1.6 and x-ray beam.
replacing it with a fixed partial denture. 4. Decreased vertical angulation of the
C. after restoring tooth 1.6 to a more x-ray beam.
normal plane of occlusion. *
D. after devitalizing and preparing tooth A. (1) (2) (3)
1.6 for a cast crown. B. (1) and (3) *
C. (2) and (4)
What is the alloy of choice for the D. (4) only
framework of a resin-bonded fixed E. All of the above.
bridge?

A. Nickel-chromium. *

Dr Abdul Naser Tamim SEHA, Dr Ghada Al Aqqad D.D.S, Dr Kamal Naser - Amrita Medical Centre, Dr Emad Wani - Shadi Dental Centre,
Dr Lina Anka SEHA, Dr Rouba Zgheibi SEHA, Maria Teresa Yongson Alejandrino SEHA 277
The average annual dose of ionizing
radiation that a patient receives from A. (1) (2) (3) *
dental radiographs is B. (1) and (3)
C. (2) and (4)
A. greater than the average amount D. (4) only
received from natural sources. E. All of the above.
B. about the same as the average
amount received from medical sources. For an acute bacterial infection, the
C. about the same as the average most valuable laboratory test(s)
amount received from cosmic radiation. is/are the
D. low when compared to the average
total amount of radiation received. * 1. Hemoglobin level.
2. White blood cell count.
A preparation for a metal-ceramic 3. Red blood cell count.
crown with a porcelain butt joint margin 4. Culture and sensitivity test.
should have a
A. (1) (2) (3)
1. 90o cavosurface margin. B. (1) and (3)
2. Subgingival finish line. C. (2) and (4) *
3. 1.2mm shoulder.
D. (4) only
4. 90o axiogingival angle.
E. All of the above.
A. (1) (2) (3)
Which of the following is/are vital
B. (1) and (3) *
signs?
C. (2) and (4)
D. (4) only A. Pulse and respiratory rate. *
E. All of the above. B. Complete blood count.
C. Pupil size and reactivity.
For a removable partial denture,
D. Height and weight.
tripoding a cast is used to
Which of the following barbiturates
A. determine the path of insertion.
can be used to reduce the
B. locate unfavorable tissue undercuts. frequency of seizures in persons
C. locate the height of contour. with grand mal seizure disorder?
D. return the cast to the surveyor. *
A. Thiopental.
Which of the following is/are vital B. Amobarbital.
signs?
C. Secobarbital.
D. Pentobarbital.
1. Blood pressure.
E. Phenobarbital. *
2. Body temperature.
3. Pulse and respiratory rate.
4. Pupil size.

Dr Abdul Naser Tamim SEHA, Dr Ghada Al Aqqad D.D.S, Dr Kamal Naser - Amrita Medical Centre, Dr Emad Wani - Shadi Dental Centre,
Dr Lina Anka SEHA, Dr Rouba Zgheibi SEHA, Maria Teresa Yongson Alejandrino SEHA 278
Which of the following is associated
with the use of penicillin? Radiation dose to the patient is
reduced by
A. High toxicity.
B. Allergenicity. * 1. Using a high speed film.
C. Development of tolerance. 2. Using an aluminum filter.
D. Renal failure. 3. Increasing the target-film distance.
4. Using low kVp.
Minor tooth movement to correct an
inclined fixed partial denture abutment A. (1) (2) (3) *
will B. (1) and (3)
C. (2) and (4)
1. Enhance resistance form of the D. (4) only
abutment. E. All of the above.
2. Reduce the possibility of pulpal
exposure. Excessively dark radiographs are the
3. Direct occlusal forces along the long result of
axis of the tooth.
4. Improve embrasure form. 1. Underdevelopment.
2. Overexposure.
A. (1) (2) (3) 3. Backward placement of the film.
B. (1) and (3) 4. Excessive milliamperage.
C. (2) and (4)
D. (4) only A. (1) (2) (3)
E. All the above. * B. (1) and (3)
C. (2) and (4) *
Probing depth reduction following D. (4) only
scaling and root planing alone will occur E. All of the above.
if the patient's periodontal condition
includes The depth of penetration of X-rays
depends on the
A. hyperemic and edematous gingival
tissue. * 1. Milliamperage.
B. gingival hyperplasia due to phenytoin 2. Density of the subject.
therapy. 3. Exposure time.
C. chronic periodontal pockets. 4. Kilovoltage.
D. necrotizing ulcerative gingivitis.
A. (1) (2) (3)
A daily dose of 81 mg of acetylsalicylic B. (1) and (3)
acid is used for its C. (2) and (4) *
D. (4) only
A. analgesic properties.
E. All of the above.
B. antipyretic effect.
C. antiplatelet action. *
D. anti-inflammatory function.

Dr Abdul Naser Tamim SEHA, Dr Ghada Al Aqqad D.D.S, Dr Kamal Naser - Amrita Medical Centre, Dr Emad Wani - Shadi Dental Centre,
Dr Lina Anka SEHA, Dr Rouba Zgheibi SEHA, Maria Teresa Yongson Alejandrino SEHA 279
Which of the following modifications is of the sinus is removed with the tooth.
recommended when making Your immediate treatment would be to
radiographs on an 8 year old child
compared to an adult? A. place a large strip of iodoform gauze,
filling the tooth socket.
A. Increase the kVp. B. irrigate the sinus repeatedly and
B. Reduce the exposure time. * place a dressing in the tooth socket.
C. Increase the mA. C. approximate the soft tissue as
D. Use a lower speed receptor. accurately as possible without irrigating.
*
The sharpness of a radiographic image D. prescribe an antibiotic and recall
can be improved by after 24hrs.

A. increasing the object-to-film Periodontitis is differentiated from


distance. V gingivitis by the amount of
B. using a larger focal spot.
C. using a larger collimator. A. plaque accumulation.
D. increasing the focal spot-to-film B. apical migration of the epithelial
distance. * attachment. *
C. gingival inflammation.
Which of the following D. discomfort of the patient.
milliamperage/kilovoltage combinations E. subgingival calculus.
results in an X-ray beam with the most
penetration? The epithelial attachment does NOT
migrate apically in
A. 10kVp - 65ma.
B. 85kVp - 5ma. A. aggressive periodontitis.
C. 90kVp - 10ma. * B. gingival hyperplasia. *
D. 65kVp - 15ma. C. chronic periodontitis.
E. 75kVp - 40ma. D. necrotizing ulcerative periodontitis.

A 2cm, discrete, white lesion of the Gingivitis is associated with


buccal mucosa has not resolved after
elimination of all local irritants. The 1. Bone loss.
most appropriate management would 2. Cyanosis of gingival tissue.
be to 3. The presence of minimal attached
gingiva.
A. cauterize it. 4. Bleeding on probing.
B. apply toluidine blue staining.
C. perform an incisional biopsy. * A. (1) (2) (3)
D. re-examine at 6 month intervals. B. (1) and (3)
E. refer patient to family physician. C. (2) and (4)
D. (4) only *
During the extraction of an isolated E. All of the above.
maxillary second molar, the bony floor

Dr Abdul Naser Tamim SEHA, Dr Ghada Al Aqqad D.D.S, Dr Kamal Naser - Amrita Medical Centre, Dr Emad Wani - Shadi Dental Centre,
Dr Lina Anka SEHA, Dr Rouba Zgheibi SEHA, Maria Teresa Yongson Alejandrino SEHA 280
In gingivitis, the sulcular epithelium mesially tipped mandibular second
molar with fixed appliances?
1. Is a barrier to bacterial invasion.
2. is permeable to bacterial enzymes 1. Extrusion of the second mandibular
and toxins. molar.
3. May be ulcerated. 2. Lingual tipping of the second molar.
4. undergoes both degenerative and 3. Intrusion of the buccal segment.
proliferative changes. 4. Buccal tipping of the buccal
segment.
A. (1) (2) (3)
B. (1) and (3) A. (1) (2) (3)
C. (2) and (4) B. (1) and (3)
D. (4) only C. (2) and (4)
E. All of the above. * D. (4) only
E. All of the above. *
Which of the following is/are clinical
signs of gingivitis? In which of the following patients can
you safely extract a tooth in a dental
1. Loss of stippling. office setting?
2. Gingival hyperplasia.
3. Bleeding on probing. A. Two weeks after a myocardial
4. Loss of attachment. infarct.
B. Four months into pregnancy. *
A. (1) (2) (3) * C. Suffering from Factor VIII deficiency.
B. (1) and (3) D. Presenting with bilateral
C. (2) and (4) submandibular space abcess.
D. (4) only
E. All of the above. Maxillary midline diastema can be
caused by
Fixed orthodontic appliances are
generally more efficient than 1. A mesiodens.
removable appliances because 2. Congenitally missing lateral incisors.
conventional fixed appliances 3. A tongue thrust habit.
4. A thumb-sucking habit.
A. provide 3-dimensional control of
tooth movement. * A. (1) (2) (3)
B. promote improved oral hygiene. B. (1) and (3)
C. provide intermittent force C. (2) and (4)
application. D. (4) only
D. are individualized to each patient. E. All of the above. *

What are the anticipated orthodontic


side effects when uprighting a

Dr Abdul Naser Tamim SEHA, Dr Ghada Al Aqqad D.D.S, Dr Kamal Naser - Amrita Medical Centre, Dr Emad Wani - Shadi Dental Centre,
Dr Lina Anka SEHA, Dr Rouba Zgheibi SEHA, Maria Teresa Yongson Alejandrino SEHA 281
Occipital and/or cervical extra-oral
anchorage Which of the periodontal ligament
fibres are most commonly associated
1. Enhances anterior tooth movement. with orthodontic relapse?
2. Enhance mandibular forward
growth. A. Oblique.
3. Restricts posterior tooth movement. B. Diagonal.
4. Restricts maxillary forward growth. C. Horizontal.
D. Supracrestal. *
A. (1) (2) (3)
B. (1) and (3) An insulin-dependent diabetic patient
C. (2) and (4) * has rushed in late for an early morning
D. (4) only ? 1 hour long appointment. Which of the
E. All of the above. following should be verified? That

Which of the following clinical A. a low-carbohydrate snack is


approaches would reduce the available in case of an insulin reaction.
tendency to the post-extraction B. the patient has taken his regular
complication of dry socket? dose of insulin.
C. the patient has eaten his regular
1. Post-extraction socket dressing with morning breakfast. *
topical chlorhexidine. D. an extra dose of insulin is available
2. Pre-extraction control of in case of a hypoglycemic reaction.
pericoronitis.
3. Use of a pre-operative mouthrinse The rate limiting step of tooth
of 0.2% chlorhexidine gluconate. movement is
4. Use of systemic antibioprophylaxis.
A. bone deposition.
A. (1) (2) (3) * B. bone resorption. *
B. (1) and (3) C. force application.
C. (2) and (4) D. occlusal interference.
D. (4) only
E. All of the above. Tetracycline will cause dentin
discolouration when ingested at the
Surgery of irradiated bone is age of
complicated by the fact that the
1. 6 months in utero.
1. Original neoplasm may invade the 2. 2 years.
area of surgery. 3. 7 years.
2. Bone becomes brittle. 4. 14 years.
3. Patients are immunocompromised.
4. Sclerosed blood vessels A. (1) (2) (3) *
compromise healing. B. (1) and (3)
C. (2) and (4)
A. (1) (2) (3) D. (4) only
B. (1) and (3) E. All of the above.
C. (2) and (4)
D. (4) only
E. All of the above. *

Dr Abdul Naser Tamim SEHA, Dr Ghada Al Aqqad D.D.S, Dr Kamal Naser - Amrita Medical Centre, Dr Emad Wani - Shadi Dental Centre,
Dr Lina Anka SEHA, Dr Rouba Zgheibi SEHA, Maria Teresa Yongson Alejandrino SEHA 282
An 8 year old child has an 8 mm B. (1) and (3)
diastema between teeth 1.1 and 2.1. C. (2) and (4)
This could indicate the presence of D. (4) only
E. All of the above. *
1. An enlarged frenum.
2. A cyst. Which of the following is a clinical
3. One or more mesiodens. CONTRAINDICATION for an all-
4. Normal development. ceramic maxillary anterior crown?

A. (1) (2) (3) * A. Excessive overjet and overbite.


B. (1) and (3) B. Normal overjet and excessive
C. (2) and (4) overbite. *
D. (4) only C. An endodontically treated tooth with
E. All of the above. a cast post and core.
D. A low caries index.
A 4 year old child has a chronically
infected non-restorable primary first Signs and symptoms of occlusal
molar. The most appropriate traumatism are
management is to
1. Pain.
A. extract it and place a space 2. Tooth mobility.
maintainer. * 3. Radiographic evidence of increased
B. observe it until it exfoliates. periodontal ligament space.
C. extract it. 4. Loss of pulp vitality.
D. observe it until it becomes
symptomatic. A. (1) (2) (3) *
B. (1) and (3)
During the extraction of a primary tooth, C. (2) and (4)
when is it appropriate to leave root tips D. (4) only
in the socket? E. All of the above.

1. There is no infection in the area. Ionization of local anesthetics is


2. Root tip size is less than 2 mm. facilitated by a tissue pH
3. Tips will not interfere with ensuing
restorative procedures. A. above 7.4.
4. Tips will not interfere with B. at 7.4.
development or eruption of permanent C. below 7.4. *
teeth.

A. (1) (2) (3)

Dr Abdul Naser Tamim SEHA, Dr Ghada Al Aqqad D.D.S, Dr Kamal Naser - Amrita Medical Centre, Dr Emad Wani - Shadi Dental Centre,
Dr Lina Anka SEHA, Dr Rouba Zgheibi SEHA, Maria Teresa Yongson Alejandrino SEHA 283
A metal in the wrought condition differs 2. Devitalization of the teeth.
from the same metal in the cast 3. Pain.
condition in that 4. Inefficient tooth movement.

1. The grains are deformed and A. (1) (2) (3)


elongated. B. (1) and (3)
2. The yield strength and hardness are C. (2) and (4)
increased. D. (4) only
3. The resistance to corrosion is E. All of the above. *
decreased.
4. If heated sufficiently, A 50 year old obese patient was
recrystallization can occur. diagnosed with type 2 diabetes last
year and has recently started taking an
A. (1) (2) (3) oral hypoglycemic. He frequently skips
B. (1) and (3) meals in order to reduce his weight.
C. (2) and (4) During his 8:30 a.m. appointment, his
D. (4) only speech becomes slurred and he is less
E. All of the above. * alert than usual. Which of the following
is the most appropriate management of
The facial profile most often associated this patient?
with an Angle’s Class III malocclusion
is A. Make him drink 175ml of diet cola.
B. Give him 15g of glucose as tablets
A. concave. * or in a solution. *
B. convex. C. Make him eat a chocolate bar.
C. straight. D. Dismiss the patient and advise him
to eat.
Which of the following muscles causes
displacement of the condyle in a A clinical sign of an acute disc
subcondylar fracture of the mandible? dislocation without reduction in the right
TMJ is
A. Medial pterygoid.
B. Lateral pterygoid. * A. deflection of the mandible to the left
C. Masseter. at maximal mouth opening.
D. Temporalis. B. deflection of the mandible to the
E. Superior pharyngeal constrictor. right during protrusion. *
C. maximal opening of 55mm.
Prolonged heavy orthodontic forces D. reciprocal click in the right TMJ.
may result in E. limitation in right laterotrusion.

1. hyalinization of periodontal ligament.

Dr Abdul Naser Tamim SEHA, Dr Ghada Al Aqqad D.D.S, Dr Kamal Naser - Amrita Medical Centre, Dr Emad Wani - Shadi Dental Centre,
Dr Lina Anka SEHA, Dr Rouba Zgheibi SEHA, Maria Teresa Yongson Alejandrino SEHA 284
Which of the following is suggestive of D. Normal eruption pattern.
a malignant tumour of the parotid
gland? A tooth with a nonvital pulp may
occasionally present radiographically
A. Excessive salivation. with shortening or blunting of the apical
B. Seventh cranial nerve paralysis. * tip of a root. The loss of apical
C. Sudden swelling. cementum and dentin would be
D. Fluctuation. classified as what type of resorption?

An incipient carious lesion on an A. Surface.


interproximal surface is usually located B. Inflammatory. *
C. Replacement.
A. vestibular to the contact area. D. Ankylosis.
B. lingual to the contact area.
C. gingival to the contact area. * The coronoid process of the mandible
D. occlusal to the contact area. can be seen in all of the following types
of radiographs EXCEPT
The optimal time for orthodontic
treatment involving growth modification A. periapical. *
is during the B. maxillary occlusal.
C. panoramic.
A. late primary dentition. D. lateral cephalometric.
B. early mixed dentition.
C. late mixed dentition. * Total removal of subgingival calculus
D. early permanent dentition. on a tooth with pockets more than 5mm
deep is best achieved by
When a simple tipping force is applied
to the crown of a single-rooted tooth, A. ultrasonic scaling.
the centre of rotation is located B. root planing.
C. surgery and scaling. *
A. at the apex. D. subgingival irrigation.
B. at the cervical line.
C. within the apical half of the root. * Which of the following does NOT have
D. within the cervical one third of the a multilocular radiolucent appearance?
root.
A. Ameloblastoma.
An 8 year old patient has a 3mm B. Odontogenic myxoma.
diastema between the erupting C. Stafne's bone defect/static bone
permanent maxillary central incisors. cavity. *
What is the most likely cause? D. Central giant cell granuloma.
E. Cherubism.
A. Failure of fusion of the premaxillae.
B. Abnormal labial frenum. *
C. Supernumerary tooth in the midline.

Dr Abdul Naser Tamim SEHA, Dr Ghada Al Aqqad D.D.S, Dr Kamal Naser - Amrita Medical Centre, Dr Emad Wani - Shadi Dental Centre,
Dr Lina Anka SEHA, Dr Rouba Zgheibi SEHA, Maria Teresa Yongson Alejandrino SEHA 285
Thinned cortical bone, decreased
cancellous trabeculation, enlargement Migration of teeth may be associated
of the medullary cavity and decreased with
bone density are radiographic features
of 1. Lip habits.
2. Tongue habits.
A. osteomalacia. 3. Bruxism.
B. osteopetrosis. 4. Periodontitis.
C. osteoradionecrosis.
D. osteoporosis. * A. (1) (2) (3)
B. (1) and (3)
For patients with prion disease such as C. (2) and (4)
Creutzfeldt-Jacob disease, D. (4) only
E. All of the above. *
A. the disease is transmissible through
air or by casual contact. The signs of chronic periodontitis
B. no additional precautions beyond include
standard procedures are
recommended for dental treatment. 1. Inflammation.
C. the disease is usually fatal within 2. Loss of attachment.
one year. * 3. Bone resorption.
D. the disease has an incubation time 4. Periodontal pockets.
of 3-6 months.
A. (1) (2) (3)
Which properties increase the B. (1) and (3)
tendency of a drug to cross cell C. (2) and (4)
membranes?
D. (4) only
E. All of the above. *
A. Non-ionized and high lipid solubility*
B. Non-ionized and low lipid solubility. A patient with bruxism is likely to
C. Ionized and low lipid solubility. demonstrate
D. Ionized and water solubility.
1. Radiographic evidence of the
Corticosteroids may be used for the widening of the periodontal ligament.
management of 2. increased mobility of teeth.
3. Premature wear of occlusal surfaces.
1. Allergy.
4. TMJ discomfort.
2. Arthritis.
3. Asthma. A. (1) (2) (3)
4. Addison's disease. B. (1) and (3)
C. (2) and (4)
A. (1) (2) (3)
D. (4) only
B. (1) and (3)
E. All of the above. *
C. (2) and (4)
D. (4) only
E. All of the above. *

Dr Abdul Naser Tamim SEHA, Dr Ghada Al Aqqad D.D.S, Dr Kamal Naser - Amrita Medical Centre, Dr Emad Wani - Shadi Dental Centre,
Dr Lina Anka SEHA, Dr Rouba Zgheibi SEHA, Maria Teresa Yongson Alejandrino SEHA 286
Which permanent tooth usually erupts
first? 1. Gamma radiation.
2. Penetration.
A. Maxillary central incisor. 3. Collimation.
B. Maxillary canine. 4. Secondary radiation.
C. Mandibular first molar. *
D. Mandibular central incisor. A. (1) (2) (3)
B. (1) and (3)
Following very early loss of a primary C. (2) and (4) *
tooth, the eruption of the permanent D. (4) only.
successor could be E. All of the above.

1. Unaltered. Dental implants are


2. Accelerated. CONTRAINDICATED in patients who
3. Delayed.
4. Arrested. 1. Are over age 80.
2. Have unrepaired cleft palates.
A. (1) (2) (3) 3. Are taking anticoagulants.
B. (1) and (3) * 4. Have uncontrolled diabetes mellitus.
C. (2) and (4)
D. (4) only A. (1) (2) (3)
E. All of the above. B. (1) and (3)
C. (2) and (4)
Orthodontic tooth movement is D. (4) only *
generally more efficient with E. All of the above.
adolescents than in adults because of
the difference in In drug synergism

1. tissue/bone reaction. A. the effect of the combination of two


2. The rate of growth. drugs is greater than the sum of their
3. The quality of cooperation. individual effects. *
4. The crown/root ratio. B. a drug accumulates in the body at a
faster rate than it is destroyed or
A. (1) (2) (3) ? removed.
B. (1) and (3) C. a drug is concentrated in the
C. (2) and (4) kidneys.
D. (4) only D. a drug produces active metabolites.
E. All of the above. * E. the concentration of the drug
increases with time.
Increasing the kilovoltage results in
greater

Dr Abdul Naser Tamim SEHA, Dr Ghada Al Aqqad D.D.S, Dr Kamal Naser - Amrita Medical Centre, Dr Emad Wani - Shadi Dental Centre,
Dr Lina Anka SEHA, Dr Rouba Zgheibi SEHA, Maria Teresa Yongson Alejandrino SEHA 287
Tooth reduction for anterior tooth D. do not have an effect on any other
preparation for metal-ceramic and all tissue than the nervous tissue.
ceramic crowns is dictated by which of
the following factor(s)? Procaine is a local anaesthetic which
is chemically classified as an
1. Crown length for adequate
retention-resistance. A. amide.
2. Porcelain/ceramic thickness for B. ester. *
fracture resistance. C. aldehyde.
3. Clearance for occlusal function. D. ethamine.
4. Parallelism of axial walls for E. aminide.
facilitating the path of insertion.
When developer solution is old and
A. (1) (2) (3) * oxydized, radiographs will appear
B. (1) and (3)
C. (2) and (4) A. blue.
D. (4) only B. brown. *
E. All of the above. ? C. gray.
D. black.
Which of the following radiographs is
best to diagnose caries and early The periodontal probe should be
alveolar bone loss? inserted into the gingival sulcus

A. Periapical. A. parallel to the long axis of the


B. Occlusal. tooth*
C. Bitewing. * B. parallel to the root surface. ??
D. Lateral jaw. C. with a firm pushing motion.
E. Panoramic. D. with a firm lateral motion.

In an infrabony periodontal pocket, the The selection of a vasoconstrictor for


tip of the periodontal probe is located a local anesthetic depends upon the

A. at the level of the alveolar crest. 1. Duration of the procedure.


B. coronal to the level of the alveolar 2. Need for hemostasis.
crest. 3. Medical status of the patient.
C. apical to the level of the alveolar 4. Type of procedure.
crest. *
A. (1) (2) (3)
Local anesthetics B. (1) and (3)
A. do not readily pass the blood-brain C. (2) and (4)
barrier. D. (4) only
B. interfere with the propagation of E. All of the above. *
action potentials in nerve fibres. *
C. selectively interfere with the
propagation of action potentials in
nociceptive fibres.

Dr Abdul Naser Tamim SEHA, Dr Ghada Al Aqqad D.D.S, Dr Kamal Naser - Amrita Medical Centre, Dr Emad Wani - Shadi Dental Centre,
Dr Lina Anka SEHA, Dr Rouba Zgheibi SEHA, Maria Teresa Yongson Alejandrino SEHA 288
Movement of a mandibular distal B. a true unilateral crossbite.
extension (Class 1) partial denture C. occlusal interference and functional
away from the denture bearing tissues shift. *
when the patient opens is primarily
caused by If there is insufficient arch space for a
permanent tooth to erupt, the tooth may
A. group function occlusion.
B. non-passive retentive arms. A. cause resorption of the root of
C. overextended borders. * another tooth.
B. erupt out of position.
Denture stomatitis can be associated C. not erupt.
with D. All of the above. *
1. Candida albicans infection.
2. Xerostomia. The Plaque Index of Silness and Loe
3. Inadequate denture hygiene. measures
4. Ill-fitting dentures.
A. quantity of plaque at the gingival
margin. *
A. (1) (2) (3) B. colony forming units of Gram-
B. (1) and (3) negative bacteria.
C. (2) and (4) C. weight of plaque obtained from facial
D. (4) only surfaces.
E. All of the above. *
Radiographs of a periodontally related
When probing a healthy gingival osseous defect show the
sulcus with a 20g force, the tip of the
periodontal probe is most likely located A. number of bony walls.
B. measurement of the defect.
A. coronal to the junctional epithelium. C. location of the epithelial attachment.
B. at the level of the junctional D. loss of crestal cortication. *
epithelium. *
C. apical to the junctional epithelium. Radiographs of a periodontally related
osseous defect can be used to confirm
A shift from bilateral crossbite with the
coincident midlines to a right-sided
posterior crossbite with non-coincident A. number of bony walls.
midlines most likely indicates B. measurement of the defect.
C. location of the epithelial attachment.
D. presence of a furcation involvement*
A. severe temporomandibular
dysfunction.

Dr Abdul Naser Tamim SEHA, Dr Ghada Al Aqqad D.D.S, Dr Kamal Naser - Amrita Medical Centre, Dr Emad Wani - Shadi Dental Centre,
Dr Lina Anka SEHA, Dr Rouba Zgheibi SEHA, Maria Teresa Yongson Alejandrino SEHA 289
Which of the following should be B. (1) and (3) *
considered when assessing the C. (2) and (4)
difficulty of removal of an impacted D. (4) only
mandibular third molar? E. All of the above.

1. Angulation. Multiple radiolucent lesions of the jaws


2. Root width. are features of
3. Depth in the alveolus.
4. Periodontal ligament space. 1. Hyperparathyroidism.
2. Multiple myeloma.
A. (1) (2) (3) 3. Basal cell nevus syndrome.
B. (1) and (3) 4. Hyperthyroidism.
C. (2) and (4)
D. (4) only A. (1) (2) (3) *
E. All of the above. * B. (1) and (3)
C. (2) and (4)
Which of the following is/are possible D. (4) only.
side effect(s) of codeine? E. All of the above.

1. Respiratory depression. What is the most likely diagnosis of a


2. Nausea. small well-circumscribed radiolucency
3. Drowsiness. at the apex of a vital mandibular
4. Diarrhea. incisor?

A. (1) (2) (3) * A. a periapical granuloma.


B. (1) and (3) B. sclerosing osteitis.
C. (2) and (4) C. a radicular cyst.
D. (4) only D. periapical cemental dysplasia. *
E. All of the above.
Which radiographic examination
Pontic design for a metal-ceramic provides the most diagnostic
bridge should information for the replacement of a
permanent mandibular first molar with
1. Provide for a rigid restoration. an osseo-integrated implant?
2. Allow for complete coverage of the
metal by the porcelain. A. Periapical.
3. Place the porcelain metal joint away B. Occlusal.
from the soft tissues. C. Panoramic.
4. Control thermal conductivity. D. Cone beam computed tomography*

A. (1) (2) (3)

Dr Abdul Naser Tamim SEHA, Dr Ghada Al Aqqad D.D.S, Dr Kamal Naser - Amrita Medical Centre, Dr Emad Wani - Shadi Dental Centre,
Dr Lina Anka SEHA, Dr Rouba Zgheibi SEHA, Maria Teresa Yongson Alejandrino SEHA 290
Appropriate collimation of the X-ray analgesic and anti-inflammatory effect
beam for the film size reduces is

1. Image definition. A. antagonism of histamine.


2. Secondary radiation. B. inhibition of prostaglandin
3. Radiographic contrast. synthesis.*
4. Radiation dose. C. local anesthetic effect on pain
fibres.
A. (1) (2) (3) D. release of adrenal steroids from the
B. (1) and (3) adrenal cortex.
C. (2) and (4) * E. synaptic inhibition in the dorsal
D. (4) only. column.
E. All of the above.
What is the primary etiologic factor for
The most appropriate radiograph to generalized aggressive periodontitis?
assess an interproximal vertical bony
defect of the alveolar bone is the A. Altered lymphocyte activity.
B. Generalized subgingival calculus.
A. bite-wing. * C. Impaired polymorphonuclear
B. periapical. phagocytosis. *
C. occlusal. D. Bacterial plaque.
D. panoramic.
Following periodontal flap surgery, the
Overlapped interproximal contacts in most common cause of recurrence of
a bite-wing radiograph are caused by pockets is
an X-ray tube cone
A. systemic disease.
A. with incorrect vertical angulation. B. traumatic occlusion.
B. with incorrect horizontal angulation* C. failure to splint.
C. of incorrect length. D. poor oral hygiene. *
D. of incorrect shape.
Local anesthetics interfere with the
The amount of radiation is best transport of which of the following
reduced by ions?

A. using collimation. * A. Sodium. *


B. decreasing the object-film distance. B. Calcium.
C. lowering kVp and increasing C. Chloride.
milliamperage. D. Potassium.
D. decreasing the target-object E. Magnesium.
distance.

A significant mechanism by which


acetylsalicylic acid produces its

Dr Abdul Naser Tamim SEHA, Dr Ghada Al Aqqad D.D.S, Dr Kamal Naser - Amrita Medical Centre, Dr Emad Wani - Shadi Dental Centre,
Dr Lina Anka SEHA, Dr Rouba Zgheibi SEHA, Maria Teresa Yongson Alejandrino SEHA 291
The shape of the distobuccal flange of
a mandibular denture is determined by 1. Acetaminophen.
the 2. Atropine.
3. Acetylsalicylic acid.
1. Buccinator muscle. 4. Scopolamine.
2. Tendon of the temporal muscle.
3. Masseter muscle. A. (1) (2) (3)
4. External oblique ridge. B. (1) and (3)
C. (2) and (4) *
A. (1) (2) (3) D. (4) only
B. (1) and (3) * E. All of the above.
C. (2) and (4)
D. (4) only Cephalosporins
E. All of the above.
1. May be cross-allergenic with
Which patient is LEAST likely to be penicillin.
predisposed to liver toxicity following a 2. Have a narrower spectrum than
dose of 1,000 mg of acetaminophen? penicillin.
3. Have a mechanism of action similar
A. An adult with liver cirrhosis. to that of penicillin.
B. A chronic alcoholic. 4. May cause cholestatic hepatitis.
C. A diabetic. *
D. A 15kg, 4 year old child. A. (1) (2) (3)
B. (1) and (3) *
A pontic exerting too much pressure C. (2) and (4)
against the ridge may cause D. (4) only
E. All of the above.
1. Fracture of the solder joints.
2. Hypertrophy of the soft tissue. A previously well-controlled periodontal
3. Crazing of the gingival portion of the patient now demonstrates the presence
porcelain. of bleeding in 60% of sites and
4. Resorption of the alveolar bone. significantly increased probing depths.
The patient has most likely developed
A. (1) (2) (3)
B. (1) and (3) A. diabetes mellitus. *
C. (2) and (4) * B. hyperchromatosis.
D. (4) only C. osteoporosis.
E. All of the above. D. hypercalcemia.

Which of the following drugs has/have


antisialagogue properties?

Dr Abdul Naser Tamim SEHA, Dr Ghada Al Aqqad D.D.S, Dr Kamal Naser - Amrita Medical Centre, Dr Emad Wani - Shadi Dental Centre,
Dr Lina Anka SEHA, Dr Rouba Zgheibi SEHA, Maria Teresa Yongson Alejandrino SEHA 292
The most common order of serial D. (4) only
extraction in the maxillary arch is E. All of the above. *

A. primary canines, first premolars, A 10 year old patient with a lateral


primary first molars. functional shift due to a maxillary
B. primary canines, primary first bilateral posterior crossbite
molars, first premolars. *
C. First premolars, primary canines, 1. Should not be treated until growth is
primary first molars. complete.
D. primary first molars, primary 2. Should be diagnosed in maximum
canines, first premolars intercuspation.
3. Requires rapid palatal expansion.
Increased overjet, moderate facial 4. has a centric occlusion-centric
convexity and a deep labio-mental relation (CO-CR) shift caused by
sulcus is most characteristic of which occlusal interferences.
facial type?
A. (1) (2) (3)
A. Prognathic. B. (1) and (3)
B. Orthognathic. C. (2) and (4)
C. Retrognathic. * D. (4) only. *
E. All of the above.
Early orthodontic treatment of a
retrognathic mandible should occur An extreme overjet, a recessive chin
and a deep labial mento-labial sulcus
A. before age 5. are common findings in which facial
B. between the ages of 6 and 8. type?
C. just prior to the pubertal growth
spurt. * A. Prognathic.
D. after skeletal maturity. B. Mesognathic.
C. Retrognathic. *
Which of the following is/are side D. Crossbite.
effects of rapid maxillary expansion?
The occlusal parameter most useful to
1. Increase in the vertical dimension. differentiate between an overbite of
2. Mesial movement of the maxilla. dental or skeletal origin is the
3. Increase in the maxillary inter-molar
width. A. mandibular curve of Spee. *
4. Increase in the mandibular inter- B. mandibular curve of Wilson.
molar width. C. maxillary curve of Wilson.
D. maxillary incisors morphology.
A. (1) (2) (3) E. maxillary lip length.
B. (1) and (3)
C. (2) and (4)

Dr Abdul Naser Tamim SEHA, Dr Ghada Al Aqqad D.D.S, Dr Kamal Naser - Amrita Medical Centre, Dr Emad Wani - Shadi Dental Centre,
Dr Lina Anka SEHA, Dr Rouba Zgheibi SEHA, Maria Teresa Yongson Alejandrino SEHA 293
A 12 year old patient has the following Premature loss of mandibular primary
cephalometric values: cuspids in Angle Class I and Class II
SNA = 87° (N = 82°) cases results in increased
SNB = 80° (N = 80°)
Mandibular Plane Angle = 32° (N = 1. Overjet.
32°) 2. Arch width.
FMA = 26° (N = 26°) 3. Overbite.
The patient is a skeletal Angle Class II 4. Leeway space.
with a
A. (1) (2) (3)
A. normally protrusive maxilla and a B. (1) and (3) *
retrognathic mandible. C. (2) and (4)
B. protrusive maxilla and an D. (4) only
orthognathic mandible. * E. All of the above.
C. protrusive maxilla and a
retrognathic mandible. What is the most appropriate
D. retrusive maxilla and an appliance to correct an Angle Class I
orthognathic mandible. malocclusion with a labially tipped
maxillary central incisor and spacing in
What is the most appropriate a 15 year old patient?
management of an ectopically erupting
premolar with the primary predecessor A. Growth modification appliance.
firmly in place? B. Hawley with an active labial bow. *
C. Molar distalizing appliance.
A. Natural exfoliation of the primary
D. Bilateral expansion appliance.
tooth.
B. Luxation of the primary tooth. The terminal plane relationship of
C. Extraction of the primary tooth and primary second molars determines the
monitor. *
D. Extraction of the primary tooth and A. arch length between permanent
space maintenance. second molars.
B. future anteroposterior positions of
The etiology of an Angle Class II permanent first molars. *
malocclusion in a 12 year old with an
C. vertical dimensions of the mandible
SNA = 82 and an SNB = 75 is most
upon eruption of permanent first
likely
molars.
D. amount of leeway space that is
A. dental.
available for permanent premolars and
B. skeletal. * canines.
C. neuromuscular.
D. dental and neuromuscular.
E. skeletal and dental.

Dr Abdul Naser Tamim SEHA, Dr Ghada Al Aqqad D.D.S, Dr Kamal Naser - Amrita Medical Centre, Dr Emad Wani - Shadi Dental Centre,
Dr Lina Anka SEHA, Dr Rouba Zgheibi SEHA, Maria Teresa Yongson Alejandrino SEHA 294
Leeway space is most efficiently 3. Nitroglycerin.
maintained by a/an 4. Warfarin.

A. lingual arch. * A. (1) (2) (3)


B. tongue crib. B. (1) and (3)
C. anterior bite plate. C. (2) and (4)
D. functional appliance. D. (4) only
E. All of the above. *
Which of the following nerves are
anesthetized by an infraorbital nerve Adequate position of a needle in the
block? lumen of a blood vessel for a
venipuncture is confirmed by the
1. Palpebral.
2. Lateral nasal. A. presence of blood upon aspiration. *
3. Labial. B. rapidity of filling of the syringe upon
4. Zygomatico-facial. aspiration.
C. pain associated with vessel
A. (1) (2) (3) * puncture.
B. (1) and (3)
C. (2) and (4) A sedative drug should
D. (4) only
E. All of the above. A. cause excitement.
B. eliminate all sensation.
Which of the following antibiotics is the C. produce unconsciousness.
most appropriate for a patient with an D. produce a mild state of central
open fracture of the mandible? nervous system depression. *

A. Clarithromycin. The surgical risk for a patient with


B. Tetracycline. organic heart disease depends upon
C. Amoxicillin. *
D. Erythromycin. A. cardiac reserve. *
B. blood pressure.
Which of the following drugs will C. respiration.
influence your periodontal treatment D. pulse rate.
planning? E. cardiac output.

1. Insulin.
2. Cortisone.

Dr Abdul Naser Tamim SEHA, Dr Ghada Al Aqqad D.D.S, Dr Kamal Naser - Amrita Medical Centre, Dr Emad Wani - Shadi Dental Centre,
Dr Lina Anka SEHA, Dr Rouba Zgheibi SEHA, Maria Teresa Yongson Alejandrino SEHA 295
Diagnostic casts for a fixed bridge E. All of the above. *
allow the dentist to
Nitrous oxide
1. Visualize the direction of the
occlusal forces. 1. Provides good muscle relaxation.
2. Assess occlusion more accurately. 2. is non-explosive and non-
3. Plan the pontic design. flammable.
4. Assess the esthetics using a 3. Is a potent anesthetic.
diagnostic wax-up. 4. provides rapid induction and
recovery.
A. (1) (2) (3)
B. (1) and (3) A. (1) (2) (3)
C. (2) and (4) B. (1) and (3)
D. (4) only C. (2) and (4) *
E. All of the above. * D. (4) only
E. All of the above.
Advantages of resin bonded bridges
are Propoxyphene has the potential for
drug abuse because
1. Tooth structure conservation.
2. Short chair-side time. A. it is a potent analgesic with a
3. Lower cost for patient. euphoriant activity equal to morphine.
4. Improved esthetics compared to B. in combination with alcohol its
traditional bridges. effects are greatly intensified. *
C. it cannot be detected in the
A. (1) (2) (3) * bloodstream.
B. (1) and (3)
C. (2) and (4) Bioavailability of orally administered
D. (4) only drugs may be influenced by
E. All of the above.
1. Formulation of the drug.
Which of the following will produce 2. Gastrointestinal perfusion.
hemostasis when applied topically? 3. PH of the gastrointestinal tract.
4. Presence of other substances in the
1. Oxidized cellulose. gastrointestinal tract.
2. Absorbable gelatin sponge.
3. Microfibrillar collagen. A. (1) (2) (3)
4. Topical bovine thrombin. B. (1) and (2)
C. (2) and (4)
A. (1) (2) (3) D. (4) only
B. (1) and (3) E. All of the above. *
C. (2) and (4)
D. (4) only

Dr Abdul Naser Tamim SEHA, Dr Ghada Al Aqqad D.D.S, Dr Kamal Naser - Amrita Medical Centre, Dr Emad Wani - Shadi Dental Centre,
Dr Lina Anka SEHA, Dr Rouba Zgheibi SEHA, Maria Teresa Yongson Alejandrino SEHA 296
Properties of glutaraldehyde include A major clinical problem of penicillin
therapy is its
1. Rapid formation of cross linkages
which limit penetration of pulp tissue. A. high toxicity.
2. Minimal effect on pulp tissues. B. allergenicity. *
3. Excellent disinfection against oral C. rapid development of tolerance.
flora. D. narrow spectrum.
4. Minimal effectiveness against
viruses and spores. A patient with periodontal disease may
complain of
A. (1) (2) (3) *
B. (1) and (3) 1. Loose teeth.
C. (2) and (4) 2. Bleeding gingiva.
D. (4) only 3. Pain.
E. All of the above. 4. Gingival recession.

Which of the following drugs has/have A. (1) (2) (3)


sedative properties? B. (1) and (3)
C. (2) and (4)
1. Benzodiazepines. D. (4) only
2. Barbiturates. E. All of the above. *
3. Meperidine.
4. Phenytoin. Gingivitis is characterized by

A. (1) (2) (3) * A. bleeding on probing. *


B. (1) and (3) B. loss of attachment.
C. (2) and (4) C. alveolar bone loss.
D. (4) only D. apical migration of the junctional
E. All of the above. epithelium.

Which of the following anesthetic Which of the following may affect


agents is/are metabolized by plasma probing depth measurements of a
cholinesterase? periodontal pocket?

1. Prilocaine. 1. Probing force.


2. Lidocaine. 2. Diameter of the probe tip.
3. Mepivacaine. 3. Angulation of the probe.
4. Procaine. 4. Subgingival calculus.

A. (1) (2) (3) A. (1) (2) (3)


B. (1) and (3) B. (1) and (3)
C. (2) and (4) C. (2) and (4)
D. (4) only * D. (4) only
E. All of the above. E. All of the above. *

Dr Abdul Naser Tamim SEHA, Dr Ghada Al Aqqad D.D.S, Dr Kamal Naser - Amrita Medical Centre, Dr Emad Wani - Shadi Dental Centre,
Dr Lina Anka SEHA, Dr Rouba Zgheibi SEHA, Maria Teresa Yongson Alejandrino SEHA 297
The mesial furcation of the permanent of the natural teeth is required, this
maxillary first molar is best assessed should be performed
from which aspect of the tooth?
A. prior to taking the final impressions. *
A. Mesiobuccal. B. prior to registering centric relation but
B. Mesiopalatal. * after final impression taking.
C. Midmesial. C. after the framework has been
constructed.
A crown margin can be extended
subgingivally when required Which of the following should be
evaluated for surgical removal before
1. for esthetics. new complete dentures are fabricated?
2. To increase retention.
3. To reach sound tooth structure. 1. Mandibular tori.
4. for caries prevention. 2. Epulis fissuratum.
3. Papillary hyperplasia.
A. (1) (2) (3) * 4. Sharp, prominent mylohyoid ridges.
B. (1) and (3)
C. (2) and (4) A. (1) (2) (3)
D. (4) only B. (1) and (3)
E. All of the above. C. (2) and (4)
D. (4) only
The most common form of periodontal E. All of the above. *
disease is
When a removable partial denture is
A. gingivitis. * fabricated to occlude with natural teeth,
B. chronic periodontitis. the occlusal form of the artificial teeth is
C. gingival hyperplasia. influenced by the
D. aggressive periodontitis.
A. endodontic condition of the abutment
Diazepam teeth.
B. occlusal form of the remaining teeth*
1. Is a benzodiazepine. C. need to produce a fully balanced
2. Is contraindicated in a patient with occlusion.
acute narrow angle glaucoma.
3. is anxiolytic. A lingual plate is indicated as a major
4. Produces amnesia. connector for a removable partial
denture when
A. (1) (2) (3)
B. (1) and (3) A. insufficient room exists for a lingual
C. (2) and (4) bar. *
D. (4) only B. a palatal torus is present.
E. All of the above. * C. a retromylohyoid undercut is present.
D. there is a low attachment of the
When a partial denture is to be lingual frenum.
fabricated where occlusal adjustment

Dr Abdul Naser Tamim SEHA, Dr Ghada Al Aqqad D.D.S, Dr Kamal Naser - Amrita Medical Centre, Dr Emad Wani - Shadi Dental Centre,
Dr Lina Anka SEHA, Dr Rouba Zgheibi SEHA, Maria Teresa Yongson Alejandrino SEHA 298
Conversion of a flush terminal plane to
a mesial step/Class I terminal plane in The canal of a maxillary canine has
the absence of orthodontics is primarily been instrumented to within 1mm of
the result of the apex and is ready to be obturated.
A radiograph indicates that the master
A. loss of the mandibular primate cone is 2.5mm short of the apex. The
space. most appropriate management is to
B. greater maxillary than mandibular
forward growth. A. proceed with the filling as the cone
C. differences in leeway between the is within acceptable limits.
maxillary and mandibular arches. * B. fit a larger cone within 2mm of the
D. distal movement of the maxillary first apex.
permanent molars. C. cut the cone 1mm and insert.
D. discard the cone and fit a smaller
Typically, vital pulps of elderly patients one. *

A. show a decrease in the number of Which local anesthetic should be used


collagenous fibres. to achieve the longest pain relief for a
B. have decreased reparative capacity patient requiring the surgical extraction
compared to younger adults. of tooth 3.8?
C. form dentinal bridges after
appropriate pulp capping procedures. A. Articaine 4%, with epinephrine
D. show an increase in myelinated 1:100,000.
nerves when compared to pulps of B. Bupivacaine 0.5%, with epinephrine
younger adults. * 1:200,000. *
C. Lidocaine 2%, with epinephrine
For a 4 year old child, the most 1:100,000.
appropriate management for a D. Mepivacaine 3% plain.
chronically infected, non-restorable E. Prilocaine 4%, with epinephrine
primary first molar is to 1:200,000.

A. extract it and place a space Premature loss of a primary maxillary


maintainer. * second molar usually produces a
B. observe it until it exfoliates. malocclusion in the permanent
C. extract it only. dentition that is characterized by
D. observe it until it becomes
symptomatic. A. anterior crowding.
B. labially displaced maxillary canines.
For a 1 year old child with primary C. delayed eruption of the permanent
herpetic gingivostomatitis, the most first molar.
appropriate management is to D. an Angle Class II molar relationship
on the affected side. *
A. prescribe antibiotics. E. an Angle Class III molar relationship
B. immunize against chicken pox. on the affected side.
C. maintain adequate fluid intake. *
D. debride the lesions.
E. swab the lesions with chlorhexidine.

Dr Abdul Naser Tamim SEHA, Dr Ghada Al Aqqad D.D.S, Dr Kamal Naser - Amrita Medical Centre, Dr Emad Wani - Shadi Dental Centre,
Dr Lina Anka SEHA, Dr Rouba Zgheibi SEHA, Maria Teresa Yongson Alejandrino SEHA 299
The normal differential white cell 1. Act as an indirect retainer.
count for neutrophils is 2. Stabilize the abutment teeth.
3. Act as a direct retainer for the distal
A. 10-19% base.
4. Counteract any force transmitted by
B. 20-29%. the retentive arm.
C. 30-39%
D. 40-65%* A. (1) (2) (3)
E. 66-90% B. (1) and (3)
C. (2) and (4) *
In a free-end distal extension partial D. (4) only
denture, the most effective means of E. All of the above.
limiting applied loads to abutment
teeth is by Radiographic examination reveals
early evidence of internal resorption.
A. splinting abutments to adjacent The most appropriate management is
teeth.
B. using the abutment teeth without A. apical surgical intervention.
splinting. B. immediate pulp extirpation. *
C. using monoplane denture teeth. C. immediate pulpotomy.
D. maintaining a stable base-tissue D. observation and re-evaluation in 3-6
relationship. * months.
E. calcium hydroxide pulpotomy.
The inferior border of the lingual bar
of a removable partial denture A laboratory remount of processed
should dentures is done in order to correct
occlusal disharmony produced by
A. displace the lingual frenum. errors primarily in the
B. be in contact with the cingula of
the incisor teeth. A. mounting of the casts on the
C. be superior to the gingival border. articulator.
D. be as inferior as the movement of B. registration of jaw relation records.
the frenum of the tongue will permit* C. processing of acrylic dentures. *
D. registration of condylar guidance.
The function(s) of the reciprocal
clasp arm is/are to

Dr Abdul Naser Tamim SEHA, Dr Ghada Al Aqqad D.D.S, Dr Kamal Naser - Amrita Medical Centre, Dr Emad Wani - Shadi Dental Centre,
Dr Lina Anka SEHA, Dr Rouba Zgheibi SEHA, Maria Teresa Yongson Alejandrino SEHA 300
A characteristic of a group function
occlusion is A. Reduction in morphologic face
height. *
A. the teeth on the nonworking side B. Advanced anterior maxillary ridge
contact in lateral excursion. resorption.
B. the teeth on the working side C. Down growth of the maxillary
contact in lateral excursion. * tuberosities.
C. the canine and lateral incisors D. Advanced alveolar bone resorption
contact in lateral excursion. under the posterior partial denture base
D. the posterior teeth on both working areas.
and nonworking sides contact in lateral
excursion. Which of the following are the most
appropriate for use as overdenture
Which of the following could cause abutments?
clicking sounds during speech in
denture wearers? A. Central incisors.
B. Second premolars.
A. Excessive vertical dimension. * C. Canines. *
B. Nonbalanced occlusion. D. First premolars.
C. Excessive buccal flange thickness.
D. Reduced vertical overlap (overbite). Which of the following removable
partial dentures has the LEAST amount
A cast partial denture replacing teeth of rotation around the fulcrum?
3.5 - 3.8 and 4.5 - 4.8 was permanently
relined with acrylic resin. At the A. Kennedy Class I.
delivery appointment, when the rests of B. Kennedy Class II.
the framework are fully seated on the C. Kennedy Class III. *
abutment teeth, the denture base does
not contact the supporting tissues. The In an edentulous patient, the coronoid
most likely cause of the problem is process

A. the denture reline resin shrunk A. limits the distal extent of the
during polymerization. mandibular denture.
B. excess pressure was placed on the B. affects the position and arrangement
rests during the impression procedure. of posterior teeth.
C. excess pressure was placed on the C. aids in determining the location of
denture base area during the the posterior palatal seal.
impression procedure. * D. limits the thickness of the maxillary
D. the patient’s tissues have buccal denture flange. *
remodelled.

Which of the following clinical findings


is NOT associated with combination
(Kelly’s) syndrome?

Dr Abdul Naser Tamim SEHA, Dr Ghada Al Aqqad D.D.S, Dr Kamal Naser - Amrita Medical Centre, Dr Emad Wani - Shadi Dental Centre,
Dr Lina Anka SEHA, Dr Rouba Zgheibi SEHA, Maria Teresa Yongson Alejandrino SEHA 301
Which pontic type is best for a knife C. the same as the function of the curve
edge residual ridge where esthetics is of Spee.
not a major concern?
Success of an endosseous dental
A. Sanitary. implant is dependent upon
B. Conical. *
C. Ridge lap. 1. Biocompatibility of the material.
D. Modified ridge lap. 2. Design.
3. A period of non-function.
On a semi-adjustable articulator, the 4. Immediate loading.
incisal guide table represents
A. (1) (2) (3) *
A. a reference point for the B. (1) and (3)
establishment of occlusal vertical C. (2) and (4)
dimension. D. (4) only
B. the anterior equivalent of the E. All of the above.
condylar guidance.
C. a mechanical equivalent of the Bruxism may be associated with
horizontal and vertical overlap of the
anterior teeth. * 1. Premature contacts in the centric
D. the mechanical equivalent of the relation.
Curve of Wilson. 2. Balancing prematurities.
3. Stress.
Dislodgement of a maxillary complete 4. Anxiety.
denture may be caused by
A. (1) (2) (3)
1. under extension. B. (1) and (3)
2. Improper occlusion. C. (2) and (4)
3. Overextension. D. (4) only
4. Thickness of distobuccal flange. E. All of the above. *
A. (1) (2) (3) A suprabony pocket is associated with
B. (1) and (3)
C. (2) and (4) 1. Enlargement of the marginal gingiva.
D. (4) only 2. Horizontal loss of alveolar bone.
E. All of the above. * 3. Subgingival calculus.
4. Spontaneous bleeding.
The function of the compensating
curve is A. (1) (2) (3)
B. (1) and (3)
A. to help provide a balanced C. (2) and (4)
occlusion in complete dentures when D. (4) only
the mandible is protruded. *
E. All of the above. *
B. to aid in establishing an incisal
guide plane.

Dr Abdul Naser Tamim SEHA, Dr Ghada Al Aqqad D.D.S, Dr Kamal Naser - Amrita Medical Centre, Dr Emad Wani - Shadi Dental Centre,
Dr Lina Anka SEHA, Dr Rouba Zgheibi SEHA, Maria Teresa Yongson Alejandrino SEHA 302
The effects of plaque on vascularity of B. Basal and alveolar bone respond
the gingival connective tissue result in identically to pressure. *
C. Osteoid is a highly mineralized
1. Dilation of small blood vessels. bundle bone.
2. increased vascular permeability.
3. Proliferation of small blood vessels. An anterior open bite is commonly
4. Increase in gingival crevicular fluid. associated with

A. (1) (2) (3) A. a horizontal growth pattern.


B. (1) and (3) B. a functional shift.
C. (2) and (4) C. an associated habit. *
D. (4) only D. a normal swallowing reflex.
E. All of the above. *
The most common presenting feature
In chronic periodontitis, the causative of bilateral maxillary constriction is
microorganisms are found in
A. diastemas.
1. The periodontal ligament. B. deep overbite.
2. The connective tissues of the C. increased overjet.
gingiva. D. unilateral crossbite. *
3. The alveolar bone.
4. The periodontal pocket. Which of the following muscles of
mastication is associated with the
A. (1) (2) (3) condylar head and articular disc?
B. (1) and (3)
C. (2) and (4) * A. Masseter.
D. (4) only B. Temporalis.
E. All of the above. C. Medial pterygoid.
D. Lateral pterygoid. *
Marginal gingiva
The majority of nitrous oxide is
A. is demarcated from the attached eliminated from a patient's circulatory
gingiva by the free gingival groove. * system through the
B. is demarcated from the attached
gingiva by the mucogingival junction. A. lungs. *
C. is firmly attached to the tooth. B. kidneys.
C. liver enzymes.
Which of the following is/are correct? D. plasma.
E. GI tract.
A. Basal and alveolar bone are
histologically identical.

Dr Abdul Naser Tamim SEHA, Dr Ghada Al Aqqad D.D.S, Dr Kamal Naser - Amrita Medical Centre, Dr Emad Wani - Shadi Dental Centre,
Dr Lina Anka SEHA, Dr Rouba Zgheibi SEHA, Maria Teresa Yongson Alejandrino SEHA 303
Which of the following best describes study design would provide the best
drug synergism? answer to her question?

A. The effect of the combination of A. A prospective cohort involving


two drugs is equal to the sum of their pregnant women, some of whom have
individual effects. periodontal disease and some of
B. The effect of the combination of whom do not.
two drugs is greater than the sum of B. A randomized clinical trial of
their individual effects. * pregnant women with periodontal
C. The effect of the combination of disease, some of whom receive
two drugs is less than the sum of their periodontal therapy and some of whom
individual effects. do not.
D. One drug’s action blocks the effect C. Comparing the rates of pre-term low
of the second drug’s action. birth weight babies, between women
with no risk factors to those with
The following are possible effects of periodontal disease.
acetylsalicylic acid EXCEPT D. A systematic review comparing the
risk of delivering pre-term low birth
A. reduction of fever. weight babies among women with
B. shortening of bleeding time*. various risk factors, including the
C. suppression of inflammatory presence of periodontal disease. *
response.
D. bleeding from the gastrointestinal The Root Caries Index may
tract. underestimate the amount of disease
because
Increased bleeding is associated with
a prolonged administration of A. the number of surfaces at risk
cannot be determined.
1. Warfarin. B. it excludes subgingival lesions. *
2. Codeine. C. root caries is difficult to diagnose.
3. acetylsalicylic acid. D. it treats filled and decayed surfaces
4. Acetaminophen. as the same.

A. (1) (2) (3) A study examined caries in 349


B. (1) and (3) * children 2 to 6 years of age living in
C. (2) and (4) the Northwest Territories. The odds
ratio for severe caries based upon
D. (4) only
whether the child drank milk was 0.44
E. All of the above.
(95% CI = 0.23 to 0.84). Which of the
following is the most appropriate
Examination of a 32 year old
interpretation of these results?
pregnant woman reveals that she has
Drinking milk
generalized periodontal probing
depths of 4-6mm, most of which
A. was protective against caries. *
bleed on probing. She is otherwise
healthy, but asks “If I don’t get the B. was not protective against caries.
proper periodontal treatment, am I C. had no effect on caries.
more likely to deliver a pre-term, low-
birth weight baby?” Which type of

Dr Abdul Naser Tamim SEHA, Dr Ghada Al Aqqad D.D.S, Dr Kamal Naser - Amrita Medical Centre, Dr Emad Wani - Shadi Dental Centre,
Dr Lina Anka SEHA, Dr Rouba Zgheibi SEHA, Maria Teresa Yongson Alejandrino SEHA 304
To assess factors affecting the those receiving fluoride varnish. What
utilization of dental services, a cross- type of reporting error is of the most
sectional study surveyed adults on a concern when interpreting these
number of variables. Considering the results?
study design, which of the following
findings CANNOT be derived from the A. Type II (ß) error.
results? B. Type I (ß) error.
C. Type II (α) error.
A. Satisfaction with the services. D. Type I (α) error. *
B. Gender differences.
C. Caries incidence. * The findings from a meta-analysis of
D. Utilization of dental services. 24 clinical trials conducted over 20
years, involving vitamin D
A study has been conducted supplementation to prevent dental
comparing rates of dental caries in 25 caries in children aged 5-14 years,
cities with differing levels of fluoride in reveal there is wide variation in the
the water supply. This study is a/an results between the studies. Which of
the following differences is LEAST
A. cross-sectional study. likely responsible for this variation?
B. community trial.
C. ecological study. * A. Age of the children.
D. case series. B. Stage of dentition of children.
C. Years when the studies were
Patterns of exposure to a risk factor in conducted. *
a population is important because it D. Baseline caries levels of children.
helps
What is the most appropriate
A. understand where preventive medication to manage postoperative
interventions can be targeted. * pain for an asthmatic patient taking
B. define the prevalence of a disease. beclomethasone and salbutamol?
C. define the population attributable
fraction of a disease. A. Acetylsalicylic acid.
D. give an insight into what statistical B. Acetylsalicylic acid/codeine
tests to use. combination.
C. Ibuprofen.
What is the most appropriate design for D. Acetaminophen/codeine
a study that is started after the combination. *
exposure to a risk factor but before the E. Naproxen.
disease started?
Which of the following drugs is LEAST
A. Case series. likely to cause gingival hyperplasia?
B. Randomized trial.
C. Case-control. A. Cyclosporine.
D. Cohort. * B. Fluoxetine. *
C. Phenytoin.
An article reports that subjects D. Nifedipine.
receiving chlorhexidine varnish had
significantly lower (p<0.05)
Streptococcus mutans levels than did

Dr Abdul Naser Tamim SEHA, Dr Ghada Al Aqqad D.D.S, Dr Kamal Naser - Amrita Medical Centre, Dr Emad Wani - Shadi Dental Centre,
Dr Lina Anka SEHA, Dr Rouba Zgheibi SEHA, Maria Teresa Yongson Alejandrino SEHA 305
Patients with primary herpetic
gingivostomatitis should NOT receive A. normally metastasizes to the
submental lymph nodes.
A. acetaminophen. B. normally metastasizes to the
B. acyclovir. submaxillary lymph nodes.
C. benzocaine. C. normally metastasizes to the
D. prednisone. * cervical lymph nodes.
E. chlorhexidine. D. normally metastasizes to the pre-
auricular lymph nodes.
Recurrent herpes labialis is E. does not normally metastasize. *

A. caused by a different organism than What type of lesions are seen in


is primary herpetic gingivostomatitis. primary herpetic gingivostomatitis,
B. a form of disease which heals by herpes simplex, herpes zoster and
scarring. varicella?
C. occurs more frequently in children.
D. a contagious lesion. * A. Macules.
B. Papules.
A simple bone cyst/traumatic bone cyst C. Vesicles. *
is a radiolucency most frequently seen D. Pustules.

A. in the mandibular ramus. Which of the following tumours has the


B. posteriorly to the maxillary molars. best prognosis?
C. from the symphysis to the ramus of
the mandible. * A. Osteosarcoma.
D. in the maxillary premolar area. B. Melanoma.
C. Ameloblastoma. *
Difficulty in mouth opening, dysphagia, D. Adenocarcinoma.
tongue stiffness and generalized
induration of the skin are characteristic Pleomorphic adenomas are most likely
of to exhibit

A. lupus erythematosus. A. rapid growth with death of patient.


B. scleroderma (systemic sclerosis). * B. early metastasis.
C. erythema multiforme. C. slow growth but tendency to local
D. lichen planus. recurrence. *
E. malignant tumour. D. early ulceration and hemorrhage.

Basal cell carcinoma on the nose

Dr Abdul Naser Tamim SEHA, Dr Ghada Al Aqqad D.D.S, Dr Kamal Naser - Amrita Medical Centre, Dr Emad Wani - Shadi Dental Centre,
Dr Lina Anka SEHA, Dr Rouba Zgheibi SEHA, Maria Teresa Yongson Alejandrino SEHA 306
A benign neoplasm of bone is called
a/an A. Basal cell carcinoma.
B. Ameloblastoma.
A. fibrous dysplasia. C. Melanoma. *
B. osteoma. D. Verrucous carcinoma.
C. torus. *
D. sarcoma. Mottled enamel is associated with
E. osteosarcoma.
A. regional odontodysplasia.
Healing of a recurrent herpes simplex B. fluorosis. *
lesion occurs within C. amelogenesis imperfecta.
D. tetracycline therapy.
A. 7-14 days without scar formation. *
B. 7-14 days with scar formation. A dentoalveolar abscess most
C. 2-4 weeks without scar formation. frequently originates from
D. 2-4 weeks with scar formation.
A. a post-extraction infection.
The most common malignant tumour B. trauma.
of the tongue is a/an C. periodontal inflammation.
D. pulpal necrosis. *
A. adenocarcinoma.
B. fibrosarcoma. Which of the following is the most
C. squamous cell carcinoma. * common site in the oral cavity for a
D. granular cell myoblastoma. squamous cell carcinoma?

A cementoblastoma is typically a A. Floor of the mouth. *


solitary circumscribed B. Buccal mucosa.
C. Palate.
A. radiopacity with a radiolucent rim D. Gingiva.
involving the apices of a mandibular
molar. Smoker's melanosis
B. radiolucency involving the apices
of the mandibular incisors. A. is painful.
C. radiopacity with a radiolucent rim B. may be pre-cancerous.
involving the apex of a mandibular C. most commonly affects the anterior
incisor. * gingiva. *
D. radiolucency in an edentulous D. resolves within a few weeks after
area. quitting smoking.

Which of the following lesions has


the LEAST favourable prognosis?

Dr Abdul Naser Tamim SEHA, Dr Ghada Al Aqqad D.D.S, Dr Kamal Naser - Amrita Medical Centre, Dr Emad Wani - Shadi Dental Centre,
Dr Lina Anka SEHA, Dr Rouba Zgheibi SEHA, Maria Teresa Yongson Alejandrino SEHA 307
Which of the following is NOT D. periapical osseous dysplasia
associated with an aphthous ulcer? (periapical cemento-osseous
dysplasia). *
A. Pain.
B. Pseudomembrane. The most common site for breast
C. Vesicle. * carcinoma to metastasize to the
D. Inflammation. maxillofacial regions is the

A 75-year old female patient is being A. anterior maxilla.


treated for oral lichen planus with a B. anterior mandible.
topical corticosteroid. She also has low C. posterior maxilla.
vitamin D. The most likely cause of a D. posterior mandible. *
reduced alveolar bone mass in this E. hard palate.
patient is
A smooth-surfaced, solid, exophytic
A. postmenopausal osteoporosis. oral mucosal lesion with no colour
B. senile osteoporosis. change is most likely a/an
C. drug-induced osteoporosis.
D. osteomalacia. * A. mucocele.
B. papilloma.
Tissue from a multilocular radiolucent C. hemangioma.
area of the posterior mandible D. fibroma. *
histologically shows follicular areas E. intramucosal nevus.
lined with columnar cells resembling the
enamel organ. The most probable Which of the following is caused by a
diagnosis is a/an microorganism?

A. neurofibroma. A. Geographic tongue.


B. ameloblastoma. * B. Lichen planus.
C. central ossifying fibroma. C. Median rhomboid glossitis. *
D. lateral periodontal cyst. D. Pyogenic granuloma.
E. dentigerous cyst.
The most common benign tumour of
Which of the following lesions is most the salivary glands is a/an
commonly found in the anterior region
of the mandible? A. Warthin’s tumour.
B. pleomorphic adenoma. *
A. Ameloblastoma. C. canalicular adenoma.
B. Calcifying epithelial odontogenic D. mucocele.
tumour.
C. Central cementifying fibroma.

Dr Abdul Naser Tamim SEHA, Dr Ghada Al Aqqad D.D.S, Dr Kamal Naser - Amrita Medical Centre, Dr Emad Wani - Shadi Dental Centre,
Dr Lina Anka SEHA, Dr Rouba Zgheibi SEHA, Maria Teresa Yongson Alejandrino SEHA 308
Which of the following has both ectodermal and The most likely diagnosis of a 1cm mobile
mesodermal tissues? mass in the parotid is

A. Ameloblastoma. A. mucoepidermoid carcinoma.


B. Cementoblastoma. B. pleomorphic adenoma. *
C. Odontoma. * C. adenoid cystic carcinoma.
D. Odontogenic myxoma. D. acinic cell carcinoma.
E. Warthin’s tumour.
A mucocele results from
Multiple congenitally missing teeth may be
A. aplasia of the duct. characteristic of
B. hyperplasia of the duct.
C. damage to the duct. * A. cleidocranial dysplasia.
D. hypersecretion. B. ectodermal dysplasia. *
C. Gardner’s syndrome.
Which of the following lesions is most D. Gorlin-Goltz’s syndrome.
commonly found in the anterior region of the
mandible? Which of the following is NOT a true cyst?

A. Ameloblastoma. A. Dentigerous cyst.


B. Calcifying epithelial odontogenic tumor. B. Simple bone cyst/traumatic bone cyst. *
C. Central cementifying fibroma. C. Radicular cyst.
D. Periapical osseous dysplasia (periapical D. Nasopalatine cyst.
cemento-osseous dysplasia). *
What is the most appropriate management for
The most common site and patient age for a erythroleukoplakia?
solid (conventional) ameloblastoma are the
posterior A. Observation.
B. Replacement of amalgam restorations.
A. mandible; 20 years. C. Change in diet.
B. mandible; 40 years. * D. Biopsy. *
C. maxilla; 20 years. E. Carbon dioxide laser ablation
D. maxilla; 40 years.

Dr Abdul Naser Tamim SEHA, Dr Ghada Al Aqqad D.D.S, Dr Kamal Naser - Amrita Medical Centre, Dr Emad Wani - Shadi Dental Centre,
Dr Lina Anka SEHA, Dr Rouba Zgheibi SEHA, Maria Teresa Yongson Alejandrino SEHA 309
A large pericoronal radiolucency
associated with an impacted third Which of the following drugs is used in
molar tooth is most likely a/an treating opioid-dependent individuals?

A. ameloblastoma. A. Codeine.
B. traumatic bone cyst. B. Methadone. *
C. eruption cyst. C. Naloxone.
D. dentigerous cyst. * D. Pentazocine.
E. calcifying odontogenic cyst. E. Meperidine.

Nicotinic stomatitis is Which of the following increases the


risk for bronchospasm in asthmatic
A. associated with squamous cell patients?
carcinoma of the palate.
B. caused by smokeless tobacco. A. Prednisone.
C. characterized by small red dots on B. Ibuprofen. *
an elevated pale mucosa. * C. Lidocaine.
D. characterized by dysplastic D. Nitrous oxide.
changes. E. Oxycodone.
E. irreversible.
Gingival enlargement observed in
Down syndrome is associated with acute leukemia is due to

A. hypertelorism. A. reactive fibrosis.


B. papular rash. B. edema.
C. mandibular prognatism. C. tissue infiltration by neoplastic cells*
D. Hutchinson’s teeth. D. capillary fragility.
E. macroglossia. *
The most appropriate management for
A 14 year old boy presents with a gingival cyst of the newborn is
bilateral white thickening of the buccal
mucosa which has been present since A. curettage.
birth. His brother has similar lesions. B. incisional biopsy.
The most likely diagnosis is C. cytologic smear.
D. excisional biopsy.
A. leukoplakia. E. observation. *
B. lichen planus.
C. mucous patches. *
D. white sponge nevus.

Dr Abdul Naser Tamim SEHA, Dr Ghada Al Aqqad D.D.S, Dr Kamal Naser - Amrita Medical Centre, Dr Emad Wani - Shadi Dental Centre,
Dr Lina Anka SEHA, Dr Rouba Zgheibi SEHA, Maria Teresa Yongson Alejandrino SEHA 310
Which of the following is the most
appropriate management for a child Which of the following entities has the
with acute primary herpetic greatest malignant potential?
gingivostomatitis?
A. Junctional nevus. *
A. Analgesic and hydration therapy* B. Oral melanotic macule.
B. Antibiotic therapy. C. Granular cell tumour.
C. Topical corticosteroid therapy. D. White sponge nevus.
D. Topical antifungal therapy.
Exfoliative cytology can be of value in
Which of the following drugs is most the diagnosis of
appropriate first line management for
a patient experiencing an acute A. lichen planus.
asthmatic attack? B. aphthous ulceration.
C. herpes simplex. *
A. Isoproterenol. D. erythema multiforme.
B. Salbutamol. * E. benign mucous membrane
C. Epinephrine. pemphigoid.
D. Hydrocortisone.
Which of the following is necessary to
Most cases of erosive oral lichen make a diagnosis of an odontogenic
planus are effectively treated with keratocyst (keratocystic odontogenic
tumour)?
A. antifungals.
B. antibacterials. A. Aspiration cytology.
C. antimalarials. B. Exfoliative cytology.
D. corticosteroids. * C. Radiographic examination alone.
D. Histopathologic examination. *
An incisional biopsy of a clinically
suspicious malignant lesion fails to
support the clinical diagnosis. The
most appropriate management is to

A. reassure the patient that there is no


malignancy.
B. contact the pathologist to discuss
the case. *
C. recall the patient in six months.
D. perform a cytological smear.

A patient has an asymptomatic,


adherent, white patch on the oral
mucosa. Which of the following is
most appropriate to confirm a
diagnosis?

A. Biopsy. *
B. Culture.
C. Exfoliative cytology.
D. Direct visual fluorescent
examination.

Dr Abdul Naser Tamim SEHA, Dr Ghada Al Aqqad D.D.S, Dr Kamal Naser - Amrita Medical Centre, Dr Emad Wani - Shadi Dental Centre,
Dr Lina Anka SEHA, Dr Rouba Zgheibi SEHA, Maria Teresa Yongson Alejandrino SEHA 311
The most common etiology of an The mechanism of adjustment to
Angle’s Class I malocclusion is maintain the shape and proportions of
bone throughout its growth period is
A. unusual dental arch development. called
B. tooth size - arch size discrepancy.*
C. congenitally missing teeth. A. remodeling. *
D. extra teeth. B. cortical drift.
C. area relocation.
Thumbsucking does NOT usually D. translatory growth.
affect the permanent dentition if the
habit is During orthodontic therapy, the width
of the periodontal ligament
A. discontinued before four years of radiographically appears
age. *
B. discontinued before eight years of A. increased. *
age. B. decreased.
C. associated with sleeping. C. unchanged.
D. is of low intensity.
The severity of an Angle’s Class II
In the mandibular dental arch of a 12- malocclusion may be reduced by
year old boy, the permanent first
molars are in contact with the first 1. Maintaining the integrity of the
premolars and the crowns of the primary dentition.
second premolars have erupted 2. Preventing thumbsucking and lip
lingually. The likely cause is biting habits.
3. Correcting mouth breathing as early
A. ankylosis of the mandibular second as possible.
premolars. 4. The strategic removal of primary
B. lack of space. teeth.
C. teeth too large for the dental arch.
D. premature loss of deciduous A. (1) (2) (3) *
second molars. * B. (1) and (3)
E. faulty lingual eruption of the second C. (2) and (4)
premolars. D. (4) only
E. All of the above.
The most frequent cause of
malocclusion is Bone laid down by the periosteum is

A. thumbsucking. A. endochondral.
B. mouth breathing. B. cartilaginous.
C. heredity. * C. appositional. *
D. ectopic eruption. D. cancellous.

Dr Abdul Naser Tamim SEHA, Dr Ghada Al Aqqad D.D.S, Dr Kamal Naser - Amrita Medical Centre, Dr Emad Wani - Shadi Dental Centre,
Dr Lina Anka SEHA, Dr Rouba Zgheibi SEHA, Maria Teresa Yongson Alejandrino SEHA 312
A 3 year old requires the extraction
of a deciduous maxillary second A. A stainless steel crown placed at
molar. The local anesthetic the same appointment. *
technique of choice is B. A stainless steel crown placed when
a radiograph demonstrates no internal
A. a posterior superior alveolar block. resorption.
B. buccal and palatal infiltration. * C. An amalgam placed at the same
C. a tuberosity block plus appointment.
subperiosteal infiltration of the D. An amalgam placed when a
mesio-buccal root. radiograph indicates no bone
D. an infra-orbital block. destruction between the roots.

An exchange of calcium ions "Dental age" is defined as the


between saliva and enamel is
A. stage of dental maturation. *
1. Affected by fluoride. B. eruption time of a given tooth.
2. A component of remineralization C. number of years elapsed since a
and demineralization. given tooth has erupted.
3. important in maintenance of tooth
structure. A single hypoplastic defect located on
4. PH dependent. the labial surface of a maxillary central
incisor is most likely due to a/an
A. (1) (2) (3)
B. (1) and (3) A. dietary deficiency.
C. (2) and (4) B. endocrine deficiency.
D. (4) only C. tetracycline therapy.
E. All of the above. * D. trauma to the maxillary primary
central incisor. *
A crossbite of a maxillary central E. high fluoride intake.
incisor is most appropriately treated
A 12 year old patient with a complete
A. following eruption of the maxillary dentition has enamel hypoplasia of
permanent canines. tooth 1.4. This condition was most
B. following eruption of the maxillary likely caused by
permanent central incisors.
C. following eruption of the maxillary A. low calcium intake.
permanent lateral incisors. B. high fluoride intake.
D. during the eruption of the C. an abscessed tooth 5.4. *
maxillary permanent central incisors* D. systemic tetracycline.
E. a febrile illness.
Which of the following permanent
restorations is the most appropriate
after a formocresol pulpotomy has
been completed on a primary molar?

Dr Abdul Naser Tamim SEHA, Dr Ghada Al Aqqad D.D.S, Dr Kamal Naser - Amrita Medical Centre, Dr Emad Wani - Shadi Dental Centre,
Dr Lina Anka SEHA, Dr Rouba Zgheibi SEHA, Maria Teresa Yongson Alejandrino SEHA 313
Which of the following conditions may A. develop into a Class I occlusion after
develop as a result of juvenile diabetes normal exfoliation of the primary
mellitus? molars. *
B. worsen with forward growth of the
A. Ataxia. maxilla.
B. Aphasia. C. develop into a Class I occlusion with
C. Deafness. late mandibular growth.
D. Blindness. * D. develop into a skeletal malocclusion
E. Motor paralysis. with growth of the maxilla and
mandible.
Alveolar bone is undergoing E. not change as the maxilla and
remodeling mandible grow.

A. through the primary dentition. The roots of primary molars in the


B. until the end of mixed dentition. absence of their permanent successors
C. until the complete eruption of
permanent teeth. 1. Sometimes are partially resorbed
D. throughout life. * and become ankylosed.
2. May remain for years with no
The last primary tooth to be replaced significant resorption.
by a permanent tooth is usually the 3. May remain for years partially
resorbed.
A. maxillary canine. * 4. Are always resorbed.
B. mandibular canine.
C. maxillary first molar. A. (1) (2) (3) *
D. mandibular second molar. B. (1) and (3)
C. (2) and (4)
Exclusive of third molars, the D. (4) only
permanent dentition is usually E. All of the above.
completely erupted by the age of
The palate grows in length by
A. 9 to 11 years.
B. 12 to 14 years. * A. endochondral growth.
C. 15 to 17 years. B. apposition on the free edge of the
D. 18 to 21 years. palatine bone. *
C. the downward and forward growth of
A Class II dental occlusion in the mixed the nasal septum.
dentition will likely

Dr Abdul Naser Tamim SEHA, Dr Ghada Al Aqqad D.D.S, Dr Kamal Naser - Amrita Medical Centre, Dr Emad Wani - Shadi Dental Centre,
Dr Lina Anka SEHA, Dr Rouba Zgheibi SEHA, Maria Teresa Yongson Alejandrino SEHA 314
The body of the mandible increases in A. adequacy of mesio-distal space. *
length to accommodate the permanent B. developmental age of the patient.
second molar by C. sequence of eruption of the
permanent dentition.
A. apposition of bone in the condyle. D. morphology of the anterior teeth.
B. resorption of bone along the anterior
border of the ramus and apposition of Correction of an Angle Class II, division
bone on the posterior border of the 1 malocclusion has the best prognosis
ramus. * when the
C. apposition of bone on the alveolar
margin and lower border of the body of 1. Maxillary incisors are tipping labially.
the mandible. 2. Skeletal bases are harmonious.
D. apposition of bone at the symphysis 3. Dentition is mildly crowded or
and posterior border of the ramus of the spaces.
mandible. 4. Lower face height is long.
E. anterior growth.
A. (1) (2) (3) *
Mandibular condylar region grows by B. (1) and (3)
C. (2) and (4)
A. sutural and interstitial proliferation. D. (4) only
B. interstitial and appositional E. All of the above.
proliferation. *
C. appositional and sutural proliferation. Angle's classification of occlusion is
D. interstitial proliferation only. based on
E. appositional proliferation only.
A. a full complement of teeth.
Loops and helices in orthodontic arches B. antero-posterior skeletal relationship
result in of maxilla to mandible.
C. antero-posterior relationship of
1. A decreased level of force maxillary and mandibular first
application. permanent molars. *
2. A greater range of activation. D. vertical relationships in the lower
3. improved tissue response. face.
4. Easier insertion.
In the mixed dentition, an end-to-end
A. (1) (2) (3) * first permanent molar relationship is
B. (1) and (3) indicative of
C. (2) and (4)
D. (4) only A. normally developing occlusion. *
E. All of the above. B. Angle Class II malocclusion.
C. Angle Class III malocclusion.
Prior to the correction of a one tooth D. ideal molar occlusion.
anterior crossbite, the principle factor to
consider is the

Dr Abdul Naser Tamim SEHA, Dr Ghada Al Aqqad D.D.S, Dr Kamal Naser - Amrita Medical Centre, Dr Emad Wani - Shadi Dental Centre,
Dr Lina Anka SEHA, Dr Rouba Zgheibi SEHA, Maria Teresa Yongson Alejandrino SEHA 315
Space closure is LEAST likely to occur An overjet of 8mm is most often
following the loss of the deciduous associated with

A. mandibular canine. A. Class I cuspid relationship.


B. maxillary first molar. B. Class II cuspid relationship. *
C. mandibular second molar. C. Class III cuspid relationship.
D. maxillary central incisor. *
Hypothyroidism affects the dental
Inadequate space for the eruption of developmental pattern by
the maxillary second premolar is most
frequently caused by the premature A. interfering with jaw growth.
loss of the deciduous B. delaying the eruption timetable. *
C. causing sclerotic bone to form
A. First molar. over the occlusal surface of erupting
B. Second molar. * teeth.
C. central incisor. D. accelerating the eruption
D. canine. timetable.

The most significant factor in the Mandibular growth


predictable correction of an anterior
crossbite is the A. is sustained over a longer period
of time in girls.
A. patient age. B. is sustained over a longer period
B. overbite. of time in boys. *
C. tooth shape. C. occurs at the same chronologic
D. mesio-distal spacing. * age in both sexes.
D. occurs two years earlier in boys
A single tooth anterior crossbite found than in girls.
in a 9 year old should
The organisms associated with a
A. self-correct. carious pulpitis are
B. be treated with a removable
appliance. * A. streptococci. *
C. have 2 arch orthodontic treatment. B. staphylococci.
D. be treated in the complete C. spirochetes.
permanent dentition. D. viruses.
E. be observed and treated when the
cuspids have erupted.

Dr Abdul Naser Tamim SEHA, Dr Ghada Al Aqqad D.D.S, Dr Kamal Naser - Amrita Medical Centre, Dr Emad Wani - Shadi Dental Centre,
Dr Lina Anka SEHA, Dr Rouba Zgheibi SEHA, Maria Teresa Yongson Alejandrino SEHA 316
If a child's teeth do NOT form, the B. 10 years. *
primary effect will be on the growth of C. 12 years.
the D. Later than 12 years.

A. alveolar bone. * The radiographic appearance of


B. mandible. internal resorption is
C. maxilla.
D. palate. A. radiolucent enlargement of the pulp
cavity. *
The primate spaces are located B. radiolucency around the apex of the
between the root.
C. radiolucency on the surfaces of the
1. Maxillary canines and lateral root.
incisors. D. localized radiopacities in the pulp
2. Maxillary canines and first molars. cavity.
3. Mandibular canines and first molars. E. radiopacity around the apex of the
4. Mandibular canines and lateral root.
incisors.
Which of the following patients should
A. (1) (2) (3) be referred to orthodontically close a
B. (1) and (3) * maxillary midline diastema?
C. (2) and (4)
D. (4) only 1. An 8 year old with no abnormal oral
E. All of the above. habits.
2. A 14 year old with no abnormal oral
The developing permanent tooth habits.
3. A 3 year old with a 4mm overjet.
1. Lies apically and lingually to primary 4. An 8 year old with a previous thumb
teeth in the anterior region. habit.
2. May show deviated eruption times if
the primary tooth is lost prematurely. A. (1) (2) (3)
3. Has a more protrusive path of B. (1) and (3)
eruption in the anterior region. C. (2) and (4) *
4. Usually erupts earlier in girls. D. (4) only
E. All of the above.
A. (1) (2) (3)
B. (1) and (3) In cephalometry, the most stable point
C. (2) and (4) in a growing skull is the
D. (4) only
E. All of the above. * A. sella turcica. *
B. nasion.
Roots of the permanent maxillary C. Broadbent's point.
central incisors are completed by what D. Bolton point.
age?

A. 8 years.

Dr Abdul Naser Tamim SEHA, Dr Ghada Al Aqqad D.D.S, Dr Kamal Naser - Amrita Medical Centre, Dr Emad Wani - Shadi Dental Centre,
Dr Lina Anka SEHA, Dr Rouba Zgheibi SEHA, Maria Teresa Yongson Alejandrino SEHA 317
Which of the following is correct with E. MTA apexification.
respect to the hand-wrist radiograph?
In primary teeth, a pulpotomy using
A. Skeletal age may be estimated by calcium hydroxide
comparing the image to a standard.
B. It provides a precise measure of A. will cause an acute inflammatory
skeletal development. reaction.
C. It is of minimal value in orthodontic B. is successful treatment in 90 percent
diagnosis. * of cases.
D. It is only determinate of skeletal C. will cause internal resorption.*
age. D. is the treatment of choice for small
mechanical exposures.
A radiographic examination of a 10 E. will stimulate apical closure.
year old child reveals retention of
deciduous teeth and presence of many The eruption of a permanent central
unerupted supernumerary teeth. This incisor may be delayed by
is characteristic of
1. A supernumerary tooth.
A. cleidocranial dysplasia. * 2. Dense fibrous tissue.
B. ectodermal dysplasia. 3. A retained deciduous incisor.
C. dentinogenesis imperfecta. 4. Early loss of a deciduous incisor.
D. congenital hypothyroidism.
A. (1) (2) (3)
The highest incidence of congenitally B. (1) and (3)
missing lateral incisors is most likely C. (2) and (4)
seen in a patient with D. (4) only
E. All of the above. *
A. unilateral cleft lip and palate. *
B. congenital heart disease. An ankylosed deciduous molar can
C. Down's syndrome. cause
D. hyperthyroidism.
1. Delayed eruption of the underlying
A 7 year old child presents with a 3mm premolar.
coronal fracture with pulp exposure of 2. Reduction of arch length.
tooth 2.1 following a trauma 24 hours 3. Difficulty with extraction.
ago. The tooth is sensitive to hot and 4. Reduction in alveolar bone.
cold fluids. The most appropriate
management is a A. (1) (2) (3)
B. (1) and (3)
A. MTA direct pulp capping. C. (2) and (4)
B. MTA pulpotomy. * D. (4) only
C. gutta-percha pulpectomy. E. All of the above. *
D. gutta-percha pulpectomy followed
by an apical surgery.

Dr Abdul Naser Tamim SEHA, Dr Ghada Al Aqqad D.D.S, Dr Kamal Naser - Amrita Medical Centre, Dr Emad Wani - Shadi Dental Centre,
Dr Lina Anka SEHA, Dr Rouba Zgheibi SEHA, Maria Teresa Yongson Alejandrino SEHA 318
In primary molars, radiographic bony changes
from an infection are initially seen A. 2.
B. 4.
A. at the apices. C. 6.
B. in the furcation area. * D. 8. *
C. at the alveolar crest. E. 10.
D. at the base of the developing tooth.
Which of the following drugs potentiates the
Which of the following is most likely related to action of sedative drugs?
the administration of excessive
vasoconstrictor? A. Digitalis.
B. Phenothiazine. *
A. Flushing of the skin and tremors. C. Propranolol.
B. Sudden pallor, sweating, weak but regular D. Methyldopa.
pulse, occasional loss of consciousness. E. Spironolactone.
C. Convulsions and loss of consciousness.
D. Tachycardia, palpitations, headache, When sutures are used to reposition tissue
cardiac arrhythmia, elevated blood pressure. * over extraction sites, they should be

After an inferior alveolar nerve block injection, 1. Placed over firm bone where possible.
a patient would develop seventh nerve 2. interrupted, 15mm apart.
paralysis if the injection was made into the 3. Firm enough to approximate tissue flaps
without blanching.
A. internal maxillary artery. 4. tight enough to produce immediate
B. retroparotid space. * hemostasis.
C. internal pterygoid muscle.
D. retromandibular vein. A. (1) (2) (3)
E. pterygoid plexus of veins. B. (1) and (3) *
C. (2) and (4)
A patient has a proven allergy to para-amino D. (4) only
benzoic acid derivatives. Which local E. All of the above.
anesthetic solution can be used safely?
When a patient has a history of porphyrism,
A. Procaine. which of the following drugs is
B. Butethamine hydrochloride. CONTRAINDICATED for pharmaco-sedation?
C. Tetracaine.
D. Lidocaine. * A. Minor tranquilizers.
B. Antihistamines.
What is the maximum number of cartridges C. Barbiturates. *
(1.8ml) of a 2% local anesthetic solution that D. Opioids
can be administered without exceeding a total
dose of 300mg?

Dr Abdul Naser Tamim SEHA, Dr Ghada Al Aqqad D.D.S, Dr Kamal Naser - Amrita Medical Centre, Dr Emad Wani - Shadi Dental Centre,
Dr Lina Anka SEHA, Dr Rouba Zgheibi SEHA, Maria Teresa Yongson Alejandrino SEHA 319
In primary molars, radiographic bony administered without exceeding a total
changes from an infection are initially dose of 300mg?
seen
A. 2.
A. at the apices. B. 4.
B. in the furcation area. * C. 6.
C. at the alveolar crest. D. 8. *
D. at the base of the developing tooth. E. 10.

Which of the following is most likely Which of the following drugs


related to the administration of potentiates the action of sedative
excessive vasoconstrictor? drugs?

A. Flushing of the skin and tremors. A. Digitalis.


B. Sudden pallor, sweating, weak but B. Phenothiazine. *
regular pulse, occasional loss of C. Propranolol.
consciousness. D. Methyldopa.
C. Convulsions and loss of E. Spironolactone.
consciousness.
D. Tachycardia, palpitations, When sutures are used to reposition
headache, cardiac arrhythmia, elevated tissue over extraction sites, they
blood pressure. * should be

After an inferior alveolar nerve block 1. Placed over firm bone where
injection, a patient would develop possible.
seventh nerve paralysis if the injection 2. interrupted, 15mm apart.
was made into the 3. Firm enough to approximate tissue
flaps without blanching.
A. internal maxillary artery. 4. tight enough to produce immediate
B. retroparotid space. * hemostasis.
C. internal pterygoid muscle.
D. retromandibular vein. A. (1) (2) (3)
E. pterygoid plexus of veins. B. (1) and (3) *
C. (2) and (4)
A patient has a proven allergy to para- D. (4) only
amino benzoic acid derivatives. Which E. All of the above.
local anesthetic solution can be used
safely? When a patient has a history of
porphyrism, which of the following
A. Procaine. drugs is CONTRAINDICATED for
B. Butethamine hydrochloride. pharmaco-sedation?
C. Tetracaine.
D. Lidocaine. * A. Minor tranquilizers.
B. Antihistamines.
What is the maximum number of C. Barbiturates. *
cartridges (1.8ml) of a 2 local D. Opioids.
anesthetic solution that can be

Dr Abdul Naser Tamim SEHA, Dr Ghada Al Aqqad D.D.S, Dr Kamal Naser - Amrita Medical Centre, Dr Emad Wani - Shadi Dental Centre,
Dr Lina Anka SEHA, Dr Rouba Zgheibi SEHA, Maria Teresa Yongson Alejandrino SEHA 320
An acute periapical abscess unresponsive to verbal stimuli, and his
originating from a mandibular third respirations are depressed to 10 per
molar generally points and drains in minute. Appropriate treatment is to
the
A. administer ephedrine.
A. submandibular space. * B. observe the patient.
B. pterygomandibular space. C. force the patient to drink coffee.
C. buccal vestibule. D. support respiration with oxygen. *
D. buccal space.
In a standard inferior alveolar nerve
Early anoxia is characterized by block, which muscle is penetrated by
the needle?
1. Cyanosis.
2. Bradycardia. A. Buccinator. *
3. Tachycardia B. Mylohyoid.
C. Superior constrictor.
A. (1) only D. Masseter.
B. (1) and (2) E. Medial (internal) pterygoid.
C. (1) and (3) *
D. All of the above. Vestibuloplasty is a preprosthetic
surgical procedure used to
The chief mechanism by which the
body metabolizes short-acting A. facilitate reliable impression making.
barbiturates is B. provide adequate posterior inter-
arch space.
A. oxidation. C. allow placement of teeth over the
B. reduction. residual ridge.
C. hydroxylation and oxidation. * D. increase the supporting surface
D. sequestration in the body fats. area. *

The most common complication In a standard dental cartridge (carpule)


associated with the use of local containing 1.8ml 2% lidocaine with
anesthetics is epinephrine 1/100,000, the amount of
vasoconstrictor is
A. syncope. *
B. trismus. A. 18.0 mg.
C. a toxic reaction. B. 0.018 mg. *
D. an allergic reaction. C. 1.8 mg.
E. an anaphylactic reaction. D. 0.18 mg.
E. 180.0 mg.
A 57 year old man received 10mg of
diazepam intravenously. He becomes

Dr Abdul Naser Tamim SEHA, Dr Ghada Al Aqqad D.D.S, Dr Kamal Naser - Amrita Medical Centre, Dr Emad Wani - Shadi Dental Centre,
Dr Lina Anka SEHA, Dr Rouba Zgheibi SEHA, Maria Teresa Yongson Alejandrino SEHA 321
The inorganic ion that is implicated in
primary hypertension is All of the following are possible
effects of acetylsalicylic acid except
A. sodium. *
B. fluoride. A. reduction of fever.
C. potassium. B. shortening of bleeding time. *
D. magnesium. C. suppression of inflammatory
response.
The psychomotor recovery time from D. bleeding from the gastrointestinal
nitrous oxide sedation is tract.

A. immediate. Short-acting barbiturates are


B. 1 to 2 minutes. metabolized mainly in the
C. 10 minutes.
D. 30 minutes. * A. liver. *
B. kidneys.
Unconsciousness in syncope results C. small intestine.
from D. pancreas.
E. spleen.
A. electrolyte imbalance.
B. neurogenic shock. Acetaminophen in therapeutic doses
C. cerebral hyperemia.
D. cerebral hypoxia. * 1. Retards platelet function.
2. Has strong anti-inflammatory
Which of the following would you properties.
prescribe for an anxious dental patient 3. Produces CNS stimulation.
with a peptic ulcer? 4. Has antipyretic properties.

A. Reserpine. A. (1) (2) (3)


B. Scopolamine. B. (1) and (3)
C. Silica gel. C. (2) and (4)
D. Diazepam. * D. (4) only *
E. Calcium carbonate. E. All of the above.

Which of the following does NOT Trismus is most frequently caused by


influence the rate of induction during
inhalation anesthesia? A. tetanus.
B. muscular dystrophy.
A. Pulmonary ventilation. C. infection. *
B. Blood supply to the lungs. D. mandibular fracture.
C. Hemoglobin content of the blood. *
D. Concentration of the anesthetic in
the inspired mixture.
E. Solubility of the anesthetic in blood.

Dr Abdul Naser Tamim SEHA, Dr Ghada Al Aqqad D.D.S, Dr Kamal Naser - Amrita Medical Centre, Dr Emad Wani - Shadi Dental Centre,
Dr Lina Anka SEHA, Dr Rouba Zgheibi SEHA, Maria Teresa Yongson Alejandrino SEHA 322
A therapeutic advantage of penicillin A. increasing the action of histaminase.
V over penicillin G is B. altering the formation of histamine.
C. blocking the actions of histamine by
A. greater resistance to penicillinase. competitive inhibition. *
B. broader antibacterial spectrum. D. interfering with the degradation of
C. greater absorption when given histamine.
orally. *
D. slower renal excretion. The most common complication of a
E. None of the above. venipuncture is

Which of the following is NOT a sign A. syncope.


of congenital heart disease? B. hematoma. *
C. thrombophlebitis.
A. Cyanosis. D. embolus.
B. Telangiectasia. *
C. Clubbing of the fingers and toes. The major stimulator of respiration is
D. Heart murmur.
E. Right ventricular hypertrophy. A. low blood pressure.
B. high percentage of blood oxygen.
Which valve is most commonly C. low percentage of blood carbon
affected by rheumatic heart dioxide.
disease? D. high percentage of blood carbon
dioxide. *
A. Aortic.
B. Pulmonary. Which of the following drugs are classic
C. Tricuspid. antagonists for curare over-dosage?
D. Mitral. *
A. Anticholinesterases. *
Tetracyclines B. Ganglionic stimulants.
C. Ganglionic blocking agents.
1. Have no side effects. D. Alpha adrenergic blocking agents.
2. May increase susceptibility to E. Beta adrenergic blocking agents.
superinfections.
3. are safe to use during pregnancy. In general anaesthesia, the last part of
4. Have a wide spectrum of the CNS (Central Nervous System) to
antibacterial activity. be depressed is the

A. (1) (2) (3) A. medulla. *


B. (1) and (3) B. oblongata.
C. (2) and (4) * C. midbrain.
D. (4) only D. cerebellum.
E. All of the above. E. spinal cord.

Antihistamines act by

Dr Abdul Naser Tamim SEHA, Dr Ghada Al Aqqad D.D.S, Dr Kamal Naser - Amrita Medical Centre, Dr Emad Wani - Shadi Dental Centre,
Dr Lina Anka SEHA, Dr Rouba Zgheibi SEHA, Maria Teresa Yongson Alejandrino SEHA 323
It is difficult to obtain satisfactory fever and bilateral cervical
anesthesia in the presence of infection lymphadenopathy. A blood examination
near the injection site because reveals
Hb: 8.9g/100ml
A. the swelling causes increased Platelets: 82,000/mm3
pressure on the nerves. Red blood cell count: 3,900,000/mm3
B. increased blood supply carries the White blood cell count: 870,000/mm3
anesthetic solution away too fast. Normal Values:
C. acidity of the infected tissue inhibits Hb: 14-18g/100ml
action of the anesthetic agent. * Platelets: 150,000-400,000/mm3
D. alkalinity of the infected tissue Red blood cell count: 4-5million/mm3
inhibits action of the anesthetic agent. White blood cell count: 5,000-
10,000/mm3
If an odontogenic infection involves the The most likely diagnosis is
pterygomandibular space, the most
obvious clinical sign will be A. thrombocytopenic purpura.
B. acute myelogenous leukemia. *
A. trismus. * C. infectious mononucleosis.
B. facial swelling. D. acute necrotizing ulcerative gingivitis.
C. swelling in the submandibular area.
D. rise in body temperature above Healing of extraction wounds is
39 C (102ºF). sometimes complicated by a condition
known as a "dry socket". In this
Ludwig's angina may cause death by condition

A. heart failure. A. no fibrin is formed in the extraction


B. asphyxia. * socket.
C. convulsions. B. routine use of antibiotics is advised
D. paralysis of muscles of respiration. as a prophylactic measure.
E. pyemia. C. a focal osteomyelitis exists in which
the clot has disintegrated. *
A 20-year old male presents with a D. healing is painful but not delayed.
three-day history of an acute
generalized gingivitis. He has malaise,

Dr Abdul Naser Tamim SEHA, Dr Ghada Al Aqqad D.D.S, Dr Kamal Naser - Amrita Medical Centre, Dr Emad Wani - Shadi Dental Centre,
Dr Lina Anka SEHA, Dr Rouba Zgheibi SEHA, Maria Teresa Yongson Alejandrino SEHA 324
Bacterial infection may be confirmed The most likely complication
by associated with the extraction of an
isolated maxillary second molar is
1. White blood cell count.
2. Hemoglobin level. A. a dry socket.
3. Erythrocyte sedimentation rate. B. nerve damage.
4. Platelet count. C. fracture of the malar ridge.
D. fracture of the tuberosity. *
A. (1) (2) (3)
B. (1) and (3) * A periapical infection of a mandibular
C. (2) and (4) third molar may spread by direct
D. (4) only extension to the
E. All of the above.
1. Parapharyngeal space.
Reduced mobility of the 2. Submandibular space.
temporomandibular joint is called 3. Pterygomandibular space.
4. Submental space.
A. Charcot's arthritis.
B. osteoarthritis. A. (1) (2) (3) *
C. ankylosis. * B. (1) and (3)
D. arthrosis. C. (2) and (4)
D. (4) only
A patient presenting with diplopia, E. All of the above.
exophthalmos, nasal bleeding and
swelling, may suffer from a fracture of A patient suddenly becomes pale and
the sweaty after an injection of 4ml of
lidocaine 2% with epinephrine
A. neck of the condyle. l:l00,000. The radial pulse is slow and
B. body of the mandible. steady. The respiration is slow. The
C. zygomatic bone. * blood pressure is 80/60. What is the
D. maxillary tuberosity. most probable diagnosis?

Chlorpromazine produces all of the A. A toxic reaction to lidocaine.


following EXCEPT B. A toxic reaction to epinephrine.
C. An allergic reaction to the local
A. photosensitivity. anesthetic.
B. excessive salivation. * D. Incipient syncope. *
C. anticholinergic effects. E. An impending adrenal insufficiency.
D. antiadrenergic effects.

Dr Abdul Naser Tamim SEHA, Dr Ghada Al Aqqad D.D.S, Dr Kamal Naser - Amrita Medical Centre, Dr Emad Wani - Shadi Dental Centre,
Dr Lina Anka SEHA, Dr Rouba Zgheibi SEHA, Maria Teresa Yongson Alejandrino SEHA 325
The characteristic pain of trigeminal 2. The electric pulp tester.
neuralgia is 3. Percussion and palpation.
4. History and subjective symptoms.
A. dull and prolonged. 5. Thermal stimulus.
B. infrequent, sharp and prolonged.
C. regularly recurrent, dull and A. (1) and (2)
persistent. B. (1) (2) (3)
D. annoying but controlled with C. (2) (3) (4) (5)
salicylates. D. (1) (3) (4) *
E. sharp, stabbing and excruciating. * E. All of the above.

Blue sclera is characteristic of The term used to describe epithelial


changes including nuclear
A. osteopetrosis. hyperchromatism, loss of increased
B. osteogenesis imperfecta. * nuclear to cytoplasmic ratio and
C. osteitis deformans. abnormal mitoses is
D. fibrous dysplasia.
A. acanthosis.
Which one of the following would be of B. hyperkeratosis.
greatest value in determining the C. dysplasia. *
etiology of an oral ulceration? D. parakeratosis.
E. hyperparakeratosis.
A. History of the oral lesion. *
B. Cytological smear. A periradicular granuloma can be
C. Systemic evaluation. differentiated from a periradicular cyst
D. Laboratory tests. by the

A 2cm, discrete, white lesion of the A. radiographic appearance.


buccal mucosa has not resolved after B. patient’s symptoms.
elimination of all local irritants. The C. response to percussion testing.
most appropriate management is to D. results of the biopsy. *

A. cauterize it. Which of the following is/are NOT


B. apply toluidine blue staining. usually affected by hereditary
C. perform an incisional biopsy. * ectodermal dysplasia?
D. re-examine at 6 month intervals.
E. refer patient to family physician. A. Salivary glands. *
B. Teeth.
Which of the following would you use C. Sweat glands.
to determine the status of the pulp of a D. Hair.
tooth with a porcelain jacket crown? E. Fingernails.

1. Radiographic appearance.

Dr Abdul Naser Tamim SEHA, Dr Ghada Al Aqqad D.D.S, Dr Kamal Naser - Amrita Medical Centre, Dr Emad Wani - Shadi Dental Centre,
Dr Lina Anka SEHA, Dr Rouba Zgheibi SEHA, Maria Teresa Yongson Alejandrino SEHA 326
Radiographically, the nasopalatine E. plasma cell.
foramen may be mistaken for
The absence of lamina dura on a dental
1. An incisive canal cyst. radiograph is suggestive of
2. A simple bone cyst/traumatic bone
cyst. A. hyperparathyroidism.
3. A radicular cyst. B. Paget's disease.
4. A naso-alveolar cyst. C. hyperthyroidism. *
D. vitamin D deficiency.
A. (1) (2) (3) E. acromegaly.
B. (1) and (3) *
C. (2) and (4) Intermittent painful swelling in the
D. (4) only submandibular region that increases at
E. All of the above. mealtime is indicative of

Inclusion bodies in the nucleus or A. a ranula.


cytoplasm of cells are diagnostic of B. a blockage of Wharton's duct. *
C. Ludwig's angina.
A. rickettsia. D. a blockage of Stensen's duct.
B. parasitic infestations. E. an epidemic parotitis.
C. bacterial diseases.
D. viral diseases. * The cells responsible for antibody
production are called
A syphilitic gumma is most commonly
found on the A. polymorphonuclear leukocytes.
B. mast cells.
A. lip. C. plasma cells. *
B. tongue. D. macrophages.
C. palate. * E. megakaryocytes.
D. buccal mucosa.
E. gingiva. In radiography, minimum magnification
and maximum definition are achieved by
A patient with a white blood cell count
of 34,000/mm3 has a differential of A. minimum OFD (object-film distance)
lymphocytes 62%, lymphoblasts 4% and minimum FFD (focal-film distance).
and polymorphonuclear leukocytes B. minimum OFD (object-film distance)
34%. The most likely form of leukemia and maximum FFD (focal-film distance)*
is C. maximum OFD (object -film distance)
and maximum FFD (focal-film distance).
A. aleukemic. D. maximum OFD (object-film distance)
B. granulocytic. and minimum FFD (focal-film distance).
C. monocytic.
D. lymphocytic. *

Dr Abdul Naser Tamim SEHA, Dr Ghada Al Aqqad D.D.S, Dr Kamal Naser - Amrita Medical Centre, Dr Emad Wani - Shadi Dental Centre,
Dr Lina Anka SEHA, Dr Rouba Zgheibi SEHA, Maria Teresa Yongson Alejandrino SEHA 327
The earliest radiographic sign of A. lymphatics drain superiorly in this
traumatic occlusion is region.
B. bone is less porous superior to the
A. hypercementosis. root apex.
B. root resorption. C. infection has passed into the angular
C. alteration of the lamina dura. vein which has no valves.
D. widening of the periodontal ligament D. the root apex lies superior to the
space. * attachment of the caninus and levator
E. ankylosis. labii superioris muscles. *

A clinical finding common to Compared to the bisecting angle


alcoholism, poorly-controlled diabetes technique, the paralleling technique will
mellitus, uremia and liver disease is result in a

A. smooth tongue. 1. Reduced dose to the thyroid gland.


B. increased blood pressure. 2. increased magnification of the image.
C. coated tongue. 3. reduced distortion of the image.
D. labial fissures. 4. increased penumbra of the image.
E. halitosis. *
A. (1) (2) (3)
Excessive formation of scar tissue B. (1) and (3) *
beyond the wound margin is called C. (2) and (4)
D. (4) only
A. a fibroma. E. All of the above.
B. a keloid. *
C. a fibro-epithelial polyp. Pernicious anemia may cause
D. epithelial hyperplasia.
A. glossitis. *
The finding of “acid-fast” B. lingual ulceration(s).
microorganisms in sputum suggests C. parotid swelling.
the presence of D. gastric hyperacidity.

A. Mycobacterium tuberculosis. * Premature eruption of primary teeth is


B. Diplococcus pneumoniae. associated with
C. Streptococcus pyogenes.
D. Neisseria gonorrhoeae. A. congenital gingival fibromatosis.
B. congenital hypothyroidism.
The most logical explanation for C. osteopetrosis.
causing swelling beneath the eye D. cleidocranial dysplasia.
caused by an abscessed maxillary E. juvenile hyperthyroidism. *
canine is that the

Dr Abdul Naser Tamim SEHA, Dr Ghada Al Aqqad D.D.S, Dr Kamal Naser - Amrita Medical Centre, Dr Emad Wani - Shadi Dental Centre,
Dr Lina Anka SEHA, Dr Rouba Zgheibi SEHA, Maria Teresa Yongson Alejandrino SEHA 328
Which of the following is NOT a sign B. cretinism.
or symptom of the myofascial pain C. hypothyroidism.
dysfunction syndrome? D. cleidocranial dysplasia. *
E. Down's syndrome.
A. Pain.
B. Muscle tenderness. Mucoceles are most commonly found
C. Limitation of jaw motion. in the
D. "Clicking" or "popping" noise in
the joints. A. upper lip.
E. Radiographic changes of the joint* B. lower lip. *
C. tongue.
A radicular cyst D. buccal mucosa.
E. soft palate.
A. enlarges rapidly.
B. infiltrates bone. Which of the following can result in
C. contains fluid. * post-developmental jaw growth?
D. cannot cause cortical bone
expansion. A. Hyperparathyroidism.
E. is associated with a vital tooth. B. Hyperthyroidism.
C. Adult hypothyroidism.
Root resorption may be associated D. Osteitis deformans (Paget's). *
with E. Hypoparathyroidism.

1. Excessive orthodontic forces. Which gingival manifestation(s) would


2. Periapical granuloma. be expected in a patient with a blood
3. Traumatic injury. dyscrasia?
4. Periapical osseous dysplasia
(periapical cemento-osseous 1. Enlargement.
dysplasia). 2. Bleeding.
3. Ulceration.
A. (1) (2) (3) * 4. Atrophy.
B. (1) and (3)
C. (2) and (4) A. (1) (2) (3) *
D. (4) only B. (1) and (3)
E. All of the above. C. (2) and (4)
D. (4) only
Multiple supernumerary teeth are E. All of the above.
most commonly found in

A. cherubism.

Dr Abdul Naser Tamim SEHA, Dr Ghada Al Aqqad D.D.S, Dr Kamal Naser - Amrita Medical Centre, Dr Emad Wani - Shadi Dental Centre,
Dr Lina Anka SEHA, Dr Rouba Zgheibi SEHA, Maria Teresa Yongson Alejandrino SEHA 329
Which of the following bone lesions of C. lichen planus.
the mandible is/are malignant? D. aphthous stomatitis.

1. Osteosarcoma. A patient presents with apparent


2. Osteochondroma. paralysis of one side of the face which
3. Ewing's tumor. appeared the day before. What is the
4. Fibrous dysplasia. most likely diagnosis?

A. (1) (2) (3) A. Glossodynia.


B. (1) and (3) * B. Bell's palsy.
C. (2) and (4) C. Myasthenia gravis. *
D. (4) only D. Trigeminal neuralgia.
E. All of the above.
Which of the following cysts is most
An ameloblastoma is most frequently likely to undergo transformation into an
found in ameloblastoma?

A. the anterior region of the maxilla. A. Radicular.


B. the mandible, near the junction of B. Dentigerous. *
the body and the ramus. * C. Fissural.
C. the posterior region of the maxilla. D. Traumatic.
D. in the anterior region of the
mandible near the midline. In the presence of an acute bacterial
infection, laboratory tests will show an
Metastasis is most likely to occur in increase in

A. squamous cell carcinoma. * A. polymorphonuclear leukocytes. *


B. basal cell carcinoma. B. plasma cells.
C. ameloblastoma. C. lymphocytes.
D. complex odontoma. D. monocytes.
E. odontogenic fibroma. E. eosinophils.

Sickle cell anemia is In the early stage, a periradicular


abscess can be differentiated from a
A. a genetic disease. * lateral periodontal abscess by
B. caused by exposure to radiation.
C. a viral infection. A. pain.
D. a drug reaction. B. type of exudate.
E. an auto-immune disease. C. tenderness to percussion.
D. response of pulp to electrical
Coxsackie A virus is the etiologic agent stimulation. *
in E. radiographic examination.

A. thrush.
B. herpangina. *

Dr Abdul Naser Tamim SEHA, Dr Ghada Al Aqqad D.D.S, Dr Kamal Naser - Amrita Medical Centre, Dr Emad Wani - Shadi Dental Centre,
Dr Lina Anka SEHA, Dr Rouba Zgheibi SEHA, Maria Teresa Yongson Alejandrino SEHA 330
What could cause an area on an B. decrease exposure time. *
analog radiograph to be darker? C. decrease scatter radiation.
D. increase sharpness.
1. Increased processing time.
2. Reduced attenuation of the X-ray Proper collimation of the useful beam
beam in the patient. for the film size and target-film distance
3. Increased exposure time. will reduce the
4. Reduced deposition of metallic silver
on the emulsion. 1. Intensity of central beam.
2. Secondary radiation.
A. (1) (2) (3) 3. Radiographic contrast.
B. (1) and (3) 4. Radiation received by patient.
C. (2) and (4)
D. (4) only * A. (1) (2) (3)
E. All of the above. B. (1) and (3)
C. (2) and (4) *
The function of the fixer solution is to D. (4) only
E. All of the above.
1. Harden the emulsion.
2. Convert the latent image to black An increased heart rate may be
metallic silver. associated with
3. Remove unexposed silver halide.
4. Continue the action of the A. hypothyroidism.
developer. B. prolonged corticosteroid therapy.
C. hyperthyroidism. *
A. (1) (2) (3) D. Down syndrome.
B. (1) and (3) *
C. (2) and (4) *?? The apical region of a non-vital tooth
D. (4) only with a deep carious lesion may
E. All of the above. radiographically show

Selection of the appropriate kilovoltage 1. Widening of the periodontal space.


for dental films is influenced by 2. Loss of lamina dura.
3. A circumscribed radiolucency.
A. line voltage fluctuation. 4. Calcification of the periodontal
B. diameter of the primary beam of membrane.
radiation.
C. type of timer. A. (1) (2) (3) *
D. tissue density. * B. (1) and (3)
E. filter thickness. C. (2) and (4)
D. (4) only
The purpose of using intensifying E. All of the above.
screens in extra-oral radiography is to

A. improve resolution.

Dr Abdul Naser Tamim SEHA, Dr Ghada Al Aqqad D.D.S, Dr Kamal Naser - Amrita Medical Centre, Dr Emad Wani - Shadi Dental Centre,
Dr Lina Anka SEHA, Dr Rouba Zgheibi SEHA, Maria Teresa Yongson Alejandrino SEHA 331
The oral examination of the E. tuberculomas.
edentulous patient should include
digital palpation because For an adult patient, the recommended
time interval between bitewing
1. The thickness of the mucosa can radiographic examination for the
better be evaluated. detection of dental caries is
2. Undercut areas may be hard to
visualize. A. 6 months.
3. Spicules under the mucosa may be B. 12 months.
overlooked. C. 24 months.
4. The arch form can be more D. dependent upon caries risk. *
accurately evaluated
Which of the following lesions is most
A. (1) (2) (3) * likely to occur in the floor of the mouth?
B. (1) and (3)
C. (2) and (4) A. Pleomorphic adenoma.
D. (4) only B. Mucoepidermoid carcinoma.
E. All of the above. C. Lymphangioma.
D. Ranula. *
Percussion of a tooth is used to E. Granular cell myoblastoma.
evaluate all of the following EXCEPT
Loss of taste to the anterior two thirds
A. ankylosis. of the tongue and a lack of secretion of
B. pain. submandibular glands indicates nerve
C. mobility. damage to the
D. vitality. *
A. mandibular division of cranial V.
The benign neoplasm that originates B. long buccal nerve.
from squamous epithelium is called C. chorda tympani nerve. *
a/an D. cranial VII.

A. adenoma. When a patient experiences


B. choriocarcinoma. continuous pain in the maxillary
C. chondroma. premolar and molar areas and there is
D. lipoma. no evidence of dental infection, the
E. papilloma. * most likely diagnosis is

Oral peripheral giant cell lesions of the A. trigeminal neuralgia.


periodontium are most likely B. acute maxillary sinusitis. *
C. impacted maxillary canine.
A. non-neoplastic granulomatous D. impacted maxillary third molar.
lesions. * E. glossopharyngeal neuralgia.
B. precursors of sarcoma.
C. malignant neoplasms.
D. benign neoplasms.

Dr Abdul Naser Tamim SEHA, Dr Ghada Al Aqqad D.D.S, Dr Kamal Naser - Amrita Medical Centre, Dr Emad Wani - Shadi Dental Centre,
Dr Lina Anka SEHA, Dr Rouba Zgheibi SEHA, Maria Teresa Yongson Alejandrino SEHA 332
The most likely diagnosis of a 1. Subperiosteal erosion of the
patient with pain, swelling, phalanges.
numbness of the jaw and 2. Osteopetrosis.
unexplained tooth mobility is 3. Pathological fractures.
4. Renal stones.
A. hyperparathyroidism.
B. fibrous dysplasia. A. (1) (3) (4) *
C. malignant neoplasm. * B. (1) and (3)
D. giant cell reparative granuloma. C. (2) and (4)
E. syphilis. D. All of the above.

A patient who uses nitroglycerine Which of the following sites for


has squamous cell carcinoma has the best
prognosis?
A. rheumatic heart disease.
B. asthma. A. Lower lip. *
C. coronary artery disease. * B. Retromolar area.
D. high blood pressure. C. Gingiva.
E. cardiac arrhythmia. D. Buccal mucosa.
E. Hard palate.
Erythroblastosis fetalis may be a
cause of The greatest single factor in reducing
radiation exposure in dentistry is
A. supernumerary incisors.
B. pigmented teeth. * A. higher kVp.
C. peg lateral incisors. B. proper filtration.
D. Fordyce's granules. C. high speed film.
E. blue sclerae. D. collimation of the X-ray beam. *

A decrease of which of the Lancinating paroxysmal pain in the


following is indicative of posterior part of the tongue, tonsil,
hypoparathyroidism? nasopharynx and pharynx is most
likely diagnostic of
A. Serum phosphorus.
B. Serum calcium. * A. Ménière's disease.
C. Thyroid activity. B. trigeminal neuralgia.
D. Serum alkaline phosphatase. C. sphenopalatine neuralgia.
D. glossopharyngeal neuralgia. *
In hyperparathyroidism, typical E. psychotic glosso pyrosis.
features of bone involvement are

Dr Abdul Naser Tamim SEHA, Dr Ghada Al Aqqad D.D.S, Dr Kamal Naser - Amrita Medical Centre, Dr Emad Wani - Shadi Dental Centre,
Dr Lina Anka SEHA, Dr Rouba Zgheibi SEHA, Maria Teresa Yongson Alejandrino SEHA 333
An excisional biopsy of a nodule 5mm Which of the following nerves should
in diameter on the lateral border of the be anesthetized for extraction of a
tongue was diagnosed as a fibroma. maxillary lateral incisor?
This patient should have
1. Nasociliary.
A. hemisection of the tongue. 2. Nasopalatine.
B. radiotherapy to site of biopsy. 3. Sphenopalatine.
C. no additional therapy. * 4. Anterior superior alveolar.
D. re-excision with wider margins.
E. radium implantation around biopsy A. (1) (2) (3)
site. B. (1) and (3)
C. (2) and (4) *
Hyperkeratosis, acanthosis, dysplasia, D. (4) only
increased mitosis, intact basal cell E. All of the above.
layer and chronic inflammatory cells
are histologic features that may be Function(s) of the dental pulp
found in include(s)

A. squamous cell carcinoma. 1. Defensive.


B. carcinoma in situ. * 2. Sensory.
C. papillofibroma. 3. Circulatory.
D. endothelioma. 4. Dentin repair.

Enlargement of the gingiva, described A. (1) (2) (3)


as idiopathic fibromatosis, is best B. (1) and (3)
described as C. (2) and (4)
D. (4) only
A. degeneration. E. All of the above. *
B. inflammation.
C. hyperplasia. * Pain upon vertical percussion on the
D. neoplasia. incisal edge of an anterior tooth may
indicate the possible presence of
Extraction of a tooth is
CONTRAINDICATED in the dental A. chronic pulpitis.
office for a patient who B. hyperemia.
C. necrosis.
1. Had a myocardial infarct two months
D. periradicular periodontitis. *
ago.
2. is hypothyroid.
The most important principle dictating
3. Has a Factor VIII deficiency.
location and size of access to the root
4. is 4 months pregnant.
canal system is
A. (1) (2) (3)
A. preservation of tooth structure.
B. (1) and (3) *
B. removal of all caries.
C. (2) and (4)
C. straight line access to the canal. *
D. (4) only
D. removal of all pulp horns.
E. All of the above.

Dr Abdul Naser Tamim SEHA, Dr Ghada Al Aqqad D.D.S, Dr Kamal Naser - Amrita Medical Centre, Dr Emad Wani - Shadi Dental Centre,
Dr Lina Anka SEHA, Dr Rouba Zgheibi SEHA, Maria Teresa Yongson Alejandrino SEHA 334
Polycarboxylate cement may be used Which of the following muscles
as a base material beneath a metallic contribute to the protrusion of the
restoration because mandible?

A. its thermal conductivity is similar to A. Medial pterygoids.


dentin and enamel. B. Geniohyoids.
B. its thermal diffusivity is similar to C. Lateral pterygoids. *
dentin and enamel. D. Temporalis.
C. its compressive strength when set
will resist forces of condensation. The physiologic rest position of the
D. All of the above. * mandible is

A tooth with a mild pulpitis should be 1. A position determined by the


sedated with musculature.
2. A fairly constant position throughout
A. polycarboxylate cement. life.
B. composite resin. 3. used in determining occlusal vertical
C. silicophosphate cement. dimension.
D. zinc-oxide-eugenol cement.* 4. used when making a centric
interocclusal record.
The success of indirect pulp capping is
dependent upon A. (1) (2) (3) *
B. (1) and (3)
A. removal of all caries at the enamel- C. (2) and (4)
dentin junction. D. (4) only
B. use of calcium hydroxide. E. All of the above.
C. a well sealed restoration.
D. All of the above. * The masseter muscle originates from
the
Which of the following muscles
comprise the retromolar pad? A. angle of the mandible.
B. coronoid process.
1. Lateral (external) pterygoid. C. pterygoid fossa.
2. Buccinator. D. zygomatic arch. *
3. Palatoglossus.
4. Superior constrictor. The primary muscle of the cheek is the

A. (1) (2) (3) A. orbicularis oris.


B. (1) and (3) B. masseter.
C. (2) and (4) * C. zygomaticus major.
D. (4) only D. buccinator. *
E. All of the above.

Dr Abdul Naser Tamim SEHA, Dr Ghada Al Aqqad D.D.S, Dr Kamal Naser - Amrita Medical Centre, Dr Emad Wani - Shadi Dental Centre,
Dr Lina Anka SEHA, Dr Rouba Zgheibi SEHA, Maria Teresa Yongson Alejandrino SEHA 335
Which of the following cells are
characteristic of chronic inflammation A. ameloblasts.
of the dental pulp? B. undifferentiated mesenchymal
cells. *
1. Neutrophils. C. multinucleated giant cells.
2. Eosinophils. D. osteoblasts.
3. Lymphocytes.
4. Macrophages. Prior to the placement of
5. Plasma cells. polycarboxylate cement as a base for
a restoration, the cavity preparation
A. (1) (2) (3) should be
B. (1) and (2)
C. (1) (4) (5) A. painted with cavity varnish.
D. (1) and (5) B. cleaned with hydrofluoric acid.
E. (3) (4) (5) * C. thoroughly dried with warm air.
D. cleaned with water and air dried. *
In teeth with complete pulp necrosis,
the periapical area is involved if In the mandibular first premolar, the
occlusal dovetail of an ideal disto-
1. There is pain to thermal stimuli. occlusal amalgam preparation is
2. There is pain on percussion. usually not extended into the mesial
3. The tooth throbs when the patient fossa because of the
is lying down.
4. The radiograph shows an apical A. small lingual lobe.
radiolucency. B. large buccal cusp.
C. large buccal pulp horn.
A. (1) (2) (3) D. prominent transverse ridge. *
B. (1) and (3)
C. (2) and (4) * Odontoblast nuclei displacement into
D. (4) only adjacent dentinal tubuli is thought to
E. All of the above. be

Aging of the pulp is evidenced by a 1. A reversible pathologic condition.


relative increase in 2. due to increased intrapulpal tissue
pressure.
1. Vascularity. 3. due to contraction of collagen
2. Nerve tissue. fibres.
3. Cell numbers. 4. More frequent following the use of
4. Fibrous elements. air coolant rather than water coolant.
5. Calcification. 5. One of the first histological changes
following operative trauma.
A. (1) (2) (3)
B. (1) and (3) A. (1) (2) (3)
C. (3) (4) (5) B. (1) and (3)
D. (4) and (5) * C. (2) and (4)
D. (4) only
When odontoblasts are destroyed or E. All of the above. *
undergo degeneration, they are
replaced by

Dr Abdul Naser Tamim SEHA, Dr Ghada Al Aqqad D.D.S, Dr Kamal Naser - Amrita Medical Centre, Dr Emad Wani - Shadi Dental Centre,
Dr Lina Anka SEHA, Dr Rouba Zgheibi SEHA, Maria Teresa Yongson Alejandrino SEHA 336
The success of replantation of an 1. A negative reaction to the electric
avulsed tooth is dependent upon vitality tester.
2. A positive reaction of short duration
A. length of time between avulsion and to cold.
replantation. * 3. A positive reaction to percussion.
B. completion of endodontic therapy 4. Presence of a draining fistula.
before replantation.
C. immersing the tooth in fluoride A. (1) (2) (3)
solution before replantation. B. (1) and (3) *
D. using calcium hydroxide as a C. (2) and (4)
treatment root canal filling. D. (4) only
E. All of the above.
Following the removal of a vital pulp,
the root canal is medicated and Severe throbbing tooth pain which
sealed. The patient returns with apical increases when the patient lies down
periodontitis. The most common cause is a symptom of
is
A. a pulp polyp (chronic hyperplastic
A. over-instrumentation. * pulpitis).
B. lateral perforation. B. late stage of acute pulpitis (acute
C. incorrect medication. suppurative pulpitis). *
D. pulp tissue left in the root canal. C. chronic pulpitis (chronic ulcerative
E. infection. pulpitis).
D. chronic apical abscess.
Which of the following microorganisms E. pulp hyperemia.
are most frequently found in infected
root canals? Accessory canals in permanent teeth
are most commonly found in the
A. Streptococcus viridans.
B. Staphylococcus aureus. A. cervical third of the root.
C. Lactobacilli. B. middle third of the root.
D. Enterococci. * C. apical third of the root. *
E. Staphylococcus albus. D. bifurcation area.
E. trifurcation area.
What clinical evidence would support a
diagnosis of acute dento-alveolar
abscess?

Dr Abdul Naser Tamim SEHA, Dr Ghada Al Aqqad D.D.S, Dr Kamal Naser - Amrita Medical Centre, Dr Emad Wani - Shadi Dental Centre,
Dr Lina Anka SEHA, Dr Rouba Zgheibi SEHA, Maria Teresa Yongson Alejandrino SEHA 337
In composite resin restorations, D. All of the above. *
polycarboxylate cements are used as
a base because they are The initial histological appearance of a
successful apicectomy would show on
A. sedative to a hyperemic pulp. a radiograph as
B. neutral in colour.
C. biocompatible. * A. a radiolucent area. *
D. None of the above. B. woven bone.
C. cortical bone around surgical site.
Sterilization of carious dentin without D. sclerotic dentin.
pulp injury is assured by the
application of Pulpotomy is the treatment of choice
in pulp exposures of
A. phenol.
B. 70% ethyl alcohol. A. asymptomatic vital teeth with
C. chlorhexidine. completely formed apices.
D. absolute alcohol. B. asymptomatic vital teeth with
E. None of the above. * incompletely formed apices. *
C. asymptomatic necrotic teeth with
In restoring occlusal anatomy, the completely formed apices.
protrusive condylar path inclination D. asymptomatic necrotic teeth with
has its primary influence on the incompletely formed apices.
morphology of
The most efficient cutting instrument
A. cusp height. used during endodontic procedures is
B. anterior teeth only. a
C. mesial inclines of maxillary cusps
and distal inclines of mandibular A. barbed broach.
cusps. B. reamer.
D. mesial inclines of mandibular C. k-type file.
cusps and distal inclines of maxillary D. Hedstrom file. *
cusps. *
Microbial virulence factors
Pulpal response to cavity preparation
depends upon A. are produced by non-pathogenic
microbes.
1. Remaining dentin thickness. B. are always pathogenic.
2. Coolant used while cutting. C. include exotoxins, capsules,
3. Sharpness of the bur. endotoxins and enzymes. *
4. Duration of the operation. D. are caused only by Gram-positive
microbes.
A. (1) and (3)
B. (2) and (3)
C. (3) and (4)

Dr Abdul Naser Tamim SEHA, Dr Ghada Al Aqqad D.D.S, Dr Kamal Naser - Amrita Medical Centre, Dr Emad Wani - Shadi Dental Centre,
Dr Lina Anka SEHA, Dr Rouba Zgheibi SEHA, Maria Teresa Yongson Alejandrino SEHA 338
Which of the following statements
concerning root canals and their A patient telephones and tells you he
foramina is NOT true? has just knocked out his front tooth but
that it is still intact. Your instructions
A. Root canals bifurcate and have should be to
dual foramina.
B. The major foramen is precisely at A. put the tooth in water and come to
the apex of the tooth. your office at the end of the day.
C. The root canals may join and have B. wrap the tooth in tissue and come
a single foramen. to your office in a week's time.
D. The dentino-cemental junction is C. put the tooth in alcohol and come to
precisely at the apex of the tooth. * your office immediately.
E. A cross section of the canal in the D. place tooth under the tongue and
apical region is relatively round. come to your office immediately.
E. place the tooth in milk and come to
During matrix placement for a Class II your office immediately. *
cavity preparation, a wedge is placed
to Which of the following is the most
appropriate management for a tooth
1. Separate the teeth. displaying crazing of the enamel?
2. Adapt the matrix to the gingival
margin. A. Splinting of teeth.
3. Aid in the creation of a contact. B. Stainless steel band.
4. Absorb moisture. C. Endodontic treatment.
D. Periodic observation. *
A. (1) (2) (3) *
B. (1) and (3) Endodontic therapy is
C. (2) and (4) CONTRAINDICATED in teeth with
D. (4) only
E. All of the above. A. inadequate periodontal support. *
B. pulp stones.
For a cast gold restoration, a gingival C. constricted root canals.
bevel is used instead of a shoulder D. accessory canals.
because a bevel E. curved roots.

1. Protects the enamel. Which of the following may be used to


2. increases retention. disinfect gutta-percha points?
3. Improves marginal adaptation.
4. Increases the thickness of gold. A. Glass bead sterilizer.
B. Autoclave.
A. (1) (2) (3) C. Chemical solutions. *
B. (1) and (3) * D. Flame sterilization.
C. (2) and (4) E. Dry heat sterilization.
D. (4) only
E. All of the above.

Dr Abdul Naser Tamim SEHA, Dr Ghada Al Aqqad D.D.S, Dr Kamal Naser - Amrita Medical Centre, Dr Emad Wani - Shadi Dental Centre,
Dr Lina Anka SEHA, Dr Rouba Zgheibi SEHA, Maria Teresa Yongson Alejandrino SEHA 339
During endodontic treatment, which of removal of tooth 4.6 involves placing
the following microorganisms is most the beaks of the forceps
likely to cause endocarditis in a
patient with valvular heart disease? A. as far apically on the tooth root as
possible and applying apical pressure
A. Hemolytic streptococci. * during luxation. *
B. Staphylococcus aureus. B. on the lingual and buccal enamel of
C. Candida albicans. the crown and applying apical pressure
D. Bacteroïdes fusiformis. during luxation.
C. at the cementoenamel junction of
The residual mercury content of an the tooth and gently pulling upward
amalgam restoration is significantly during luxation.
affected by D. as far apically on the tooth root as
possible and gently pulling upward
A. size of the preparation. during luxation.
B. amount of amalgam used. E. at the cementoenamel junction of
C. type of amalgam used. * the tooth and applying a rotational
D. burnishing technique. force during luxation.

An acute oral infection is of most What should be the immediate


concern in a patient with management of an acute anginal
episode?
A. pemphigus.
B. Crohn's disease. A. Oral ibuprofen.
C. otitis media. B. Sublingual nitroglycerin. *
D. a prosthetic heart valve. * C. Subcutaneous epinephrine.
D. Inhaled salbutamol.
All the following medications can be
used for the treatment of Candida In which of the following situations can
albicans EXCEPT topical corticosteroids be used?

A. nystatin. A. Angular cheilitis.


B. fluconazole. B. Candidiasis.
C. chlorhexidine. C. Herpes labialis.
D. amoxicillin. * D. Erosive lichen planus. *
E. Necrotizing ulcerative gingivitis.
For acute dental pain, the daily
maximum cumulative dose of Which of the following impression
acetylsalicylic acid is materials is the LEAST flexible when
set?
A. 2,400 mg.
B. 3,200 mg. A. Polyvinyl siloxane.
C. 3,600 mg. B. Condensing silicone.
D. 4,000 mg. * C. Polyether. *
D. Polysulfide.
The proper placement of a lower
universal extraction forceps for the

Dr Abdul Naser Tamim SEHA, Dr Ghada Al Aqqad D.D.S, Dr Kamal Naser - Amrita Medical Centre, Dr Emad Wani - Shadi Dental Centre,
Dr Lina Anka SEHA, Dr Rouba Zgheibi SEHA, Maria Teresa Yongson Alejandrino SEHA 340
In a normally developing occlusion, A. Hematoma.
spaces between primary incisors are B. Ulceration. *
called C. Erythema.
D. Fibromatosis.
A. physiological spaces. *
B. primate spaces. What statement related to self-
C. leeway spaces. threaded pins is FALSE?
D. freeway spaces.
A. The ideal depth for the pin hole is
When performing a frenectomy, a 3mm. *
minimal amount of anesthetic solution is B. The pin does not increase fracture
used to prevent resistance of the restoration.
C. The pin can increase retention of
A. distortion of the tissues. * the restoration.
B. sloughing. D. The drill has a smaller diameter
C. secondary bleeding. than the pin.
D. irritation.
Which disorder presents with all
During the intravenous administration of permanent teeth exhibiting shortened
diazepam, which of the following roots, obliterated pulp canals, small
arteries may accidentally be entered in crescent-shaped pulp chambers and
the antecubital fossa? apical radiolucencies?

A. Ulnar. A. Amelogenesis imperfecta.


B. Radial. B. Dentinogenesis imperfecta.
C. Brachial. * C. Dentin dysplasia type I. *
D. Deep brachial. D. Dentin dysplasia type II.
E. Radial recurrent.
Which drug should be administered as
Which of the following dental materials the initial management for a patient
is the most radiolucent? with chest pain consistent with a
myocardial infarction?
A. Porcelain.
B. Gutta-percha. A. Epinephrine.
C. Acrylic. * B. Atropine.
D. Zinc oxide. C. Diphenhydramine.
D. Lidocaine.
Which of the following is the most E. Acetylsalicylic acid. *
characteristic gingival feature of
agranulocytosis?

Dr Abdul Naser Tamim SEHA, Dr Ghada Al Aqqad D.D.S, Dr Kamal Naser - Amrita Medical Centre, Dr Emad Wani - Shadi Dental Centre,
Dr Lina Anka SEHA, Dr Rouba Zgheibi SEHA, Maria Teresa Yongson Alejandrino SEHA 341
During dental treatment, a 45 year old has completed the series of
male patient complains of a tight vaccinations against hepatitis B
constriction of his chest, becomes pale reveals that their anti-HBsAg is less
and sweaty, feels nauseous and than the value required for immunity.
attempts to vomit. The most likely The health care worker should
diagnosis is
A. receive one additional vaccination
A. pulmonary embolus. followed by post-immunization testing*
B. stroke. B. repeat the full series of hepatitis B
C. pneumonia. vaccinations followed by post-
D. myocardial infarction. * immunization testing.
C. refrain from performing any
A dentist infected with Hepatitis C virus exposure-prone procedures for a
but without disease symptoms should period of 3-6 months followed by a full
series of hepatitis B vaccinations.
A. not be allowed to practice. D. have liver function tests performed
B. be allowed to practice but should be to assess liver damage from a
excluded from performing exposure previous hepatitis B infection.
prone procedures.
C. be allowed to practice but should be In the pterygomandibular space, the
excluded from performing exposure inferior alveolar nerve passes
prone procedures after assessment
and agreement by an expert review A. anterior to the deep tendon of the
panel and if there is compliance with temporal muscle.
standard precautions (routine B. lateral to the sphenomandibular
practices). ligament. *
D. be allowed to practice after C. medial to the medial pterygoid
assessment and agreement by an muscle.
expert review panel and if there is D. medial to the pterygomandibular
compliance with standard precautions raphe.
(routine practices). *
What is the name of the process by
Naloxone reverses respiratory which carbamide peroxide bleaches
depression caused by the teeth?

A. meperidine. * A. Oxidation. *
B. lorazepam. B. Addition.
C. alcohol. C. Subtraction.
D. phenobarbital. D. Hydrogenation.

Post-immunization serological test


results for a health care worker who

Dr Abdul Naser Tamim SEHA, Dr Ghada Al Aqqad D.D.S, Dr Kamal Naser - Amrita Medical Centre, Dr Emad Wani - Shadi Dental Centre,
Dr Lina Anka SEHA, Dr Rouba Zgheibi SEHA, Maria Teresa Yongson Alejandrino SEHA 342
Polyether impression materials should
be used with caution for full arch The accuracy of alginate impression
impressions of dentate patients materials will be improved if
because they
A. the space between the tray and the
A. exhibit viscoelasticity. teeth is 1-2mm.
B. exhibit a high elastic modulus. * B. the space between the tray and the
C. are thixotropic. teeth allows 4-5mm of alginate. *
D. are hydrophilic. C. the impression is removed slowly
E. are exothermic. from the undercuts around the teeth.
D. the impression is immersed in
Which is the most appropriate method disinfectant for one hour before
to minimize loss of dental amalgam pouring.
and mercury from dental offices into
sewage systems? A conical pontic replacing a
mandibular first molar should be
A. Use of ISO approved amalgam designed so that
separators. *
B. Storage of amalgam capsules in A. it seals the mucogingival fold.
sealed containers. B. it has open gingival embrasures. *
C. Use of mercury vapour scavengers. C. the porcelain to metal junction is on
D. Disposal of scrap amalgam in a its gingival surface.
landfill site. D. the gingival surface is concave
buccolingually.
A patient presents with pain from tooth
4.7 which is an abutment for a 4 unit During the administration of local
bridge from 4.4 to 4.7. Clinical and anesthesia, positive aspiration of blood
radiographic examinations reveal tooth will occur most often in a/an
4.7 has extensive distal caries and
apical rarefying osteitis. The most A. mental or incisive block.
appropriate initial management is to B. posterior superior alveolar block.
C. inferior alveolar block. *
A. prescribe an antibiotic and an D. anterior superior alveolar block.
analgesic and reappoint the patient. E. long buccal nerve block.
B. perform endodontic therapy through
the 4.7 crown. A Class II amalgam preparation on a
C. section the bridge at 4.4, remove primary tooth does NOT require a
4.7 crown and assess 4.7. * gingival bevel because the enamel
D. remove entire bridge and assess rods in the area incline
restorability of abutments.
A. gingivally.
A reciprocal clasp arm on a removable B. horizontally.
partial denture will provide C. occlusally. *
D. vertically.
A. support.
B. indirect retention. *
C. stabilization. *
D. direct retention.

Dr Abdul Naser Tamim SEHA, Dr Ghada Al Aqqad D.D.S, Dr Kamal Naser - Amrita Medical Centre, Dr Emad Wani - Shadi Dental Centre,
Dr Lina Anka SEHA, Dr Rouba Zgheibi SEHA, Maria Teresa Yongson Alejandrino SEHA 343
For sterilization to occur in an A. Afternoon appointments should be
autoclave, the packaged instruments scheduled.
are subjected to pressurized B. Insulin should be increased to
offset the length of the appointment.
A. steam. * C. A medical consult is required prior
B. chemical vapour. to treatment.
C. boiling water. D. The patient should follow normal
D. heated air. dietary and insulin regimen. *

Which of the following is NOT An oroantral communication occurs


associated with cancer chemotherapy
in an adult? A. more often with the removal of
maxillary second premolars.
A. Candida infection. B. less often when the maxillary sinus
B. Gingival bleeding. has pneumatized into the alveolus.
C. Enamel staining. * C. more often with the removal of
D. Dysgeusia. maxillary first molars. *
D. less often in elderly patients.
The daily maximum dose of ibuprofen
for an adult is Which of the following does NOT
occur when epinephrine is
A. 1,200 mg. administered intravenously?
B. 1,600 mg.
C. 2,000 mg. A. Increased systolic pressure.
D. 2,400 mg. * B. Increased heart rate.
C. Arrhythmias.
The antithrombotic effects of aspirin D. Respiratory depression. *
may be compromised by
Which of the following has the
A. codeine. potential for malignant transformation?
B. acetaminophen.
C. ibuprofen. * A. Osteomalacia.
D. tramadol. B. Albright's syndrome.
C. Paget's disease. *
Which of the following is the most D. Osteogenesis imperfecta.
appropriate management for a well-
controlled, type 1 diabetic patient?

Dr Abdul Naser Tamim SEHA, Dr Ghada Al Aqqad D.D.S, Dr Kamal Naser - Amrita Medical Centre, Dr Emad Wani - Shadi Dental Centre,
Dr Lina Anka SEHA, Dr Rouba Zgheibi SEHA, Maria Teresa Yongson Alejandrino SEHA 344
Increased tooth mobility and the B. conservative preparation for an
absence of lamina dura are signs of amalgam restoration.
C. preventive regimen to be
A. hyperthyroidism. implemented to arrest the
B. hyperpituitarism. demineralization and remineralize. *
C. hyperparathyroidism. * D. conservative preparation for a
D. scleroderma. resin-modified glass ionomer
restoration.
Which of the following statements
regarding Informed Consent is Of the following structures, which
FALSE? It is would be projected closest to the
occlusal plane when taking a
A. always necessary. mandibular posterior periapical
B. given only by the person receiving radiograph?
the treatment. *
C. able to be withdrawn. A. External oblique ridge. *
D. procedure specific. B. Mandibular canal.
C. Submandibular salivary gland
A 10-15 second application of 37% fossa.
phosphoric acid on prepared dentin will D. Mental foramen.
result in all of the following EXCEPT
A periodontal screening and
A. elimination of the smear layer. recording (PSR) score of 3 for a
B. opening of the dentinal tubules. sextant indicates that probing depth
C. demineralization of the superficial does NOT exceed
dentin.
D. elimination of the collagen fibres. * A. 3.0mm.
B. 3.5mm.
Facial paralysis following an inferior C. 4.0mm.
alveolar nerve block is a result of D. 5.5mm. *
injecting the solution too far E. 6.0mm.

A. inferiorly. A surgical flap for the extraction of a


B. superiorly. tooth should be designed to
C. posteriorly. *
D. anteriorly. A. avoid attached gingiva.
B. remain coronal to the mucogingival
Early detection of a noncavitated junction.
smooth surface caries lesion allows for C. be partial thickness.
a D. have a wide base. *

A. conservative preparation for a


composite resin restoration.

Dr Abdul Naser Tamim SEHA, Dr Ghada Al Aqqad D.D.S, Dr Kamal Naser - Amrita Medical Centre, Dr Emad Wani - Shadi Dental Centre,
Dr Lina Anka SEHA, Dr Rouba Zgheibi SEHA, Maria Teresa Yongson Alejandrino SEHA 345
An impacted mandibular third molar
can be displaced into the A. The sterilization process effectively
submandibular space during its removes latex proteins from
surgical removal when the instruments handled with latex gloves.
B. Latex protein antigens can exist in
A. attachment of the mylohyoid the ambient air for a maximum of 10
muscle is inferior to the level of the minutes following operative
roots. procedures.
B. roots of the mandibular third molar C. Emergency kits should be available
lie close to the buccal cortex. that contain latex-free materials. *
C. buccal cortical bone in the D. Patients should be scheduled at
mandibular third molar area is the end of the day to avoid any
extremely thin. exposure to latex products in the
D. bone on the lingual surface is dental office.
fenestrated inferior to the mylohyoid
muscle. * A 60 year old patient in chronic renal
failure has bilateral radiolucent
Creutzfeldt-Jacob disease is caused mandibular lesions. Histological
by (a) analysis reveals that these are giant
cell lesions. This patient should be
A. virus. evaluated for
B. bacteria.
C. fungus. A. hyperparathyroidism.
D. prion. * B. hyperthyroidism. *
C. hyperpituitarism.
Which virus is the most likely to cause D. hypoparathyroidism.
an infection in a healthcare worker
following exposure to blood from an With respect to the use of gloves in
individual infected with the virus? dentistry,

A. Hepatitis A. A. gloves may be reused on the same


B. Hepatitis B. * patient if they can be removed without
C. Hepatitis C. contaminating the operator.
D. Human immunodeficiency virus. B. utility gloves are appropriate for
patient care when no other options
Which is the LEAST common type of are available.
adverse reaction associated with the C. gloves may be used on more than
use of latex gloves? one patient as long as there are no
identified risks in that patient’s
A. Immediate type I allergic reaction. * medical history.
B. Delayed hypersensitivity. D. sterile gloves should be used
C. Type IV immunologic reaction. during a surgical procedure. *
D. Irritant contact dermatitis.

When managing patients with true


latex allergies, which of the following
statements is true?

Dr Abdul Naser Tamim SEHA, Dr Ghada Al Aqqad D.D.S, Dr Kamal Naser - Amrita Medical Centre, Dr Emad Wani - Shadi Dental Centre,
Dr Lina Anka SEHA, Dr Rouba Zgheibi SEHA, Maria Teresa Yongson Alejandrino SEHA 346
Which of the following is the most likely Which of the following statements is
cause of osteoporosis, glaucoma, true regarding local anesthetic syringes
hypertension and peptic ulcers in a 65 and needles for dental anesthesia?
year old with Crohn’s disease?
A. Bending a needle is an acceptable
A. Uncontrolled diabetes. practice for injections when the needle
B. Systemic corticosteroid therapy. * is inserted more than 5mm into soft
C. Chronic renal failure. tissue.
D. Prolonged NSAID therapy. B. To avoid percutaneous injury,
E. Malabsorption syndrome. needles may be left uncapped away
from the working area after use.
The most appropriate indication for C. A new anesthetic needle should be
double gloving is used when the elapsed time between
multiple injections is more than 30
A. patient-specific. minutes.
B. for a procedure on a patient with D. Needles should be recapped after
AIDS. use, using a scoop method or
C. procedure-specific. * mechanical device. *
D. for a procedure that requires a high
degree of tactile sensitivity. Acute anaphylactic reactions to
penicillin are LEAST likely to occur
Primary personal protective equipment
includes all of the following EXCEPT A. in patients with a negative skin test
to penicillin. *
A. protective clothing. B. within minutes after drug
B. gloves. administration.
C. masks. C. in patients who have already
D. protective eyewear. experienced an allergic reaction to the
drug.
E. glove liners. *
D. when the drug is administered
If post-exposure prophylaxis is parenterally.
recommended following a significant
percutaneous injury from an HIV- A syphilitic gumma is most commonly
positive patient, the antiviral drugs found on the
should ideally be administered within
A. lip.
A. 1-2 hours of the injury. * B. tongue.
B. 1-2 days of the injury. C. palate. *
C. 2 weeks of the injury. D. buccal mucosa.
D. 4 weeks of the injury. E. gingiva.

Dr Abdul Naser Tamim SEHA, Dr Ghada Al Aqqad D.D.S, Dr Kamal Naser - Amrita Medical Centre, Dr Emad Wani - Shadi Dental Centre,
Dr Lina Anka SEHA, Dr Rouba Zgheibi SEHA, Maria Teresa Yongson Alejandrino SEHA 347
A patient presents complaining of B. aggressive periodontitis. *
spontaneous pain from a tooth. Pain C. gingivitis.
persists following thermal testing by D. gingival hyperplasia.
cold. What is the most likely
diagnosis? The etiologic agent for necrotizing
ulcerative gingivitis (NUG) is
A. Normal pulp.
B. Reversible pulpitis. A. bacterial. *
C. Irreversible pulpitis. * B. viral.
D. Pulpal necrosis. C. fungal.
D. protozoan.
During guided tissue regeneration
therapy, the regenerative cells After the elimination of occlusal trauma,
originate primarily from the even in the presence of inflammation,
which of the following is most likely to
A. lamina propria. result?
B. periodontal ligament. *
C. cellular cementum. A. Reduction in tooth mobility. *
D. collagen membrane. B. Regeneration of the periodontal
ligament.
Which tooth has the best prognosis in C. Restoration of lost alveolar bone.
a patient with generalized periodontal D. Gain of clinical attachment.
disease?
Metronidazole has been prescribed for
A. 1.1. a patient taking warfarin daily for the
B. 1.3. * last 2 years. Which of the following
C. 1.4. must be closely monitored?
D. 1.6.
A. Platelet count.
The normal position of the alveolar B. Hematocrit.
crest in healthy periodontium is C. Bleeding time.
D. INR. *
A. 1 to 2mm coronal to the CEJ.
B. at the CEJ. Primary occlusal trauma can cause
C. 1 to 2mm apical to the CEJ. *
D. 3 to 4mm apical to the CEJ. A. gingival recession.
B. furcation involvement.
Systemic antibiotics may be indicated C. horizontal bone loss.
for patients presenting with D. tooth sensitivity. *
generalized

A. chronic periodontitis.

Dr Abdul Naser Tamim SEHA, Dr Ghada Al Aqqad D.D.S, Dr Kamal Naser - Amrita Medical Centre, Dr Emad Wani - Shadi Dental Centre,
Dr Lina Anka SEHA, Dr Rouba Zgheibi SEHA, Maria Teresa Yongson Alejandrino SEHA 348
A patient has been taking a systemic Which of the following will result from a
corticosteroid for 10 years. Which 2 week regimen of tooth whitening
skeletal disorder would the patient using a 10% carbamide peroxide gel in
most likely have as a result of this a custom tray for 8 hours each night?
medication?
A. Moderate demineralization of
A. Osteopetrosis. enamel.
B. Osteogenesis imperfecta. B. Significant incidence of irreversible
C. Skeletal hyperostosis. pulpitis.
D. Osteoporosis. * C. Decreased bonding potential to
E. Osteoarthritis. enamel. *
D. Decreased enamel surface porosity.
Compared to a full thickness flap, a
partial-thickness (split-thickness) flap Proper lip support for a maxillary
will complete denture is provided primarily
by which of the following?
A. increase the loss of marginal bone.
B. reduce infraosseous defects. A. Labial surface of the teeth and
C. provide improved surgical access. simulated gingiva. *
D. increase the amount of attached B. Thickness of the border in the
gingiva. vestibule.
E. reduce healing time.* C. Festooned carvings on the labial
surface of the simulated gingiva.
Which of the following is the most D. Convex surface of the labial flange.
appropriate indication for resective
osseous periodontal surgery? What is the purpose of having a
radiometer in a dental office?
A. Advanced attachment loss.
B. Class III furcation defect. A. To track the number of x-rays taken
C. Inadequate clinical crown length. * in a month.
D. Vertical root fracture. B. To measure the output of the visible
light-curing unit. *
Tooth 3.6 had endodontic treatment C. To measure the wavelength of the
completed 10 years ago. It is x-ray machine.
asymptomatic but a periapical D. To measure the wavelength of the
radiograph reveals a 5mm visible light-curing unit.
radiolucency associated with the distal
root apex. The surrounding soft tissues Which of the following is NOT a
are within normal limits. The most likely characteristic of cavity liners?
diagnosis for tooth 3.6 is a/an
A. They are placed with minimal
A. acute periradicular periodontitis. thickness.
B. acute periradicular abscess. B. They provide some type of
C. chronic periradicular periodontitis. * therapeutic benefit.
D. chronic suppurative periradicular C. They are used as a dentin
periodontitits. replacement. *
D. They promote pulpal health.

Dr Abdul Naser Tamim SEHA, Dr Ghada Al Aqqad D.D.S, Dr Kamal Naser - Amrita Medical Centre, Dr Emad Wani - Shadi Dental Centre,
Dr Lina Anka SEHA, Dr Rouba Zgheibi SEHA, Maria Teresa Yongson Alejandrino SEHA 349
Incomplete tooth fracture The advantage of a high copper
amalgam is
A. can readily be diagnosed using
transillumination. * A. high early strength. *
B. most commonly involves the B. low post operative sinsitivity.
supporting cusps. C. high formation of the 2 phase.
C. is associated with medium to large- D. increased polishability.
sized restorations.
D. elicits dull, prolonged pain on Undercontoured restorations on the
chewing. vestibular and lingual surfaces of
posterior teeth can immediately lead to
An 86 year old patient with poor oral
hygiene has a cavitated lesion with A. deflection of food particles.
active caries. The lesion is on the B. gingival recession.
vestibular root surface of a maxillary C. increased root sensitivity.
posterior tooth, and the patient does D. food impaction. *
not want any mercury in their mouth.
The most appropriate restorative The proximal surfaces of two adjacent
material for this lesion is a/an teeth in contact form the borders of the

A. microfill resin composite. A. interdental space.


B. hybrid resin composite. B. embrasures. *
C. resin-modified glass ionomer. * C. interdental col.
D. amalgam. D. line angles.
The effects of polymerization A patient presents with pain from tooth
shrinkage of composite resin must be 4.7 which is an abutment for a 4 unit
taken into account for all the following bridge from 4.4 to 4.7. Clinical and
EXCEPT the placement of a/an radiographic examination reveal tooth
4.7 has extensive distal caries and
A. occlusal restoration. apical rarefying osteitis. The most
B. mesiocclusal restoration. appropriate initial management is to
C. vestibular restoration.
D. direct veneer restoration. * A. prescribe an antibiotic and an
analgesic and reappoint the patient.
A smooth-surface proximal carious B. perform endodontic therapy through
lesion the 4.7 crown.
C. section the bridge at 4.4, remove
A. initially begins as an enamel defect 4.7 crown and assess 4.7. *
in the contact area. D. remove entire bridge and assess
B. in enamel, can be depicted as a restorability of abutments.
cone with the apex of the cone at the
dentino-enamel junction. *
C. in enamel, can be depicted as a
cone with the base of the cone at the
dentino-enamel junction.
D. initially begins as a subgingival
enamel defect covered with plaque.

Dr Abdul Naser Tamim SEHA, Dr Ghada Al Aqqad D.D.S, Dr Kamal Naser - Amrita Medical Centre, Dr Emad Wani - Shadi Dental Centre,
Dr Lina Anka SEHA, Dr Rouba Zgheibi SEHA, Maria Teresa Yongson Alejandrino SEHA 303
Which of the following is a sign of
local anesthetic overdose?
All of the following appear as midline
A. Rash. structures on periapical radiographs
B. Wheezing. EXCEPT
C. Fainting.
D. Convulsions. * A. nasopalatine/incisive canal.
E. Swelling. B. anterior nasal spine.
C. nasal septum.
Which of the following muscles is a D. zygomatic process of the maxilla. *
depressor of the mandible?
Which of the following dental
A. Temporalis. procedures could be performed with
B. Lateral pterygoid. * minimal risk for a 35 year old patient
C. Masseter. with a severe bleeding disorder?
D. Medial pterygoid.
A. Mandibular block anesthesia.
Post-immunization serological test B. Supragingival calculus removal. *
results for a health care worker who C. Incisional biopsy.
has completed the series of D. Subgingival restoration.
vaccinations against hepatitis B is
informed that their anti-HBsAg is less Polyvinylsiloxane impression materials
than the value required for immunity. have high
The health care worker should
A. polymerization shrinkage.
A. receive one additional vaccination B. dimensional stability. *
followed by post-immunization testing* C. by-product formation.
B. undergo the full series of hepatitis D. linear expansion.
B vaccinations followed by post-
immunization testing. When restoring an endodontically
C. refrain from performing any treated tooth, the post
exposure-prone procedures for a
period of 3-6 months followed by a full A. reinforces the root.
series of hepatitis B vaccinations. B. needs to end within 1mm of the
D. have liver function tests performed apex.
to assess liver damage from a C. retains the core. *
previous hepatitis B infection.

The greatest cariogenic potential is


exhibited by

A. cheese.
B. apples.
C. chewing gum.
D. raisins. *

Dr Abdul Naser Tamim SEHA, Dr Ghada Al Aqqad D.D.S, Dr Kamal Naser - Amrita Medical Centre, Dr Emad Wani - Shadi Dental Centre,
Dr Lina Anka SEHA, Dr Rouba Zgheibi SEHA, Maria Teresa Yongson Alejandrino SEHA 304
At the wax rim stage of jaw relation records for
complete dentures, phonetic tests can be used Which of the following provide the longest
to determine duration of anesthesia?

A. flange extension. A. Lidocaine 2% with 1:100,000 epinephrine.


B. protrusion. B. Prilocaine 4% with 1:200,000 epinephrine.
C. esthetics. C. Bupivacaine 0.5% with 1:200,000
D. centric relation. epinephrine. *
E. vertical dimension of occlusion. * D. Articaine 4% with 1:100,000 epinephrine.

Which of the following would require a custom A patient complains of fatigue, abdominal pain
incisal guide table for a patient with mutually and lack of appetite. The clinical examination
protected occlusion? shows that the sclera of the eyes are yellow.
There is also a yellowish diffuse discolouration
A. A fixed partial denture from tooth 3.5-3.7. of the oral mucosa. What is the most likely
B. An all ceramic crown on tooth 4.7. diagnosis?
C. A fixed partial denture from tooth 1.1-1.3. *
D. A single ceramometal crown on tooth 1.4. A. Viral hepatitis. *
B. Iron deficiency anemia.
Which of the following materials is most likely C. Hypercarotenemia.
to initiate a hypersensitivity reaction? D. Thrombocytopenic purpura.

A. Titanium. Which of the following bone lesions of the


B. Gold. mandible is/are malignant?
C. Nickel-chromium. *
D. Chrome-cobalt. 1. Osteosarcoma.
E. Silver amalgam. 2. Osteochondroma.
3. Ewing's tumor.
The youngest age at which tetracycline 4. Fibrous dysplasia.
ingestion will NOT cause discolouration of any
tooth crowns except third molars is A. (1) (2) (3)
B. (1) and (3) *
A. 6 months in utero. C. (2) and (4)
B. 5 years. D. (4) only
C. 10 years. * E. All of the above.
D. 15 years.

Dr Abdul Naser Tamim SEHA, Dr Ghada Al Aqqad D.D.S, Dr Kamal Naser - Amrita Medical Centre, Dr Emad Wani - Shadi Dental Centre,
Dr Lina Anka SEHA, Dr Rouba Zgheibi SEHA, Maria Teresa Yongson Alejandrino SEHA 305
Which anatomical structures form the inverted During the pharyngeal phase of swallowing,
Y (Y line) in maxillary periapical radiographs? motor neurons in the swallowing center are
activated to
A. Nasopalatine/incisive canal and floor of the
nasal fossa. A. open the lower esophageal sphincter.
B. Anterior nasal spine and B. inhibit respiration. *
nasopalatine/incisive canal. C. initiate the secondary peristaltic wave.
C. Floor of the nasal fossa and maxillary sinus D. open the palatopharyngeal folds.
border. *
D. Zygomatic process of the maxilla and
maxillary sinus border.
Which of the following is a sign of an allergic
An adult female patient presents to the dental reaction to penicillin?
office with fractured teeth, a lacerated lip and a
black eye. She is with her 6 year old son, who A. Dizziness.
is crying and upset. The dentist’s receptionist B. Nausea.
discretely reports that the child was asking his C. Oliguria.
mother “Why was Daddy hurting you?” Upon D. Dermatitis. *
questioning, the patient confides that her
E. Diarrhea.
husband was the source of her injuries and
indicates that she would not be pursuing any
Which of the following is characteristic of a
action. She is referred to a specialist due to the
maxillary sinus retention cyst/antral
complexity of her dental injuries, but she
pseudocyst?
requests that the source of her injuries not be
disclosed. What is the dentist’s obligation?
A. Pain and soreness of the face.
A. Respect the patient’s request regarding B. Dome-shaped appearance on a radiograph.
confidentiality. * *
B. Report her injuries to an adult protection C. Buccal expansion of the maxillary sinus.
agency.
The mesial furcation of maxillary permanent
C. Forward all information, including the source
first molars is best probed from the
of the injuries, to the specialist.
D. Report the situation to a relevant child
A. buccal.
protection agency.
B. buccal or lingual.
In addition to iron, calcium and folate, which of C. lingual. *
the following nutrients is of special concern
during pregnancy? Elevated serum parathyroid hormone levels
result in Ca2+ being released from bone
A. Vitamin B12. through receptor-mediated actions on
B. Pyridoxine.
C. Vitamin D. * A. osteoclasts.
D. Ascorbic acid. B. osteoblasts. *
C. osteocytes.
D. chondroblasts.

Dr Abdul Naser Tamim SEHA, Dr Ghada Al Aqqad D.D.S, Dr Kamal Naser - Amrita Medical Centre, Dr Emad Wani - Shadi Dental Centre,
Dr Lina Anka SEHA, Dr Rouba Zgheibi SEHA, Maria Teresa Yongson Alejandrino SEHA 306
During the pharyngeal phase of Elevated serum parathyroid hormone
swallowing, motor neurons in the levels result in Ca2+ being released
swallowing center are activated to from bone through receptor-mediated
actions on
A. open the lower esophageal
sphincter. A. osteoclasts.
B. inhibit respiration. * B. osteoblasts. *
C. initiate the secondary peristaltic C. osteocytes.
wave. D. chondroblasts.
D. open the palatopharyngeal folds.
Overlapping contacts on a bitewing
Which of the following is a sign of an radiograph result from
allergic reaction to penicillin?
1. Malalignment of teeth.
A. Dizziness. 2. Incorrect vertical angulation of the x-
B. Nausea. ray beam.
C. Oliguria. 3. Incorrect horizontal angulation of
D. Dermatitis. * the x-ray beam.
E. Diarrhea. 4. Patient movement during the
exposure.
Which of the following is characteristic
of a maxillary sinus retention A. (1) (2) (3)
cyst/antral pseudocyst? B. (1) and (3) *
C. (2) and (4)
A. Pain and soreness of the face. D. (4) only
B. Dome-shaped appearance on a E. All of the above.
radiograph. *
C. Buccal expansion of the maxillary Tell-show-do technique when used for
sinus. behaviour management of pediatric
patients
The mesial furcation of maxillary
permanent first molars is best probed A. works best for children under 3
from the years of age.
B. involves using scientific dental
A. buccal. terminology in all explanations.
B. buccal or lingual. C. will decrease a child’s fear of the
C. lingual. * unknown and their anticipation of pain*

Dr Abdul Naser Tamim SEHA, Dr Ghada Al Aqqad D.D.S, Dr Kamal Naser - Amrita Medical Centre, Dr Emad Wani - Shadi Dental Centre,
Dr Lina Anka SEHA, Dr Rouba Zgheibi SEHA, Maria Teresa Yongson Alejandrino SEHA 307
Which of the following should be
performed to ensure a well-adapted
and functional stainless steel crown?
Assuming there is adequate tooth
A. Prepare the tooth with sharp line structure remaining, composite resins
angles. can be used as a core material for
B. Break interproximal contacts using endodontically treated teeth to be
a tapered bur. * crowned provided
C. Prepare a well-defined chamfer
margin. A. the resin has a high contrast colour
D. Cement the crown with rubber dam with tooth structure.
in place. B. there is an adequate ferrule. *
C. the resin is autopolymerizing.
Which of the following is a D. subsequent crown margins are not
CONTRAINDICATION for placement located on cementum.
of a fissure sealant on a permanent
molar? Which of the following drugs controls
and reduces inflammation?
A. Tooth erupted more than one year.
B. Deep, narrow fissures. A. Codeine.
C. Inadequate moisture control. * B. Acetaminophen.
D. Community water supply C. Ibuprofen. *
fluoridated at 1.0ppm. D. Meperidine HCl.

A 4 year old child presents for an For a cast gold restoration, a gingival
emergency examination. The mother bevel is used instead of a shoulder
is concerned about white patches on because a bevel
the child’s tongue. The child has no
pain, eats and drinks normally and 1. Protects the enamel.
has a history of repeated use of 2. increases retention.
amoxicillin for otitis media. What is the 3. Improves marginal adaptation.
most likely diagnosis? 4. Increases the thickness of gold.
A. Primary herpetic gingivostomatitis. A. (1) (2) (3)
B. Geographic tongue. B. (1) and (3) *
C. Candidiasis. * C. (2) and (4)
D. Hairy tongue. D. (4) only
E. All of the above.
The tooth preparation for a porcelain
veneer must have a/an

A. incisal reduction of 0.5mm.


B. space for the veneer material. *
C. margin at least 1mm
supragingivally.
D. butt joint gingival margin.

Dr Abdul Naser Tamim SEHA, Dr Ghada Al Aqqad D.D.S, Dr Kamal Naser - Amrita Medical Centre, Dr Emad Wani - Shadi Dental Centre,
Dr Lina Anka SEHA, Dr Rouba Zgheibi SEHA, Maria Teresa Yongson Alejandrino SEHA 308
A 45 year old patient has 32 unrestored teeth. An incipient lesion on an interproximal surface
The only defects are deeply stained grooves in is usually located
the posterior teeth. Clinical examination reveals
no evidence of caries in the grooves. The most A. at the contact area.
appropriate management is B. facial to the contact area.
C. lingual to the contact area.
A. application of pit and fissure sealants. D. gingival to the contact area. *
B. preventive resin restorations. E. occlusal to the contact area.
C. conservative Class I amalgams.
D. prophylactic odontotomy. The residual mercury content of the amalgam
E. no treatment. * restoration is significantly affected by

In an ideal Class I occlusion, the cusp of which A. cavity outline.


mandibular tooth is in contact with the central B. amount of amalgam used.
fossa of the maxillary second molar? C. condensation technique. *
D. carving technique.
A. Mesiobuccal cusp of the first molar.
B. Distobuccal cusp of the first molar. The most accurate indicator of caries activity in
C. Mesiobuccal cusp of the second molar. root caries lesions is to
D. Distobuccal cusp of the second molar. *
A. assess the colour.
The minimum time to wait before placing B. evaluate the hardness. *
composite restorations after the completion of C. use bite-wing radiographs.
a bleaching (whitening) treatment is D. apply caries detector dyes.
A. 1 to 2 hours. The recommended cavity access for a Class III
B. 24 to 48 hours. carious lesion is from the lingual because it
C. 1 to 2 weeks. *
D. 4 to 5 weeks. A. permits less removal of intact enamel.
B. preserves the labial enamel for esthetic
The minimal occlusal reduction for a porcelain purposes. *
onlay is C. is easier than from the vestibular.
D. permits the use of a larger bur.
A. 1mm.
B. 2mm. * A radiopaque area within the alveolar process
C. 3mm. containing several rudimentary teeth suggests
a/an
Composite resins bond to tooth structure
through A. periapical cemento-osseous dysplasia.
B. ameloblastoma.
A. Van der Waals forces. C. compound odontoma. *
B. micromechanical retention. * D. complex odontoma.
C. chemical bonding. E. Pindborg tumor.

Dr Abdul Naser Tamim SEHA, Dr Ghada Al Aqqad D.D.S, Dr Kamal Naser - Amrita Medical Centre, Dr Emad Wani - Shadi Dental Centre,
Dr Lina Anka SEHA, Dr Rouba Zgheibi SEHA, Maria Teresa Yongson Alejandrino SEHA 309
Under normal conditions, the most C. fluoroaluminosilicate powder and
definitive test to confirm the loss of pulp orthophosphoric acid.
vitality is D. fluoroaluminosilicate powder and
polyacrylic acid. *
A. applying warm gutta percha to the
crown. A known insulin-dependent diabetic
B. cutting into the dentin without patient feels unwell following the
anesthetic. * administration of a local anesthetic and
C. applying ethyl chloride to the crown. becomes pale and sweaty. This
D. performing a radiographic condition does not respond to placing
examination of the tooth. the patient in a supine position. The
E. performing an electric pulp test. most likely cause is

Which of the following conditions would A. syncope.


NOT require antibiotic premedication B. adrenal insufficiency.
before endodontic therapy? C. hyperglycemia.
D. hypoglycemia. *
A. Valvular heart disease. E. carotid sinus reflex.
B. Cardiac prosthesis.
C. Persistent odontogenic fistula. * Epinephrine should NOT be used as a
D. Immunosuppressive therapy. vasoconstrictor for patients with
E. Organ transplant. uncontrolled

A fracture in an all-ceramic crown may A. hyperthyroidism. *


be caused by B. hyperparathyroidism.
C. myxedema.
1. Inadequate ceramic thickness. D. asthma.
2. Sharp line angles in the tooth
preparation. To prevent mesial drift of a permanent
3. Excessive occlusal load. first molar, the ideal time to place a
4. Use of an inappropriate luting distal extension space maintainer is
material.
A. as soon as the tooth erupts through
A. (1) (2) (3) the gingival tissue.
B. (1) and (3) B. after the permanent second molar
C. (2) and (4) has erupted.
D. (4) only C. immediately after extraction of the
E. All of the above. * primary second molar. *
D. as soon as the extraction site of the
Generally, glass ionomer cements primary second molar has completely
contain healed.

A. zinc oxide and distilled water.


B. zinc oxide and polyacrylic acid.

Dr Abdul Naser Tamim SEHA, Dr Ghada Al Aqqad D.D.S, Dr Kamal Naser - Amrita Medical Centre, Dr Emad Wani - Shadi Dental Centre,
Dr Lina Anka SEHA, Dr Rouba Zgheibi SEHA, Maria Teresa Yongson Alejandrino SEHA 310
An 8 year old patient with all primary A. regular assessment of arch
molars still present exhibits a cusp-to- development.
cusp relationship of permanent B. to perform space analysis.
maxillary and mandibular first molars. C. insertion of a space maintainer. *
The management of this patient should D. extraction of the contra-lateral
be to molar.
E. extraction of the opposing molar.
A. plan serial extractions for more
normal adjustment of the occlusion. In a 4 year old child, the primary central
B. refer the patient to an orthodontist incisor has discoloured following a
for consultation. traumatic injury. The treatment of
C. place a cervical headgear to choice is
reposition maxillary molars.
D. disk the distal surfaces of primary A. pulpotomy.
mandibular second molars to allow B. pulpectomy.
normal adjustment of permanent C. observation. *
molars. D. extraction.
E. observe. *
A large carious exposure occurs on a
Which of the following will impede permanent first molar of a 7 year old.
healing following the surgical closure of There is no periapical involvement and
an oroantral fistula? the tooth is vital. The treatment should
be to
1. Poor flap design.
2. Excessive tissue tension. A. cap the exposure with calcium
3. Blowing the nose. hydroxide and place zinc-oxide and
4. Sinus infection. eugenol.
B. perform a pulpotomy and place
A. (1) (2) (3) calcium hydroxide. *
B. (1) and (3) C. perform a pulpectomy.
C. (2) and (4) D. extract the tooth and place a space
D. (4) only maintainer.
E. All of the above. *
In children, the most common cause of
A Le Fort I or Guerin fracture is a a fistula is a/an

A. fracture of the zygomatic arch. A. acute periradicular abscess.


B. horizontal fracture of the maxilla. * B. suppurative periradicular
C. fracture of the malar complex periodontitis. *
involving the floor of the orbit. C. acute periodontal abscess.
D. pyramidal fracture of the maxilla. D. dentigerous cyst.
E. craniofacial dysjunction.

The most appropriate treatment


following the extraction of a first
primary molar in a 4 year old child is

Dr Abdul Naser Tamim SEHA, Dr Ghada Al Aqqad D.D.S, Dr Kamal Naser - Amrita Medical Centre, Dr Emad Wani - Shadi Dental Centre,
Dr Lina Anka SEHA, Dr Rouba Zgheibi SEHA, Maria Teresa Yongson Alejandrino SEHA 311
A 6 year old patient has a larger than
average diastema between the A. is nonvital.
maxillary central incisors. The B. has a periodontal pocket.
radiographic examination shows a C. has a hyperemic pulp. *
mesiodens. In order to manage the D. has chronic proliferative pulpitis.
diastema, you should extract the
mesiodens A patient complains of acute pain 24
hours after the insertion of a restoration
A. after its complete eruption. in a tooth with no preexisting periapical
B. once the patient has reached the pathology. The tooth is vital and tender
age of 12. to percussion. The radiograph will
C. only if it develops into a cystic show
lesion.
D. as soon as possible. * A. an apical radiolucency.
B. acute osteitis.
One week after an amalgam C. root resorption.
restoration is placed in the mandibular D. condensing osteitis.
first premolar, the patient returns E. normal lamina dura. *
complaining of a sharp pain of short
duration when eating or drinking A well circumscribed 3mm radiolucent
something cold. Teeth respond lesion is present in the apical region of
normally to electric pulp testing and the mandibular second premolar. The
heat and the radiographs are normal. tooth responds normally to vitality tests.
The most likely diagnosis is The radiolucency is most likely
A. hypercementosis. A. a periradicular periodontitis.
B. reversible pulpitis. * B. a dentigerous cyst.
C. pulpal microabscess. C. a rarefying osteitis.
D. acute periradicular periodontitis. D. the mental foramen. *
The most appropriate radiographic On a bite-wing radiograph of posterior
examination for a 4 year old without teeth, which of the following is most
visible or clinically detectable caries or likely to be misdiagnosed as proximal
anomalies, and with open proximal caries?
contacts is
A. Cemento-enamel junction. *
A. maxillary and mandibular anterior B. Marginal ridge.
occlusals.
C. Carabelli cusp.
B. a pair of posterior bite-wings.
D. Calculus.
C. maxillary and mandibular posterior
E. Cemental tear.
periapicals.
D. no radiographic examination. *

A cold stimulus applied to a tooth will


produce a hypersensitive response if
the tooth

Dr Abdul Naser Tamim SEHA, Dr Ghada Al Aqqad D.D.S, Dr Kamal Naser - Amrita Medical Centre, Dr Emad Wani - Shadi Dental Centre,
Dr Lina Anka SEHA, Dr Rouba Zgheibi SEHA, Maria Teresa Yongson Alejandrino SEHA 312
Radiographically, the opening of the
incisive canal may be misdiagnosed A. nonvital.
as a B. associated with a root fracture.
C. infraerupted. *
1. Branchial cyst. D. found in the permanent dentition.
2. Nasopalatine cyst.
3. Nasolabial cyst. Which of the following is/are
4. Periradicular cyst. associated with an unerupted tooth?

A. (1) (2) (3) 1. Odontogenic adenomatoid tumor.


B. (1) and (3) 2. Periapical cemento-osseous
C. (2) and (4) * dysplasia.
D. (4) only 3. Calcifying epithelial odontogenic
E. All of the above. tumor.
4. Cementoblastoma.
An ameloblastoma can develop from
the epithelial lining of which of the A. (1) (2) (3)
following cysts? B. (1) and (3) *
C. (2) and (4)
A. Periradicular. D. (4) only
B. Dentigerous. * E. All of the above.
C. Residual.
D. Lateral periodontal. Which of the following results from a
necrotic pulp?
The microscopic appearance of the
central giant cell granuloma of the A. Dentigerous cyst.
jaws is similar to that of lesions which B. Lateral periodontal cyst.
occur in C. Chronic periradicular periodontitis*
D. Pulp polyp.
A. hyperparathyroidism. *
B. Paget's disease. Root resorption of permanent teeth
C. cleidocranial dysplasia. may be associated with
D. hyperpituitarism.
1. Excessive orthodontic forces.
For which of the following 2. Chronic periradicular periodontitis.
pathological conditions would a lower 3. Traumatic injury.
central incisor tooth be expected to 4. Periapical cemento-osseous
respond to heat, cold and electric dysplasia.
pulp test?
A. (1) (2) (3) *
A. Apical cyst. B. (1) and (3)
B. Acute apical abscess. C. (2) and (4)
C. Periapical cemento-osseous D. (4) only
dysplasia. * E. All of the above.
D. Chronic apical periodontitis.

An ankylosed tooth is usually

Dr Abdul Naser Tamim SEHA, Dr Ghada Al Aqqad D.D.S, Dr Kamal Naser - Amrita Medical Centre, Dr Emad Wani - Shadi Dental Centre,
Dr Lina Anka SEHA, Dr Rouba Zgheibi SEHA, Maria Teresa Yongson Alejandrino SEHA 313
A 4 year old child has a normal A. insufficient parathyroid hormone.
complement of deciduous teeth, but in B. excessive parathyroid hormone.
appearance they are grayish and C. insufficient thyroid hormone. *
exhibit extensive occlusal and incisal D. excessive thyroid hormone.
wear. Radiographic examination
indicates some extensive deposits of Which of the following is most often
secondary dentin in these teeth. This associated with a nonvital tooth?
condition is typical of
A. Chronic periradicular periodontitis. *
A. cleidocranial dysplasia. B. Internal resorption.
B. amelogenesis imperfecta. C. Periapical cemento-osseous
C. neonatal hypoplasia. dysplasia.
D. dentinogenesis imperfecta. * D. Hyperplastic pulpitis.

Which of the following features would An end result of ionizing radiation used
be most indicative of a cracked tooth? to treat oral malignancies is

A. Periapical radiolucency. A. deformity of the jaws.


B. Hypersensitivity to thermal stimuli. B. reduced vascularity of the jaws. *
C. Pain upon biting pressure. * C. increased vascularity of the jaws.
D. Absent vitalometric response. D. increased brittleness of the jaws.

A 15 year old presents with hypoplastic If an alginate impression must be


enamel on tooth 1.5. All other teeth are stored for a few minutes before the
normal. This was most probably cast is poured, it should be placed in
caused by a/an
A. water.
A. vitamin D deficiency. B. 100% relative humidity. *
B. generalized calcium deficiency. C. a 1% aqueous calcium sulfate
C. high fever encountered by the solution.
patient when he had measles at age 3.
D. infection of tooth 5.5 during the When a radiographic examination is
development of tooth 1.5. * warranted for a 10 year old child, the
E. hereditary factor. most effective way to decrease
radiation exposure is to
Condensing osteitis in the periapical
region is indicative of a/an A. use a thyroid collar and lead apron.*
B. apply a radiation protection badge.
A. acute inflammation of the pulp. C. use high speed film.
B. pulpal abscess. D. decrease the kilovoltage to 50kVp.
C. chronic inflammation of the pulp.* E. take a panoramic film only.
D. early apical abscess formation.

Myxedema is associated with

Dr Abdul Naser Tamim SEHA, Dr Ghada Al Aqqad D.D.S, Dr Kamal Naser - Amrita Medical Centre, Dr Emad Wani - Shadi Dental Centre,
Dr Lina Anka SEHA, Dr Rouba Zgheibi SEHA, Maria Teresa Yongson Alejandrino SEHA 314
Which of the following drugs is used in A. All carbohydrates are equally
the treatment of mild allergic reactions? cariogenic.
B. More frequent consumption of
A. Isoproterenol. carbohydrates increases the risk. *
B. Meperidine hydrochloride. C. The rate of carbohydrate clearance
C. Diphenhydramine hydrochloride. * from the oral cavity is not significant.
D. Propoxyphene. D. Increased dietary fat increases the
risk.
A protective mechanism of the dental
pulp to external irritation or caries is the Which of the following is/are clinical
formation of signs of gingivitis?

A. pulp stones. 1. Loss of stippling.


B. tertiary dentin. * 2. Gingival hyperplasia.
C. secondary cementum. 3. Decreased pocket depth.
D. primary dentin. 4. Bleeding on probing.

Procaine (Novocaine®) is an example A. (1) (2) (3)


of a local anesthetic which is B. (1) and (3)
chemically classified as an C. (2) and (4) *
D. (4) only
A. amide. E. All of the above.
B. ester. *
C. aldehyde. The most likely diagnosis for a child
D. ethamine. with a painful, fiery-red, diffuse
E. aminide. gingivitis is

Lidocaine (Xylocaine®) is an example A. primary herpetic gingivostomatitis. *


of a local anesthetic which is B. aggressive periodontitis.
chemically classified as an C. idiopathic fibromatosis.
D. aphthous stomatitis.
A. amide. *
B. ester. In an infrabony pocket, the epithelial
C. aldehyde. attachment is located
D. ethamine.
E. aminide. A. within basal bone.
B. coronal to alveolar bone crest.
Regarding dental caries, which of the C. apical to alveolar bone crest. *
following is correct?

Dr Abdul Naser Tamim SEHA, Dr Ghada Al Aqqad D.D.S, Dr Kamal Naser - Amrita Medical Centre, Dr Emad Wani - Shadi Dental Centre,
Dr Lina Anka SEHA, Dr Rouba Zgheibi SEHA, Maria Teresa Yongson Alejandrino SEHA 315
The location and extent of subgingival Platelets: 82,000/mm3
calculus is most accurately determined Red blood cell count: 3,900,000/mm3
clinically by White blood cell count: 870,000/mm3
Normal Values:
A. radiopaque solution used in Hb: 14-18g/100ml
conjunction with radiographs. Platelets: 150,000-400,000/mm3
B. disclosing solution. Red blood cell count: 4-5million/mm3
C. probing with a fine instrument. * White blood cell count: 5,000-
D. visual inspection. 10,000/mm3
The most likely diagnosis is
Trauma from occlusion may
A. thrombocytopenic purpura.
A. initiate marginal gingivitis. B. acute myelogenous leukemia. *
B. affect the blood supply to gingiva. C. infectious mononucleosis.
C. initiate periodontitis. D. necrotizing ulcerative gingivitis.
D. affect the progression of
periodontitis. * Which disorder is associated with
hypercementosis of teeth?
Following the administration of a right
inferior alveolar nerve block, right facial A. Paget’s disease. *
paralysis is noted. This condition was B. Fibrous dysplasia.
caused by inadvertent injection into the C. Cherubism.
D. Hyperparathyroidism.
A. pterygopalatine fossa.
B. facial canal. The term used to describe epithelial
changes including nuclear
C. parotid gland. *
hyperchromatism, alteration of
D. submandibular region.
nuclear/cytoplasmic ratio and abnormal
E. sublingual gland. mitoses is
A 20 year old male presents with a
A. acanthosis.
three-day history of an acute
B. hyperparakeratosis.
generalized gingivitis. He has malaise,
fever and bilateral cervical C. dysplasia. *
lymphadenopathy. A blood D. acantholysis.
examination reveals
Hb: 8.9g/100ml

Dr Abdul Naser Tamim SEHA, Dr Ghada Al Aqqad D.D.S, Dr Kamal Naser - Amrita Medical Centre, Dr Emad Wani - Shadi Dental Centre,
Dr Lina Anka SEHA, Dr Rouba Zgheibi SEHA, Maria Teresa Yongson Alejandrino SEHA 316
Excessively dark radiographs will result E. type 2 diabetic.
from
Which of the following has
A. underdevelopment. anticonvulsant properties?
B. overexposure. *
C. backward placement of the film. A. Acetaminophen.
D. too little milliamperage. B. Codeine.
C. Diazepam. *
The full palatal major connector is D. Diphenhydramine.
indicated where E. Nitrous oxide.

A. there is a high, narrow palatal vault. Local anesthetic containing


B. a well-defined, undercut palatal epinephrine is CONTRAINDICATED
torus is present. for a patient with
C. very few teeth remain in a flat or U-
shaped arch. * A. Addison’s disease.
D. palatal tissue is soft and B. congenital methemoglobinemia.
compressible. C. diabetes mellitus.
D. pseudocholinesterase deficiency.
A 70 year old insulin-dependent patient E. sulfite sensitivity. *
has just completed a 7 day course of
ampicillin for a respiratory infection. He Which of the following local
presents with signs and symptoms anesthetics is classified as an ester?
consistent with a diagnosis of oral
candidiasis. Which of the following is A. Articaine.
the most appropriate management for B. Bupivacaine.
this patient? C. Lidocaine.
D. Mepivacaine.
A. Double the patient’s insulin dose. E. Procaine. *
B. Reduce the patient’s insulin dose.
C. Prescribe a topical steroid. During general anesthesia, all of the
D. Prescribe clindamycin. following should be monitored
E. Prescribe nystatin. * EXCEPT

Acetylsalicylic acid should be avoided A. pupil diameter. *


in each of the following EXCEPT B. blood pressure.
C. heart rate.
A. gastric ulcer. D. oxygen saturation.
B. gout. E. respiratory rate.
C. severe asthma.
D. hyperlipidemia. *

Dr Abdul Naser Tamim SEHA, Dr Ghada Al Aqqad D.D.S, Dr Kamal Naser - Amrita Medical Centre, Dr Emad Wani - Shadi Dental Centre,
Dr Lina Anka SEHA, Dr Rouba Zgheibi SEHA, Maria Teresa Yongson Alejandrino SEHA 317
Which of the following should NOT be Which of the following does NOT
administered to a patient with chest describe the energy of x-ray photons
pain consistent with a myocardial exiting the x-ray unit?
infarction?
A. Normally distributed.
A. Epinephrine. * B. Mono-energetic. *
B. Nitroglycerin. C. Proportional to frequency.
C. Oxygen. D. Inversely proportional to
D. Morphine. wavelength.
E. Acetylsalicylic acid.
An opioid, like Fentanyl, can be
The first drug used for the prescribed in conjunction with
management of anaphylaxis is
A. benzodiazepines. *
A. atropine. B. gabapentin.
B. diphenhydramine. C. muscle relaxants.
C. epinephrine. * D. NSAIDs.
D. hydrocortisone.
E. nitroglycerin. Which form of hepatitis does NOT
have a known carrier state?
A 50 year old woman has a history of
rheumatoid arthritis, bilateral A. Hepatitis A. *
enlargement of one or more salivary B. Hepatitis B.
glands and lacrimal glands, as well as C. Hepatitis C.
dryness of the eyes, nose, mouth and D. Hepatitis D.
throat. The diagnosis is
Objects that absorb x-ray radiation are
A. erythema multiforme. referred to on a radiographic image as
B. Reiter's syndrome. being
C. Gardner's syndrome.
D. Sjögren's syndrome. * A. radiodense.
E. Plummer-Vinson syndrome. B. radiolucent.
C. radiopaque. *
Which of the following describes the
radiation produced by high voltage? Which source delivers the highest
dose of radiation to humans?
A. Short wavelength, low frequency.
B. Short wavelength, high energy. * A. Cosmic.
C. Long wavelength, high frequency. B. Medically-related.
D. Long wavelength, low energy. C. Radon. *
D. Terrestrial.

Dr Abdul Naser Tamim SEHA, Dr Ghada Al Aqqad D.D.S, Dr Kamal Naser - Amrita Medical Centre, Dr Emad Wani - Shadi Dental Centre,
Dr Lina Anka SEHA, Dr Rouba Zgheibi SEHA, Maria Teresa Yongson Alejandrino SEHA 318
Ankylosis of primary teeth is most A. nuclear receptors.
frequently observed in B. ionotropic receptors.
C. metabotropic receptors.
A. maxillary molars. D. ionotropic and metabotropic
B. mandibular molars. * receptors. *
C. maxillary canines.
D. mandibular incisors. GABA is a/an

The descending (or repolarizing) A. drug binding to benzodiazepine


phase of the action potential is receptor.
caused by B. neurotrophic factor.
C. inhibitory neurotransmitter. *
A. opening potassium channels* D. excitatory neurotransmitter.
B. closing potassium channels.
C. opening chloride channels. What is the threshold count of S.
D. closing calcium channels. mutans in mixed saliva at which a
patient is deemed “high risk” for
The risk of a health care worker caries?
contracting hepatitis B through a
needlestick injury from a chronic A. 100/ml.
hepatitis B carrier is increased when B. 10,000/ml.
the patient’s serology report indicates C. 1,000,000/ml. *
the presence of D. 100,000,000/ml.

A. HBsAg. A 47 year old female patient


B. anti-HBcAg. complains of a “burning sensation” of
C. HBeAg. * the tongue. Examination reveals
D. anti-HBsAg. angular cheilitis and a smooth
redness on the entire dorsal surface
The form of hepatitis that poses the of the tongue. The most likely
greatest risk of transmission from diagnosis is
patient to dental health care worker is
A. anemia. *
A. hepatitis A. B. epithelial dysplasia.
B. hepatitis B. * C. squamous cell carcinoma.
C. hepatitis C. D. primary herpes.
D. hepatitis D. E. median rhomboid glossitis.

In neurons, glutamate is an amino


acid that binds only to

Dr Abdul Naser Tamim SEHA, Dr Ghada Al Aqqad D.D.S, Dr Kamal Naser - Amrita Medical Centre, Dr Emad Wani - Shadi Dental Centre,
Dr Lina Anka SEHA, Dr Rouba Zgheibi SEHA, Maria Teresa Yongson Alejandrino SEHA 319
Which microorganism does NOT B. attach the cells to the basement
contribute significantly to the membrane.
progression of dentinal caries? C. seal off the dentin from the pulp.
D. permit lateral cell-cell
A. Actinomyces naeslundii. communication. *
B. Lactobacillus casei.
C. Actinomyces viscosus. The periodontal ligament is constantly
D. Streptococcus salivarius. * remodeled due to the activity of

One of the mechanisms of bacterial A. osteoclasts.


adherence to the dental pellicle is B. fibroblasts. *
through C. macrophages.
D. mesenchymal cells.
A. positively charged bacteria with the
pellicle. In the keyhole model of the enamel
B. interaction of cations such as prism
calcium.
C. hydrophilic interactions. A. crystallites in the tail are angled
D. interaction of salivary anions * compared to the head. *
B. the water is located at the prism
When odontoblasts are destroyed as a edges.
result of cavity preparation C. proteins are only present in the tail.
D. crystallites have an identical
A. adjacent fibroblasts differentiate into molecular structure.
new odontoblasts.
B. a permanent defect results in the The protective role of junctional
odontoblast layer. epithelium is aided by its increased
C. surrounding odontoblasts are number of
stimulated to divide.
D. new cells differentiate from pulpal A. cell layers.
mesenchyme cells. * B. intercellular spaces.
C. cell-cell adhesions. *
Failure of bone resorption over an D. collagen fibres.
erupting tooth is due to lack of
Lack of ramus height is caused by
A. fibroblasts. faulty development of
B. osteocytes.
C. osteoclasts * A. membranous bone.
D. neutrophils. B. endochondral bone. *
C. Meckel’s cartilage.
Odontoblast gap junctions D. the temporomandibular joint.
A. adhere the cells to one another.

Dr Abdul Naser Tamim SEHA, Dr Ghada Al Aqqad D.D.S, Dr Kamal Naser - Amrita Medical Centre, Dr Emad Wani - Shadi Dental Centre,
Dr Lina Anka SEHA, Dr Rouba Zgheibi SEHA, Maria Teresa Yongson Alejandrino SEHA 320
Abnormal development of the first pharyngeal
arch may produce defects in the Which of the following statements is FALSE
with respect to rounded internal line angles in
A. zygomatic bones and the external ears* preparations for Class II composite resins?
B. mandible and the external nose.
C. maxilla and the muscles of facial A. Tooth structure is conserved.
expression. B. Stress concentration is reduced.
D. palate and the hyoid bone. C. Resistance form is compromised. *
D. Adaptation of the resin during placement is
Accessory root canals develop because root enhanced.
odontoblasts fail to
When light-cured composite resins are placed
A. produce matrix.
B. survive. A. surface polymerization is inhibited by carbon
C. divide. dioxide (CO2).
D. differentiate* B. the degree of conversion is 85-95%.
C. increments of resin should not exceed 2mm*
A small hinge articulator was used for the D. polymerization shrinkage increases with
fabrication of a cast gold onlay for tooth 4.6. filler content.
Which of the following movements will result in
the greatest discrepancy between the An amalgam coronal-radicular core build-up for
articulator and the patient? endodontically treated molar teeth requires

A. Laterotrusive. A. an adequate pulp chamber and ferrule*


B. Mediotrusive* B. a pulp chamber, ferrule and amalgam
C. Protrusive. bonding.
D. Retrusive. C. the presence of a post.
D. the use of retentive threaded pins.
Which statement is true with respect to Class II
composite resin preparations? Pins for cusp replacement should ideally be
placed
A. Extension for prevention is a key element.
B. Fissures are only included when carious. * A. within enamel.
C. Including occlusal grooves makes the B. at the dentino-enamel junction.
restoration more fracture-resistant compared to C. a minimum of 0.5-1.0mm from the dentino-
a slot preparation. enamel junction*
D. The preparation has a “standard” shape for D. a minimum of 1.5-2.0mm from the dentino-
each individual tooth. enamel junction.

Dr Abdul Naser Tamim SEHA, Dr Ghada Al Aqqad D.D.S, Dr Kamal Naser - Amrita Medical Centre, Dr Emad Wani - Shadi Dental Centre,
Dr Lina Anka SEHA, Dr Rouba Zgheibi SEHA, Maria Teresa Yongson Alejandrino SEHA 321
Mercury content in an amalgam can be C. is composed of bacteria and their
reduced by using by-products. *
D. allows only S. mutans species to
A. fast setting amalgam. flourish.
B. adequate condensation*
C. high zinc content alloy. A smooth surface coronal white spot
D. low copper content alloy. carious lesion that is visible when the
tooth is both wet and dry indicates that
Tooth 1.1 has a small fractured the
mesioincisal corner into dentin. Which
of the following is true with respect to A. lesion is less than halfway through
the preparation for the restoration? the enamel.
B. enamel is stained and not
A. Pins are usually necessary to demineralized. *
achieve adequate retention. C. lesion is more than halfway through
B. A lingual dovetail is usually required the enamel.
for retention. D. caries involves the inner half of the
C. An enamel bevel of 1mm is placed dentin.
where enamel thickness allows. *
D. Dentin coves are the primary form The periodontal ligament
of retention for the restoration.
A. only has oblique fibres during the
A 4 year old child presents with a eruptive stage. *
history of trauma and an asymptomatic B. increases in width with age.
discoloured primary maxillary left C. achieves its final structural form
incisor. A periapical radiograph reveals after complete eruption.
no abnormalities. The most appropriate D. has osteoblasts as its principle
management is a/an cells.

A. regular recall for observation* The most likely diagnosis for a 5 year
B. pulpotomy. old patient with multiple well-defined
C. pulpectomy. multilocular radiolucencies of the
D. extraction. maxilla and mandible is

Plaque accumulation is an etiologic A. ameloblastic fibromas.


factor in caries as it B. ameloblastomas.
C. cherubism*
A. is a good source of fermentable D. hyperthyroidism.
carbohydrates. E. hypophosphatasia.
B. is an acidic environment.

Dr Abdul Naser Tamim SEHA, Dr Ghada Al Aqqad D.D.S, Dr Kamal Naser - Amrita Medical Centre, Dr Emad Wani - Shadi Dental Centre,
Dr Lina Anka SEHA, Dr Rouba Zgheibi SEHA, Maria Teresa Yongson Alejandrino SEHA 322
A post cemented with zinc phosphate D. clinical attachment loss.
cement is used in an endodontically
treated tooth to Which of the following is NOT
suggestive of a diagnosis of
A. obturate the canal. necrotizing ulcerative gingivitis (NUG)?
B. strengthen the root.
C. reinforce the remaining crown. A. Bleeding from the gingiva.
D. retain the restoration. * B. “Punched-out” papillae with necrotic
slough.
A 70 year old insulin-dependent patient
C. Bad breath.
has just completed a 7 day course of
ampicillin for a respiratory infection. He D. Metallic taste.
presents with signs and symptoms E. Periodontal pocketing. *
consistent with a diagnosis of oral
candidiasis. Which of the following Spontaneous hemorrhage from the
drugs is/are appropriate to manage this gingiva may be indicative of
condition?
A. parotitis.
1. Fluconazole. B. Hodgkin’s disease.
2. Nystatin.
C. diabetes.
3. Ketoconazole.
4. Clindamycin. D. leukemia. *

A. (1) (2) (3) * Management of a “dry socket” should


include
B. (1) and (3)
C. (2) and (4)
A. hydrogen peroxide irrigation of
D. (4) only
socket.
E. All of the above.
B. vigorous curettage of the socket.
C. placement of a dressing in the
Which of the following is a reason to
socket. *
perform initial periodontal debridement
before periodontal surgery? D. a prescription for antibiotics.

It is ethical to replace amalgam


A. Increase the attachment levels.
restorations
B. Reduce infrabony pockets.
C. Reduce bleeding during the surgery*
A. on request from an informed
D. Increase the width of keratinized patient*
tissue.
B. to relieve symptoms of multiple
sclerosis.
Clinically, the progression of
C. to reduce the risk of developing
periodontitis can be determined best by
Alzheimer’s disease.
the increasing
D. to eliminate toxins from the patient.
A. number of bleeding sites on probing.
B. severity of gingival inflammation.
C. depth of periodontal probing. *

Dr Abdul Naser Tamim SEHA, Dr Ghada Al Aqqad D.D.S, Dr Kamal Naser - Amrita Medical Centre, Dr Emad Wani - Shadi Dental Centre,
Dr Lina Anka SEHA, Dr Rouba Zgheibi SEHA, Maria Teresa Yongson Alejandrino SEHA 323
It is ethical for a dentist to decline to extraction of a permanent maxillary first
treat a patient due to molar?

A. a difference in religious beliefs. A. Greater palatine.


B. the patient’s infectious status. B. Nasopalatine. *
C. Middle superior alveolar.
C. a patient being physically
D. Posterior superior alveolar.
challenged.
D. a patient being mentally challenged. With respect to local anesthetic, which
E. the dentist’s lack of skill or of the following will elicit the most rapid
knowledge* response in a patient?

Which of the following may be A. Too large a dose.


associated with a fracture of the B. Rapid absorption.
mandible? C. Slow elimination.
D. Intravascular injection. *
A. Diplopia. E. Slow biotransformation.
B. Malocclusion. *
C. Swelling of the orbit. Which of the following medications can
D. Bleeding from the nose. cause gingival enlargement?

Using a high speed dental handpiece A. Diuretics.


WITHOUT water coolant will B. Beta blockers.
C. Calcium channel blockers. *
A. produce a smoother surface. D. Angiotensin converting enzyme
B. decrease pulpal damage if used inhibitors.
with light pressure.
C. reduce clogging of dental bur. A 29 year old patient has had multiple
D. reduce debris accumulation. painful ulcerated lesions on the buccal
E. increase frictional heat. * attached gingiva for one day. No fever,
malaise or lymphadenopathy is
All of the following are signs of present. The most appropriate initial
occlusal trauma EXCEPT management is (a)

A. tooth mobility. A. topical steroid.


B. tooth sensitivity. B. palliative mouth rinse *
C. radiographic evidence of increased C. systemic antibiotic.
periodontal ligament space. D. systemic antiviral therapy.
D. loss of pulp vitality *

Which of the following nerves does


NOT require anesthesia for the

Dr Abdul Naser Tamim SEHA, Dr Ghada Al Aqqad D.D.S, Dr Kamal Naser - Amrita Medical Centre, Dr Emad Wani - Shadi Dental Centre,
Dr Lina Anka SEHA, Dr Rouba Zgheibi SEHA, Maria Teresa Yongson Alejandrino SEHA 324
Guided tissue regeneration is a surgical
procedure to A. Weight.
B. Medical history.
A. repair non-mineralized connective C. Age.
tissue. D. Gender. *
B. repair cemental defects.
C. regenerate long junctional epithelial Tetracycline therapy instituted either
attachment. in the second trimester or post partum
D. regenerate the periodontium. * to the infant is responsible for all the
following EXCEPT
A patient that has been prescribed
metronidazole should avoid A. discolouration of deciduous teeth
and permanent teeth.
A. alcohol. * B. minor changes in the
B. antacids. hydroxyapatite of the enamel. *
C. caffeine. C. predisposing the infant to candidal
D. cheese. infections.
E. grapefruit juice. D. forming a complex with the
developing tooth material.
The subgingival microbial flora isolated
from sites of peri-implantitis is most Which of the following drugs should
similar to the flora of NOT be administered to a patient in
order to alleviate symptoms of an
A. periradicular abscess. acute asthmatic attack?
B. gingivitis.
C. periodontitis. * A. Isoproterenol.
B. Metaproterenol.
A patient who is a hepatitis B carrier C. Epinephrine.
presents for an extraction. The D. Hydrocortisone. *
extraction should be delayed and
The pulpal floor of an occlusal
A. rescheduled at the end of the day for amalgam preparation on a mandibular
infection control. first premolar should slope apically
B. an antibiotic prescribed from
prophylactically.
C. the patient referred to a hospital A. mesial to distal.
dental department. B. buccal to lingual. *
D. an evaluation of liver function C. distal to mesial.
performed. * D. lingual to buccal.

Which one of the following factors is


LEAST important in determining the
appropriate dose of drug for a patient?

Dr Abdul Naser Tamim SEHA, Dr Ghada Al Aqqad D.D.S, Dr Kamal Naser - Amrita Medical Centre, Dr Emad Wani - Shadi Dental Centre,
Dr Lina Anka SEHA, Dr Rouba Zgheibi SEHA, Maria Teresa Yongson Alejandrino SEHA 325
Aspiration prior to a local anesthetic
injection reduces the A. decrease of the freeway space.
B. increase of the freeway space *
A. toxicity of local anesthetic. C. increase of the vertical dimension.
B. toxicity of vasoconstrictor. D. extrusion of the posterior teeth.
C. possibility of intravascular E. downward and backward mandibular
administration. * rotation.
D. possibility of paresthesia.
The occlusal parameter that is most
The pterygomaxillary fissure is formed useful to differentiate between an
by the maxilla and which other bone? overbite of dental or skeletal origin is
the
A. Temporal.
B. Sphenoid. * A. mandibular curve of Spee *
C. Frontal. B. mandibular curve of Wilson.
D. Occipital. C. molar sagittal relationship.
D. mandibular anterior lack of space.
Following radiation therapy to the E. maxillary curve of Wilson.
mandible, extraction of mandibular teeth
is most likely to result in Which of the following applies to gutta-
percha?
A. fracture.
B. actinomycosis. A. It can be thermoplasticized. *
C. osteomyelitis * B. It is a good thermal conductor.
D. soft tissue necrosis. C. It can be inserted easily into fine
E. development of malignancy. canals.
D. It is soluble in periapical exudate.
In which of the following pathological
conditions would a lower central incisor Which statement is FALSE regarding
tooth be expected to respond to heat, the use of a barbed broach?
cold and an electric pulp test?
A. Removal of vital or non-vital pulp
A. Chronic apical abscess (chronic tissue.
periradicular abscess). B. Removal of food debris from the
B. Acute apical abscess (acute canal.
periradicular abscess). C. Removal of paper points and cotton
C. Periapical osseous dysplasia pellets.
(periapical cemento-osseous dysplasia)* D. Removal of gutta-percha during non-
D. Asymptomatic apical periodontitis surgical retreatment. *
(chronic periradicular periodontitis).

An anterior bite plane will NOT result in

Dr Abdul Naser Tamim SEHA, Dr Ghada Al Aqqad D.D.S, Dr Kamal Naser - Amrita Medical Centre, Dr Emad Wani - Shadi Dental Centre,
Dr Lina Anka SEHA, Dr Rouba Zgheibi SEHA, Maria Teresa Yongson Alejandrino SEHA 326
The most appropriate way to disinfect
gutta-percha cones prior to obturation Which of the following is consistent
is to with a diagnosis of complete pulpal
necrosis?
A. immerse in a 5.25% sodium
hypochlorite solution. * A. Poorly localized spontaneous pain.
B. immerse in ethyl alcohol. B. Positive response to hot and cold
C. autoclave for a full cycle. tests.
D. wipe with an alcohol soaked gauze. C. No response to electric pulp testing*
D. Extreme pain elicited by palpation
Which of the following agents is most and percussion tests.
effective in cold testing?
Which of the following statements is
true regarding endodontically treated
A. Ice water. teeth?
B. Air jet.
C. CO2 (dry ice). * A. These teeth are more brittle than
D. Ethyl chloride. teeth with vital pulps due to
desiccation.
A thermal (hot or cold) test is used to B. Fracture of these teeth are usually
test the response of nerves in the due to loss of coronal tooth structure *
C. These teeth require full coverage to
A. pulp. * prevent fracture.
B. alveolar bone. D. A post provides strength for these
C. attached gingiva. teeth.
D. periodontal ligament.
E. mucosa. Which of the following teeth is most
likely to have two roots and two
Hyperplastic pulpitis is canals?

A. an acute condition. A. Maxillary second premolar.


B. a proliferative reaction of the pulp. * B. Mandibular second premolar.
C. frequently found in elderly patients. C. Maxillary first premolar. *
D. accompanied by severe pain. D. Mandibular first premolar.

Which permanent maxillary molar root All afferent impulses from the pulp
has a higher incidence of two canals? result in the sensation of

A. Distobuccal root of the first molar. A. heat.


B. Distobuccal root of the second B. pain. *
molar. C. proprioception.
C. Mesiobuccal root of the first molar. * D. cold.
D. Mesiobuccal root of the second
molar.

Dr Abdul Naser Tamim SEHA, Dr Ghada Al Aqqad D.D.S, Dr Kamal Naser - Amrita Medical Centre, Dr Emad Wani - Shadi Dental Centre,
Dr Lina Anka SEHA, Dr Rouba Zgheibi SEHA, Maria Teresa Yongson Alejandrino SEHA 327
Two weeks following the placement of
a restoration, a patient complains of A rubber dam should be used in
pain to hot and cold in the restored
tooth. The most likely diagnosis is A. pulp capping procedures.
B. amalgam placement.
A. galvanic shock. C. composite placement.
B. reversible pulpitis. * D. removing carious dentin from deep
C. gingival irritation. lesions.
E. all of the above. *
Dentinal pain is explained by
Which of the following statements
A. hydraulic pressure theory. is/are true when using forceps for
B. hydrodynamic theory. * extraction of a maxillary first molar?
C. mechanical deformation theory.
D. osmotic pressure theory. 1. Palatal bone is thinner than buccal
bone.
The preparation of an anterior tooth for 2. Buccal bone is easier to expand.
a metal-ceramic crown should provide 3. Forcep movement should be
principally in the palatal direction.
1. Adequate length for retention and 4. Forcep movement should be
resistance form. principally in the buccal direction.
2. Space for thickness of metal that
will resist deformation. A. (1) (2) (3)
3. Space for thickness of porcelain. B. (1) and (3)
4. A single path of insertion. C. (2) and (4) *
D. (4) only
A. (1) (2) (3) E. All of the above.
B. (1) and (3)
C. (2) and (4) A patient presents with a dislocated
D. (4) only mandible after an accident. After
E. All of the above. * reduction, the management of this
patient should be to
One week after receiving a complete
denture a patient returns with an A. inject the joint with hydrocortisone.
isolated sore spot. The most likely B. refer for joint surgery.
cause is C. advise vigorous exercise of the
mandible.
A. incorrect vertical dimension. D. recommend mandibular movement
B. localized pressure. * be minimized. *
C. an inaccurate centric relation
record.
D. decreased tissue tolerance.

Dr Abdul Naser Tamim SEHA, Dr Ghada Al Aqqad D.D.S, Dr Kamal Naser - Amrita Medical Centre, Dr Emad Wani - Shadi Dental Centre,
Dr Lina Anka SEHA, Dr Rouba Zgheibi SEHA, Maria Teresa Yongson Alejandrino SEHA 328
Oral signs and/or symptoms of vitamin
B2 (riboflavin) deficiency may include A. periodontal pocket depth.
B. the height of the bone on the facial
1. Glossitis. surfaces of the teeth.
2. Angular cheilitis. C. the extent of furcation involvements.
3. Pain. D. infrabony pocket topography.
4. Erythematous oral mucosa. E. None of the above. *

A. (1) (2) (3) An enameloma is


B. (1) and (3)
C. (2) and (4) A. an odontoma.
D. (4) only B. an ameloblastoma.
E. All of the above. * C. a pearl of enamel. *
D. an enamel hypocalcification.
The major advantage of glass
ionomer cement as a restorative A patient wearing complete dentures
material is that it is complains of tingling and numbness in
the lower lip bilaterally. This is often an
A. highly translucent. indication of
B. a fluoride releasing material. *
C. highly esthetic. A. allergy to denture base material.
D. unaffected by moisture during the B. impingement of denture on the
setting reaction. mandibular nerve.
C. defective occlusal contacts.
If the lining cement is left on the D. impingement of denture on the
gingival cavosurface margin of a mental nerve *
Class II amalgam restoration, E. neoplastic invasion of the inferior
mandibular nerve.
A. cement dissolution will lead to
leakage. * Which of the following is NOT
B. the preparation will lack retention correlated to early implant failure?
form.
C. the preparation will lack resistance A. Age of the patient. *
form to bulk fracture. B. Type III/IV bone around the implant.
D. the preparation will lack C. Poorly controlled diabetes.
appropriate outline form. D. Regular smoking habit.

A dental radiograph will accurately


indicate

Dr Abdul Naser Tamim SEHA, Dr Ghada Al Aqqad D.D.S, Dr Kamal Naser - Amrita Medical Centre, Dr Emad Wani - Shadi Dental Centre,
Dr Lina Anka SEHA, Dr Rouba Zgheibi SEHA, Maria Teresa Yongson Alejandrino SEHA 329
When light cured composite resin
restorations are polymerized A. Dental impression materials.
B. Acrylic resin for denture bases.
A. a conversion rate of 80-95% is C. Uncured composite resins.
achieved. D. Hand instruments for placing
B. oxygen improves polymerization of resins. *
the surface. E. Trituration of amalgam.
C. cross-linking increases the stiffness
of the material. * Voids on a dental cast could be
D. shrinkage decreases as conversion caused by the high
rate increases.
A. surface tension of a silicone
Hardening of Type IV cast gold dental impression material. *
alloys by heat treatment increases B. surface tension of an irreversible
hydrocolloid.
A. ductility. C. wettability of the dental stone.
B. yield strength. * D. wettability of the impression
C. coring. material.
D. elastic modulus.
E. malleability. The size of pores or cracks in a
material
The yield strength of an orthodontic
wire is A. determines a material’s fracture
toughness. *
A. the same as the proportional limit. B. influences the strength of metals
B. decreased by work hardening. more than ceramics.
C. the same as the stress at fracture. C. decreases with cyclic or fatigue
D. higher than the proportional limit* loading.
D. is not typically a function of a
Ceramics used in dentistry exhibit material’s processing.

A. greater strength in tension than The apex of the heart lies deep to the
compression.
B. a tendency for tensile fracture * A. 2nd left intercostal space.
B. 3rd left intercostal space.
C. chemical instability.
C. 5th left intercostal space.
D. high thermal coefficients of
D. 7th left intercostal space.
expansion.
E. 9th left intercostal space.
Wettability is NOT a desirable property
for which of the following?

Dr Abdul Naser Tamim SEHA, Dr Ghada Al Aqqad D.D.S, Dr Kamal Naser - Amrita Medical Centre, Dr Emad Wani - Shadi Dental Centre,
Dr Lina Anka SEHA, Dr Rouba Zgheibi SEHA, Maria Teresa Yongson Alejandrino SEHA 330
A gluteal intramuscular injection may
be safely administered in which A. asthma.
quadrant? B. emphysema.
C. rhinophyma.
A. Lower medial. D. cardiac insufficiency *
B. Upper medial.
C. Lower lateral. In determining the ideal proximal
D. Upper lateral. * outline form for a Class II amalgam
cavity preparation in a molar the
Enamel spindles are
1. Axial wall should be 1.5mm deep.
A. aberrant dentinal tubules that cross 2. Gingival cavosurface margin must
the dentinoenamel junction * clear contact with the adjacent tooth.
B. structural faults that span the entire 3. Proximal walls diverge occlusally.
thickness of the enamel. 4. Facial and lingual proximal
C. responsible for the incremental lines cavosurface margins must just clear
in enamel. contact with the adjacent tooth.
D. seen as perikymata on the surface
of newly erupted teeth. A. (1) (2) (3)
B. (1) and (3)
Which thermal property is most C. (2) and (4) *
important in selecting a restorative D. (4) only
material to protect the pulp from E. All of the above.
excessive temperature changes?
To achieve optimum strength and
A. conductivity * esthetics, a metal ceramic restoration
B. diffusivity. with a porcelain butt joint margin
C. expansion coefficient. should really have a
D. modulus.
A. 0.8 – 1.0mm shoulder reduction.
A primary molar, in the absence of its B. 0.8 – 1.0mm incisal reduction.
permanent successor, C. 90 cavosurface margin *
D. finish line that is 2mm subgingival.
A. should be treated endodontically to
prevent root resorption. Saliva is most effective in minimizing
B. may remain for years with no an acid challenge by its
significant resorption. *
C. will undergo normal root resorption. A. lubrication function.
D. should be extracted. B. antimicrobial effect.
E. is more susceptible to dental caries. C. buffering action *
D. fluoride concentration.
A patient has a history of shortness of
breath and ankle edema. You would
suspect

Dr Abdul Naser Tamim SEHA, Dr Ghada Al Aqqad D.D.S, Dr Kamal Naser - Amrita Medical Centre, Dr Emad Wani - Shadi Dental Centre,
Dr Lina Anka SEHA, Dr Rouba Zgheibi SEHA, Maria Teresa Yongson Alejandrino SEHA 331
The primary stimulus for growth of the
mandible is A child on antibiotic therapy would be
more likely to develop
1. Genetic.
2. Epigenetic. A. herpangina.
3. Functional. B. pemphigus.
4. Environmental. C. candidiasis *
D. herpetic gingivostomatitis.
A. (1) (2) (3)
B. (1) and (3) * A drug that affects alkaline
C. (2) and (4) phosphatase activity would target
D. (4) only
E. All of the above. A. muscles and bones.
B. skin and teeth.
A drug inhibiting ATP release at a site C. bones and teeth. *
of injury could be a potential analgesic D. muscles and skin.
because ATP
High telomerase activity is associated
A. inhibits nociceptors. with
B. activates nociceptors. *
C. causes vasoconstriction. A. diabetes mellitus.
D. prevents vasoconstriction. B. malignant tumours. *
C. hyperthyroidism.
The dentino-enamel junction is the D. cystic fibrosis.
most sensitive portion of a tooth
because The principal use of the rubber dam
during placement of a composite resin
A. free nerve endings terminate on restoration is to provide
odontoblasts at this region.
B. odontoblastic processes branch A. interproximal gingival retraction.
considerably at this region. * B. contrast between the tooth and the
C. ameloblasts make synaptic operating field.
connections with odontoblasts at this C. access to the operating field.
junction. D. protection from fluid contamination *
D. odontoblastic tubules help convey
hydrostatic forces to the pulp cells. A pontic should

In the formation of the tooth, Tome’s A. exert no pressure on the ridge. *


processes are responsible for B. be contoured by scraping the master
cast.
A. laying down the enamel prisms. * C. have a large surface area in contact
B. forming the odontoblastic tubules. with the ridge.
C. reorganizing the collagen fibres. D. contact nonkeratinized tissue.
D. generating the incremental lines.
E. secreting the calcospherites.

Dr Abdul Naser Tamim SEHA, Dr Ghada Al Aqqad D.D.S, Dr Kamal Naser - Amrita Medical Centre, Dr Emad Wani - Shadi Dental Centre,
Dr Lina Anka SEHA, Dr Rouba Zgheibi SEHA, Maria Teresa Yongson Alejandrino SEHA 332
When prescribing antibiotics for an
orofacial infection in a healthy elderly A. excessive etching of enamel
patient, the usual adult dose and margins.
duration of the prescription should be B. inadequate peripheral seal *
written using the following guidelines. C. excessive primer resin.
The dose is D. inadequate polymerization of primer
resin.
A. decreased by one half, duration
unchanged. A carious lesion on tooth 1.6 appears
B. decreased by one third, duration close to the pulp on the bitewing
unchanged. radiograph. A diagnosis of irreversible
C. unchanged, duration unchanged. * pulpitis can be made based on
D. increased by one third, duration
unchanged. A. proximity of the radiolucency to the
E. unchanged, duration extended by pulp.
one half. B. a lower electric pulp test reading
compared to the control.
A facebow record provides an C. the symptoms reported by the
approximation of the patient. *

A. horizontal transverse axis and During the fabrication of a removable


defines the position of the maxillary complete denture, block out and relief
cast. * is placed on the
B. horizontal transverse axis and
defines the position of the mandibular A. diagnostic cast. *
cast. B. master cast.
C. vertical axis and defines the C. refractory cast.
position of the maxillary cast. D. remount cast.
D. vertical axis and defines the
position of the mandibular cast. The single most important measure to
reduce the risk of transmitting
Absence of occlusal contacts on a organisms to patients is
provisional restoration may result in
the definitive restoration exhibiting A. use of personal protective barriers:
masks, eyewear, outerwear and
A. heavy occlusal contact * gloves.
B. no occlusal contact. B. sterilization of instruments and
C. tight proximal contacts. disinfection of the operatory.
D. open proximal contacts. C. handwashing. *?
D. introduction of single use
The most likely cause of postoperative instruments and disposables.
sensitivity following the placement of a
posterior composite resin restoration
with ideal occlusion is

Dr Abdul Naser Tamim SEHA, Dr Ghada Al Aqqad D.D.S, Dr Kamal Naser - Amrita Medical Centre, Dr Emad Wani - Shadi Dental Centre,
Dr Lina Anka SEHA, Dr Rouba Zgheibi SEHA, Maria Teresa Yongson Alejandrino SEHA 333
Unbagged sterilized instruments
A. Position of the flap after suturing.
A. can be stored for up to 24 hours if B. Extent of flap reflection.
placed in an airtight container after C. Level of plaque control. *
sterilization. D. Type of initial incision.
B. can be stored for up to 7 days if
placed in sterile bags after During periodontal disease activity, the
sterilization. loss of clinical attachment
C. can be stored for up to 1 year if
wrapped after sterilization. A. precedes alveolar bone loss. *
D. must not be stored after B. follows alveolar bone loss.
sterilization. * C. is concomitant with alveolar bone
loss.
Which of the following is the dominant
inflammatory cell type in the initial Which of the following types of bone
lesion of gingivitis? contain the insertions of the periodontal
ligament fibres?
A. Neutrophil. *
B. Plasma cell. A. Woven.
C. Macrophage. B. Bundle. *
D. Lymphocyte. C. Lamellar.
D. Cortical.
Which of the following is NOT a sign of
occlusal trauma? In a post-endodontic restoration, the
function of the post is to insure
A. Fremitus.
B. Gingival recession. * A. distribution of forces along the long
C. Widened periodontal ligament. axis.
D. Tooth migration. B. resistance of the tooth to fracture.
C. sealing of the root canal.
A 5 year old child is diagnosed with D. retention of the definitive restoration.
leukocyte adherence deficiency and is *
also affected with generalized severe
bone loss adjacent to his primary The purpose of a post and core
teeth. What is the diagnosis? restoration is to

A. Generalized aggressive A. seal the root canal treatment.


periodontitis. B. reinforce the remaining tooth
B. Generalized chronic periodontitis. structure.
C. Gingival diseases modified by C. retain the crown. *
systemic factors. D. prevent root discolouration.
D. Periodontitis as a manifestation of
systemic disease. *

Which of the following is the best


predictor for successful periodontal
flap surgery?

Dr Abdul Naser Tamim SEHA, Dr Ghada Al Aqqad D.D.S, Dr Kamal Naser - Amrita Medical Centre, Dr Emad Wani - Shadi Dental Centre,
Dr Lina Anka SEHA, Dr Rouba Zgheibi SEHA, Maria Teresa Yongson Alejandrino SEHA 334
Guided tissue regeneration surgery E. Chronic periodontitis.
selectively promotes the growth of all
of the following EXCEPT In comparing ANSI D and ANSI F
speed intra-oral radiographic films,
A. epithelial cells * ANSI F speed film emulsion is
B. endothelial cells. approximately
C. osteoblasts.
D. cementoblasts. A. 20% more sensitive than ANSI D
speed film.
A patient fails to demonstrate effective B. 40% more sensitive than ANSI D
plaque control during initial speed film.
periodontal therapy for moderate C. 60% more sensitive than ANSI D
periodontitis. The most appropriate speed film. *
management is D. 80% more sensitive than ANSI D
speed film.
A. continued initial therapy. *
B. gingival curettage. When compared to dental amalgams
C. gingivectomy. made from lathe cut particles, dental
D. an apically positioned flap. amalgams made from spherical
particles
In gingivitis, the sulcular epithelium
has the following characteristics A. require more mercury.
EXCEPT it B. set more quickly. *
C. are more difficult to adapt to the
A. is a barrier to bacterial invasion. * cavity preparation.
B. is permeable to bacterial enzymes D. require higher condensation forces.
and toxins.
C. may be ulcerated. The bond between porcelain and metal
D. undergoes both degenerative and in a ceramometal (porcelain bonded to
proliferative changes. metal) crown is

Which has the WORST prognosis? A. chemical. *


B. mechanical.
A. Occlusal traumatism. C. equally chemical and mechanical.
B. Gingivitis. D. neither chemical nor mechanical.
C. Aggressive periodontitis. *
D. Periodontal atrophy.

Dr Abdul Naser Tamim SEHA, Dr Ghada Al Aqqad D.D.S, Dr Kamal Naser - Amrita Medical Centre, Dr Emad Wani - Shadi Dental Centre,
Dr Lina Anka SEHA, Dr Rouba Zgheibi SEHA, Maria Teresa Yongson Alejandrino SEHA 335
An altered cast impression technique
for free-end extension mandibular A rubber dam should be used in
partial denture cases is done primarily
to A. pulp capping procedures.
B. amalgam placement.
1. Capture soft tissue in a supporting C. composite placement.
form. D. removing carious dentin from deep
2. Capture the retromylohyoid area. lesions.
3. Prevent displacement of the E. all of the above. *
retromolar pad.
4. Allow jaw relation records to be In periodontal flap surgery, the initial
made simultaneously with impression incision is made to
making. A. expose the sulcular lining of the
pocket. *
A. (1) (2) (3) * B. aid in healing.
B. (1) and (3) C. sever the attachment of the oblique
C. (2) and (4) fibres of the periodontal ligament.
D. (4) only D. excise the keratinized gingiva.
E. All of the above.
Which of the following medications
For an acid-etched Class III composite increases a patient’s risk for intraoral
resin, the cavosurface margin of the candidiasis?
cavity can be bevelled to
A. Warfarine (Coumadin®).
1. Eliminate the need for internal B. Cyclosporine. *
retention. C. Pentobarbital.
2. Improve convenience form. D. Ibuprofen.
3. Aid in finishing. E. Pilocarpine.
4. Increase the surface area for
etching. Dental implants are
CONTRAINDICATED in patients who
A. (1) (2) (3)
B. (1) and (3) 1. Are over age 80.
C. (2) and (4) 2. Have unrepaired cleft palates.
D. (4) only * 3. Are taking anticoagulants.
E. All of the above. 4. Have uncontrolled diabetes
mellitus.
For a mandibular denture impression,
the muscle determining the form of the A. (1) (2) (3)
lingual flange in the molar region is B. (1) and (3)
C. (2) and (4)
A. mylohyoid. * D. (4) only *
B. geniohyoid. E. All of the above.
C. medial pterygoid.
D. lateral pterygoid.
E. genioglossus.

Dr Abdul Naser Tamim SEHA, Dr Ghada Al Aqqad D.D.S, Dr Kamal Naser - Amrita Medical Centre, Dr Emad Wani - Shadi Dental Centre,
Dr Lina Anka SEHA, Dr Rouba Zgheibi SEHA, Maria Teresa Yongson Alejandrino SEHA 336
A patient is currently on warfarin. crowns, cervical constriction and
Before a planned extraction of tooth obliterated pulp canals and chambers?
3.4, the patient’s coagulation
mechanism should be evaluated A. Amelogenesis imperfecta.
using which test? B. Dentinogenesis imperfecta. *
C. Dentin dysplasia type I.
A. Bleeding time. D. Dentin dysplasia type II.
B. Partial thromboplastin time.
C. Prothrombin time or INR. * Which radiograph best depicts the
D. Von Willebrand’s Factor. buccal cortex of the mandible?

All of the following are parts of the A. Bite-wing.


temporal bone EXCEPT the B. Periapical.
C. Panoramic.
A. mastoid. D. Occlusal. *
B. Hamulus. *
C. tympanic. During radiographic film processing,
D. zygomatic. silver halide is removed from the
emulsion during the
Which of the following conditions is
most likely to result in new periosteal A. developing stage.
bone formation? B. post-developing rinse stage.
C. fixing stage. *
A. Fibrous dysplasia. D. post-fixing wash stage.
B. Paget’s disease of bone. *
In comparison to visible light, X-rays
C. Chronic osteomyelitis.
D. Hyperparathyroidism. A. have a longer wave length.
B. have higher energy. *
The purpose of phenidone in
C. travel faster.
radiographic developing solution is to
chemically D. can be focused.

A ghost-like opaque image in a


A. oxidize silver halide to metallic
panoramic radiograph caused by a
silver in the emulsion.
metal earring worn in the lobe of the left
B. reduce silver halide to metallic
ear will be superimposed over the
silver in the emulsion. *
C. remove silver halide from the
A. left mandibular ramus.
emulsion that has been exposed to
B. right mandibular ramus.
radiation.
C. left posterior maxilla.
D. remove silver halide from the
emulsion that has not been exposed D. right posterior maxilla. *
to radiation.

Which disorder presents with all


permanent teeth exhibiting bulbous

Dr Abdul Naser Tamim SEHA, Dr Ghada Al Aqqad D.D.S, Dr Kamal Naser - Amrita Medical Centre, Dr Emad Wani - Shadi Dental Centre,
Dr Lina Anka SEHA, Dr Rouba Zgheibi SEHA, Maria Teresa Yongson Alejandrino SEHA 337
The most appropriate antibiotic for a A clinical sign of unilateral fracture of
dental patient with a history of the body of the zygoma is
penicillin allergy is
A. cerebrospinal rhinorrhea.
A. methicillin. B. impaired hearing.
B. clindamycin * C. subconjunctival haemorrhage*
C. streptomycin. D. otorrhea.
D. tetracycline.
Which of the following is most likely to
To ensure maximum marginal displace the adjacent teeth?
strength for an amalgam restoration
the cavosurface angle should A. Lateral periodontal cyst.
B. Dentigerous cyst. *
A. approach 45 degrees. C. Periapical cemental dysplasia.
B. approach 90 degrees. * D. Periapical abscess.
C. be beveled. E. Radicular cyst.
D. be chamfered.
The roots of the first permanent molar
Which of the following principle fibre should be completely formed by the
groups of the periodontal ligament is age of
the most numerous and provides the
main support for the tooth? A. six years.
B. seven years.
A. Horizontal. C. nine years.
B. Transseptal. D. eleven years. *
C. Oblique. * E. thirteen years.
D. Gingival.
Xerostomia can result from
The tooth preparation for a porcelain
veneer must have a 1. Sjögren’s syndrome.
2. Radiation therapy for oral cancer.
1. Coarse diamond finish.
2. Space for the veneer material. 3. Antidepressant drug therapy.
3. Margin at least 1mm 4. Anticholinergics (Atropine).
supragingivally.
4. Definite gingival finish line. A. (1) (2) (3)
B. (1) and (3)
A. (1) (2) (3) C. (2) and (4)
B. (1) and (3) D. (4) only
C. (2) and (4) * E. All of the above. *
D. (4) only
E. All of the above.

Dr Abdul Naser Tamim SEHA, Dr Ghada Al Aqqad D.D.S, Dr Kamal Naser - Amrita Medical Centre, Dr Emad Wani - Shadi Dental Centre,
Dr Lina Anka SEHA, Dr Rouba Zgheibi SEHA, Maria Teresa Yongson Alejandrino SEHA 338
In alginate impression materials, The desirable relationship between
sodium phosphate (Na3PO4) is the the coefficients of thermal expansion
of an alloy (ε metal ) and a ceramic
A. reactor. (εceramic) used for a metallo-
B. catalyst. ceramic restoration is
C. retarder. *
D. disinfectant. A. ε metal significantly higher than
E. cross linking agent. εceramic.
B. ε metal significantly lower than
Light-cured dental composites set εceramic.
when exposed to light. Light is the C. ε metal equal to εceramic.
D. ε metal slightly higher than
A. initiator. εceramic. *
E. ε metal slightly lower than
B. reactor.
εceramic.
C. catalyst.
D. activator. * Hydrocolloid impressions are
E. terminator. removed from the mouth with a snap
because they exhibit
Zinc oxide eugenol cement is a/an
A. syneresis.
A. phosphate cement. B. imbibition.
B. phenolic cement. * C. viscoelasticity. *
C. resin modified glass ionomer D. low elastic recovery.
cement.
E. low tear strength.
D. polyalkenoic acid cement.
E. adhesive resin cement. When compared with admixed
amalgams, spherical amalgams
The type of amalgam requiring the
least volume of mercury for its A. require less condensation
setting reaction is pressure. *
B. require shorter trituration time.
A. low copper.
C. are stronger (24 hour
B. high copper. * compressive strength).
C. admixed. D. have better resistance to
D. lathe cut. marginal fracture.
E. spherical. E. tarnish more.

Dr Abdul Naser Tamim SEHA, Dr Ghada Al Aqqad D.D.S, Dr Kamal Naser - Amrita Medical Centre, Dr Emad Wani - Shadi Dental Centre,
Dr Lina Anka SEHA, Dr Rouba Zgheibi SEHA, Maria Teresa Yongson Alejandrino SEHA 339
A vital canine is to be used as the A. ameloblasts migrate apically down
anterior abutment of a four unit fixed the root.
partial denture and it has 2.0mm B. cells of the epithelial root sheath do
remaining coronal tooth structure. The not migrate away from the dentin. *
most acceptable foundation C. cells of the dental follicle fail to
restoration would be develop.
D. epithelial rests transform into
A. bonded amalgam core build-up. ameloblast vesicles.
B. a pin retained amalgam core build-
up. Cementicles
C. a pin retained composite resin core
build-up. A. contain pulp tissue.
D. intentional devitalization followed B. has the same composition as
by a post and core restoration. * cementum. *
C. are a response to trauma.
For which of the following is nystatin D. develop from excess dental follicle
oral suspension an appropriate cells.
treatment?
A. Herpetic gingivostomatitis. Which of the following should be
B. Nicotinic stomatitis. performed to confirm a diagnosis of
C. Denture stomatitis. * pseudomembranous candidiasis?
D. Aphthous stomatitis.
A. An incisional biopsy.
The small bubble normally seen in a B. A cytological smear. *
local anesthetic cartridge is C. An excisional biopsy.
D. A complete blood count.
A. nitrogen. *
B. air. A patient with Alzheimer’s dementia
C. oxygen. presents with his personal care
D. a breakdown product. worker. His daughter, who is his legal
guardian, is unavailable. What is
A 50 year old patient, who is a heavy required to obtain informed consent for
smoker, has developed a barrel chest, an elective invasive procedure?
has difficulty breathing and has a
bluish tinge to his complexion. The A. The patient’s presence implies
most likely diagnosis is consent.
B. Obtain written consent from the
A. emphysema. * patient.
B. acute upper respiratory infection. C. Obtain written consent from the
C. primary cancer of the lung. personal care worker.
D. cardiac insufficiency. D. Obtain consent from the patient’s
daughter. *
Enamel pearls form when

Dr Abdul Naser Tamim SEHA, Dr Ghada Al Aqqad D.D.S, Dr Kamal Naser - Amrita Medical Centre, Dr Emad Wani - Shadi Dental Centre,
Dr Lina Anka SEHA, Dr Rouba Zgheibi SEHA, Maria Teresa Yongson Alejandrino SEHA 340
A bite-wing radiograph of tooth 1.4 A. control polymerization shrinkage in
reveals caries penetrating one third composite resins.
into the mesial enamel. The most B. enhance the bond between a
appropriate management of tooth 1.4 porcelain veneer and the resin cement*
is to C. reduce the surface tension when
investing a wax pattern.
A. place an amalgam restoration. D. facilitate the soldering of gold
B. place a porcelain inlay. castings.
C. place a direct composite
restoration. The primary retention of a Class II gold
D. apply fluoride and improve oral inlay is achieved by
hygiene. *
1. Adding an occlusal dovetail.
In order to achieve a proper 2. Increasing the parallelism of walls.
interproximal contact when using a 3. Lengthening the axial walls.
spherical alloy, which of the following 4. Placing a gingival bevel.
is/are essential?
A. (1) (2) (3) *
1. A larger sized condenser. B. (1) and (3)
2. A thinner matrix band. C. (2) and (4)
3. A properly placed wedge. D. (4) only
4. Use of mechanical condensation. E. All of the above.

A. (1) (2) (3) * Possible side effects of therapeutic


B. (1) and (3) doses of codeine are
C. (2) and (4)
D. (4) only 1. Constipation.
E. All of the above. 2. Drowsiness.
3. Nausea.
The tooth preparation for a porcelain 4. Respiratory depression.
veneer must have a
A. (1) (2) (3 *
1. Rough surface. B. (1) and (3)
2. Space for the veneer material. C. (2) and (4)
3. Definite finish line. D. (4) only
4. Margin at least 1mm E. All of the above. ?
supragingivally.
The primary stress bearing area of the
A. (1) (2) (3) * maxillary complete denture is the
B. (1) and (3)
C. (2) and (4) A. hard palate.
D. (4) only B. alveolar ridge. *
E. All of the above. C. soft palate.
D. zygoma.
A silane coupling agent is used to

Dr Abdul Naser Tamim SEHA, Dr Ghada Al Aqqad D.D.S, Dr Kamal Naser - Amrita Medical Centre, Dr Emad Wani - Shadi Dental Centre,
Dr Lina Anka SEHA, Dr Rouba Zgheibi SEHA, Maria Teresa Yongson Alejandrino SEHA 341
Xerostomia can be A. Place amalgam restorations over
the next few months.
1. Found in the elderly. B. Excavate caries and place
2. Drug induced. temporary restorations within the next
3. Associated with diabetes. few weeks. *
4. Predisposing to dental diseases. C. Delay any treatment until the
hygiene improves.
A. (1) (2) (3) D. Restore all teeth with composite
B. (1) and (3) resin over the next few months.
C. (2) and (4)
D. (4) only In Angle's classification, a Class II,
E. All of the above. * division 1 malocclusion may describe
a combination of skeletal problems,
Root resorption of permanent teeth such as
may be associated with
1. Maxillary protrusion, normal
1. Excessive orthodontic forces. mandibular position.
2. Periapical granuloma. 2. Normal maxillary position,
3. Traumatic injury. mandibular retrusion.
4. Periapical osseous dysplasia 3. Maxillary protrusion, mandibular
(periapical cemento-osseous retrusion.
dysplasia). 4. Bimaxillary protrusion, with the
maxilla more protrusive than the
A. (1) (2) (3) * mandible.
B. (1) and (3)
A. (1) (2) (3)
C. (2) and (4)
B. (1) and (3)
D. (4) only
C. (2) and (4)
E. All of the above.
D. (4) only
Which of the following clinical E. All of the above. *
conditions is the most serious?
Displacement of fractures is
A. Acute periapical abscess of a influenced by
mandibular central incisor.
B. Middle face cellulitis. * 1. Direction of the blow.
C. Chronic periapical abscess of a 2. Muscle attachments.
mandibular third molar. 3. Direction of fracture line.
D. Infected dentigerous cyst. 4. Hemorrhage.

After initiating preventive A. (1) (2) (3) *


management for a 16 year old patient B. (1) and (3)
with multiple extensive carious C. (2) and (4)
lesions, which of the following D. (4) only
restorative treatments is most E. All of the above.
appropriate?

Dr Abdul Naser Tamim SEHA, Dr Ghada Al Aqqad D.D.S, Dr Kamal Naser - Amrita Medical Centre, Dr Emad Wani - Shadi Dental Centre,
Dr Lina Anka SEHA, Dr Rouba Zgheibi SEHA, Maria Teresa Yongson Alejandrino SEHA 342
The principles of closed fracture Alginate impression material
management are
A. is a reversible hydrocolloid.
1. Reduction of fracture. B. sets by condensation
2. Immobilization of fracture. polymerization.
3. Restoration of occlusion. C. is a rigid material.
4. Incision and debridement at fracture D. is an irreversible material. *
site. E. is a thermoplastic material.
A. (1) (2) (3) * For an acid-etched Class III composite
B. (1) and (3) resin, the cavosurface margin of the
C. (2) and (4) cavity can be bevelled to
D. (4) only
E. All of the above. A. eliminate the need for internal
retention.
Alteration of the intestinal flora by B. improve convenience form.
some chemotherapeutic agents can C. aid in finishing.
interfere with reabsorption of a D. increase the surface area for
contraceptive steroid thus preventing etching. *
the recirculation of the drug through
the enterohepatic circulation. Which of Amalgam is condensed to
the following can interfere with this
mechanism? 1. Further break down the alloy
particles.
1. Codeine. 2. Force the alloy particles together.
2. Penicillin V. 3. Complete the trituration process.
3. Acetaminophen. 4. Express excess mercury content.
4. Tetracycline.
A. (1) (2) (3)
A. (1) (2) (3) B. (1) and (3)
B. (1) and (3) C. (2) and (4) *
C. (2) and (4) * D. (4) only
D. (4) only E. All of the above.
E. All of the above.
The primary stimulus for growth of the
The primary purpose(s) of relining a mandible is
distal extension base of a removable
cast framework partial denture is/are to 1. Genetic.
improve
2. Epigenetic.
3. Functional.
1. Fit of the framework.
4. Environmental.
2. Occlusion.
3. Function. A. (1) (2) (3)
4. Tissue adaptation. B. (1) and (3)
C. (2) and (4) *
A. (1) (2) (3)
D. (4) only
B. (1) and (3)
E. All of the above.
C. (2) and (4)
D. (4) only *
E. All of the above.

Dr Abdul Naser Tamim SEHA, Dr Ghada Al Aqqad D.D.S, Dr Kamal Naser - Amrita Medical Centre, Dr Emad Wani - Shadi Dental Centre,
Dr Lina Anka SEHA, Dr Rouba Zgheibi SEHA, Maria Teresa Yongson Alejandrino SEHA 343
An Angle Class II dental malocclusion
in the mixed dentition will most likely Proper collimation of the useful beam
for the film size and target-film distance
A. develop into an Angle Class I will reduce
malocclusion with normal exfoliation of
the primary molars. * 1. Image definition.
B. worsen with forward growth of the 2. Secondary radiation.
maxilla. 3. Radiographic contrast.
C. develop into an Angle Class I 4. Radiation received by patient.
malocclusion with late mandibular
growth. A. (1) (2) (3)
D. develop into a skeletal B. (1) and (3)
malocclusion with growth of the C. (2) and (4) *
maxilla and mandible. D. (4) only
E. not change as the maxilla and E. All of the above.
mandible grow.
Which conditions are associated with
The amount of radiation to a patient Acquired Immunodeficiency Syndrome
can be reduced by (AIDS)?

1. Using a high speed film. 1. Acute marginal periodontitis.


2. Using an aluminum filter. 2. Hairy leukoplakia.
3. Increasing target-film distance. 3. Candidiasis.
4. Using low kVp.
4. Geographic tongue.
A. (1) (2) (3) *
A. (1) (2) (3) *
B. (1) and (3)
B. (1) and (3)
C. (2) and (4)
C. (2) and (4)
D. (4) only
D. (4) only
E. All of the above.
E. All of the above.
In x-ray equipment, kilovoltage
The primary use of nitrous oxide and
controls
oxygen in dentistry today is as a(n)
1. Contrast.
A. substitute agent for local
2. Speed of electrons. anesthesia.
3. Penetrating power of radiation. B. general anesthetic agent.
4. Amount of radiation produced. C. agent for conscious sedation. *
D. agent for the management of
A. (1) (2) (3) *
chronic obstructive pulmonary disease.
B. (1) and (3)
C. (1) and (4)
D. (4) only
E. All of the above.

Dr Abdul Naser Tamim SEHA, Dr Ghada Al Aqqad D.D.S, Dr Kamal Naser - Amrita Medical Centre, Dr Emad Wani - Shadi Dental Centre,
Dr Lina Anka SEHA, Dr Rouba Zgheibi SEHA, Maria Teresa Yongson Alejandrino SEHA 344
Amphetamines D. a deep fungal infection.
E. tuberculosis.
1. Increase mental alertness.
2. Increase salivation. In mucous membrane pemphigoid, a
3. Decrease fatigue. positive Nikolsky sign is the result of
4. are useful in controlling a/an
arrhythmias.
A. separation at the basement
A. (1) (2) (3) membrane *
B. (1) and (3) * B. intraepithelial separation.
C. (2) and (4) C. separation of the lamina propria
D. (4) only and submucosal.
E. All of the above. D. intraepithelial bulla formation.

The physical properties of alginate The direct immunofluorescence


impression materials will be adversely pattern seen in pemphigus vulgaris
affected by has been described as

A. "tumbling" the alginate container A. target-like.


prior to filling the dispensing scoop. B. chicken wire *
B. adding powder to the water in the C. soap-bubble.
mixing bowl. D. corrugated.
C. using room temperature water. E. cotton wool.
D. mixing beyond the recommended
time * In general, a pigmented macule
measuring 4mm in diameter should be
Overhangs on restorations predispose treated by

1. enhanced plaque retention. A. incisional biopsy.


2. restricted plaque removal. B. excisional biopsy *
3. enhanced food retraction. C. exfoliative cytology.
4. increased caries susceptibility. D. carbon dioxide laser.
E. electrocautery.
A. (1) (2) (3)
B. (1) and (3) All of the following are common
C. (2) and (4) features of a malignant neoplasm of
D. (4) only the jaws EXCEPT
E. All of the above. *
A. moth-eaten radiolucencies.
The most likely diagnosis of a non- B. irregular radiopacities.
healing indurated ulcer on the lateral C. widening of periodontal ligament
border of the tongue in a 60 year old spaces.
patient is D. well-circumscribed periapical
radiolucencies *
A. a traumatic ulcer. E. paresthesia and tooth loosening.
B. major aphthous ulcer.
C. squamous cell carcinoma *

Dr Abdul Naser Tamim SEHA, Dr Ghada Al Aqqad D.D.S, Dr Kamal Naser - Amrita Medical Centre, Dr Emad Wani - Shadi Dental Centre,
Dr Lina Anka SEHA, Dr Rouba Zgheibi SEHA, Maria Teresa Yongson Alejandrino SEHA 345
The test(s) diagnostic for primary
hyperparathyroidism in a patient with A patient with pain, fever and unilateral
multiple brown tumours is/are parotid swelling following a general
anesthetic most likely has
A. parathyroid biopsy.
B. multiple jaw biopsies. A. Mumps.
C. radiographic skeletal survey. B. sialolithiasis.
D. serum calcium and PTH level* C. acute bacterial sialadenitis *
E. creatine clearance and BUN. D. Sjögren’s syndrome.
E. sarcoidosis.
Multiple osteomas and supernumerary
teeth may be associated with The most common location of a lateral
periodontal cyst is in the area of the
A. Gorlin’s syndrome.
B. Rubenstein – Taybi syndrome. A. maxillary incisors.
C. Gardner’s syndrome. * B. maxillary molars.
D. Cleidocranial dysplasia. C. mandibular premolars *
E. Ectodermal dysplasia. D. mandibular molars.
Which of the following diseases may All of the following are well
cause an enlargement of the jaws, documented initiating factors of hairy
development of diastemas and/or a tongue EXCEPT
poorly fitting denture?
A. candidiasis.
A. Phantom bone disease. B. mouth rinses.
B. Rickets. C. antibiotics.
C. Paget’s disease. * D. systemic corticosteroids
D. Osteoporosis. (Prednisone)*
E. Hypophosphatasia. E. radiotherapy to the head and neck.
The inherited defect of teeth that may Trauma from occlusion may
be associated with osteogenesis
imperfecta is A. initiate marginal gingivitis.
B. affect the blood supply to gingiva.
A. amelogenesis imperfecta. C. initiate periodontitis.
B. dentin dysplasia. D. affect the progression of
C. dentinogenesis imperfecta * periodontitis. *
D. taurodontism.
E. regional odontodysplasia.

Dr Abdul Naser Tamim SEHA, Dr Ghada Al Aqqad D.D.S, Dr Kamal Naser - Amrita Medical Centre, Dr Emad Wani - Shadi Dental Centre,
Dr Lina Anka SEHA, Dr Rouba Zgheibi SEHA, Maria Teresa Yongson Alejandrino SEHA 346
Corticosteroids may be used for the
management of The vibrating line of the palate is

1. Allergy. 1. Always on the hard palate.


2. Arthritis. 2. An area which marks the movement of the
3. Asthma. soft palate.
4. Addison's disease. 3. Easily located on a cast.
4. A useful landmark in complete denture
A. (1) (2) (3) fabrication.
B. (1) and (3)
C. (2) and (4) A. (1) (2) (3)
D. (4) only B. (1) and (3)
E. All of the above. * C. (2) and (4) *
D. (4) only
Which of the following is/are essential when E. All of the above.
using a spherical rather than an admix alloy for
a routine amalgam restoration?

1. A larger diameter condenser tip. Doubling the diameter of a round stainless steel
2. An anatomical wedge. orthodontic wire decreases its springiness how
3. A thinner matrix band. many times?
4. A serrated condenser tip.
A. 4.
A. (1) (2) (3) * B. 8.
B. (1) and (3) C. 12.
C. (2) and (4) D. 16. *
D. (4) only
E. All of the above.

A 7 year old patient is missing tooth 5.5 and tooth 7.5. Space maintainers were not placed. A current
mixed dentition analysis yields the following data:
R L
-5mm 3mm
-3mm 4mm

The actual space loss is


A. 4.5mm in the mandible.
B. 7.5mm in the mandible.
C. 8mm in the maxilla.
D. 2mm in the maxilla *

Dr Abdul Naser Tamim SEHA, Dr Ghada Al Aqqad D.D.S, Dr Kamal Naser - Amrita Medical Centre, Dr Emad Wani - Shadi Dental Centre,
Dr Lina Anka SEHA, Dr Rouba Zgheibi SEHA, Maria Teresa Yongson Alejandrino SEHA 347
What is the earliest age that the An angina attack in a patient with a
diagnosis of a congenitally missing known cardiac problem immediately
mandibular second bicuspid can be after a tooth extraction was most
confirmed? probably precipitated by
A. 2 years. A. a sudden repositioning of the
B. 4 years. * patient into an upright position.
C. 6 years. B. a delayed allergic reaction to the
D. 8 years. anesthetic agent.
C. the stress resulting from the
A survey of the master cast shows treatment. *
that the 3.5 and 3.7 abutments for a D. the use of a vasoconstrictor in the
fixed partial denture have different anesthetic agent.
paths of insertion with respect to 3.7.
A semi-precision attachment is After many caries free years a 78 year
chosen rather than preparing the teeth old patient develops multiple root
again. Where should the male part of surface caries. This is most likely the
the attachment ideally be located? result of
A. Distal of the 3.5 retainer. A. changes in cementum composition.
B. Distal of the 3.6 pontic. B. exposure of the cementoenamel
C. Mesial of the 3.7 retainer. junctions.
D. Mesial of the 3.6 pontic. * C. decreased salivary flow. *
D. changes in dietary pattern.
A fracture in an all ceramic crown may
be caused by Which of the following has analgesic,
1. Inadequate ceramic thickness. antipyretic and anti-inflammatory
2. Sharp line angles in the tooth effects?
preparation. A. Acetaminophen.
3. Excessive occlusal load. B. Acetylsalicylic acid. *
4. Use of an inappropriate luting C. Bradykinin.
material. D. Diazepam.

A. (1) (2) (3) An unerupted supernumerary tooth


B. (1) and (3) between the permanent maxillary
C. (2) and (4) central incisors is commonly
D. (4) only associated with which of the following
E. All of the above. * clinical observations?
A. Dental diastema. *
The angle SNA can be used to B. Hypertrophied labial frenum.
evaluate the C. Delayed exfoliation of primary
A. maxillary protrusion. * maxillary lateral incisors.
B. overbite. D. Absence of permanent maxillary
C. upper incisor inclination. lateral incisors.
D. facial height.
E. mandibular angle.

Dr Abdul Naser Tamim SEHA, Dr Ghada Al Aqqad D.D.S, Dr Kamal Naser - Amrita Medical Centre, Dr Emad Wani - Shadi Dental Centre,
Dr Lina Anka SEHA, Dr Rouba Zgheibi SEHA, Maria Teresa Yongson Alejandrino SEHA 348
The redness of inflamed gingiva is due 2. A moist retraction cord.
to 3. A copper band that is removed
A. the degree of keratinization. when the impression is made.
B. subgingival deposits. 4. Using a heavy body material in the
C. increased collagen fiber density. impression tray.
D. increased vasodilation. *
A. (1) (2) (3) *
Healthy attached gingiva B. (1) and (3)
A. has no basal cell layer. C. (2) and (4)
B. is closely bound to underlying D. (4) only
periosteum.* E. All of the above.
C. contains elastic fibers.
D. has no rete pegs. If a polyvinyl siloxane material is used
to make the final impression for a
Aging pulps show a relative increase maxillary cast restoration, the
in impression tray must
1. Fibrous elements. 1. be rigid.
2. Cell numbers. 2. Have occlusal stops.
3. Calcification. 3. Be coated with an appropriate
4. Vascularity. adhesive.
4. Cover the hard palate.
A. (1) (2) (3)
B. (1) and (3) * A. (1) (2) (3) *
C. (2) and (4) B. (1) and (3)
D. (4) only C. (2) and (4)
E. All of the above. D. (4) only
E. All of the above.
Which of the following could cause
phonetic problems for patients with
removable dentures? A Bolton relationship has determined
1. Posterior teeth placed in a buccal a
position. maxillary “12” excess of 3.5mm
2. Excessive bulk in the palatal area. maxillary “6” excess of 3.0mm
3. Anterior teeth that are too long.
4. Advanced ridge resorption. What effect(s) could this Bolton
relationship have on a Class I
A. (1) (2) (3) * malocclusion?
B. (1) and (3) 1. Deeper overbite.
C. (2) and (4) 2. Maxillary crowding.
D. (4) only 3. Reduced overjet.
E. All of the above. 4. Increased overjet.

Which of the following is/are (an) A. (1) (2) (3)


acceptable mean(s) to obtain the B. (1) and (3)
gingival retraction that is needed when C. (2) and (4) *???? check
using an elastomeric impression D. (4) only
material for making a final impression E. All of the above
in fixed prosthodontics?
1. Electrosurgery.

Dr Abdul Naser Tamim SEHA, Dr Ghada Al Aqqad D.D.S, Dr Kamal Naser - Amrita Medical Centre, Dr Emad Wani - Shadi Dental Centre,
Dr Lina Anka SEHA, Dr Rouba Zgheibi SEHA, Maria Teresa Yongson Alejandrino SEHA 349
Which of the following can increase the The survey of the diagnostic cast that is done
chances of successful osseointegration of a as part of a removable partial denture design
dental implant? procedure is needed to
1. An atraumatic surgical approach. 1. Determine the path of insertion of the
2. The availability of dense cancellous bone. removable partial denture.
3. A good initial stability of the implant. 2. Evaluate the retention potential of the
4. Immediate loading of the implant. abutment teeth.
3. Evaluate the modifications needed to create
A. (1) (2) (3) * parallel guiding planes.
B. (1) and (3) 4. Locate the height of contour abutment teeth.
C. (2) and (4)
D. (4) only A. (1) (2) (3)
E. All of the above. B. (1) and (3)
C. (2) and (4)
Which of the following could cause clicking D. (4) only
sounds during speech in denture wearers? E. All of the above. *
1. A nonbalanced occlusion.
2. An excessive occlusal vertical dimension. When using a zinc phosphate cement to lute a
3. A reduced horizontal overlap. full crown, it is recommended to
4. A lack of denture retention. A. mix the cement on a waxed paper pad.
B. leave the tooth moist but not wet.
A. (1) (2) (3) C. apply continuous occlusal loading while the
B. (1) and (3) cement sets. *
C. (2) and (4) * D. remove any excess before the cement is set.
D. (4) only
E. All of the above Gingival enlargement may result from the
administration of
1. Nifedipine.
The best way to protect the abutments of a
Class I removable partial denture from the 2. Cyclosporine.
negative effects of the additional load applied to 3. Phenytoin sodium.
them is by 4. Prednisolone.
A. splinting abutments with adjacent teeth.
B. keeping a light occlusion on the distal A. (1) (2) (3) *
extensions. B. (1) and (3)
C. placing distal rests on distal abutments. C. (2) and (4)
D. using cast clasps on distal abutments. D. (4) only
E. regular relining of the distal extensions. * E. All of the

Dr Abdul Naser Tamim SEHA, Dr Ghada Al Aqqad D.D.S, Dr Kamal Naser - Amrita Medical Centre, Dr Emad Wani - Shadi Dental Centre,
Dr Lina Anka SEHA, Dr Rouba Zgheibi SEHA, Maria Teresa Yongson Alejandrino SEHA 350
The risk of latex allergy increases with 3. Incorrect horizontal angulation of the
a/an x-ray beam *
4. Patient movement during the
1. Increased exposure to latex. exposure.
2. History of spina bifida.
3. History of allergy to bananas, A. (1) (2) (3)
chestnuts or avocado. B. (1) and (3) *
4. History of eczema. C. (2) and (4)
D. (4) only
A. (1) (2) (3) E. All of the above.
B. (1) and (3)
C. (2) and (4) Which of the following is necessary for
D. (4) only collagen formation?
E. All of the above. *
A. Vitamin A.
The washing of hands must be B. Vitamin C. *
performed before putting on and after C. Vitamin D.
removing gloves because it D. Vitamin E.
E. Vitamin K.
1. Reduces the number of skin
bacteria which multiply and cause Which of the following is a possible
irritation. cause for a low density radiograph
2. Completely eliminates skin bacteria. (light film)?
3. Minimizes the transient bacteria
which could contaminate hands A. Cold developer. *
through small pinholes. B. Over exposure.
4. Allows gloves to slide on easier C. Improper safety light.
when the hands are moist. D. Excessive developing time.
A. (1) (2) (3) A patient presents with hypodontia,
B. (1) and (3) * conical teeth, fine, scanty, fair hair, and
C. (2) and (4) an intolerance to hot weather. The
D. (4) only most likely diagnosis is
E. All of the above.
A. achondroplasia.
Overlapping contacts on a bitewing B. malignant hyperthermia.
radiograph result from C. ectodermal dysplasia *
D. cystic fibrosis.
1. Malalignment of teeth.
2. Incorrect vertical angulation of the x-
ray beam.

Dr Abdul Naser Tamim SEHA, Dr Ghada Al Aqqad D.D.S, Dr Kamal Naser - Amrita Medical Centre, Dr Emad Wani - Shadi Dental Centre,
Dr Lina Anka SEHA, Dr Rouba Zgheibi SEHA, Maria Teresa Yongson Alejandrino SEHA 351
A bitewing radiograph of an early 1. Medial pterygoid muscles.
mixed dentition should include the 2. Masseter muscles.
following proximal surfaces. 3. Lateral pterygoid muscles.
4. Digastric muscles.
A. Distal of the primary canine to distal
of the permanent first molar. * A. (1) (2) (3) *
B. Distal of the primary canine to B. (1) and (3)
mesial of the permanent first molar. C. (2) and (4)
C. Mesial of the primary first molar to D. (4) only
mesial of the permanent first molar. E. All of the above.
D. Mesial of the primary first molar to
distal of the permanent first molar. Which of the following are possible
causes of Bell’s Palsy?
The most appropriate time to remove a
supernumerary tooth that is disturbing 1. Incorrect injection technique.
the eruption of a permanent tooth is 2. Viral infection.
3. Inflammation of the facial nerve.
A. as soon as possible * 4. Surgical trauma.
B. after two thirds of the permanent
root has formed. A. (1) (2) (3)
C. after the apex of the permanent root B. (1) and (3)
has closed. C. (2) and (4)
D. after the crown appears calcified D. (4) only
radiographically.
E. All of the above. *
A full coverage all-ceramic anterior
Which of the following statements
crown requires
concerning the airway is correct?
A. a sloping shoulder (long bevel)
A. Sympathetic fibres constrict the
margin.
bronchioles.
B. a minimum margin depth of 0.5mm
B. The trachea is membranous
lingually.
posteriorly to accommodate the
C. rounded internal line angles * pulsations of the aorta.
D. a minimum incisal reduction of C. The right primary bronchus forms
1.0mm. two secondary bronchi.
D. An aspirated foreign body would
During a normal chewing cycle, which
likely fall into the right primary
of the following has/have maximum
bronchus. *
EMG activity when the teeth are in
maximum intercuspation?

Dr Abdul Naser Tamim SEHA, Dr Ghada Al Aqqad D.D.S, Dr Kamal Naser - Amrita Medical Centre, Dr Emad Wani - Shadi Dental Centre,
Dr Lina Anka SEHA, Dr Rouba Zgheibi SEHA, Maria Teresa Yongson Alejandrino SEHA 352
One function of the striated ducts in chest compressions are given per
the parotid and submandibular glands minute?
is to
A. 40.
A. add Na+ ions to saliva. B. 60.
B. help form a hypertonic saliva ( C. 80.
relative to serum ). D. 100. *
C. add bicarbonate ions to saliva *
D. remove K+ ions from saliva. The risk of transmission of Hepatitis B
E. add salivary amylase to saliva. Virus (HBV) is greater than that of
Human Immunodeficiency Virus (HIV)
Which of the following sweeteners because HBV is
used in sugarless gum is most
effective in preventing caries? 1. More resistant than HIV.
2. transmissible through saliva.
A. Xylitol. * 3. in higher numbers in blood than HIV.
B. Sorbitol. 4. Autoclave resistant.
C. Mannitol.
D. Glycerol. A. (1) (2) (3) *
B. (1) and (3)
In the treatment of necrotizing C. (2) and (4)
ulcerative gingivitis with associated D. (4) only
lymphadenopathy, which of the E. All of the above.
following medications is the most
effective? A 12 year old female patient has a
developmental age of 8 years.
A. An anti-inflammatory. Radiographic examination reveals
B. A topical antibiotic. * mandibular dysplasia, delayed growth
C. A systemic antibiotic. of the cranial vault and reduced facial
D. An analgesic. height. Dental development is
equivalent to 10 years. Panoramic
Yellow or brown stains appearing on radiographs reveal an abnormal tooth
radiographs months after processing eruption pattern, crowding of the
result from primary and permanent teeth, delayed
eruption of the permanent canines and
A. processing at an excessive absence of the premolars. The most
temperature. likely diagnosis is
B. storing radiographs at 30°C.
C. incomplete fixing and/or washing of A. hypogonadism.
radiographs. * B. hypoparathyroidism.
D. using expired film. C. hypothyroidism *
D. hyposecretion of growth hormone.
With two rescuers performing
cardiopulmonary resuscitation (CPR)
on an adult patient, how many external

Dr Abdul Naser Tamim SEHA, Dr Ghada Al Aqqad D.D.S, Dr Kamal Naser - Amrita Medical Centre, Dr Emad Wani - Shadi Dental Centre,
Dr Lina Anka SEHA, Dr Rouba Zgheibi SEHA, Maria Teresa Yongson Alejandrino SEHA 353
Which of the following “in office” B. macrophages.
preventive procedures are most C. lymphocytes.
practical and effective for an D. polymorphonuclear leukocytes *
uncooperative 4-year old patient from a
non-compliant family? A patient has a displaced right
subcondylar fracture of the mandible.
A. Oral hygiene instruction. On opening, the mandible deflects to
B. Dietary counseling. the right. Which muscle is prevented
C. Fluoride varnish every six months. * from functioning appropriately?
D. Fluoride supplements.
E. Pit and fissure sealants on all A. Medial pterygoid.
primary molars. B. Temporalis.
C. Masseter.
A smooth, elevated, red patch devoid D. Lateral pterygoid. *
of filiform papillae, located in the
midline of the dorsum of the tongue By definition, a compound fracture of
immediately anterior to the the mandible must have
circumvallate papillae is indicative of
A. multiple bone fragments.
A. benign migratory glossitis. B. exposure to the external
B. median rhomboid glossitis * environment *
C. a granular cell tumor. C. a tooth in the line of fracture.
D. iron deficiency anemia. D. displacement of the fractured
E. a fibroma. segments.

An 8 year old patient with all primary The pain associated with pulpitis often
molars still present exhibits a cusp-to- disappears when a patient visits a
cusp relationship of permanent dental office. This occurs due to which
maxillary and mandibular first molars of the following events?
and good alignment of the lower
incisors. The management of this A. Stress-induced sympathetic activity
patient should be to inhibits pulpal sensory fibres that cause
pain. *
A. refer for orthodontic consultation. B. Stress-induced sympathetic activity
B. use a cervical headgear to causes vasodilatation of pulpal blood
reposition maxillary molars. vessels.
C. disk the distal surfaces of primary C. Local mediators of pulpal pain are
mandibular second molars. not released when a stress-induced
D. place patient on appropriate recall rise in blood pressure occurs.
schedule * D. Pulpal pain receptors undergo
fatigue.
The most prevalent inflammatory cells
found in gingival tissue 24 hours
following flap surgery are
A. monocytes.

Dr Abdul Naser Tamim SEHA, Dr Ghada Al Aqqad D.D.S, Dr Kamal Naser - Amrita Medical Centre, Dr Emad Wani - Shadi Dental Centre,
Dr Lina Anka SEHA, Dr Rouba Zgheibi SEHA, Maria Teresa Yongson Alejandrino SEHA 354
Decreased alveolar bone density is examination shows linear mucosal
associated with decreased levels of ulcers with hyperplastic margins in the
buccal vestibule. What is the most
A. thyroxin. likely diagnosis?
B. hydrocortisone.
C. parathyroid hormone. A. Crohn’s disease. *
D. estrogen * B. Leukemia.
C. AIDS.
Which of the following presents with D. Diabetes mellitus.
high serum calcium levels, thinning of
cortical bone and giant cell osteoclasts A patient complains of irritability,
in the jaw and drifting teeth? fatigue and weakness. She is losing
weight and has diarrhea. The clinical
A. Hyperthyroidism. examination shows diffuse brown
B. Hyperparathyroidism. * macular pigmentation of the oral
mucosa. The pigmentation appeared
C. Hypothyroidism.
recently. The most likely diagnosis is
D. Hypoparathyroidism.
A. iron deficiency anemia.
Enamel hypoplasia, blunted roots and
B. Addison’s disease. *
abnormal dentin are indicative of
C. acute myeloid leukemia.
D. Crohn’s disease.
A. hyperparathyroidism.
B. hyperthyroidism. A 52 year old patient presents with a
C. hypoparathyroidism. limitation of mouth opening. The
D. hypothyroidism * patient has loss of attached gingiva
and multiple areas of gingival
Which of the following dental recession. A panoramic radiograph
procedures could be performed with shows diffuse widening of the
minimal risk for a 35-year old patient periodontal ligament. The most likely
with a severe bleeding disorder? diagnosis is

A. Mandibular block anesthesia. A. scleroderma *


B. Supragingival calculus removal. * B. hyperparathyroidism.
C. Incisional biopsy. C. cicatricial pemphigoid.
D. Subgingival restoration. D. erythema multiforme.
E. advanced adult periodontitis.
A 24 year old patient complains of
abdominal pain, frequent diarrhea and
weight loss. The oral clinical

Dr Abdul Naser Tamim SEHA, Dr Ghada Al Aqqad D.D.S, Dr Kamal Naser - Amrita Medical Centre, Dr Emad Wani - Shadi Dental Centre,
Dr Lina Anka SEHA, Dr Rouba Zgheibi SEHA, Maria Teresa Yongson Alejandrino SEHA 355
A 22 year old patient has been A. Polysulfide.
experiencing general malaise, fever, B. Condensation reaction silicone.
sore throat and coughing for one week. C. Reversible hydrocolloid.
There are multiple ulcerations of the D. Polyvinylsiloxane. *
oral mucosa, crusting of the lips and E. Irreversible hydrocolloid.
red circular lesions on the palms of the
hands. The most likely diagnosis is Which one of the following luting
agents has been shown to reduce the
A. gonorrhea. incidence of fracture in an all-ceramic
B. infectious mononucleosis. restoration?
C. acute herpetic gingivostomatitis.
D. AIDS. A. Resin modified glass ionomer
E. erythema multiforme * cement.
B. Zinc phosphate cement.
A 32 year old female patient complains C. Composite resin cement. *
of fever, weight loss and general D. Glass ionomer cement.
malaise. She has a rash on the malar
area and nose, as well as some The primary objective of periodontal
irregularly shaped ulcerations on the debridement is removal of
buccal mucosa. The most likely
diagnosis is A. calculus.
B. plaque *
A. lichen planus. C. cementum.
B. lupus erythematosus * D. dentin.
C. erythema multiforme.
D. bullous pemphigoid. The current recommended regimen of
E. pemphigus. antibiotic prophylaxis for a patient with
a prosthetic heart valve and an allergy
A 72 year old male patient complains to penicillin is
of bone pain, fever and fatigue. Clinical
examination shows petechiae on the A. amoxicillin 3g orally one hour before
skin and oral mucosa. The tongue has procedure; then 1.5g six hours after
a nodular appearance. A craniofacial initial dose.
radiograph shows punched-out B. amoxicillin 2g orally one hour before
radiolucencies. The most likely procedure only.
diagnosis is C. clindamycin 300mg orally one hour
before procedure; then 150mg six
A. Burkitt’s lymphoma. hours after initial dose.
B. chondrosarcoma. D. clindamycin 600mg orally one hour
C. acute lymphocytic leukemia. before procedure only *
D. multiple myeloma *
E. erythromycin stearate, 2g orally two
hours before procedure only.
Which impression material can be
stored for more than 24 hours before
being poured and still produce
accurate dies?

Dr Abdul Naser Tamim SEHA, Dr Ghada Al Aqqad D.D.S, Dr Kamal Naser - Amrita Medical Centre, Dr Emad Wani - Shadi Dental Centre,
Dr Lina Anka SEHA, Dr Rouba Zgheibi SEHA, Maria Teresa Yongson Alejandrino SEHA 356
Methemoglobinemia is a potential A fistula found in association with a
complication of an excessive dose of non-vital tooth should be

A. bupivacaine. 1. Cauterized to remove the


B. lidocaine. epithelium.
C. mepivacaine. 2. Traced to source with a gutta-
D. prilocaine * percha point on a radiograph.
3. Treated with combined surgical
Local anesthetic injected into dental and nonsurgical root canal therapy.
abscesses is rarely effective because 4. Treated with nonsurgical root canal
therapy.
A. bacteria can metabolize the local
anesthetic. A. (1) (2) (3)
B. edema dilutes the local anesthetic. B. (1) and (3)
C. the tissue is too acidic. * C. (2) and (4) *
D. there is excessive vasoconstriction. D. (4) only
E. All of the above.
Following administration of a posterior
superior alveolar nerve block, a Which of the following may affect the
hematoma occurs. Which of the results of electric pulp testing?
following statements is correct?
1. Patient anxiety.
A. The formation of this hematoma 2. Pain threshold.
indicates poor injection technique. 3. Analgesics.
B. This nerve block is not commonly 4. Recent trauma.
associated with hematoma formation.
C. Management of this hematoma A. (1) (2) (3)
includes immediate application of heat B. (1) and (3)
for at least the first 6 hours. C. (2) and (4)
D. The patient may experience trismus D. (4) only
the next day. * E. All of the above. *

The most appropriate treatment of a Fibrous dysplasia of the jaws


true combined endodontic-periodontal
lesion is A. is of known etiology.
B. is premalignant.
A. periodontal surgical therapy only. C. has monostotic and polyostotic
B. nonsurgical root canal therapy only. forms. *
C. periodontal surgical therapy before D. begins in the fifth decade.
non-surgical endodontic treatment. E. is bilaterally symmetrical.
D. nonsurgical root canal therapy
before periodontal therapy *

Dr Abdul Naser Tamim SEHA, Dr Ghada Al Aqqad D.D.S, Dr Kamal Naser - Amrita Medical Centre, Dr Emad Wani - Shadi Dental Centre,
Dr Lina Anka SEHA, Dr Rouba Zgheibi SEHA, Maria Teresa Yongson Alejandrino SEHA 357
The micro-organisms of dental caries Oral signs and/or symptoms of
are advanced vitamin C deficiency include

1. Streptococcus mutans. 1. Pain.


2. Staphylococcus aureus. 2. Angular cheilitis.
3. Lactobacillus acidophilus. 3. Spontaneous hemorrhage of the
4. β-hemolytic streptococci. gingiva.
4. Xerostomia.
A. (1) (2) (3)
B. (1) and (3) * A. (1) (2) (3)
C. (2) and (4) B. (1) and (3) *
D. (4) only C. (2) and (4)
E. All of the above. D. (4) only
E. All of the above.
While the teeth are set in wax, dentures
are tried in to A laboratory-fabricated composite
resin inlay compared to a direct
A. verify the maxillomandibular records. composite resin restoration has
B. verify the vertical dimension of increased
occlusion.
C. evaluate esthetics. A. colour stability.
D. All of the above. * B. surface smoothness.
C. control of polymerization shrinkage*
Which of the following is NOT D. bondability to tooth structure.
associated with Cushing’s disease?
Oral lesions that do NOT cross the
A. Buffalo hump. midline are most likely
B. Osteoporosis.
C. Hirsutism. A. herpes zoster *
D. Hypertension. B. varicella.
E. Diabetes insipidus. * C. an allergic response.
D. recurrent herpes.
Following root canal therapy, the most
desirable form of tissue response at the The collagen fibres of the periodontal
apical foramen is ligament that insert into bone are
called
A. cementum deposition *
B. connective tissue capsule formation. A. Müller’s fibres.
C. epithelium proliferation from the B. reticular fibres.
periodontal ligament. C. Sharpey’s fibres. *
D. dentin deposition. D. oxytalan fibres.

Dr Abdul Naser Tamim SEHA, Dr Ghada Al Aqqad D.D.S, Dr Kamal Naser - Amrita Medical Centre, Dr Emad Wani - Shadi Dental Centre,
Dr Lina Anka SEHA, Dr Rouba Zgheibi SEHA, Maria Teresa Yongson Alejandrino SEHA 358
Epinephrine is added to local The major advantage of glass ionomer
anesthetic formulations in order to cement as a restorative material is that
it is
1. Increase duration of anesthesia.
2. Increase depth of anesthesia. A. highly translucent.
3. Reduce likelihood of systemic B. a fluoride releasing material. *
toxicity. C. highly esthetic.
4. Reduce likelihood of allergic D. unaffected by moisture during the
reaction. setting reaction.

A. (1) (2) (3) * Epinephrine should be administered


B. (1) and (3) immediately upon recognizing which
C. (2) and (4) one of the following emergencies?
D. (4) only
E. All of the above. A. A severe angina attack.
B. A mild asthmatic attack.
Periapical cemental dysplasia is C. Severe anaphylaxis. *
D. Severe hypotension.
A. painful.
B. expansile. Nitrous oxide
C. associated with vital teeth. *
D. premalignant. A. may supplement behavioural
management of an anxious patient. *
Which of the following mucosae is B. is a substitute for behavioural
normally keratinized? management of an anxious patient.
C. is a substitute for local anesthesia
A. Soft palate. for a dental extraction.
B. Hard palate. * D. would have no analgesic effect at a
C. Lateral tongue. dose of 60%.
D. Ventral tongue.
Which of the following is most likely to
After placement of the rubber dam you be a squamous cell carcinoma?
notice that the interdental papilla is
protruding from beneath the rubber A. Sore shallow ulcer, present for a few
dam. The reason for this is that days.
B. Burning red plaque, present for
A. a rubber dam frame was used. several weeks.*
B. the holes were placed too far apart. C. Asymptomatic gray macule, present
C. a light weight dam was used. for several months.
D. the holes were placed too close D. Occasionally tender normal
together. * coloured nodule, present for several
E. the teeth were not individually years.
ligated.

Dr Abdul Naser Tamim SEHA, Dr Ghada Al Aqqad D.D.S, Dr Kamal Naser - Amrita Medical Centre, Dr Emad Wani - Shadi Dental Centre,
Dr Lina Anka SEHA, Dr Rouba Zgheibi SEHA, Maria Teresa Yongson Alejandrino SEHA 359
Which one of the following effects is The principal internal retention for a
expected with a therapeutic dose of Class V amalgam cavity preparation is
acetaminophen? established at the

A. Gastrointestinal irritation. A. occluso-axial and gingivo-axial line


B. Increased bleeding. angles *
C. Anti-inflammatory action. B. mesio-axial and disto-axial line
D. Antipyretic action. * angles.
C. mesio-gingival and disto-gingival
Which one of the following is NOT a line angles.
contraindication to ibuprofen? D. None of the above.

A. Concurrent use of alcohol. An open proximal contact on an


B. Asthmatic reaction to acetylsalicylic amalgam restoration could have been
acid. caused by
C. An allergy to acetaminophen. *
D. A gastric ulcer. 1. Inadequate wedging.
2. Overtightening the matrix band.
Which of the following impression 3. Inadequate condensing forces.
materials has the best dimensional 4. Simultaneous placement of adjacent
stability? proximal restorations.

A. Polysulfide rubber. A. (1) (2) (3) *


B. Condensation silicone. B. (1) and (3)
C. Polyvinylsiloxane. * C. (2) and (4)
D. Irreversible hydrocolloid. D. (4) only
E. All of the above.
Which of the following phases in the
setting reaction of dental amalgam is Which of the following is NOT a
weak and corrosion-prone? common side effect from a tricyclic
antidepressant?
A. α1.
B. α2. A. Orthostatic hypotension.
C. y. B. Increased urination. *
D. y1. C. Xerostomia.
E. y2. * D. Confusion.

Dr Abdul Naser Tamim SEHA, Dr Ghada Al Aqqad D.D.S, Dr Kamal Naser - Amrita Medical Centre, Dr Emad Wani - Shadi Dental Centre,
Dr Lina Anka SEHA, Dr Rouba Zgheibi SEHA, Maria Teresa Yongson Alejandrino SEHA 360
Which of the following procedures
should be used to reduce the risk of 1. Apply a cold compress.
exposure to mercury vapour? 2. Administer 0.3mg epinepherine
sublingually.
1. Staff education. 3. Apply pressure.
2. Rubber dam. 4. Refer for immediate medical
3. High velocity suction. treatment.
4. Surgical mask.
A. (1) (2) (3)
A. (1) (2) (3) B. (1) and (3) *
B. (1) and (3) C. (2) and (4)
C. (2) and (4) D. (4) only
D. (4) only E. All of the above.
E. All of the above.*
The presurgical assessment of a
If a patient loses a permanent permanent maxillary first molar reveals
maxillary first molar before the age of roots which are trifurcated and widely
11, the divergent. The maxillary antrum
encroaches into the area of trifurcation.
1. Premolar drifts distally. Which of the following should be done
2. Maxillary second molar erupts and during extraction of this tooth?
moves mesially.
3. Opposing tooth erupts into the 1. Reflect a flap.
space created. 2. Reduce the palatal bone to the level
4. Overbite increases. of the trifurcation.
3. Reduce the buccal bone to the level
A. (1) (2) (3) * of the trifurcation.
B. (1) and (3) 4. Amputate the crown and section the
C. (2) and (4) roots.
D. (4) only
E. All of the above. ?? A. (1) (2) (3)
B. (1) and (3)
Immediately following a left posterior C. (2) and (4)
superior alveolar nerve block injection, D. (4) only *
the patient’s face becomes quickly and E. All of the above.
visibly swollen on the left side. The
immediate treatment should be to

Dr Abdul Naser Tamim SEHA, Dr Ghada Al Aqqad D.D.S, Dr Kamal Naser - Amrita Medical Centre, Dr Emad Wani - Shadi Dental Centre,
Dr Lina Anka SEHA, Dr Rouba Zgheibi SEHA, Maria Teresa Yongson Alejandrino SEHA 361
In an acute apical abscess (acute B. the age of onset is younger.
periapical abscess), which of the C. it is usually associated with
following teeth is most likely to spread abrasion.
infection to the submandibular space? D. it progresses more quickly. *

A. Mandibular second bicuspid. Which of the following procedures


B. Maxillary third molar. must be done to ensure acceptable
C. Mandibular first molar. mercury hygiene in a dental office?
D. Mandibular third molar. *
E. Mandibular lateral incisor. A. Use of high volume evacuation
when working with amalgam. *
Which of the following structures lies B. Use of air spray when condensing,
inferior to the mylohyoid muscle at the polishing or removing amalgam.
level of the mandibular second molar? C. Storage of amalgam scrap in a dry
container with a lid.
A. Lingual artery. D. A quarterly mercury assessment for
B. Lingual vein. * office personnel.
C. Lingual nerve.
D. Submandibular duct. The maxillary cast partial denture
major connector design with the
The layer of intermingled collagen and greatest potential to cause speech
resin located beneath a restoration is problems is
called the
A. a thick narrow major connector *
A. smear layer. B. an anterior and a posterior bar.
B. hybrid layer * C. a thin broad palatal strap.
C. Weil layer. D. narrow horseshoe shaped.
D. decalcification layer.
A reciprocal clasp arm on a removable
A direct or indirect pulp cap has the partial denture will provide
greatest chance of clinical success
when there is a 1. Resistance to horizontal force.
2. Indirect retention.
A. history of spontaneous pain 3. Stabilization.
B. prolonged response to cold 4. Direct retention.
stimulus.
C. apical lesion A. (1) (2) (3) *
D. vital pulp * B. (1) and (3)
C. (2) and (4)
The initiation of cemental caries differs D. (4) only
from enamel caries because E. All of the above.

A. dental plaque is not involved.

Dr Abdul Naser Tamim SEHA, Dr Ghada Al Aqqad D.D.S, Dr Kamal Naser - Amrita Medical Centre, Dr Emad Wani - Shadi Dental Centre,
Dr Lina Anka SEHA, Dr Rouba Zgheibi SEHA, Maria Teresa Yongson Alejandrino SEHA 362
Which muscle is LEAST likely to C. immediately following complete
dislodge a mandibular denture? eruption of the first permanent molars.
D. several months prior to the pre-
A. Masseter. pubertal growth spurt. *
B. Buccinator. * E. after skeletal maturity.
C. Mentalis.
D. Mylohyoid. In its classic form, serial extraction is
best applied to patients with Class I
Which of the following hemostatic occlusions with crowding of
agents is most likely to create a
systemic reaction? A. less than 10mm in each of the
upper and lower arches and 35%
A. Aluminum sulphate (Pascord®). overbite.
B. Aluminum chloride (Hemodent®). B. 10mm or more in each of the upper
C. Epinephrine (Racord®). * and lower arches and 35% overbite *
D. Ferric sulphate (Astringedent®). C. less than 10mm in each of the
upper and lower arches and 70%
The surgical removal of tooth 3.6 overbite.
requires anesthesia of the inferior D. 10mm or more in each of the upper
alveolar nerve as well as which of the and lower arches and 70% overbite.
following nerves?
Which of the following statements
A. Lingual, cervical plexus. applies to the submandibular lymph
B. Cervical, long buccal. nodes?
C. Lingual, long buccal. *
D. Mental, long buccal. 1. They discharge into the internal
jugular nodes.
Which of the following is NOT a 2. When draining an area of acute
component of a dental cartridge infection, they are enlarged,
containing 2% lidocaine with 1:100,000 nontender, soft, well defined and
epiniphrine? movable.
3. They are found medially to the
A. Methylparaben. * mandible.
B. Water. 4. They drain the anterior palatine
C. Sodium metabisulphite. pillar, soft palate, posterior third of the
D. Sodium chloride. tongue.

The best time to begin interceptive A. (1) (2) (3)


orthodontic treatment for a patient with B. (1) and (3) *
a skeletal Class II malocclusion is C. (2) and (4)
D. (4) only.
A. as soon as the malocclusion is E. All of the above.
diagnosed.
B. immediately following complete
eruption of the deciduous dentition.

Dr Abdul Naser Tamim SEHA, Dr Ghada Al Aqqad D.D.S, Dr Kamal Naser - Amrita Medical Centre, Dr Emad Wani - Shadi Dental Centre,
Dr Lina Anka SEHA, Dr Rouba Zgheibi SEHA, Maria Teresa Yongson Alejandrino SEHA 363
What percentage of 5 - 17 year olds in A. Reflexes between jaw closing and
North America are caries free? opening muscles.
B. Swallowing and respiration
A. 15 - 25 neuronal activity.
B. 30 - 45 C. Periodontal receptor stimulation.
C. 50 - 60 * D. Reticular formation neuronal
D. 65 - 75 activity. *

Which of the following would The parasympathetic post ganglionic


maximize vitamin E intake following fibers leaving the otic ganglion will
osseous surgery? travel along which cranial nerve?

A. Lettuce. A. Glossopharyngeal.
B. Wheat germ. * B. Branch of the mandibular. *
C. Eggs. C. Temporal.
D. Fish. D. Facial.

A 23 year old female complains of Continued smoking will impair wound


bilateral stiffness and soreness in the healing following a surgical procedure
preauricular region. Her symptoms because of
have been present for the past week
and are most pronounced in the A. stain development.
morning. The most likely cause is B. increased rate of plaque formation.
C. increased rate of calculus
A. fibrous ankylosis of the formation.
temporomandibular joints. D. contraction of peripheral blood
B. nocturnal bruxism * vessels *
C. early osteoarthritis. E. superficial irritation to tissues by
D. mandibular subluxation. smoke.

On bite-wing radiographs, the normal On bite-wing radiographs of adults


alveolar crest on a young adult is under the age of 30, the normal
alveolar crest is
A. at the cemento-enamel junction.
B. 1-2mm apical to the cemento- A. at the cementoenamel junction.
enamel junction. * B. 1-2mm apical to the
C. 3-4mm apical to the cemento- cementoenamel junction. *
enamel junction. C. 3-4mm apical to the
cementoenamel junction.
Chewing “automatism” is dependent D. not clearly distinguishable.
on which mechanism?

Dr Abdul Naser Tamim SEHA, Dr Ghada Al Aqqad D.D.S, Dr Kamal Naser - Amrita Medical Centre, Dr Emad Wani - Shadi Dental Centre,
Dr Lina Anka SEHA, Dr Rouba Zgheibi SEHA, Maria Teresa Yongson Alejandrino SEHA 364
For an otherwise healthy patient, with Which of the following cells are
an acute localized periodontal involved in an immune response to
abscess, initial treatment must include plaque in the periodontal pocket?

A. scaling and root planing * 1. Macrophages.


B. occlusal adjustment. 2. Plasma cells.
C. prescription of an antibiotic. 3. Lymphocytes.
D. prescription of an analgesic. 4. Neutrophils.

Eliminating periodontal pockets by A. (1) (2) (3) *


gingivectomy results in B. (1) and (3)
C. (2) and (4)
A. healing by primary intention. D. (4) only
B. adequate access to correct E. All of the above.
irregular osseous contours.
C. retention of all or most of the The difference between gingivitis and
attached gingiva. periodontitis is
D. healing by secondary intention *
A. radiographic changes are present
The predominant organism(s) only in gingivitis.
associated with chronic (adult) B. radiographic changes are present
periodontitis is/are only in periodontitis. *
C. changes in gingival colour are
1. Prevotella intermedia. present only in gingivitis.
2. Pseudomonas aeruginosa. D. changes in gingival colour are
3. Porphyromonas gingivalis. present only in periodontitis.
4. Heliobacter pilori.
The most common clinical
A. (1) (2) (3) characteristic/s of a buccolingual
B. (1) and (3) * functional crossbite is/are
C. (2) and (4)
D. (4) only 1. Mandibular shift from initial contact
E. All of the above. to maximum intercuspation.
2. Asymmetrical arches.
Gingival connective tissue fibres are 3. Midline deviation.
primarily composed of 4. Several missing teeth.
A. (1) (2) (3)
A. collagen. * B. (1) and (3) *
B. reticulin. C. (2) and (4)
C. elastin. D. (4) only
D. oxytalin. E. All of the above.

Dr Abdul Naser Tamim SEHA, Dr Ghada Al Aqqad D.D.S, Dr Kamal Naser - Amrita Medical Centre, Dr Emad Wani - Shadi Dental Centre,
Dr Lina Anka SEHA, Dr Rouba Zgheibi SEHA, Maria Teresa Yongson Alejandrino SEHA 365
The most likely cause of tooth loss A periodontal dressing is placed
following a tunneling procedure to following a gingivectomy to:
provide complete access for a
mandibular Class III furcation A. Promote wound healing.
involvement is B. prevent microbial colonization of
the wound.
A. root caries * C. protect the wound from mechanical
B. root sensitivity. injury. *
C. pulpal involvement. D. achieve hemostasis.
D. recurrent pocketing.
A 45 year old, overweight man reports
In periodontal therapy, “guided tissue that his wife complains that he
regeneration” is most successful in snores. The initial management of the
treating patient’s snoring problem is to

1. Horizontal bone loss. A. fabricate an appliance to reduce


2. A 3-walled infrabony defect. snoring.
3. A mandibular Class III furcation B. fabricate restorations to increase
involvement. the patient’s vertical dimension of
4. A mandibular Class II furcation occlusion.
involvement. C. refer for an orthognathic surgery
consultation.
A. (1) (2) (3) D. refer for a sleep assessment. *
B. (1) and (3)
C. (2) and (4) * Following root planing, a patient
experiences thermal sensitivity. This
D. (4) only.
pain is associated with which of the
E. All of the above.
following?
The most appropriate treatment of
A. Golgi receptor.
necrotizing ulcerative periodontitis
(NUP) in a patient with no fever and no B. Free nerve endings.
lymphadenopathy is C. Odontoblastic processes. *
D. Cementoblasts.
1. Periodontal debridement.
2. Antibiotic therapy. Which two muscles are involved in
sucking?
3. Oral hygiene instruction.
4. Topical steroid therapy.
A. Caninus and depressor angularis.
A. (1) (2) (3) B. Risorius and buccinator.
B. (1) and (3) * C. Buccinator and orbicularis oris. *
C. (2) and (4) D. Levator labii superioris and
zygomaticus major.
D. (4) only
E. All of the above.

Dr Abdul Naser Tamim SEHA, Dr Ghada Al Aqqad D.D.S, Dr Kamal Naser - Amrita Medical Centre, Dr Emad Wani - Shadi Dental Centre,
Dr Lina Anka SEHA, Dr Rouba Zgheibi SEHA, Maria Teresa Yongson Alejandrino SEHA 366
A patient with a tumor in the right 2. To relieve symptoms of multiple
infratemporal fossa shows a significant sclerosis.
shift of the mandible to the right when 3. in highly esthetic areas of the mouth.
opening. Which nerve is involved? 4. To eliminate toxins from the patient.

A. Facial nerve VII. A. (1) (2) (3)


B. Glossopharyngeal nerve IX. B. (1) and (3) *
C. Trigeminal nerve V. * C. (2) and (4)
D. Hypoglossal nerve XII. D. (4) only
E. All of the above.
A healthy 78 year old patient presents
with three new carious lesions on root Dentigerous cysts are usually found
surfaces. This is most likely the result
of A. periapically.
B. pericoronally. *
A. age related changes in cementum C. interradicularly.
composition. D. mid-root.
B. the architecture at the CEJ.
C. age related decrease in salivary When two teeth have Class III lesions
flow. * adjacent to each other, the operator
D. changes in dietary pattern. should prepare the
E. chronic periodontal disease.
A. larger lesion first and restore the
The following conditions can be the smaller one first. *
result of ill-fitting complete dentures. B. smaller lesion first and restore the
smaller one first.
1. Soft tissue hyperplasia. C. smaller lesion first and restore the
2. Alveolar ridge resorption. larger one first.
3. Angular cheilitis. D. large lesion first and restore the
4. Carcinoma. larger one first.

A. (1) (2) (3) * Benign (reactive) lymphoid hyperplasia


B. (1) and (3) of cervical lymph nodes is typically
C. (2) and (4)
D. (4) only A. tender, fixed.
E. All of the above. ?? B. tender, mobile *
C. nontender, fixed.
It is ethical to replace amalgam D. nontender, matted.
restorations

1. on request from an informed patient.

Dr Abdul Naser Tamim SEHA, Dr Ghada Al Aqqad D.D.S, Dr Kamal Naser - Amrita Medical Centre, Dr Emad Wani - Shadi Dental Centre,
Dr Lina Anka SEHA, Dr Rouba Zgheibi SEHA, Maria Teresa Yongson Alejandrino SEHA 367
Focal periapical osteopetrosis (dense A. Hyperplasia of the gingiva. *
bone island) differs from focal B. Alteration of the trabecular bone
sclerosing osteomyelitis in that it is pattern.
C. Generalized loss of lamina dura.
A. expansile. D. Giant cell tumours of bone.
B. periapical.
C. radiopaque. Which articular disease most often
D. caused by pulpitis. * accompanies Sjögren’s syndrome?

Smooth surface caries begins at A. Suppurative arthritis.


localized areas on the B. Rheumatoid arthritis. *
C. Degenerative arthrosis.
A. outer surface of enamel and dentin* D. Psoriatic arthritis.
B. inner surface of the enamel. ?? E. Lupus arthritis.
C. outer surface of the dentin.
D. outer surface of the enamel. Which of the following is the most
E. inner surface of the dentin. effective pharmacologic treatment for
angular cheilosis?
Melanoma differs from physiologic
pigmentation because it A. Penicillin.
B. Erythomycin.
A. is macular. C. Tetracycline.
B. contains melanin. D. Clindamycin.
C. affects the gingiva. E. Nystatin. *
D. changes clinically. *
A patient wants all his remaining teeth
Which of the following is the most extracted and dentures fabricated. He
frequent major congenital has carious lesions involving the dentin
malformation of the head and neck? on all remaining teeth. The
periodontium is sound. The most
A. Cystic hygroma colli. appropriate management is to
B. Cleft palate. *
C. Encephalotrigeminal angiomatosis. 1. Respect the patient's decision.
D. Double lip. 2. Advise the patient to consult a
E. Commissural pits. specialist.
3. Discuss all of the appropriate
Which disease could cause both renal treatment options.
and cardiac complications? 4. Refuse to refer or to treat this patient
since it is unethical.
A. Measles.
B. Scarlet fever. * A. (1) (2) (3)
C. Rubella. B. (1) and (3) *
D. Rosacea. C. (2) and (4)
D. (4) only
In primary or secondary E. All of the above.
hyperparathyroidism, which of the
following will NOT be found?

Dr Abdul Naser Tamim SEHA, Dr Ghada Al Aqqad D.D.S, Dr Kamal Naser - Amrita Medical Centre, Dr Emad Wani - Shadi Dental Centre,
Dr Lina Anka SEHA, Dr Rouba Zgheibi SEHA, Maria Teresa Yongson Alejandrino SEHA 368
Aphthous ulcers are characterized by 2. Diabetes mellitus.
3. Acquired immunodeficiency
1. Pain. syndrome.
2. Pseudomembranes. 4. Hereditary hypohydrotic ectodermal
3. Inflammation. dysplasia.
4. Vesicle formation.
A. (1) (2) (3)
A. (1) (2) (3) * B. (1) and (3)
B. (1) and (3) C. (2) and (4)
C. (2) and (4) D. (4) only *
D. (4) only E. All of the above.
E. All of the above.
Which of the following procedures is
The features of aggressive (rapidly NOT indicated for the management of
progressive) periodontitis are infrabony defects?

1. Rapid attachment loss. A. Gingivectomy. *


2. suspected periodontal microbial B. Regenerative surgery.
pathogens. C. Flap surgery.
3. Onset before the age of 35. D. Gingival graft.
4. Ulcerations of the gingiva.
According to the principles of ethics
A. (1) (2) (3) * that are generally accepted, a dentist
B. (1) and (3) may refuse to treat a patient with HIV
C. (2) and (4) infection for a specific procedure for the
following reason(s).
D. (4) only
E. All of the above.
1. Inadequate experience in the
specific procedure.
A lateral cephalometric radiograph for a
patient with a 3mm anterior functional 2. Inadequate knowledge of the
shift should be taken with the patient in specific procedure.
3. Lack of instruments or equipment for
A. maximum intercuspation. this procedure.
B. initial contact. * 4. Infection control procedures that are
not designed for infectious patients.
C. normal rest position.
D. maximum opening.
A. (1) (2) (3) *
E. protrusive position.
B. (1) and (3)
C. (2) and (4)
Which of the following systemic
diseases does/do NOT predispose a D. (4) only
patient to periodontitis? E. All of the above.

1. Cyclic neutropenia.

Dr Abdul Naser Tamim SEHA, Dr Ghada Al Aqqad D.D.S, Dr Kamal Naser - Amrita Medical Centre, Dr Emad Wani - Shadi Dental Centre,
Dr Lina Anka SEHA, Dr Rouba Zgheibi SEHA, Maria Teresa Yongson Alejandrino SEHA 369
As a dentist, it is ethical to refuse to A 22 year old presents with a fracture of
treat a patient on the basis of the incisal third of tooth 2.1 exposing a
small amount of dentin. The fracture
1. Religious beliefs. occurred one hour previously. There is
2. Physical handicap. no mobility of the tooth but the patient
3. Infectious disease. complains that it is rough and sensitive
4. Recognition of lack of skill or to cold. The most appropriate
knowledge. emergency treatment is to

A. (1) (2) (3) A. open the pulp chamber, clean the


B. (1) and (3) canal and temporarily close with zinc
C. (2) and (4) oxide and eugenol.
D. (4) only * B. smooth the surrounding enamel and
E. All of the above. apply glass ionomer cement. *
C. smooth the surrounding enamel and
To achieve optimum strength and apply a calcium hydroxide cement.
esthetics, a porcelain fused to metal D. place a provisional (temporary)
restoration with a porcelain butt joint crown.
margin requires a
A centric relation record must be used to
1. 0.8 - 1.2mm shoulder. articulate casts in which of the following
2. Sharp, well defined axiogingival line cases?
angle.
3. 90 - 100 cavosurface margin. A. More than one third of the patients
occlusal contacts are to be restored. *
4. Finish line that is 2mm subgingival.
B. The vertical dimension of occlusion
A. (1) (2) (3) will be modified on the articulator.
B. (1) and (3) * C. The patient shows signs of a
temporomandibular disorder.
C. (2) and (4)
D. The patient's centric occlusion and
D. (4) only
centric relation do not match.
E. All of the above.
The best way to increase the working
Zinc phosphate cement, when used as
time of a polyvinylsiloxane is to
a luting agent for cast restorations, has
which of the following properties?
A. change the catalyst/base ratio.
1. Insolubility. B. refrigerate the material. *
2. Anticariogenicity. C. add oleic acid.
3. Chemical adhesion. D. reduce mixing time.
4. Mechanical retention.
The "smear layer" is an important
A. (1) (2) (3) consideration in
B. (1) and (3)
C. (2) and (4) A. plaque accumulation.
D. (4) only * B. caries removal.
E. All of the above. C. pulp regeneration.
D. dentin bonding *

Dr Abdul Naser Tamim SEHA, Dr Ghada Al Aqqad D.D.S, Dr Kamal Naser - Amrita Medical Centre, Dr Emad Wani - Shadi Dental Centre,
Dr Lina Anka SEHA, Dr Rouba Zgheibi SEHA, Maria Teresa Yongson Alejandrino SEHA 370
Which of the following should NOT be 2. A thinner matrix band.
corrected with a porcelain veneer? 3. An anatomical wedge.
4. Use of mechanical condensation.
A. Peg lateral incisor.
B. Diastema between 1.1 and 2.1. A. (1) (2) (3) *
C. Cross bite on tooth 1.3. * B. (1) and (3)
D. Enamel hypoplasia. C. (2) and (4)
D. (4) only
Twenty-four hours after placement of a E. All of the above.
Class II amalgam restoration, a patient
returns complaining of discomfort when Resin bonding of composites to acid-
"biting". There are no other symptoms. etched enamel results in
It is most likely that the
A. decreased polymerization shrinkage
A. pulp was exposed. of the resin.
B. restoration is leaking. B. decreased crack formation in the
C. restoration is in supra-occlusion.* enamel.
D. amount of base material is C. reduced microleakage. *
inadequate. D. elimination of post-operative
sensitivity.
Which of the following cements can E. improved wear resistance of the
chemically bond to enamel? composite.

1. Zinc phosphate cement. Which of the following affect(s)


2. Polycarboxylate cement. polymerization of visible light cured
3. Ethoxy benzoic acid cement. composite resins?
4. Glass ionomer cement.
1. Intensity of the light source.
A. (1) (2) (3) 2. Thickness of composite resin.
B. (1) and (3) 3. Proximity of light source.
C. (2) and (4) * 4. Shade of composite resin.
D. (4) only
E. All of the above. A. (1) (2) (3)
B. (1) and (3)
In order to achieve a proper C. (2) and (4)
interproximal contact when using a D. (4) only
spherical alloy, which of the following E. All of the above. *
is/are essential?

1. A larger sized condenser.

Dr Abdul Naser Tamim SEHA, Dr Ghada Al Aqqad D.D.S, Dr Kamal Naser - Amrita Medical Centre, Dr Emad Wani - Shadi Dental Centre,
Dr Lina Anka SEHA, Dr Rouba Zgheibi SEHA, Maria Teresa Yongson Alejandrino SEHA 371
Composite resin is The tooth preparation for a porcelain
CONTRAINDICATED as a posterior veneer must create a/an
restorative material in cases of
A. rough surface for improved bonding.
1. Cusp replacement. B. space for an appropriate thickness of
2. Bruxism. the veneering material.
3. Lack of enamel at the gingival cavo- C. margin well below the gingival crest.
surface margin. D. definite finish line *
4. Inability to maintain a dry operating
field. Aspiration is carried out prior to a local
anaesthetic injection in order to reduce
A. (1) (2) (3) the
B. (1) and (3)
C. (2) and (4) ????!!! A. toxicity of local anaesthetic.
D. (4) only B. toxicity of vasoconstrictor.
E. All of the above* C. possibility of intravascular
administration *
For amalgam restorations, a 90 D. possibility of paraesthesia.
cavosurface angle accommodates the
Which one of the following describes the
1. Condensing of amalgam. position of the needle tip during
2. Compressive strength of amalgam. administration of local anesthetic for the
3. Tensile strength of amalgam. inferior alveolar nerve block?
4. Compressive strength of enamel.
A. Anterior to the pterygomandibular
A. (1) (2) (3) raphe.
B. (1) and (3) B. Medial to the medial pterygoid
C. (2) and (4) * muscle.
D. (4) only C. Superior to the lateral pterygoid
E. All of the above. muscle.
D. Lateral to the sphenomandibular
The dimensional stability of polyether ligament. *
impression material is considered to be
good EXCEPT if the material is Which of the following constituents of a
local anesthetic cartridge is most likely
A. dehydrated. to be allergenic?
B. allowed absorbing water after
setting. * A. Lidocaine.
C. used in uneven thickness. B. Epinephrine.
D. distorted by rapid removal of the C. Metabisulfite. *
impression from the mouth. D. Hydrochloric acid.
E. contaminated with latex.

Dr Abdul Naser Tamim SEHA, Dr Ghada Al Aqqad D.D.S, Dr Kamal Naser - Amrita Medical Centre, Dr Emad Wani - Shadi Dental Centre,
Dr Lina Anka SEHA, Dr Rouba Zgheibi SEHA, Maria Teresa Yongson Alejandrino SEHA 372
Which of the following is/are essential C. (2) and (4)
when using spherical rather than D. (4) only
admix alloy for a routine amalgam E. All of the above.
restoration
An alveoplasty is performed to
1. A larger diameter condenser.
2. An anatomical wedge. 1. Facilitate removal of teeth.
3. Decreased condensing pressure. 2. Correct irregularities of alveolar
4. A dead soft matrix band. ridges following tooth removal.
3. Prepare the residual ridge for
A. (1) (2) (3) dentures.
B. (1) and (3) *
C. (2) and (4) A. (1) and (2)
D. (4) only B. (1) and (3)
E. All of the above. C. (2) and (3) *
D. All of the above.
A dental laboratory has returned a
removable partial denture framework. Hydroxyapatite used in dental surgery
The framework fit the master cast well is
but when tried in the mouth, a stable
fit could not be achieved. The A. used to fill osseous defects. *
possible cause(s) of the problem B. derived from tooth enamel.
is/are C. resistant to fracture.
D. osteogenic.
1. Distortion in the final impression.
2. Insufficient retention. Which of the following patients has the
3. Improper pour of the master cast. POOREST prognosis when placing
4. Casting error. dental implants? A patient who

A. (1) (2) (3) A. is over age 80 years old.


B. (1) and (3) * B. is on thyroid replacement therapy.
C. (2) and (4) C. smokes 2 packs of cigarettes daily *
D. (4) only D. has generalized osteoporosis.
E. All of the above.
In the surgical removal of an impacted
Sutures can be mandibular third molar, which of the
following would be considered to be
1. Used to hold soft tissues into the most difficult?
position.
2. Used to control bleeding. A. Mesio-angular.
3. Resorbable or non-resorbable. B. Horizontal.
4. Used to stretch soft tissues into C. Vertical.
position. D. Disto-angular. *

A. (1) (2) (3) *


B. (1) and (3)

Dr Abdul Naser Tamim SEHA, Dr Ghada Al Aqqad D.D.S, Dr Kamal Naser - Amrita Medical Centre, Dr Emad Wani - Shadi Dental Centre,
Dr Lina Anka SEHA, Dr Rouba Zgheibi SEHA, Maria Teresa Yongson Alejandrino SEHA 373
The design of a mucoperiosteal flap E. All of the above.
should
Recurring tooth rotations occur most
1. Provide for visual access. frequently after orthodontic correction
2. Provide for instrument access. due to
3. Permit repositioning over a solid
bone base. A. density of the cortical bone.
4. be semilunar in shape. B. persistence of tongue and finger
habits.
A. (1) (2) (3) * C. free gingival and transseptal fibres *
B. (1) and (3) D. oblique fibres of the periodontal
C. (2) and (4) ligament.
D. (4) only
E. All of the above. If the norm for the cephalometric angle
SNA is 82 degrees and a patient's SNA
Which of the following drug groups can is 90 degrees, this would likely indicate
cause xerostomia?
A. protrusive maxillary incisors.
1. Diuretics. B. dysplasia of the anterior cranial base.
2. Antibiotics. C. maxillary prognathism *
3. Antidepressants. D. mandibular prognathism.
4. Non-steroidal anti-inflammatory
agents. Mouth breathing is most commonly
associated with Angle's malocclusion
A. (1) (2) (3) Class
B. (1) and (3) *
C. (2) and (4) A. I.
D. (4) only B. II, Division 1. *
E. All of the above. C. II, Division 2.
D. III.
Which of the following conditions can E. None of the above.
make an older patient short of breath
on mild exertion? The best space maintainer to prevent
the lingual collapse that often occurs
1. Anemia. following the early loss of a mandibular
2. Cardiac failure. primary canine is a
3. Obesity.
4. Osteoarthritis. A. Nance expansion arch.
B. lingual arch *
A. (1) (2) (3) * C. band and loop space maintainer.
B. (1) and (3) D. distal shoe space maintainer.
C. (2) and (4)
D. (4) only

Dr Abdul Naser Tamim SEHA, Dr Ghada Al Aqqad D.D.S, Dr Kamal Naser - Amrita Medical Centre, Dr Emad Wani - Shadi Dental Centre,
Dr Lina Anka SEHA, Dr Rouba Zgheibi SEHA, Maria Teresa Yongson Alejandrino SEHA 374
Following loss of a permanent which the growth velocity reaches its
mandibular first molar at age 8, which peak. That difference is
of the following changes are likely to
occur? A. boys six months ahead of girls.
B. girls six months ahead of boys.
1. Distal drift of second premolar. C. girls one year ahead of boys.
2. No movement of second premolar. D. girls two years ahead of boys *
3. Mesial drift of second permanent
molar. The condyle of the mandible is unique
4. No movement of second permanent because
molar.
A. (1) (2) (3) A. it develops from Meckel's cartilage.
B. (1) and (3) * B. both interstitial and appositional
C. (2) and (4) bone formation are present. *
D. (4) only C. both primary and secondary
E. All of the above. cartilage growth centres are present.

The predominant type of movement After the age of 6 years, growth of the
produced by a finger spring on a mandible is greatest
removable appliance is
A. at the symphysis.
A. torque. B. between canines.
B. tipping * C. along the lower border.
C. rotation. D. posterior to first molars *
D. translation.
Compared to unfilled resins, composite
Excessive orthodontic force used to resins have
move a tooth may
1. Reduced thermal dimensional
1. Cause hyalinization. changes.
2. Cause root resorption. 2. increased strength.
3. Crush the periodontal ligament. 3. Reduced polymerization shrinkage.
4. Impair tooth movement. 4. Better polishability.

A. (1) (2) (3) A. (1) (2) (3) *


B. (1) and (3) B. (1) and (3)
C. (2) and (4) C. (2) and (4)
D. (4) only D. (4) only
E. All of the above. * E. All of the above.

There is a differential between girls


and boys with respect to the age at

Dr Abdul Naser Tamim SEHA, Dr Ghada Al Aqqad D.D.S, Dr Kamal Naser - Amrita Medical Centre, Dr Emad Wani - Shadi Dental Centre,
Dr Lina Anka SEHA, Dr Rouba Zgheibi SEHA, Maria Teresa Yongson Alejandrino SEHA 375
A hinge axis facebow records 1. Apical migration of junctional
epithelium.
A. Bennett angle. 2. Fibrotic enlargement of marginal
B. centric relation. tissue.
C. lateral condylar inclination. 3. Bleeding upon probing.
D. horizontal condylar inclination. 4. Increased depth upon probing.
E. opening and closing axis of the
mandible * A. (1) (2) (3)
B. (1) and (3)
In a cavity preparation which closely C. (2) and (4)
approximates the pulp, you would D. (4) only *
protect the pulp with E. All of the above.

A. a zinc phosphate cement base. Periodontitis


B. a calcium hydroxide cement base.
C. a calcium hydroxide wash and cavity 1. Develops from gingivitis.
varnish. 2. is associated with continuous
D. a calcium hydroxide cement liner destruction of the alveolar bone.
and a glass ionomer cement base. * 3. Goes through stages of tissue
destruction and quiescence.
If the lining cement is left on the gingival 4. Results in occlusal traumatism.
cavosurface margin of a Class II
amalgam restoration, A. (1) (2) (3)
B. (1) and (3) *
A. cement dissolution will lead to C. (2) and (4)
leakage. * D. (4) only
B. the preparation will lack retention E. All of the above.
form.
C. the preparation will lack resistance Detection of periodontal pockets is done
form to bulk fracture. by
D. the preparation will lack appropriate
outline form. A. visual examination.
B. radiographic examination.
Patients with occlusal parafunctional C. testing for mobility of teeth.
habits may have D. probing *
A. soreness in masticatory muscles. Infrabony defects occur most frequently
B. occlusal wear facets. in
C. tooth mobility.
D. All of the above. * A. cancellous bone.
B. cortical bone.
Which of the following is/are common to C. bundle bone.
both gingival and periodontal pockets? D. interseptal bone. *

Dr Abdul Naser Tamim SEHA, Dr Ghada Al Aqqad D.D.S, Dr Kamal Naser - Amrita Medical Centre, Dr Emad Wani - Shadi Dental Centre,
Dr Lina Anka SEHA, Dr Rouba Zgheibi SEHA, Maria Teresa Yongson Alejandrino SEHA 376
When using the periodontal probe to D. (4) only
measure pocket depth, the E. All of the above.
measurement is taken from the
Which of the following pharmacokinetic
A. base of the pocket to the change(s) occur(s) with aging?
cementoenamel junction.
B. free gingival margin to the 1. Absorption is altered by a decrease
cementoenamel junction. in the gastric pH.
C. base of the pocket to the crest of 2. Metabolism is decreased by a
the free gingiva * reduced liver mass.
D. base of the pocket to the 3. Distribution is altered by a decrease
mucogingival junction. in total body fat.
4. Excretion is reduced because of
Hypercementosis at the root apex is lessened renal blood flow.
often associated with
A. (1) (2) (3)
A. hypothyroidism. B. (1) and (3)
B. Paget's disease. * C. (2) and (4) *
C. orthodontic tooth movement. D. (4) only
D. normal occlusal function. E. All of the above.
E. hyperparathyroidism.
Which one of the following cements is
The first sign of a toxic reaction to an anticariogenic because of fluoride ion
injected local anesthetic solution release?
would be
A. Resin.
A. convulsions. B. Polycarboxylate.
B. erythematous rash. C. Zinc phosphate.
C. asthmatic attack. D. Glass ionomer. *
D. excitement. *
A hardened gold alloy will exhibit
Which of the following should NOT be
prescribed for a patient receiving A. less plastic deformation per unit of
warfarin (Coumadin®)? stress than the same alloy in a
softened condition *
1. Acetylsalicylic acid. B. greater plastic deformation per unit
2. Oxycodone. of stress than the same alloy in a
3. Ketorolac. softened condition.
4. Codeine. C. no difference in the plastic
deformation per unit of stress of the
A. (1) (2) (3) alloy in hard or soft condition.
B. (1) and (3) *
C. (2) and (4)

Dr Abdul Naser Tamim SEHA, Dr Ghada Al Aqqad D.D.S, Dr Kamal Naser - Amrita Medical Centre, Dr Emad Wani - Shadi Dental Centre,
Dr Lina Anka SEHA, Dr Rouba Zgheibi SEHA, Maria Teresa Yongson Alejandrino SEHA 377
In dental radiography, the most 4. Act as enzymes.
effective method of reducing patient
somatic exposure is to use A. (1) (2) (3)
B. (1) and (3)
A. a lead apron ??!!! C. (2) and (4)
B. high speed film.* D. (4) only
C. added filtration. E. All of the above. *
D. collimation.
Wolff's Law states that bone elements
Excessively dark radiographs will
result from A. rearrange themselves in the
direction of functional pressures.
A. underdevelopment. B. increase their mass to reflect
B. overexposure. functional stress.
C. backward placement of the film. C. decrease their mass to reflect
D. excessive milliamperage. functional stress.
D. All of the above. *
A. (1) (2) (3)
B. (1) and (3) The epithelial attachment
C. (2) and (4) *
D. (4) only A. in health, is located at the cemento-
E. All of the above. enamel junction.
B. with periodontitis, moves apically
Attached gingival tissue is primarily along the root surface as periodontal
composed of disease progresses.
C. is composed of stratified squamous
A. collagenous fibres. non-keratinized epithelium.
B. keratinized squamous epithelium. D. All of the above. *
C. elastic fibres.
D. A. and B. * Which of the following are
E. A., B. and C. mechanisms of growth of the naso-
maxillary complex?
The inflammatory response in
periodontal pathology is caused by A. Sutural.
bacterial products from plaque which B. Cartilaginous.
C. Appositional.
1. Act as chemotactic products. D. All of the above. *
2. Activate the kallikrein system.
3. Initiate an immune response.

Dr Abdul Naser Tamim SEHA, Dr Ghada Al Aqqad D.D.S, Dr Kamal Naser - Amrita Medical Centre, Dr Emad Wani - Shadi Dental Centre,
Dr Lina Anka SEHA, Dr Rouba Zgheibi SEHA, Maria Teresa Yongson Alejandrino SEHA 378
The maxilla is formed from 2. Foreign body inflammatory reaction.
3. Wound dehiscence.
A. bundle bone. 4. Necrosis of bone.
B. endochondral bone.
C. membranous bone. * A. (1) (2) (3)
D. lamellar bone. B. (1) and (3) *
C. (2) and (4)
A removable orthodontic appliance, D. (4) only
producing a light force on the labial of E. All of the above.
a proclined maxillary central incisor will
cause Which of the following is the LEAST
likely primary site for the development
A. lingual movement of the crown and of oral squamous cell carcinoma in the
lingual movement of the root apex. elderly?
B. intrusion of the central incisor and
lingual movement of the crown. A. Dorsum of the tongue.*
C. lingual movement of the crown and B. Floor of the mouth.
labial movement of the root apex. C. Lateral border of the tongue.
D. intrusion of the central incisor. D. Tonsillar fossa.

Benign neoplasms Inefficient progress in permanent


molar uprighting is most likely due to
1. Grow slowly.
2. are generally painless. A. chronic periodontitis.
3. Can be managed conservatively. B. heavy force application.
4. Can metastasize. C. lack of anchorage.
D. occlusal interference. *
A. (1) (2) (3) *
B. (1) and (3) Forces for orthodontic tooth
C. (2) and (4) movement ideally should be
D. (4) only
E. All of the above. 1. Intermittent.
2. Continuous.
Which of the following basic forcep 3. Heavy.
movements is NOT used for extracting 4. Light.
teeth?
A. (1) (2) (3)
A. Apical. B. (1) and (3)
B. Rotational. C. (2) and (4) *
C. Mesial. * D. (4) only
D. Lingual (palatal). E. All of the above.

A surgical flap not repositioned over a


bony base will result in
1. Slower healing.

Dr Abdul Naser Tamim SEHA, Dr Ghada Al Aqqad D.D.S, Dr Kamal Naser - Amrita Medical Centre, Dr Emad Wani - Shadi Dental Centre,
Dr Lina Anka SEHA, Dr Rouba Zgheibi SEHA, Maria Teresa Yongson Alejandrino SEHA 379
The undesirable side-effect most permanent canines and primary
commonly associated with use of a canines all in contact and anterior to
finger spring to tip the crown of an the premolars. The most likely cause is
anterior permanent tooth is
A. ankylosed permanent canines.
A. pain. B. ankylosed primary canines.
B. gingival irritation. C. impacted permanent lateral incisors.
C. tendency for the tooth to intrude* D. congenitally missing permanent
D. severe mobility of the tooth. lateral incisors *

In cephalometric analysis of children Water irrigation devices have been


with malocclusion, the angle ANB is shown to
frequently used. In patients with
severe Class II malocclusion, this A. eliminate plaque.
angle is B. dislodge food particles from
between teeth. *
A. greater than the normal * C. disinfect pockets for up to 18 hours.
B. less than the normal. D. prevent calculus formation.
C. normal.
The interdental gingival col is the area
Cephalometrics is useful in assessing between the
which of the following relationships?
A. facial and lingual interdental
A. Tooth-to-tooth. gingiva*
B. Bone-to-bone. B. facial and lingual attached gingiva.
C. Tooth-to-bone. C. distal and mesial interdental gingiva.
D. All of the above. *
In horizontal alveolar bone loss, the
Cephalometrics is used in pathway of inflammation is
orthodontics to
A. through the periodontal ligament.
A. treat malocclusions. B. through the epithelial attachment.
B. study growth changes. C. through the cortical bone of the
C. aid in diagnosis and case analysis. alveolar process.
D. B. and C. * D. perivascularly, into the marrow
E. All of the above. spaces of the crestal bone. *

Clinical examination of a 15 year old


girl shows permanent central incisors,

Dr Abdul Naser Tamim SEHA, Dr Ghada Al Aqqad D.D.S, Dr Kamal Naser - Amrita Medical Centre, Dr Emad Wani - Shadi Dental Centre,
Dr Lina Anka SEHA, Dr Rouba Zgheibi SEHA, Maria Teresa Yongson Alejandrino SEHA 380
The Frankfort-horizontal is a D. (4) only
reference plane constructed by E. All of the above.
joining which of the following
landmarks? Which of the following is the most
severe adverse effect of antibiotic
A. Porion and sella. therapy?
B. Porion and nasion.
C. Porion and orbitale * A. Urticaria.
D. Nasion and sella. B. Angioneurotic edema.
C. Diarrhea.
Which of the following should NOT be D. Anaphylactic reaction. *
prescribed to a pregnant patient? E. Flatulence.

A. Erythromycin. In an infection caused by non-


B. Cloxacillin. penicillinase producing staphylococcus,
C. Tetracycline. * the drug of choice is
D. Cephalosporins.
A. penicillin V. *
Antibiotic prophylaxis is B. cephalexin.
recommended for patients with which C. tetracycline.
of the following? D. vancomycin.

1. Mitral valve prolapse with The most appropriate oral drugs for
regurgitation. control of anxiety in a dental patient are
2. Cardiac pacemaker.
3. Prosthetic heart valves. A. benzodiazepines. *
4. All heart murmurs. B. phenothiazines.
C. barbiturates.
A. (1) (2) (3) D. tricyclic antidepressants.
B. (1) and (3) *
C. (2) and (4) Hypoglycemia in a diabetic patient can
D. (4) only be characterized by
E. All of the above.
1. Nausea.
Intravenous administration of 2. Palpitations.
epinephrine results in 3. Sweating.
4. Mental confusion
1. increased systolic pressure. A. (1) (2) (3)
2. increased heart rate. B. (1) and (3)
3. Palpitations. C. (2) and (4)
4. Respiratory depression. D. (4) only
E. All of the above. *
A. (1) (2) (3) *
B. (1) and (3)
C. (2) and (4)

Dr Abdul Naser Tamim SEHA, Dr Ghada Al Aqqad D.D.S, Dr Kamal Naser - Amrita Medical Centre, Dr Emad Wani - Shadi Dental Centre,
Dr Lina Anka SEHA, Dr Rouba Zgheibi SEHA, Maria Teresa Yongson Alejandrino SEHA 381
The surgical procedure indicated for Incomplete polymerization of composite
odontogenic cysts is resin will occur when the resin comes in
contact with
A. enucleation *
B. cauterization. A. zinc phosphate cement.
C. incision and drainage. B. calcium hydroxide lining.
C. zinc oxide eugenol base *
Prophylactic antibiotic therapy is D. glass-ionomer lining.
indicated for patients with E. polycarboxylate cement.

A. a functional heart murmur. The best means of extending the


B. mitral valve prolapse * working time of an irreversible
hydrocolloid impression material is to
C. an osseointegrated dental implant.
D. an organic heart murmur.
A. extend spatulation time.
B. add additional water.
Side effects of chemotherapeutic
treatment for malignancy include C. use cold water. *
D. add a small amount of borax.
A. atrophic thinning of the oral E. add potassium sulfate.
mucosa.
B. ulceration. Using less water for mixing plaster of
Paris will result in set plaster that
C. necrosis.
D. spontaneous bleeding.
A. contracts.
E. All of the above. *
B. is stronger. *
The use of complete dentures by C. is more porous.
older people most frequently causes D. is less brittle.

1. Denture induced hyperplasia. If a complete mandibular denture


causes a burning sensation in the
2. Alveolar ridge resorption.
premolar region, this is due to the
3. Angular cheilitis.
denture exerting pressure in/on the
4. Stomatitis.
5. Leukoplakia. A. fibres of the buccinator muscle.
B. lingual branch of the mandibular
A. (1) (4) (5)
division of the trigeminal nerve.
B. (2) (3) (5)
C. underlying bone.
C. (1) (2) (3) (4) *
D. mental branch of the inferior alveolar
D. All of the above. nerve. *
E. buccal frenum area.

Dr Abdul Naser Tamim SEHA, Dr Ghada Al Aqqad D.D.S, Dr Kamal Naser - Amrita Medical Centre, Dr Emad Wani - Shadi Dental Centre,
Dr Lina Anka SEHA, Dr Rouba Zgheibi SEHA, Maria Teresa Yongson Alejandrino SEHA 382
In patients wearing complete dentures, D. (4) only
the most frequent cause of tooth E. All of the above.
contact (clicking) during speaking is
A patient with new complete dentures
A. nervous tension. complains of a burning sensation in the
B. incorrect centric relation position. palate. The most likely area requiring
C. excessive occlusal vertical relief is at the
dimension. *
D. lack of vertical overlap. A. frenum.
E. unbalanced occlusion. B. post dam.
C. incisive foramen. *
A patient with complete dentures D. foveae palatinae.
complains of clicking. The most
common causes are Which of the following is/are (a) useful
guide(s) in determining a patient’s
A. reduced vertical dimension and occlusal vertical dimension?
improperly balanced occlusion.
B. excessive vertical dimension and 1. Appearance.
poor retention. * 2. Phonetics.
C. use of too large a posterior tooth 3. Observation of the rest position.
and too little horizontal overlap. 4. Pre-extraction profile records.
D. improper relation of teeth to the
ridge and excessive anterior vertical A. (1) (2) (3)
overlap. B. (1) and (3)
C. (2) and (4)
In partial denture design, the major D. (4) only.
connector should E. All of the above. *

A. rigidly connect the bilateral Planing the enamel at the gingival


components. * cavosurface of a Class II amalgam
B. act as a stress-breaker. preparation on a permanent tooth
C. not interfere with lateral forces.
D. dissipate vertical forces. A. should result in a long bevel.
B. is contraindicated because of the
The retention form of a full crown low edge strength of amalgam.
preparation can be improved by C. is unnecessary since the tooth
structure in this area is strong.
1. Reducing its taper. D. should remove unsupported enamel
2. Increasing its length. which may fracture. *
3. Utilizing grooves or boxes. E. should result in a sharp
4. Polishing the preparation. gingivoproximal line angle.

A. (1) (2) (3) *


B. (1) and (3)
C. (2) and (4)

Dr Abdul Naser Tamim SEHA, Dr Ghada Al Aqqad D.D.S, Dr Kamal Naser - Amrita Medical Centre, Dr Emad Wani - Shadi Dental Centre,
Dr Lina Anka SEHA, Dr Rouba Zgheibi SEHA, Maria Teresa Yongson Alejandrino SEHA 383
A bevel is CONTRAINDICATED on the To ensure maximum marginal strength
cavosurface margins of a Class I for an amalgam restoration the
amalgam cavity preparation because cavosurface angle should

A. this type of margin would tend to A. approach 45 degrees.


leak. B. approach 90 degrees. *
B. as the tooth undergoes natural C. be bevelled.
attrition the bevel would be worn away. D. be chamfered.
C. a thin flange of the amalgam
restorative material is prone to fracture* A characteristic sign of aggressive
D. the restoration is more difficult to periodontitis in an adolescent (juvenile
polish. periodontitis) is

Whenever possible, the margins of a A. marginal gingivitis.


restoration should be placed B. painful, burning gingivae.
C. hyperplastic gingivitis.
1. Subgingivally. D. drifting of the teeth. *
2. Supragingivally.
3. on cementum. On an edentulous patient, a panoramic
4. on enamel. radiograph is used to

A. (1) (2) (3) A. determine the level of muscle


B. (1) and (3) attachments.
C. (2) and (4) * B. identify systemic problems affecting
D. (4) only the soft tissues of the mouth.
E. All of the above. C. reveal retained roots or residual
areas of infection. *
For an acid-etched Class III composite D. assess the vertical dimension.
resin, the cavosurface margin of the
cavity can be bevelled to Desquamative gingivitis is seen in

1. Eliminate the need for internal 1. Pemphigus vulgaris.


retention. 2. Mucous membrane pemphigoid.
2. Improve convenience form. 3. Erosive lichen planus.
3. Aid in finishing. 4. Erythema multiforme.
4. Increase the surface area for
etching. A. (1) (2) (3)
B. (1) and (3)
A. (1) (2) (3) C. (2) and (4)
B. (1) and (3) D. (4) only
C. (2) and (4) E. All of the above. *
D. (4) only *
E. All of the above.

Dr Abdul Naser Tamim SEHA, Dr Ghada Al Aqqad D.D.S, Dr Kamal Naser - Amrita Medical Centre, Dr Emad Wani - Shadi Dental Centre,
Dr Lina Anka SEHA, Dr Rouba Zgheibi SEHA, Maria Teresa Yongson Alejandrino SEHA 384
Prognosis for a patient with oral The elastic limit of a material is the
squamous cell carcinoma depends greatest load to which a material can
upon which of the following factors? be subjected to in order that it will

1. Size of the tumor. A. fracture.


2. Location of the tumor. B. remain distorted.
3. Lymph node involvement. C. return to its original dimension*
4. Symptoms. D. return to a point beyond its original
dimension.
A. (1) (2) (3) *
B. (1) and (3) If an alginate impression must be
C. (2) and (4) stored for a few minutes before the
D. (4) only cast is poured, it should be placed in
E. All of the above.
A. water.
World epidemiological data indicates B. 100% relative humidity.*
that periodontal disease is the most C. A 1% aqueous calcium sulfate
likely cause of tooth loss in the solution.
following age group: D. None of the above.

A. 10 - 20 years. The amount of radiation to a patient


B. 20 - 30 years. can be reduced by
C. 30 - 50 years.
D. 65 - 75 years. * 1. Using a high speed film.
E. over 75 years. 2. Using an aluminum filter.
3. Using low kVp.
Particulate hydroxyapatite, when 4. Increasing target-film distance.
placed subperiostially, 5. Decreasing target-film distance.

1. is highly biocompatible. A. (1) (2) (3) (4)


2. Has a low incidence of secondary B. (1) (2) (4) *
infection following surgery. C. (1) (3) (5)
3. has a tendency to migrate following D. (2) (3) (4)
insertion. E. (2) (3) (5)
4. Induces bone formation throughout
the implanted material. Filters are placed in the path of the x-
ray beam to
A. (1) (2) (3)*
B. (1) and (3) A. increase contrast.
C. (2) and (4) B. reduce film density.
D. (4) only C. reduce exposure time.
E. All of the above. ??? D. reduce patient radiation dose. *

Dr Abdul Naser Tamim SEHA, Dr Ghada Al Aqqad D.D.S, Dr Kamal Naser - Amrita Medical Centre, Dr Emad Wani - Shadi Dental Centre,
Dr Lina Anka SEHA, Dr Rouba Zgheibi SEHA, Maria Teresa Yongson Alejandrino SEHA 385
Which of the following will increase C. Bradykinin.
image sharpness on radiographs? D. A. and B.
E. None of the above.
A. Small focal spot.
B. Long source-film distance. Amphetamines
C. Short object-film distance.
D. All of the above. * 1. Increase mental alertness and
decrease fatigue.
Using current radiographic techniques, 2. Are analeptics.
a satisfactory intraoral radiograph can 3. Have no effect on psychomotor
be produced with skin exposures as activity.
low as 4. are useful in controlling
arrhythmias.
A. 1 - 10mR.
B. 100 - 600mR. * A. (1) and (3) *
C. 1 - 5R. B. (2) and (4)
D. 150 - 1,000R. C. (4) only
D. All of the above.
The depth of penetration of any object
by x-rays is determined by The lowest level of fluoride in drinking
water which will cause enamel
1. Milliamperage. mottling is
2. Density of the object.
3. Exposure time. A. 0.5ppm.
4. Kilovoltage. B. 1.0ppm. *
C. 3.0ppm.
A. (1) (2) (3) D. 5.0ppm.
B. (1) and (3)
C. (2) and (4) * A 70-year old female has consumed
D. (4) only optimal levels of fluorides all her life.
E. All of the above. You would expect to find a decreased
incidence of
Which of the following is most often
associated with a non-vital tooth? 1. Paget's disease of bone.
2. Osteoporosis.
A. Periapical cyst. * 3. Hyperparathyroidism.
B. Internal resorption. 4. Dental caries.
C. Periapical cementoma.
D. Hyperplastic pulpitis. A. (1) (2) (3)
B. (1) and (3)
Which of the following has/have C. (2) and (4)
analgesic, antipyretic and anti- D. (4) only *
inflammatory effects? E. All of the above.

A. Acetominophen.
B. Acetylsalicylic acid. *

Dr Abdul Naser Tamim SEHA, Dr Ghada Al Aqqad D.D.S, Dr Kamal Naser - Amrita Medical Centre, Dr Emad Wani - Shadi Dental Centre,
Dr Lina Anka SEHA, Dr Rouba Zgheibi SEHA, Maria Teresa Yongson Alejandrino SEHA 386
Endotoxin is A. tooth morphology.
B. occlusal function. *
A. a cell wall component of gram- C. cementum.
negative bacteria. D. alveolar bone.
B. a potent inflammatory agent.
C. present in diseased root cementum. Normal sulcular epithelium in man is
D. All of the above. *
1. Nonkeratinized.
Gingival inflammation may result from 2. Squamous.
3. Stratified.
A. trauma. 4. Nonpermeable.
B. chemical irritation.
C. plaque. A. (1) (2) (3) *
D. All of the above. * B. (1) and (3)
C. (2) and (4)
Juvenile periodontitis D. (4) only
E. All of the above.
A. is associated with gram-negative
anaerobic flora. Which of the following statements is
B. is associated with gram-positive correct?
anaerobic flora. *
C. is associated with root caries. A. Epithelial cells are united to the
D. has a definite predilection toward basal lamina by hemidesmosomes.
males. B. The basal lamina is divided into
lamina lucida and lamina densa.
Hypoglycemia is characterized by C. The basal lamina is a product of
epithelial cells.
1. Mental confusion. D. All of the above. *
2. Tachycardia.
3. Sweating. Before performing surgery on a patient
4. Nausea. who is taking warfarin, which of the
following should be evaluated?
A. (1) (2) (3)
B. (1) and (3) A. Bleeding time.
C. (2) and (4) B. Clotting time.
D. (4) only C. Prothrombin time. *
E. All of the above. * D. Coagulation time.

The arrangement and character of the


principal fibres of the periodontal
ligament are modified by

Dr Abdul Naser Tamim SEHA, Dr Ghada Al Aqqad D.D.S, Dr Kamal Naser - Amrita Medical Centre, Dr Emad Wani - Shadi Dental Centre,
Dr Lina Anka SEHA, Dr Rouba Zgheibi SEHA, Maria Teresa Yongson Alejandrino SEHA 387
In patients with cleft palates there may
be A. the use of a separating medium. *
B. soaking the die in cold water.
A. an increase in supernumerary teeth. C. burnishing the wax to the die.
B. an increase in congenitally missing D. melting the wax onto the die.
teeth.
C. a higher incidence of crown defects. Extreme resorption of an edentulous
D. All of the above. * mandible can bring the alveolar ridge to
the level of the attachment of the
Permanent first molars begin to calcify
at A. buccinator, styloglossus and
geniohyoid muscles.
A. 1 to 4 months in utero. B. mylohyoid, buccinator and
B. birth. * styloglossus muscles.
C. 3 to 6 months. C. superior constrictor, mylohyoid and
D. 7 to 11 months. buccinator muscles.
E. 12 to 15 months. D. mylohyoid, buccinator and
genioglossus muscles. *
The absence of a pulp chamber in a
deciduous maxillary incisor is most High copper amalgam alloys are
likely due to superior to conventional alloys in that
they have
A. amelogenesis imperfecta.
B. hypophosphatasia. 1. Lower creep.
C. trauma. * 2. Less corrosion.
D. ectodermal dysplasia. 3. Less marginal breakdown.
E. cleidocranial dysostosis. 4. Higher 1 hour compressive strength.

Which of the following maxillary teeth A. (1) (2) (3)


are more likely to present with a palatal B. (1) and (3)
abscess? C. (2) and (4)
D. (4) only
1. Lateral incisors. E. All of the above. *
2. Central incisors.
3. First molars. Upon setting, a mixture of plaster of
4. First premolars. Paris and water will exhibit

A. (1) (2) (3) A. loss in compressive strength.


B. (1) and (3) * B. expansion. *
C. (2) and (4) C. gain in moisture content.
D. (4) only D. contraction.
E. All of the above.

When making an indirect wax pattern


on a stone die, improved adaptation
will result from

Dr Abdul Naser Tamim SEHA, Dr Ghada Al Aqqad D.D.S, Dr Kamal Naser - Amrita Medical Centre, Dr Emad Wani - Shadi Dental Centre,
Dr Lina Anka SEHA, Dr Rouba Zgheibi SEHA, Maria Teresa Yongson Alejandrino SEHA 388
To improve denture stability, 3. A non-rigid connector is placed in
mandibular molar teeth should the fixed partial denture.
normally be placed 4. Its long axis is within 25° of the long
axis of the other abutments.
A. over the crest of the mandibular
ridge * A. (1) (2) (3)
B. buccal to the crest of the mandibular B. (1) and (3)
ridge. C. (2) and (4)
C. over the buccal shelf area. D. (4) only
D. lingual to the crest of the mandibular E. All of the above. *
ridge.
Radiographically, the lamina dura is a
Irreversible hydrocolloid materials are
best removed from the mouth by A. thick layer of bone forming the inner
surface of the alveolus.
A. a quick snap. * B. thin radiolucent line around the roots
B. a slow teasing motion. of the teeth.
C. twisting and rocking. C. thick layer of cortical bone.
D. having the patient create a positive D. thin radiopaque line around the
pressure. roots of the teeth. *

The anatomical landmarks used to help Widening of the periodontal space is


establish the location of the posterior NOT seen radiographically in
palatal seal of a maxillary complete
denture include the A. trauma from occlusion.
B. orthodontic tooth movement.
A. pterygomaxillary notches and the C. scleroderma.
fovea palatinae. * D. Paget's disease. *
B. pterygomaxillary notches and the
posterior nasal spine. In the bisecting angle principle of
C. posterior border of the tuberosities intraoral radiography, the radiopacity
and the posterior border of the palatine that can obliterate the apices of
bone. maxillary molars is the
D. anterior border of the tuberosities,
the palatine raphe and the posterior A. maxillary sinus.
border of the palatine bone. B. palatine bone and the zygoma.
C. orbital process of the zygomatic
A tilted molar can be used as a fixed bone.
partial denture abutment if D. zygoma and the zygomatic process
of the maxilla. *
1. It undergoes orthodontic uprighting
first.
2. A coping and telescopic crown are
used on the abutment.

Dr Abdul Naser Tamim SEHA, Dr Ghada Al Aqqad D.D.S, Dr Kamal Naser - Amrita Medical Centre, Dr Emad Wani - Shadi Dental Centre,
Dr Lina Anka SEHA, Dr Rouba Zgheibi SEHA, Maria Teresa Yongson Alejandrino SEHA 389
Hyperplastic lingual tonsils may
resemble which of the following? A. are found at the apices of nonvital
teeth.
A. Epulis fissuratum. B. occur in infants.
B. Lingual varicosities. C. are a nonlipid reticulo-endotheliosis*
C. Squamous cell carcinoma. * D. are caused by a virus.
D. Median rhomboid glossitis.
E. Prominent fungiform papillae. Osteogenesis imperfecta is manifested
by
Diabetes mellitus is the result of
A. punched-out radiolucencies in the
A. hypersecretion of the posterior jaws.
pituitary. B. numerous unerupted supernumerary
B. atrophy of the islands of teeth.
Langerhans. * C. osteoporosis and anemia.
C. destruction of the adrenal cortex. D. multiple fractures and blue sclera. *
D. destruction of the posterior pituitary
or associated hypothalamic centres. Which of the following has the highest
rate of recurrence?
The most likely origin of a metastic
carcinoma of the mandible is a primary A. Odontogenic keratocyst. *
lesion of the B. Nasoalveolar cyst.
C. Median palatal cyst.
1. Lung. D. Incisive canal cyst.
2. Breast.
3. Prostate. A positive pulp response to the
4. Nasopharynx. application of cold indicates

A. (1) (2) (3) * A. necrosis.


B. (1) and (3) B. periodontal involvement.
C. (2) and (4) C. an acute alveolar abscess.
D. (4) only D. a periodontal abscess.
E. All of the above. E. vitality. *

Which of the following is the most Which of the following conditions is


frequent cause of ankylosis of the characterized by abnormally large pulp
temporomandibular joint? chambers?

A. Intra-articular injection of steroids. A. Amelogenesis imperfecta.


B. Chronic subluxation. B. Regional odontodysplasia. *
C. Trauma. * C. Dentinogenesis imperfecta.
D. Anterior disc dislocation. D. Dentinal dysplasia Type I.

Eosinophilic granulomas

Dr Abdul Naser Tamim SEHA, Dr Ghada Al Aqqad D.D.S, Dr Kamal Naser - Amrita Medical Centre, Dr Emad Wani - Shadi Dental Centre,
Dr Lina Anka SEHA, Dr Rouba Zgheibi SEHA, Maria Teresa Yongson Alejandrino SEHA 390
In placing an amalgam, interproximal D. location of the carious lesion.
wedging will be most effective in
controlling the In pin-retained restorations, the pin
holes should be parallel to the
A. buccal contour.
B. gingival contour. A. long axis of the tooth.
C. lingual contour. B. nearest external surface. *
D. marginal ridge contour. * C. pulp chamber.
D. axial wall.
The principal reason for a cavosurface
bevel on an inlay preparation is to Using pins to retain amalgam
restorations increases the risk of
A. remove undermined enamel.
B. improve marginal adaptation. * 1. Cracks in the teeth.
C. decrease marginal percolation. 2. Pulp exposures.
D. increase resistance and retention 3. Thermal sensitivity.
forms. 4. Periodontal ligament invasion.

Sclerotic dentin is A. (1) (2) (3)


B. (1) and (3)
A. soft. C. (2) and (4)
B. hypersensitive. D. (4) only
C. resistant to caries. * E. All of the above. *
D. injurious to the pulp.
A patient has a smooth, red,
Enlargement of the thyroid gland can protruding lesion at the tip of the
be caused by tongue. Microscopic examination
reveals a stratified squamous
A. insufficient fluoride. epithelium covering loose, fibrous
B. excess iodine. connective tissue with many thin-
C. insufficient iodine. * walled, vascular spaces. The most
D. excess calcium. appropriate diagnosis is
E. excess sodium.
A. fibroma.
What is the most important factor to B. papilloma.
consider when deciding whether or not C. hemangioma. *
to use pulp protection? D. pleomorphic adenoma.
E. granular cell tumour
A. depth of the pulpal floor. (myoblastoma).
B. thickness of the remaining dentin. *
C. amount of carious material
removed.

Dr Abdul Naser Tamim SEHA, Dr Ghada Al Aqqad D.D.S, Dr Kamal Naser - Amrita Medical Centre, Dr Emad Wani - Shadi Dental Centre,
Dr Lina Anka SEHA, Dr Rouba Zgheibi SEHA, Maria Teresa Yongson Alejandrino SEHA 391
Which of the following should be done 1. Relaxes bronchial muscles.
prior to pin hole placement in an 2. Stimulates heart muscle and
extensive amalgam preparation? increases heart rate.
3. increases systolic blood pressure.
1. Examine the radiograph. 4. Produces vasoconstriction in many
2. Determine the subgingival anatomic vascular beds.
contours.
3. Remove caries and unsupported A. (1) (2) (3)
enamel. B. (1) and (2)
4. Place a pilot hole at the dentino- C. (2) and (4)
enamel junction. D. (4) only
E. All of the above. *
A. (1) (2) (3) *
B. (1) and (3) When used for conscious sedation,
C. (2) and (4) nitrous oxide may
D. (4) only
E. All of the above. 1. Produce signs of inherent
myocardial depression.
In X-ray equipment, kilovoltage 2. Produce an indirect
controls sympathomimetic action.
3. Cause the patient to sweat.
1. Contrast. 4. Produce numbness of the
2. Speed of electrons. extremities.
3. Amount of radiation produced.
4. Penetrating power of radiation. A. (1) (2) (3)
5. Temperature of the cathode B. (1) and (3)
filament. C. (2) and (4)
D. (4) only
A. (1) and (2) E. All of the above. *
B. (1) (2) (4) *
C. (1) (3) (5) Streptomycin
D. (1) and (4)
1. Enhances the activity of some
Which of the following, if left untreated, neuromuscular blocking agents.
is most likely to result in a periapical 2. Can induce 8th cranial nerve
lesion? damage.
3. Is a broad-spectrum antibiotic.
A. Internal resorption. 4. Is used to treat tuberculosis.
B. Reversible pulpitis.
C. Acute suppurative pulpitis. * A. (1) (2) (3)
D. Chronic hyperplastic pulpitis. B. (1) and (3)
E. Diffuse calcification of the pulp. C. (2) and (4)
D. (4) only
Epinephrine is one drug used in the E. All of the above. *
management of an acute anaphylactic
reaction (Type I allergic reaction)
because it

Dr Abdul Naser Tamim SEHA, Dr Ghada Al Aqqad D.D.S, Dr Kamal Naser - Amrita Medical Centre, Dr Emad Wani - Shadi Dental Centre,
Dr Lina Anka SEHA, Dr Rouba Zgheibi SEHA, Maria Teresa Yongson Alejandrino SEHA 392
Warfarin (Coumadin®) acts by 4. Decrease the effectiveness of local
anesthetics.
A. preventing formation of
thromboplastin. A. (1) (2) (3)
B. preventing fibrinogen conversion to B. (1) and (3)
fibrin. C. (2) and (4)
C. inhibiting the synthesis of D. (4) only
prothrombin in the liver. * E. All of the above. *
D. incorporating ionic calcium.
Which is the LEAST effective method
Which of the following is the greatest of instrument sterilization?
risk factor for rampant caries in
children? A. Chemical solutions. *
B. Dry heat.
A. Frequent ingestion of C. Chemical autoclave.
polysaccharides. D. Steam autoclave.
B. Frequent ingestion of high sucrose-
containing foods. * Which of the following bacterial types
C. Severe enamel hypoplasia. is implicated in the initiation of
D. Deficiency of vitamin D. gingivitis?

The main functions of fat soluble A. Streptococcus salivarius.


vitamins are: B. Streptococcus mutans.
C. Leptothrix buccalis.
1. Vitamin E is an important D. Actinomyces viscosus. *
antioxidant.
2. Vitamin A is important in the The success of remineralization of
formation of visual purple. "white spot" lesions is dependent upon
3. Vitamin D promotes intestinal the
calcium and phosphate absorption.
4. Vitamin K catalyzes the synthesis of 1. PH of the saliva.
prothrombin. 2. Frequency of the cariogenic
5. Vitamin A maintains the integrity of challenge.
mucous membranes. 3. Availability of mineral ions in saliva.
4. Viscosity of the saliva.
A. (1) (2) (3)
B. (1) (4) (5) A. (1) (2) (3)
C. (2) (3) (4) B. (1) and (3)
D. All of the above. * C. (2) and (4)
D. (4) only
Chronic alcoholism will E. All of the above. *

1. Cause impairment of liver function.


2. Increase tendency to hemorrhage.
3. Delay healing.

Dr Abdul Naser Tamim SEHA, Dr Ghada Al Aqqad D.D.S, Dr Kamal Naser - Amrita Medical Centre, Dr Emad Wani - Shadi Dental Centre,
Dr Lina Anka SEHA, Dr Rouba Zgheibi SEHA, Maria Teresa Yongson Alejandrino SEHA 393
Collagen E. create a smooth finish.

A. is most common in hard tissues. The principal purpose of amalgam


B. forms insoluble high tensile strength trituration is to
fibres.
C. has a triple helical structure. A. coat the alloy particles with mercury*
D. All of the above. * B. dissolve all the alloy particles in the
mercury.
A lowering of serum calcium is the C. reduce the size of the crystals as
stimulus for the endogenous release of rapidly as they form.
D. reduce mercury content of the
A. thyroid hormone. restoration.
B. adrenocortical hormone.
C. insulin. The location of a crown margin is
D. parathyroid hormone. * determined by
E. adrenalin.
1. Esthetic requirements.
Which of the following is classified as a 2. Clinical crown length.
muscle of mastication? 3. Presence of caries.
4. Presence of an existing restoration.
A. Temporalis. * A. (1) (2) (3)
B. Buccinator. B. (1) and (3)
C. Digastric. C. (2) and (4)
D. Mylohyoid. D. (4) only
E. All of the above. *
Which of the following muscles has two
separate functions in mandibular The retention of an indirect, extra-
movement? coronal restoration can be improved by

A. Masseter. 1. Decreasing the taper of the


B. Geniohyoid. preparation wall.
C. External (lateral) pterygoid. * 2. Lengthening the clinical crown.
D. Buccinator. 3. Adding grooves.
4. An antirotation key.
After initial setting, glass ionomer
cements should have a coating agent A. (1) (2) (3)
applied in order to B. (1) and (3)
C. (2) and (4)
A. hasten the final set. D. (4) only
B. protect the cement from moisture. * E. All of the above. *
C. retard the final set.
D. protect the cement from ultraviolet
light.

Dr Abdul Naser Tamim SEHA, Dr Ghada Al Aqqad D.D.S, Dr Kamal Naser - Amrita Medical Centre, Dr Emad Wani - Shadi Dental Centre,
Dr Lina Anka SEHA, Dr Rouba Zgheibi SEHA, Maria Teresa Yongson Alejandrino SEHA 394
Which of the following is NOT a direct Varying the mercury content of an
physiological response to additional amalgam results in which of the
forces placed on abutment teeth? following?

A. Resorption of bone. A. The higher the mercury content the


B. Increase thickness of cementum. greater the strength.
C. Increased density in cribiform plate. B. The higher the mercury content the
D. Decrease in width of periodontal less the flow.
ligament. * C. The lower the mercury content the
greater the flow.
The retentive arm of a combination D. The lower the mercury content the
clasp (wrought wire retentive arm and greater the strength. *
cast lingual arm) is better than a cast
arm because it The prime advantage of vacuum firing
of porcelain is
A. has a lower yield strength.
B. produces less stress on the A. better colour.
abutment tooth during removal and B. less shrinkage.
insertion. * C. more translucency.
C. can be used to engage deeper D. increased strength. *
undercuts due to a high modulus of
elasticity. Glass ionomer cement restorations are
D. is economical to fabricate. indicated for

After initial setting, a chemically cured A. root caries. *


glass ionomer cement restoration B. incisal edge fractures.
should have a coating agent applied to C. Class II lesions in adults.
D. locations where esthetics are
A. hasten the final set. important.
B. protect the cement from moisture.*
C. retard the final set. A maxillary complete denture exhibits
D. protect the cement from ultraviolet more retention and stability than a
light. mandibular one because it
E. create a smooth finish.
1. Covers a greater area.
Dental amalgam restorations 2. Incorporates a posterior palatal seal.
3. Is not subject to as much muscular
A. show decreased flow when the tin displacement.
mercury phase is increased. 4. Is completely surrounded by soft
B. contain nickel to increase the yield tissue.
strength.
C. show decreased corrosion and A. (1) (2) (3) *
marginal breakdown when the copper B. (1) and (3)
tin phase is increased. * C. (2) and (4)
D. contain zinc to reduce galvanic D. (4) only
corrosion. E. All of the above.

Dr Abdul Naser Tamim SEHA, Dr Ghada Al Aqqad D.D.S, Dr Kamal Naser - Amrita Medical Centre, Dr Emad Wani - Shadi Dental Centre,
Dr Lina Anka SEHA, Dr Rouba Zgheibi SEHA, Maria Teresa Yongson Alejandrino SEHA 395
The greatest dimensional change in C. syneresis.
denture bases will occur D. absorption.

A. after the dentures have been in the Inlay wax patterns should be invested
mouth 24 hours. as soon as possible in order to
B. when the dentures have been decrease distortion caused by
stored in tap water at room
temperature. A. reduced flow.
C. when a complete denture opposes B. drying-out of the wax.
natural teeth. C. release of internal stress. *
D. at the time the dentures are D. continued expansion of the wax.
removed from the flask. *
Gold alloy can be given a white colour
Distortion of a wax pattern is mainly by introducing sufficient
due to
A. zinc.
A. insufficient plasticity during B. copper.
manipulation. C. cadmium.
B. insufficient bulk of material. D. palladium. *
C. relaxation of stresses introduced
during manipulation. * Gold contributes which of the
D. insufficient paraffin ingredient. following properties to a gold-copper
E. refrigeration. alloy?

The modulus of elasticity of a material A. Corrosion resistance. *


is determined by B. Increased strength.
C. Lowered specific gravity.
A. dividing stress by strain below D. Increased hardness.
elastic limit. *
B. dividing strain by stress. A decrease in the particle size of the
C. multiplying proportional limit by amalgam alloy will affect the
strain. amalgam by
D. squaring proportional limit and
dividing by strain. A. increasing flow.
B. decreasing expansion.
Molecular attraction between unlike C. retarding setting rate.
substances is called D. increasing early strength. *

A. adhesion. *
B. cohesion.

Dr Abdul Naser Tamim SEHA, Dr Ghada Al Aqqad D.D.S, Dr Kamal Naser - Amrita Medical Centre, Dr Emad Wani - Shadi Dental Centre,
Dr Lina Anka SEHA, Dr Rouba Zgheibi SEHA, Maria Teresa Yongson Alejandrino SEHA 396
The polishing of an amalgam The main reason for adding copper to
restoration a dental amalgam alloy is to

A. should not be performed before 24 A. increase expansion.


hours after insertion. * B. reduce tarnish resistance.
B. completely prevents tarnish from C. make amalgamation easier.
occurring. D. increase lustre.
C. removes the mercury rich surface E. reduce the tin-mercury phase. *
layer of the amalgam.
D. is more permanent if the surface is Dental amalgam
heated during the procedure.
A. is subject to solubility and
Which of the following would occur if a disintegration in the oral fluids.
zinc containing amalgam is B. has a satisfactory resistance to
contaminated with saliva during compressive stress. *
condensing? C. adheres chemically to the walls of
the cavity.
1. No change in compressive strength
but lower tensile strength. Where cavity preparations are
2. Increased expansion. extensive, polycarboxylate cement can
3. Reduced flow or creep. be used as a base material because
4. Increased surface pitting.
A. its pH stimulates secondary dentin
A. (1) (2) (3) formation.
B. (1) and (3) B. it interacts with setting amalgam to
C. (2) and (4) * form a weak chemical union.
D. (4) only C. it is biocompatible with the pulp. *
E. All of the above. D. it is compressible when set.

Dental amalgams that are made from Which of the following are
alloys containing 6% copper, compared characteristics of restorative glass
to those made from alloys containing ionomer cements?
13% copper
1. Release of fluoride.
1. Are more resistant to tarnish and 2. Bonding to enamel.
corrosion. 3. Setting not affected by a well-
2. Demonstrate less creep or flow. controlled moist environment.
3. Demonstrate less marginal 4. Irritating to pulpal tissues.
breakdown in clinical service.
4. Generally have lower compressive A. (1) (2) (3) *
strength. B. (1) and (3)
C. (2) and (4)
A. (1) (2) (3) D. (4) only
B. (1) and (3) E. All of the above.
C. (2) and (4)
D. (4) only *
E. All of the above.

Dr Abdul Naser Tamim SEHA, Dr Ghada Al Aqqad D.D.S, Dr Kamal Naser - Amrita Medical Centre, Dr Emad Wani - Shadi Dental Centre,
Dr Lina Anka SEHA, Dr Rouba Zgheibi SEHA, Maria Teresa Yongson Alejandrino SEHA 397
Effective tissue displacement with
elastic impression materials can be For a patient with new complete
accomplished by dentures, errors in centric occlusion are
best adjusted by
1. Injection of the material into the
gingival sulcus. A. directing the patient to close the
2. Placement of chemical-impregnated jaws, bringing the teeth into occlusion.
cords into the gingival sulcus. B. having the patient close in centric
3. Electrosurgical means. occlusion and making a transfer record
4. A firm tray material. to the articulator.
C. having the patient leave the
A. (1) (2) (3) dentures out of the mouth for 24 hours.
B. (1) and (3) D. remounting the dentures in the
C. (2) and (4) articulator using remount casts and
D. (4) only new interocclusal records. *
E. All of the above. *
A distal extension removable partial
Upon examination of an edentulous denture is one that receives its support
patient, it is observed that the
tuberosities contact the retromolar A. from the tissues only.
pads at the correct occlusal vertical B. mostly from the tissues. *
dimension. The treatment of choice is C. mostly from the abutment teeth.
to D. equally from the abutment teeth and
the tissues.
A. reduce the retromolar pads
surgically to provide the necessary An excessively thick palatal bar of a
clearance. maxillary partial denture will
B. reduce the tuberosities surgically to
provide the necessary clearance. * A. cause injury to the abutment teeth.
C. construct new dentures at an B. distort under occlusal stress.
increased occlusal vertical dimension C. irritate the palatal tissues.
to gain the necessary clearance. D. cause difficulty in swallowing. *
D. proceed with construction of the
denture and reduce the posterior Polysulfide impression materials
extension of the mandibular denture to
eliminate interferences. A. have a lower "tear strength" than
condensation silicone.
Which of the following structures B. have a better dimensional stability
affects the thickness of the flange of a than addition cured silicones.
maxillary complete denture? C. have a better detail reproduction
than both condensation and addition
A. Malar process. polymerization silicones.
B. Coronoid process. * D. demonstrate, after polymerization,
C. Mylohyoid ridge. cross-linking chains. *
D. Zygomatic process.
E. Genial tubercle.

Dr Abdul Naser Tamim SEHA, Dr Ghada Al Aqqad D.D.S, Dr Kamal Naser - Amrita Medical Centre, Dr Emad Wani - Shadi Dental Centre,
Dr Lina Anka SEHA, Dr Rouba Zgheibi SEHA, Maria Teresa Yongson Alejandrino SEHA 398
The accuracy of the polysulfide and
silicone rubber impression materials 1. Low compressive strength.
2. High hardness.
A. is better than that of polyethers. 3. High tensile strength.
B. compares favorably with reversible 4. Low impact strength.
hydrocolloids. *
C. is inversely proportional to A. (1) (2) (3)
temperature and humidity. B. (1) and (3)
C. (2) and (4) *
Dental polysulfide rubber impression D. (4) only
materials are polymerized with the E. All of the above.
following initiator:
In comparison to gold alloys, base
A. lead peroxide. * metal alloy removable partial denture
B. sodium peroxide. frameworks can be made
C. carbon disulfide.
D. hydrogen peroxide. A. more corrosion resistant.
E. phosphorus pentoxide. B. lighter. *
C. more ductile.
After processing, complete dentures D. with greater casting accuracy.
on the original stone casts are
rearticulated in order to correct The stiffness of a material can best be
occlusal disharmony produced by described by the

A. flasking and processing A. modulus of elasticity. *


procedures.* B. percentage elongation.
B. inaccurate jaw relation records. C. modulus of resilience.
C. errors in registering of centric D. elastic limit.
relation record.
In an edentulous maxilla, the direction
If hydroquinone is added to the of resorption of the alveolar ridge is
monomer to, it will
A. upward and palatally. *
A. inhibit polymerization. * B. upward and facially.
B. initiate polymerization. C. uniform in all directions.
C. activate polymerization. D. upward only.
D. None of the above.

Dental porcelain has

Dr Abdul Naser Tamim SEHA, Dr Ghada Al Aqqad D.D.S, Dr Kamal Naser - Amrita Medical Centre, Dr Emad Wani - Shadi Dental Centre,
Dr Lina Anka SEHA, Dr Rouba Zgheibi SEHA, Maria Teresa Yongson Alejandrino SEHA 399
To evaluate an existing occlusion, 1. Terminate in retentive undercuts
diagnostic casts should be mounted lying gingival to the height of contour.
on an articulator in 2. Originate from the framework and
approach the tooth undercut area from
A. centric relation. * a gingival direction.
B. balancing occlusion. 3. Provide retention by the resistance
C. either centric relation or balancing of metal to deformation rather than
occlusion. frictional resistance of parallel walls.
D. horizontal protrusive relation. 4. are circular in cross-section.

The crown-root ratio is A. (1) (2) (3)


B. (1) and (3) *
1. The comparison of the length of C. (2) and (4)
root retained in bone to the amount of D. (4) only
tooth external to it. E. All of the above.
2. An important factor in abutment
tooth selection. A removable partial denture rest should
3. Determined from radiographs.
4. Determined during surveying of the A. be extended for retention.
diagnostic cast. B. increase retention of the partial
denture.
A. (1) (2) (3) * C. direct forces parallel to the long axis
B. (1) and (3) of the abutment. *
C. (2) and (4) D. be located on a centric contact.
D. (4) only
E. All of the above. Following the insertion of complete
dentures, a generalized soreness over
A removable partial denture rest the entire mandibular alveolar ridge
should be placed on the lingual can be caused by
surface of a canine rather than on the
incisal surface because A. Inadequate interocclusal distance.*
B. impingement on the buccal frenum.
A. less leverage is exerted against the C. high muscle attachments.
tooth by the rest. * D. excess border thickness.
B. the enamel is thicker on the lingual
surface. Unsupported, hyperplastic tissue in an
C. visibility and access are better. edentulous maxilla is most often found
D. the cingulum of the canine provides
a natural recess. A. near the tuberosities.
B. in the vault.
I-bar clasp arms and circumferential C. in the anterior segment of the arch. *
clasp arms both

Dr Abdul Naser Tamim SEHA, Dr Ghada Al Aqqad D.D.S, Dr Kamal Naser - Amrita Medical Centre, Dr Emad Wani - Shadi Dental Centre,
Dr Lina Anka SEHA, Dr Rouba Zgheibi SEHA, Maria Teresa Yongson Alejandrino SEHA 400
A facebow is used to record the Which of the following are effects of cold
working a metal?
1. Vertical dimension of occlusion.
2. Intercondylar distance. 1. Increase in ductility.
3. Horizontal condylar inclination. 2. Decrease in ductility.
4. Relationship of the maxilla to the hinge axis. 3. Increase in hardness.
4. Increase in resistance to corrosion.
A. (1) (2) (3)
B. (1) and (3) A. (1) and (2)
C. (2) and (4) B. (1) and (4)
D. (4) only. * C. (2) and (3) *
E. All of the above. D. (3) and (4)

Reversible hydrocolloids transform from sol to In the preparation of gypsum products, an


gel and gel to sol as a function of the increase in the water/powder ratio will

A. concentration of the fillers and plasticizers. A. increase the surface hardness.


B. percent composition by weight of water. B. increase the compressive strength.
C. concentration of potassium sulfate. C. accelerate the setting reaction.
D. temperature. * D. None of the above. *

Which of the following should be checked first


when a cast gold crown that fits on its die
cannot be seated on its abutment?
A patient wearing complete dentures has
A. The occlusal contacts. angular cheilitis. The most likely cause is
B. The taper of the preparation.
C. The proximal contacts. * A. increased vertical dimension.
D. The impression used to pour the cast. B. insufficient horizontal overlap (overjet).
C. decreased vertical dimension. *
Centric relation is a D. excessive vertical overlap (overbite).

A. relation of the maxilla to the rest of the skull. Rests on terminal abutment teeth for a cast
B. vertical relationship of the mandible to the metal removable partial denture provides
maxilla.
C. horizontal relationship of the mandible to the A. primary retention.
maxilla. * B. indirect retention.
D. rest position of the mandible. C. vertical stability. *
D. lateral force transmission.
Compared with zinc-phosphate cement,
polycarboxylate cement has

A. longer working time.


B. lower film thickness.
C. increased compressive strength.
D. superior biologic compatibility. *

The line drawn through the occlusal removable partial denture is the
rests of two principal abutments for a

Dr Abdul Naser Tamim SEHA, Dr Ghada Al Aqqad D.D.S, Dr Kamal Naser - Amrita Medical Centre, Dr Emad Wani - Shadi Dental Centre,
Dr Lina Anka SEHA, Dr Rouba Zgheibi SEHA, Maria Teresa Yongson Alejandrino SEHA 401
A. survey line. abutment teeth when the retentive
B. terminal line. arms pass over the height of contour in
C. axis of rotation. * order to
D. line of greatest torque.
A. prevent distortion of the clasps.
In the design of a removable partial B. assure complete seating of the
denture, guiding planes are made framework.
C. provide needed support to abutment
A. parallel to the long axis of the tooth. teeth during a period of added stress. *
B. parallel to the path of insertion. *
C. at a right angle to the occlusal Wrought gold alloy clasps are superior
plane. to cast gold clasps of the same cross-
D. at a right angle to the major sectional area because they
connector.
1. Are coarser grained.
In removable partial denture design, 2. Exhibit greater flexibility
the surface of the abutment tooth most 3. are more accurately adapted to the
often altered to provide clasp tooth.
reciprocity is 4. Have a higher proportional limit.

A. mesial. A. (1) (2) (3)


B. distal. B. (1) and (3)
C. occlusal. C. (2) and (4) *
D. buccal. D. (4) only
E. lingual. * E. All of the above.

In designing a removable partial The muscle of the floor of the mouth in


denture, the location of clasps is the molar region which requires special
determined on attention in the final mandibular
denture impression is the
A. a master cast after tooth
preparations are completed. A. genioglossus.
B. a surveyed diagnostic cast prior to B. geniohyoid.
preparation of occlusal rest areas. * C. mylohyoid. *
C. the refractory cast. D. hyoglossus.

Clasps should be designed so that


upon insertion or removal of a partial
denture the reciprocal arms contact the

Dr Abdul Naser Tamim SEHA, Dr Ghada Al Aqqad D.D.S, Dr Kamal Naser - Amrita Medical Centre, Dr Emad Wani - Shadi Dental Centre,
Dr Lina Anka SEHA, Dr Rouba Zgheibi SEHA, Maria Teresa Yongson Alejandrino SEHA 402
During the fabrication of new complete A. the retainers can be so prepared as
dentures, which of the following can be to have equal retentive qualities.
modified to achieve the desired B. 2 or 3 teeth are to be replaced.
occlusion? C. constructing a mandibular fixed
prosthesis.
1. The compensating curve. D. the abutments cannot be prepared
2. The orientation of the occlusal plane. in parallel without excessive removal of
3. The cusp inclination. tooth structure. *
4. The condylar inclination.
The gingival margin of the preparation
A. (1) (2) (3) * for a full crown on a posterior tooth,
B. (1) and (3) with a clinical crown that satisfies the
C. (2) and (4) requirements for retention and
D. (4) only resistance, should be placed
E. All of the above.
A. 0.5mm subgingivally.
The vertical relation of rest is B. on the enamel.
C. at least 1mm supragingivally. *
A. the same as the vertical relation of D. at the cemento-enamel junction.
occlusion. E. at the gingival margin.
B. greater than the vertical relation of
occlusion. * A cast post and core is used to
C. less than the vertical relation of
occlusion. 1. Provide intraradicular venting.
D. the same as the interocclusal 2. Strengthen a weakened tooth.
distance. 3. Redirect the forces of occlusion.
4. Provide retention for a cast crown.
Adjustment of the occlusal plane of
natural teeth opposed by a complete or A. (1) (2) (3)
partial denture should be completed B. (1) and (3)
C. (2) and (4)
A. after the teeth have been set on the D. (4) only *
trial denture. E. All of the above.
B. immediately after making the final
casts. An epinephrine-containing retraction
C. upon delivery of the denture. cord has the potential of
D. after the diagnosis and treatment
plan has been established. * A. interfering with the setting of the
impression material.
A "broken stress" or "non-rigid" B. causing tissue necrosis.
connector is indicated for a fixed partial C. producing a systemic reaction. *
denture when D. discolouring gingival tissue.

Dr Abdul Naser Tamim SEHA, Dr Ghada Al Aqqad D.D.S, Dr Kamal Naser - Amrita Medical Centre, Dr Emad Wani - Shadi Dental Centre,
Dr Lina Anka SEHA, Dr Rouba Zgheibi SEHA, Maria Teresa Yongson Alejandrino SEHA 403
"Cuspid guided occlusion" occurs when The major disadvantage of zinc
the phosphate used to cement crowns is
the
A. teeth on the nonworking side make
contact in lateral excursions. A. development of heat during setting.
B. teeth on the working side make B. pulp irritation. *
contact in lateral excursions. C. lack of edge strength.
C. canine and lateral incisors make D. low crushing strength.
contact in lateral excurison.
D. posterior teeth make no contact in The principal ingredient of a zinc
lateral excursions on the working side * phosphate cement powder is

A fixed bridge pontic should A. zinc phosphate.


B. silica.
1. Restore tooth function. C. magnesium oxide.
2. Reduce thermal conductivity. D. zinc-oxide. *
3. be biologically acceptable. E. calcium hydroxide.
4. Reduce galvanic reactions between
abutments and other restorations. Which of the following prevents
distortion of a reversible hydrocolloid
A. (1) (2) (3) impression material?
B. (1) and (3) *
C. (2) and (4) A. Slow removal from undercuts.
D. (4) only B. Storage in 100% humidity for 30
E. All of the above. minutes. *
C. Storage in air.
A pontic replacing a mandibular first D. Storage in 2% solution of potassium
molar should be designed so that sulfate for 60 minutes.

1. It seals the muco-gingival field. The immersion of a hydrocolloid


2. It has opened gingival embrasures. impression in 2% potassium sulphate
3. It conceals the porcelain to metal for 2 to 5 minutes will
junction on its gingival surface.
4. Its gingival surface is convex in all A. retard the set of the stone.
directions. B. accelerate the set of the stone. *
C. inhibit the formation of bubbles in the
A. (1) (2) (3) stone.
B. (1) and (3) D. minimize the distortion of the
C. (2) and (4) * hydrocolloid material during the storage
D. (4) only time.
E. All of the above.

Dr Abdul Naser Tamim SEHA, Dr Ghada Al Aqqad D.D.S, Dr Kamal Naser - Amrita Medical Centre, Dr Emad Wani - Shadi Dental Centre,
Dr Lina Anka SEHA, Dr Rouba Zgheibi SEHA, Maria Teresa Yongson Alejandrino SEHA 404
Heated impression modeling compound The main purpose of flux in soldering
is "tempered" in warm water before is to
placement in the mouth in order to
A. dissolve surface oxides and prevent
A. avoid burning the soft tissues. * further oxidation. *
B. reduce contraction error. B. prevent recrystallization and grain
C. initiate a chemical reaction. growth.
D. minimize distortion. C. prevent oxidation and lower the
melting range of the solder.
In soldering nickel-cobalt-chromium D. dissolve surface oxides and lower
alloys and stainless steel, the function the melting range.
of the fluoride flux is
The addition of platinum to a dental
A. to lower the melting temperature of gold alloy results in increased
the solder.
B. to reduce the copper-oxide content 1. Strength.
of the alloy. 2. Hardness.
C. to stop the flow of the molten solder 3. Melting point.
onto undesired areas. 4. Resistance to corrosion.
D. to reduce the formation of chromium
oxide during soldering. * A. (1) (2) (3)
B. (1) and (3)
During the setting phase, a dental C. (2) and (4)
stone mixture will exhibit D. (4) only
E. All of the above. *
A. expansion. *
B. contraction. The gypsum material used for
C. loss in compressive strength. fabrication of dies exhibits a higher
D. gain in moisture content. crushing strength than does regular
stone because of
While the teeth are set in wax, dentures
are tried in to A. difference in particle shape and
density. *
1. Verify the maxillomandibular records. B. difference in the chemical
2. Verify the vertical dimension of composition of the powder particles.
occlusion. C. exothermic heat of setting.
3. Evaluate esthetics.
4. Assess facial contour and lip High humidity in a room where zinc
support. oxide and eugenol impression paste is
being mixed will
A. (1) (2) (3)
B. (1) and (3) A. increase the setting time.
C. (2) and (4) B. not affect the setting.
D. (4) only C. prevent any setting.
E. All of the above. * D. decrease the setting time. *

Dr Abdul Naser Tamim SEHA, Dr Ghada Al Aqqad D.D.S, Dr Kamal Naser - Amrita Medical Centre, Dr Emad Wani - Shadi Dental Centre,
Dr Lina Anka SEHA, Dr Rouba Zgheibi SEHA, Maria Teresa Yongson Alejandrino SEHA 405
From Type I to Type IV gold alloys
there is 1. Determine the path of insertion.
2. Locate tooth surfaces that can act
A. an increase in gold content. as guiding planes.
B. no change in the gold content. 3. Locate retention areas.
C. a reduction in gold content. * 4. Locate the height of contour.
D. a reduction in platinum content.
A. (1) (2) (3)
The higher modulus of elasticity of a B. (1) and (3)
chromium-cobalt-nickel alloy, C. (2) and (4)
compared to a Type IV gold alloy, D. (4) only
means that chromium-cobalt-nickel E. All of the above. *
partial denture clasp will require
Residual soft tissue interdental
A. a heavier cross section for a clasp craters not associated with underlying
arm. bony changes are eliminated by
B. a shorter retentive arm.
C. more taper. A. root planing.
D. a shallower undercut. * B. subgingival curettage.
C. flap operation.
In taking an impression with D. gingivoplasty. *
polysulfide or silicone materials, if the E. None of the above.
heavy bodied tray material begins to
set before seating, the resultant die Correction of an inadequate zone of
will attached gingiva on several adjacent
teeth is best accomplished with a/an
A. not be affected dimensionally.
B. be overall smaller. * A. apically repositioned flap.
C. be overall larger. B. laterally positioned sliding flap.
D. develop a rough surface texture. C. double-papilla pedicle graft.
E. develop bubbles at the interface of D. coronally positioned flap.
the syringe and tray material. E. free gingival graft. *
In the design of a removable partial
denture, the objectives of surveying
the diagnostic cast are to

Dr Abdul Naser Tamim SEHA, Dr Ghada Al Aqqad D.D.S, Dr Kamal Naser - Amrita Medical Centre, Dr Emad Wani - Shadi Dental Centre,
Dr Lina Anka SEHA, Dr Rouba Zgheibi SEHA, Maria Teresa Yongson Alejandrino SEHA 406
The benefits of open flap debridement
alone include A removable full-arch occlusal splint is
used to
A. direct access for thorough
debridement. A. reduce pocket formation.
B. pocket reduction. B. allow for individual tooth movement.
C. increased opportunity for new C. reduce unfavorable forces on teeth. *
attachment. D. permit eruption or elongation of
D. A. and B. teeth.
E. All of the above. *
The tissues of the epithelial attachment
The coronal collagen fibres of the
periodontium are A. are dynamic rather than static.
B. can be reconstituted by repair.
A. circular. C. exhibit a high rate of biologic
B. transeptal. turnover.
C. supraperiosteal. D. All of the above. *
D. All of the above. * E. None of the above.

The primary objective of initial Following root planing, the amount of


periodontal therapy is to gingival shrinkage depends upon

A. reduce occlusal trauma. A. the thickness of the gingiva.


B. make adequate dietary and B. the degree of gingival edema
nutritional adjustments. present.
C. remove the colonized masses of C. whether the pocket orifice is broad or
microorganisms and calculus. * narrow.
D. eliminate crowded and tilted teeth. D. the degree of suppuration present.
E. All of the above. *
Destructive occlusal forces can be
reduced by Chronic gingival inflammation is best
eliminated by
1. Selective grinding.
2. Orthodontics. A. gingival surgery.
3. Restorative treatment. B. regular use of a water-irrigating
4. Tooth extraction. device.
C. root planing and curettage. *
A. (1) (2) (3) D. occlusal correction.
B. (1) and (3) ?? E. splinting.
C. (2) and (4)
D. (4) only
E. All of the above. *

Dr Abdul Naser Tamim SEHA, Dr Ghada Al Aqqad D.D.S, Dr Kamal Naser - Amrita Medical Centre, Dr Emad Wani - Shadi Dental Centre,
Dr Lina Anka SEHA, Dr Rouba Zgheibi SEHA, Maria Teresa Yongson Alejandrino SEHA 407
The instrument best suited for root 3. Interproximal areas.
planing is a/an 4. Bifurcations and trifurcations.

A. hoe. A. (1) (2) (3)


B. file. B. (1) and (3)
C. curette. * C. (2) and (4)
D. sickle scaler. D. (4) only
E. ultrasonic scaler. E. All of the above. *

Primary herpetic gingivostomatitis The epithelial attachment does not


most frequently occurs migrate apically in

A. before age 10. * A. juvenile periodontitis.


B. between l0 and 20 years of age. B. hyperplastic gingivitis. *
C. between 20 and 30 years of age. C. chronic periodontitis.
D. after age 30. D. rapidly progressive periodontitis.
E. at any age.
In periodontics, the best prognosis for
Necrotizing ulcerative gingivitis bone regeneration follows the surgical
(NUG) and acute herpetic treatment of
gingivostomatitis can be
differentiated clinically by (the) A. suprabony pockets.
B. one-wall infrabony pockets.
A. location of the lesions. * C. two-wall infrabony pockets.
B. temperature of the patient. D. three-wall infrabony pockets. *
C. pain.
D. lymphadenopathy. The predominant organisms associated
with active periodontitis are
The absence of adequate drainage
in a periodontal pocket may result in 1. Cocci.
2. Rods.
A. cyst formation. 3. Spirochetes.
B. abscess formation. * 4. Motile rods.
C. epithelial hyperplasia.
D. increased calculus formation. A. (1) and (2)
B. (3) and (4) *
Infraosseous defects may occur at C. (1) only
the D. (1) and (3)
E. All of the above.
1. Palatal surface of maxillary
anterior teeth.
2. Buccal and lingual surfaces of
molars.

Dr Abdul Naser Tamim SEHA, Dr Ghada Al Aqqad D.D.S, Dr Kamal Naser - Amrita Medical Centre, Dr Emad Wani - Shadi Dental Centre,
Dr Lina Anka SEHA, Dr Rouba Zgheibi SEHA, Maria Teresa Yongson Alejandrino SEHA 408
An increase of immunoglobulins is C. incisal edges.
consistent with increased numbers of D. facial surfaces of teeth. *

A. fibroblasts. Regarding dental caries, which of the


B. neutrophils. following is correct?
C. lymphocytes.
D. plasma cells. ?* A. All carbohydrates are equally
cariogenic.
The most important objective of B. More frequent consumption of
occlusal adjustment of a natural carbohydrates increases the risk. *
dentition is to C. The rate of carbohydrate clearance
from the oral cavity is not significant.
A. prevent temporomandibular joint D. Increased dietary fat increases the
syndrome. risk.
B. increase the shearing action in
mastication. A clenching habit may be a factor in
C. improve oral hygiene by preventing
food impaction. A. suprabony periodontal pocket
D. achieve a more favorable direction formation.
and distribution of forces of occlusion* B. marginal gingivitis.
C. increased tooth mobility. *
Carious lesions are most likely to D. generalized recession.
develop if a patient has
DMF-S is an index for expressing
A. a high lactobacillus count.
B. saliva with low buffering capacity* A. dental needs.
C. plaque on his teeth. B. tooth mortality.
D. lactic acid in his mouth. C. extent of dental neglect.
D. dental caries. *
The physiologic wear of hard dental
tissue resulting from mastication is In a young patient living in an area with
known as communal water fluoridation, the
fluoride concentration of an erupted
A. decalcification. tooth is greatest
B. attrition. *
C. abrasion. A. at the dentino-enamel junction.
D. erosion. B. on the surface of the clinical crown.
C. at the layer of dentin nearest the
Abrasion is most commonly seen on pulp chamber *
the D. evenly throughout the enamel.

A. lingual surface of posterior teeth.


B. occlusal surface of posterior teeth.

Dr Abdul Naser Tamim SEHA, Dr Ghada Al Aqqad D.D.S, Dr Kamal Naser - Amrita Medical Centre, Dr Emad Wani - Shadi Dental Centre,
Dr Lina Anka SEHA, Dr Rouba Zgheibi SEHA, Maria Teresa Yongson Alejandrino SEHA 409
Vitamin C is essential for
Irregularly distributed shallow to
A. formation of collagen. moderate craters in the interseptal
B. osteoid. bone are best eliminated by
C. dentin.
D. cementum. A. osteoplasty. *
E. All of the above. * B. gingivoplasty.
C. deep scaling.
The principal component of the fibres D. bone grafting.
of the periodontal ligament is
Which treatment procedure is
A. elastin. indicated for a patient with
B. reticulin. asymptomatic age related gingival
C. fibronectin. recession?
D. collagen. *
A. Connective tissue graft.
In normal gingiva, the predominant B. Gingivoplasty.
microflora of gingival plaque are C. Lateral sliding flap.
D. Gingival graft.
A. gram-positive cocci. * E. No treatment. *
B. gram-negative cocci.
C. gram-negative facultative and A protective bite plate is indicated in
anaerobic rods. all of the following EXCEPT to
D. spirochetes.
A. reduce tooth mobility.
The tooth surfaces LEAST susceptible B. prevent excessive tooth wear.
to caries are C. control a bruxism habit.
D. manage temporomandibular joint
A. mesial of the maxillary arch. pain dysfunction syndrome.
B. lingual of the mandibular arch. * E. prevent migration of teeth. *
C. distal of the maxillary arch.
D. occlusal of the mandibular arch. Gingival hyperplasia may be
E. lingual of the maxillary arch.
A. familial.
Vitamin D is a factor in B. idiopathic.
C. drug induced.
A. caries susceptibility. D. All of the above. *
B. calculus formation. E. None of the above.
C. calcium absorption. *
D. repair of hypoplastic defects of the
enamel.

Dr Abdul Naser Tamim SEHA, Dr Ghada Al Aqqad D.D.S, Dr Kamal Naser - Amrita Medical Centre, Dr Emad Wani - Shadi Dental Centre,
Dr Lina Anka SEHA, Dr Rouba Zgheibi SEHA, Maria Teresa Yongson Alejandrino SEHA 410
A characteristic of a periodontal Plaque accumulation on tooth surfaces
pocket is is affected by

A. gingival edema. A. the anatomy, position and surface


B. gingival hyperplasia. characteristics of the teeth.
C. alveolar bone loss. * B. the architecture of the gingival
tissues and their relationship to the
Caries in older persons is most teeth.
frequently found on which of the C. friction at the tooth surface from the
following locations? diet, lips and tongue.
D. All of the above. *
A. Pits and fissures.
B. Proximal enamel. Which of the following root surfaces
C. Root surfaces. * are most likely to have concavities that
D. Incisal dentin. will make root planing difficult?

Overhangs on restorations initiate 1. Mesial surfaces of maxillary first


chronic inflammatory periodontal premolars.
disease by 2. Mesial surfaces of mandibular
incisors.
A. increasing plaque retention. * 3. Mesial surfaces of maxillary incisors.
B. increasing food retention. 4. Distal surfaces of mandibular
C. causing traumatic occlusion. second premolars.
D. causing pressure atrophy.
A. (1) and (2) *
Abnormalities in blood clotting may B. (1) and (3)
be associated with a deficiency of C. (1) and (4)
vitamin D. (2) and (4)
E. (3) and (4)
A. B12.
B. C. The radiographic term used to describe
C. E. the dense bone of the socket and
D. K. * septal crest is

Calculus contributes to gingival A. periodontal ligament space.


inflammation by B. cancellous bone.
C. cribriform plate.
A. having a porous surface. D. lamina dura. *
B. having cytotoxic bacterial products. E. cortical bone.
C. promoting bacterial colonization.
D. all of the above. *

Dr Abdul Naser Tamim SEHA, Dr Ghada Al Aqqad D.D.S, Dr Kamal Naser - Amrita Medical Centre, Dr Emad Wani - Shadi Dental Centre,
Dr Lina Anka SEHA, Dr Rouba Zgheibi SEHA, Maria Teresa Yongson Alejandrino SEHA 411
Supragingival calculus is most often A. desquamated epithelial cells.
found on the B. components from oral secretions.
C. bacteria and their products.
A. lingual of mandibular anterior teeth* D. cuticle or pellicle.
B. buccal of mandibular anterior teeth. E. All of the above. *
C. palatal of maxillary molars.
D. lingual of mandibular molars. Dietary deficiency of vitamin D can
result in
The color of gingiva is influenced by
A. abnormal formation of osteoid.
1. The degree of keratinization. B. osteitis fibrosa cystica. *
2. Connective tissue vascularity. C. Paget's disease.
3. Amount of melanin pigmentation. D. myositis ossificans.
4. Subgingival deposits. ? E. osteogenesis imperfecta.

A. (1) (2) (3) During tooth development, vitamin A


B. (1) and (3) deficiency may result in
C. (2) and (4)
D. (4) only A. peg-shaped teeth.
E. All of the above. * B. partial anodontia (hypodontia).
C. Hutchinson's incisors.
In the development of gingivitis, the D. enamel hypoplasia. *
fibre groups first lost are E. dentinogenesis imperfecta.

A. oblique. As gingival inflammation progresses to


B. horizontal. marginal periodontitis, the associated
C. transeptal. changes are:
D. free gingival. *
A. Apical migration and disintegration
The periodontium is best able to of the epithelial attachment.
tolerate forces directed to a tooth B. Resorption of the alveolar crest.
C. Destruction of the alveolar crest and
A. horizontally. periodontal ligament fibres.
B. laterally. D. All of the above. *
C. obliquely. E. None of the above.
D. vertically. *

Dental plaque is composed of

Dr Abdul Naser Tamim SEHA, Dr Ghada Al Aqqad D.D.S, Dr Kamal Naser - Amrita Medical Centre, Dr Emad Wani - Shadi Dental Centre,
Dr Lina Anka SEHA, Dr Rouba Zgheibi SEHA, Maria Teresa Yongson Alejandrino SEHA 412
Which of the following is essential for B. protect the wound. *
successful periodontal treatment? C. stabilize the teeth.
D. protect the sutures.
A. Scaling.
B. Final evaluation and maintenance If a periodontal probe is inserted 4 or
on a one-year recall. 5mm to the base of a pocket on the
C. Periodontal flap surgery. mesialbuccal of a tooth and then
D. Elimination of local etiologic pushed facially causing blanching, this
factors. * indicates that

Maximum shrinkage after gingival A. gingival hyperplasia exists.


curettage can be expected from B. there is an inadequate zone of
tissue that is attached gingiva.
C. the lateral wall of the pocket does
A. fibroedematous. not consist of bone. *
B. edematous. *
C. fibrotic. A gingivectomy may be performed
D. formed within an infrabony when there is/are
pocket.
E. associated with exudate A. horizontal bone loss.
formation. B. no intrabony defects.
C. an adequate zone of attached
Occlusal (night) guards are used to gingiva.
D. a gingival pocket.
A. treat bruxism. * E. All of the above. *
B. reduce pocket formation.
C. prevent pulpitis. Following periodontal surgery, the
D. permit eruption or elongation of most important factor to promote
teeth. healing is

In periodontal flap surgery, the A. a salt water rinse.


design of the incision is influenced B. thorough plaque control. *
by the C. gingival massage.
D. leaving the site undisturbed for a
A. frenum attachment. period of 3 months.
B. depth of the vestibule.
C. amount of attached gingiva. Which of the following contains
D. presence of infrabony defects. microorganisms?
E. All of the above. *
A. Acquired pellicle.
The primary reason for placing a B. Calculus.
surgical dressing after a C. Dental plaque.
gingivectomy is to D. B. and C. *
E. All of the above.

A. prevent hemorrhage.

Dr Abdul Naser Tamim SEHA, Dr Ghada Al Aqqad D.D.S, Dr Kamal Naser - Amrita Medical Centre, Dr Emad Wani - Shadi Dental Centre,
Dr Lina Anka SEHA, Dr Rouba Zgheibi SEHA, Maria Teresa Yongson Alejandrino SEHA 413
Which of the following factors may affect 1. Inflammatory exudate that can
probing depth measurements? involve neutrophils, lymphocytes and
plasma cells.
1. Probing force. 2. Proliferative and degenerative
2. Probe type. changes of the junctional epithelium.
3. Angulation of probing. 3. Collagen destruction subjacent to
4. Periodontal health. the junctional epithelium.
4. Deepening of the gingival sulcus.
A. (1) (2) (3)
B. (1) and (3) A. (1) (2) (3)
C. (2) and (4) B. (1) and (3)
D. (4) only C. (2) and (4)
E. All of the above. * D. (4) only
E. All of the above. *
When analyzing occlusion, which of the
following findings is/are potentially Which cells migrate into the gingival
damaging? sulcus in the largest numbers in
response to the accumulation of
1. Marginal ridge discrepancies. plaque?
2. Extruded teeth.
3. Wide occlusal tables resulting from A. Plasma cells and monocytes.
excessive wear. B. Polymorphonuclear leukocytes. *
4. Deep overbite with minimal overjet. C. Macrophages.
D. Lymphocytes.
A. (1) (2) (3) E. Mast cells.
B. (1) and (3)
C. (2) and (4) Acquired pellicle
D. (4) only
E. All of the above. * A. is composed of salivary
glycoproteins. *
Mobility of teeth WITHOUT loss of bone B. takes 24 hours to establish.
support suggests C. is difficult to remove.
D. causes inflammation.
A. a primary occlusal trauma. *
B. a secondary occlusal trauma. Deposition of plaque on teeth occurs
C. an atrophic condition of the in
periodontium.
A. less than 24 hours *
Histopathologic alterations associated B. 24 to 48 hours.
with the pathogenesis of periodontal C. 2 to 4 days.
disease include D. 5 to 7 days.

Dr Abdul Naser Tamim SEHA, Dr Ghada Al Aqqad D.D.S, Dr Kamal Naser - Amrita Medical Centre, Dr Emad Wani - Shadi Dental Centre,
Dr Lina Anka SEHA, Dr Rouba Zgheibi SEHA, Maria Teresa Yongson Alejandrino SEHA 414
In health, the crest of the alveolar bone,
as seen in a radiograph, is situated A. (1) (2) (3)
1~to~2mm apical to the cemento- B. (1) and (3)
enamel junction. .sp 1 Radiographically, C. (2) and (4)
the normal alveolar crest should parallel D. (4) only
an imaginary line drawn between the E. All of the above. *
cemento-enamel junction of adjacent
teeth. With the development of gingivitis, the
sulcus becomes predominantly
A. The first statement is true, the populated by
second is false.
B. The first statement is false, the A. gram-positive organisms.
second is true. B. gram-negative organisms. *
C. Both statements are true. * C. diplococcal organisms.
D. Both statements are false. D. spirochetes.
A characteristic of the periodontium Which of the following oral diseases are
which allows safe temporary separation largely preventable through lifestyle
of the teeth is the adjustments?

A. nature of acellular cementum. 1. Dental caries.


B. elasticity of bone. * 2. Periodontal disease.
C. modified continuous eruption of the 3. Oral malignancies.
teeth. 4. Cleft lip and palate.
D. passive eruption.
A. (1) (2) (3) *
In a fully erupted tooth with a healthy B. (1) and (3)
periodontium, the apical end of the C. (2) and (4)
junctional epithelium is located
D. (4) only
E. All of the above.
A. in the cervical third of the crown.
B. at the cemento-enamel junction. * Epidemiology of disease is best
C. 1.5mm below the cemento-enamel described as the
junction.
D. in the cervical third of the root. A. data obtained from sickness surveys.
B. usual low level of disease normally
The colour of normal gingiva is affected found within a population.
by the
C. control of disease.
D. study of disease patterns in a
1. Vascularity of the gingiva.
population. *
2. Epithelial keratinization.
3. Thickness of the epithelium.
4. Melanin pigmentation.

Dr Abdul Naser Tamim SEHA, Dr Ghada Al Aqqad D.D.S, Dr Kamal Naser - Amrita Medical Centre, Dr Emad Wani - Shadi Dental Centre,
Dr Lina Anka SEHA, Dr Rouba Zgheibi SEHA, Maria Teresa Yongson Alejandrino SEHA 415
The Fluorosis Index is used to measure
the A. Total carbohydrate consumption.
B. Frequency of fermentable
A. degree of protection offered against carbohydrate consumption. *
dental caries by fluoride supplements. C. Total protein consumption.
B. concentration of fluoride in public D. Daily calcium intake.
water supplies.
C. degree or severity of mottled enamel* After tooth eruption, which of the
D. opposition to fluoridation by citizens' following materials gradually decreases
groups. in concentration from the enamel
E. total amount of fluoride ingested. surface ?

In chewing, maximum contact between A. Carbonate. *


teeth occurs in the position of B. Protein.
C. Fluoride.
A. habitual occlusion * D. Calcium.
B. lateral excursion on the non-working E. Chloride.
side
C. protrusive excursion Salivary secretion
D. All of the above
A. increases with age.
The fluoride ion B. decreases with age. ??
C. shows no difference with age. *
1. Is excreted rapidly by the kidney. D. shows a decrease in mineral
2. Passes the placental barrier. content in the elderly.
3. Is deposited in teeth.
4. Is deposited in bone. The Silness-Löe Index measures

A. (1) (2) (3) A. periodontal disease.


B. (1) and (3) B. oral hygiene. *
C. (2) and (4) C. attachment level.
D. (4) only D. probing depth.
E. All of the above. *
Which oral condition predisposes to
Which of the following foods is the most caries?
cariogenic?
A. Xerostomia. *
A. Cheese. B. Leukoplakia.
B. Dark chocolate. C. Pharyngitis.
C. Jam. D. Stomatitis medicamentosa.
D. Toffee. *

A diet survey can provide which of the


following information for the prevention
of dental caries?

Dr Abdul Naser Tamim SEHA, Dr Ghada Al Aqqad D.D.S, Dr Kamal Naser - Amrita Medical Centre, Dr Emad Wani - Shadi Dental Centre,
Dr Lina Anka SEHA, Dr Rouba Zgheibi SEHA, Maria Teresa Yongson Alejandrino SEHA 416
Which one of the following statements 2. Oblique.
is FALSE? 3. Alveolar crest.
4. Circular.
A. Stippling in healthy gingiva varies
with location. A. (1) (2) (3) *
B. Healthy gingiva does not bleed. B. (1) and (3)
C. The interdental papillae in the C. (2) and (4)
posterior regions are broader than in D. (4) only.
the anterior region. E. All of the above.
D. Healthy gingiva is bright red in
color* Highly filled, hybrid, posterior
composite resins are
The oral mucosa covering the base of CONTRAINDICATED as a posterior
the alveolar bone restorative material in cases of

A. is normally non-keratinized but can 1. Cusp replacement.


become keratinized in response to 2. Bruxism.
physiological stimulation. 3. Lack of enamel at the gingival cavo-
B. is closely bound to underlying surface margin.
muscle and bone. 4. Inability to maintain a dry operating
C. does not contain elastic fibres. field.
D. merges with the keratinized gingiva
at the mucogingival junction. * A. (1) (2) (3)
E. has a tightly woven dense B. (1) and (3)
collagenous corium. C. (2) and (4) *
D. (4) only
In a healthy periodontium, attached E. All of the above.
gingiva
Which of the following is/are essential
1. Is bound firmly by the periosteum to when using spherical rather than admix
the alveolar bone. alloy for a routine amalgam
2. Is attached by collagen fibres to the restoration?
cementum.
3. displays varying degrees of 1. A larger diameter condenser.
stippling. 2. An anatomical wedge.
4. Has a keratinized surface. 3. Decreased condensing pressure.
4. A dead soft matrix band.
A. (1) (2) (3)
B. (1) and (3) A. (1) (2) (3)
C. (2) and (4) B. (1) and (3) *
D. (4) only C. (2) and (4)
E. All of the above. * D. (4) only
E. All of the above.
Which of the following fibre groups are
attached to bone?

1. Apical.

Dr Abdul Naser Tamim SEHA, Dr Ghada Al Aqqad D.D.S, Dr Kamal Naser - Amrita Medical Centre, Dr Emad Wani - Shadi Dental Centre,
Dr Lina Anka SEHA, Dr Rouba Zgheibi SEHA, Maria Teresa Yongson Alejandrino SEHA 417
The interocclusal relationship of the D. an infra-orbital block.
primary second molars
A 7 year old child who complains of
1. Does not affect the resultant pain when eating, has a large carious
permanent molar relationship. lesion on a permanent molar. A
2. is normal if there is a flush terminal radiograph reveals no periapical
plane. change. The most appropriate
3. Determines the amount of leeway treatment is a/an
space.
4. May aid in the prediction of A. indirect pulp capping. *
permanent tooth malocclusion. B. direct pulp capping.
C. partial pulpotomy.
A. (1) (2) (3) D. pulpotomy.
B. (1) and (3)
C. (2) and (4) * An indirect pulp capping procedure for
D. (4) only. primary molar teeth is indicated when
E. All of the above.
A. caries involves dentin to a depth of
The most frequent cause of 1mm.
malocclusion is B. there is continuous pain.
C. there is radiolucency in the
A. thumbsucking. bifurcation.
B. mouth breathing. D. removal of caries will lead to pulp
C. heredity. * exposure. *
D. ectopic eruption.
The term "dental age" refers to the
The mechanism of adjustment to
maintain the shape and proportions A. state of dental maturation *
of bone throughout its growth period B. eruption time of a given tooth
is called C. number of years elapsed since a
given tooth erupted.
A. remodeling. * D. None of the above.
B. cortical drift.
C. area relocation. In a 5 year old, a small mechanical
D. translatory growth. exposure in a vital primary molar would
be treated by
A 3 year old requires the extraction of
a deciduous maxillary second molar. A. extraction of the tooth.
The local anesthetic technique of B. a pulp capping with calcium
choice is hydroxide *
C. a routine amalgam restoration
A. a posterior superior alveolar block. without any specific treatment for the
B. buccal and palatal infiltration. * exposed pulp.
C. a tuberosity block plus D. the use of a cavity liner.
subperiosteal infiltration of the mesio-
buccal root.

Dr Abdul Naser Tamim SEHA, Dr Ghada Al Aqqad D.D.S, Dr Kamal Naser - Amrita Medical Centre, Dr Emad Wani - Shadi Dental Centre,
Dr Lina Anka SEHA, Dr Rouba Zgheibi SEHA, Maria Teresa Yongson Alejandrino SEHA 418
The most appropriate treatment for an In an 11 year old with an otherwise
11 year old who has intermittent acceptable occlusion, an impacted
swelling and pain associated with a maxillary canine
central incisor which was traumatized
6 months ago is A. could be extracted.
B. could be retained and the first
A. pulpotomy. premolar removed to allow the canine
B. pulpectomy. * to erupt.
C. extraction. C. could be surgically exposed to
D. observation. speed its eruption.
D. could constitute a problem requiring
A single hypoplastic defect located on consultation with an orthodontist.
the labial surface of a maxillary central E. All of the above. *
incisor is most likely due to a/an
Following the premature loss of the
A. dietary deficiency. deciduous molars, the Angle
B. endocrine deficiency. classification is most accurately
C. tetracycline therapy. determined using the
D. trauma to the maxillary primary
central incisor. * A. facial profile.
E. high fluoride intake. B. permanent molars. *
C. permanent canines.
Poor oral hygiene during orthodontic D. permanent incisors.
treatment may result in
Alveolar bone is undergoing
1. Edema. remodeling
2. Bleeding.
3. Loss of stippling. A. through the primary dentition.
4. Gingival desquamation. B. until the end of mixed dentition.
C. until the complete eruption of
A. (1) (2) (3) * permanent teeth.
B. (1) and (3) D. throughout life. *
C. (2) and (4)
D. (4) only Following premature deciduous tooth
E. All of the above. loss, space loss occurs most
frequently in the area of the
Primary herpetic lesions of the oral
cavity are most likely to occur during A. maxillary lateral incisor.
B. mandibular central incisor.
A. 1 to 5 years. * C. mandibular second premolar. *
B. 6 to 12 years. D. maxillary first premolar.
C. 13 to 16 years.
D. Any age.

Dr Abdul Naser Tamim SEHA, Dr Ghada Al Aqqad D.D.S, Dr Kamal Naser - Amrita Medical Centre, Dr Emad Wani - Shadi Dental Centre,
Dr Lina Anka SEHA, Dr Rouba Zgheibi SEHA, Maria Teresa Yongson Alejandrino SEHA 419
Which of the following processes is 2. May remain for years with no
NOT active in causing tooth significant resorption.
eruption? 3. May remain for years partially
resorbed.
A. Growth of the dentin. 4. Are always resorbed.
B. Growth of the root.
C. Growth of the enamel. * A. (1) (2) (3) *
D. Pressure from periapical tissue. B. (1) and (3)
C. (2) and (4)
A patient who is jaundiced because D. (4) only
of liver disease has an increased E. All of the above.
risk of
Bone tissue grows by
1. Postextraction bleeding.
2. Cardiac arrest. A. interstitial growth. *
3. Postoperative infection. B. osteoclastic activity.
4. Anaphylactic shock. C. proliferation of endodermal tissue.
D. differentiation of cartilaginous
A. (1) (2) (3) tissue.
B. (1) and (3) *
C. (2) and (4) Cartilage grows by
D. (4) only
E. All of the above. A. interstitial growth.
B. appositional growth.
The primary stimulus for growth of C. both appositional and interstitial
the mandible is growth. *
D. None of the above.
A. genetic.
B. epigenetic. As the mandible grows downward and
C. environmental. forward, bone deposition takes place
D. functional.
E. A. and D. * A. on all surfaces of the mandible.
B. on the posterior border of the
The roots of primary molars in the ramus.
absence of their permanent C. on the anterior border of the ramus.
successors D. on the alveolar margins.
E. B. and D. *
1. Sometimes are partially resorbed
and become ankylosed.

Dr Abdul Naser Tamim SEHA, Dr Ghada Al Aqqad D.D.S, Dr Kamal Naser - Amrita Medical Centre, Dr Emad Wani - Shadi Dental Centre,
Dr Lina Anka SEHA, Dr Rouba Zgheibi SEHA, Maria Teresa Yongson Alejandrino SEHA 420
The roots of the first permanent molar D. (4) only
should be completely formed by the E. All of the above.
age of
The most significant factor in
A. six years. determining the prognosis of anterior
B. seven years. crossbite correction is the
C. nine years.
D. eleven years. * A. age of patient.
E. thirteen years. B. depth of the overbite. *
C. shape of the tooth involved.
A space maintainer in the posterior D. space available mesiodistally.
segment will
A single tooth anterior crossbite found
A. prevent extrusion of opposing in a 9 year old should
teeth.
B. prevent the eruption of the A. self-correct.
permanent teeth. B. be treated with a removable
C. retard eruption of the permanent appliance. *
teeth. C. have 2 arch orthodontic treatment.
D. maintain arch length. * D. be treated in the complete
permanent dentition.
A 6 year old has an open bite caused E. be observed and treated when the
by active thumbsucking. The most cuspids have erupted.
appropriate management is to
The greatest period of cranial growth
A. insert a habit-breaking appliance. occurs between
B. refer to a psychologist for
evaluation. A. birth and 5 years *
C. encourage habit cessation and B. 6 and 8 years.
observe. * C. 10 and 12 years.
D. bond limited fixed bracket. D. 14 and 16 years.

Loss of a permanent maxillary first Hypothyroidism affects the dental


molar may result in developmental pattern by

1. Distal drift of the adjacent premolar. A. interfering with jaw growth.


2. Mesial drift of the adjacent molar. B. delaying the eruption timetable. *
3. Overeruption of the opposing tooth. C. causing sclerotic bone to form over
4. Increase in overbite. the occlusal surface of erupting teeth.
D. accelerating the eruption timetable.
A. (1) (2) (3) *
B. (1) and (3)
C. (2) and (4)

Dr Abdul Naser Tamim SEHA, Dr Ghada Al Aqqad D.D.S, Dr Kamal Naser - Amrita Medical Centre, Dr Emad Wani - Shadi Dental Centre,
Dr Lina Anka SEHA, Dr Rouba Zgheibi SEHA, Maria Teresa Yongson Alejandrino SEHA 421
Mandibular growth An endomorph is characterized as a
person who
A. is sustained over a longer period
of time in girls. A. is short and fat. *
B. is sustained over a longer period B. is tall and thin.
of time in boys. * C. is muscular.
C. occurs at the same chronologic D. matures early.
age in both sexes. E. matures late.
D. occurs two years earlier in boys
than in girls. The developing permanent tooth

The principal growth sites of the A. lies apically and lingually to primary
maxilla in a downward and forward teeth in the anterior region.
direction include the B. may show deviated eruption times if
the primary tooth is lost prematurely.
1. Frontomaxillary suture. C. has a more protrusive path of
2. Zygomatic maxillary suture. eruption in the anterior region.
3. Pterygopalatine suture. D. All of the above. *
4. Median palatine suture.
Roots of the permanent maxillary
A. (1) (2) (3) * central incisors are completed by what
B. (1) and (3) age?
C. (2) and (4)
D. (4) only. A. 8 years.
E. All of the above. B. 10 years. *
C. 12 years.
Cleidocranial dysostosis is D. Later than 12 years.
distinguished by
In a 7 year old, the intrusion of a
A. usually shortened skull. permanent central incisor can cause
B. delayed suture closure.
C. persistence of deciduous teeth. 1. Laceration of the periodontal
D. clavicles absent or maldeveloped. membrane.
E. All of the above. * 2. Loss of pulp vitality.
3. Ankylosis.
In a normal eruption pattern, the last 4. Root resorption.
primary tooth to be lost is the
A. (1) (2) (3)
A. maxillary canine * B. (1) and (3)
B. mandibular canine. C. (2) and (4)
C. maxillary first molar. D. (4) only
D. mandibular second molar. E. All of the above. *
E. maxillary second molar.

Dr Abdul Naser Tamim SEHA, Dr Ghada Al Aqqad D.D.S, Dr Kamal Naser - Amrita Medical Centre, Dr Emad Wani - Shadi Dental Centre,
Dr Lina Anka SEHA, Dr Rouba Zgheibi SEHA, Maria Teresa Yongson Alejandrino SEHA 422
The normal gingiva of the child patient is
diagnosed on the basis of In cephalometry, the most stable point
in a growing skull is the
1. Contour.
2. Stippling. A. sella turcica. *
3. Sulcus depth. B. nasion.
4. Tight fitting gingival collar. C. Broadbent's point.
D. Bolton point.
A. (1) (2) (3)
B. (1) and (3) * Which of the following is correct
C. (2) and (4) regarding hand-wrist radiographs?
D. (4) only
E. All of the above. A. Skeletal age is estimated via
comparsion with standard values.*
The radiographic appearance of internal B. They are a precise measure of
resorption is skeletal development.
C. They are of little diagnosic value.
A. radiolucent enlargement of the pulp D. They accurately determine skeletal
cavity. * age.
B. radiolucency around the apex of the
root. A radiographic examination of a 10
C. radiolucency on the surfaces of the year old child reveals retention of
root. deciduous teeth and presence of
D. localized radiopacities in the pulp many unerupted supernumerary
cavity. teeth. This is characteristic of
E. radiopacity around the apex of the
root. A. cleidocranial dysplasia. *
B. ectodermal dysplasia.
Which of the following patients should C. dentinogenesis imperfecta.
be referred for orthodontic treatment to D. congenital hypothyroidism.
close a diastema between maxillary
central incisors? The facial and lingual walls of the
occlusal portion of a Class II cavity
1. An 8-year old with no abnormal oral preparation for an amalgam in
habits. deciduous teeth should
2. A 14-year old with no abnormal oral
habits. A. be parallel to each other.
3. A 3-year old with a 4mm overjet. B. diverge toward the occlusal
4. An 8-year old with a previous thumb surface.
habit. C. converge toward the occlusal
surface. *
A. (1) (2) (3) D. not follow the direction of the
B. (1) and (3) enamel rods.
C. (2) and (4) *
D. (4) only
E. All of the above.

Dr Abdul Naser Tamim SEHA, Dr Ghada Al Aqqad D.D.S, Dr Kamal Naser - Amrita Medical Centre, Dr Emad Wani - Shadi Dental Centre,
Dr Lina Anka SEHA, Dr Rouba Zgheibi SEHA, Maria Teresa Yongson Alejandrino SEHA 423
The eruption of a permanent central A. without encroaching upon the space
incisor may be delayed by for tooth 1.1.
B. encroaching upon the space for
A. a supernumerary tooth. tooth 1.1. *
B. dense fibrous tissue. C. distally to the space for tooth 1.2.
C. a retained deciduous incisor.
D. All of the above. * After an inferior alveolar nerve block
injection, a patient would develop
An ankylosed deciduous molar can seventh nerve paralysis if the injection
cause was made into the

A. delayed eruption of the succeeding A. internal maxillary artery.


premolar. B. retroparotid space *
B. alteration of arch length. C. internal pterygoid muscle.
C. difficulty with extraction. D. retromandibular vein.
D. All of the above. * E. pterygoid plexus of veins.

In primary molars, radiographic bony Which of the following may be used as


changes from an infection are initially a local anesthetic in a patient allergic to
seen both amide and ester-type local
anesthetics?
A. at the apices.
B. in the furcation area. * A. Nitrous oxide.
C. at the alveolar crest. B. Bupivacaine.
D. at the base of the developing tooth. C. Phenylephrine.
D. Diphenhydramine. *
The cells responsible for root E. Ethyl aminobenzoate.
resorption are
Which of the following nerves should be
A. fibroblasts. anesthetized for the removal of a
B. cementoblasts. maxillary first molar?
C. osteoblasts.
D. osteoclasts. * 1. Greater palatine.
2. Naso palatine.
During normal growth, the gnathion, as 3. Middle superior alveolar.
viewed on successive cephalograms, 4. Anterior superior alveolar.
will move 5. Posterior superior alveolar.

A. downward and backward. A. (1) (2) (4)


B. downward and forward. * B. (1) (3) (4)
C. backward and upward. C. (1) (3) (5) *
D. forward only. D. (2) (3) (5)
E. (2) (4) (5)
An 8 year old has lost tooth 1.1. Tooth
1.2 will most likely erupt

Dr Abdul Naser Tamim SEHA, Dr Ghada Al Aqqad D.D.S, Dr Kamal Naser - Amrita Medical Centre, Dr Emad Wani - Shadi Dental Centre,
Dr Lina Anka SEHA, Dr Rouba Zgheibi SEHA, Maria Teresa Yongson Alejandrino SEHA 424
Cardiovascular collapse caused by a When sutures are used to reposition
high circulating dose of a local tissue over extraction sites, they
anesthetic is due to should be

A. vagal stimulation. 1. Placed over firm bone where


B. histamine release. possible.
C. myocardial depression * 2. interrupted, 15mm apart.
D. medullary stimulation. 3. Firm enough to approximate tissue
flaps without blanching.
Hydrochlorothiazide (Hydrodiuril) is 4. tight enough to produce immediate
used to treat hemostasis.

1. Hypertension. A. (1) (2) (3) *


2. Angina pectoris. B. (1) and (3)
3. Atrial fibrillation. C. (2) and (4)
4. Ventricular fibrillation. D. (4) only
5. Congestive heart failure. E. All of the above.

A. (1) and (2) Which of the following lower(s) the


B. (2) and (3) body temperature increased by
C. (3) and (4) bacterial pyrogens?
D. (1) and (5) *
1. Acetaminophen.
What is the maximum number of 2. Bradykinin.
cartridges (1.8ml) of a 2 local 3. Acetylsalicylic acid.
anesthetic solution that can be 4. Codeine.
administered without exceeding a total
dose of 300mg? A. (1) (2) (3)
B. (1) and (3) *
A. 2. C. (2) and (4)
B. 4. D. (4) only
C. 6. E. All of the above.
D. 8. *
E. 10. If a patient is allergic to penicillin, the
most appropriate antibiotic for an
Use of nitrous oxide analgesia odontogenic infection would be
produces tinnitus as a result of
A. ampicillin.
A. central nervous system reaction. B. cephalexin.
B. peripheral action on the eardrum. C. clindamycin. *
C. increased pressure in the middle D. declomycin.
ear. * E. streptomycin.
D. cochlea effect.
E. dysphoria.

Dr Abdul Naser Tamim SEHA, Dr Ghada Al Aqqad D.D.S, Dr Kamal Naser - Amrita Medical Centre, Dr Emad Wani - Shadi Dental Centre,
Dr Lina Anka SEHA, Dr Rouba Zgheibi SEHA, Maria Teresa Yongson Alejandrino SEHA 425
During extraction of a maxillary third C. increased vascular permeability.
molar, the tuberosity is fractured. The D. accelerated healing.
tooth with the tuberosity remains
attached to the surrounding soft tissue. In the treatment of an acute
You should anaphylactic reaction, the first drug that
should be administered is
A. remove both and suture.
B. leave both and stabilize, if possible.* A. hydroxyzine.
C. remove both, fill the defect with B. epinephrine. *
Gelfoam and suture. C. hydrocortisone.
D. reflect the mucoperiosteum, remove D. diphenhydramine.
the tooth, leaving the tuberosity in
place and suture. Early anoxia is characterized by

An acute periapical abscess originating A. cyanosis. *


from a mandibular third molar generally B. bradycardia.
points and drains in the C. bronchospasm.
D. amnesia.
A. submandibular space. *
B. pterygomandibular space. The chief mechanism by which the
C. buccal vestibule. body metabolizes short-acting
D. buccal space. barbiturates is

Bilateral dislocated fractures of the A. oxidation.


mandibular condyles result in B. reduction.
C. hydroxylation and oxidation. *
1. Anterior open bite. D. sequestration in the body fats.
2. Anesthesia of the mental nerves.
3. Inability to protrude the mandible. With respect to local anaesthetics,
4. Inability to bring the molars into which of the following statements is/are
contact. correct?

A. (1) (2) (3) A. Certain nerve fibers are more


B. (1) and (3) * susceptible.
C. (2) and (4) B. In mixed nerves, sensory fibers are
D. (4) only more susceptible.
E. All of the above. C. They are marketed as water-soluble
acid salts.
In achieving hemostasis, external cold D. They are capable of blocking every
application produces type of nerve tissue.
E. All of the above. *

A. positive chemotaxis.
B. a transient vasoconstriction. *

Dr Abdul Naser Tamim SEHA, Dr Ghada Al Aqqad D.D.S, Dr Kamal Naser - Amrita Medical Centre, Dr Emad Wani - Shadi Dental Centre,
Dr Lina Anka SEHA, Dr Rouba Zgheibi SEHA, Maria Teresa Yongson Alejandrino SEHA 426
An infected root is accidentally E. allergy.
displaced into the maxillary sinus.
Examination of the socket reveals Which of the following is the strongest
perforation of the sinus lining. Therapy stimulus to respiration?
should consist of
A. Decrease in arterial pH.
1. Acrylic template to cover socket B. Increase in arterial oxygen.
opening and saline rinses. C. Decrease in arterial oxygen.
2. Closure of oro-antral D. Increase in arterial carbon dioxide. *
communication and antibiotic E. Decrease in arterial carbon dioxide.
coverage.
3. Antibiotic coverage and Which of the following nerves should be
observation. anesthetized for extraction of a
4. Antrostomy for retrieval of root. maxillary lateral incisor?

A. (1) (2) (3) 1. Nasociliary.


B. (1) and (3) 2. Nasopalatine.
C. (2) and (4) * 3. Sphenopalatine.
D. (4) only. 4. Anterior superior alveolar.
E. All of the above.
A. (1) and (2)
A 57 year old man received 10mg of B. (1) (3) (4)
diazepam intravenously. He becomes C. (2) (3) (4)
unresponsive to verbal stimuli, and his D. (2) and (4) *
respirations are depressed to 10 per E. (3) and (4)
minute. Appropriate treatment is to
High plasma levels of local anesthetics
A. administer ephedrine. may cause
B. observe the patient.
C. force the patient to drink coffee. A. inhibition of peristalsis.
D. support respiration with oxygen. * B. stimulation of the central nervous
system. *
Death from barbiturates is the result of C. inhibition of the vagus nerve to the
heart.
A. alkalosis. D. depression of the central nervous
B. irreversible hypotension. system.
C. toxic effects on the liver.
D. depression of the centres of
respiration. *

Dr Abdul Naser Tamim SEHA, Dr Ghada Al Aqqad D.D.S, Dr Kamal Naser - Amrita Medical Centre, Dr Emad Wani - Shadi Dental Centre,
Dr Lina Anka SEHA, Dr Rouba Zgheibi SEHA, Maria Teresa Yongson Alejandrino SEHA 427
Lidocaine
Vestibuloplasty is a preprosthetic
1. Is a local anesthetic agent. surgical procedure used to
2. has topical anesthetic properties.
3. Is an antiarrhythmic agent. A. facilitate reliable impression making.
4. has anticonvulsant properties. B. provide adequate posterior inter-arch
space.
A. (1) (2) (3) C. allow placement of teeth over the
B. (1) and (3) residual ridge.
C. (2) and (4) D. increase the supporting surface area*
D. (4) only
E. All of the above. * The mode of action of the sulfonamides
upon susceptible bacteria is by
Patients with a history of rheumatic
fever and known heart valve damage A. inhibiting the biosynthesis of p-
should be given prophylactic antibiotic aminobenzoic acid.
coverage before dental extractions B. competing for nutrients in the tissue
because of the risk of environment of the microorganisms.
C. interfering with the synthesis of cell
A. myocardial infarction. wall protein.
B. subacute bacterial endocarditis* D. interfering with the synthesis of folic
C. cardiac arrest. acid. *
D. All of the above.
In a standard dental cartridge (carpule)
In a standard inferior alveolar nerve containing 1.8ml 2% lidocaine with
block, which muscle is penetrated by epinephrine 1/100,000, the amount of
the needle? vasoconstrictor is

A. Buccinator. * A. 18.0 mg.


B. Mylohyoid. B. 0.018 mg. *
C. Superior constrictor. C. 1.8 mg.
D. Masseter. D. 0.18 mg.
E. Medial (internal) pterygoid. E. 180.0 mg.

Immediate toxic reactions to local Which one of the following tests is used
anesthetic administration are most to confirm the presence of an acute
commonly due to infection?

A. deterioration of the anesthetic A. Erythrocyte sedimentation rate.


agent. B. Urinalysis.
B. hypersensitivity to the C. Differential white cell count. *
vasoconstrictor. D. Serum alkaline phosphatase.
C. hypersensitivity to the anesthetic
agent.
D. excessive blood level of the
anesthetic agent. *

Dr Abdul Naser Tamim SEHA, Dr Ghada Al Aqqad D.D.S, Dr Kamal Naser - Amrita Medical Centre, Dr Emad Wani - Shadi Dental Centre,
Dr Lina Anka SEHA, Dr Rouba Zgheibi SEHA, Maria Teresa Yongson Alejandrino SEHA 428
Which of the following is the most 1. Its long duration of action is partly
important factor in the preoperative due to active metabolites.
evaluation of a patient? 2. It does not produce antianxiety
effects after intramuscular
A. Medical history. * administration.
B. Laboratory data. 3. Intravenous administration is more
C. Electrocardiogram. reliable than oral.
D. Pulse and blood pressure. 4. Its sedative effect can be reversed
by naloxone.
Displacement of mandibular fractures
is dependent upon A. (1) (2) (3)
B. (1) and (3) *
A. proprioceptor nerve action. C. (2) and (4)
B. TMJ mobility. D. (4) only
C. tooth in line of fracture. E. All of the above.
D. direction of blow. *
The appearance of a rash, itching,
Which of the following can be broncho-constriction and fever after
mistaken on a radiograph for a chronic the administration of a drug are the
alveolar abscess? result of

1. Mental foramen. A. allergy. *


2. Periapical cemento-osseous B. tolerance.
dysplasia. C. idiosyncrasy.
3. Posterior palatine foramen. D. teratogenicity.
4. Nasopalatine foramen.
In facial injury management, the most
A. (1)(2)(3) important first aid measure is to
B. (1) et (3)
C. (2) et (4) A. control the bleeding.
D. (4) B. prevent shock.
E. All of the above * C. establish and maintain an airway. *
D. control infection.
The inorganic ion that is implicated in
primary hypertension is Unconsciousness in syncope results
from
A. sodium. *
B. fluoride. A. electrolyte imbalance.
C. potassium. B. neurogenic shock.
D. magnesium. C. cerebral hyperemia.
D. cerebral hypoxia. *
Which of the following statements
is/are true regarding diazepam?

Dr Abdul Naser Tamim SEHA, Dr Ghada Al Aqqad D.D.S, Dr Kamal Naser - Amrita Medical Centre, Dr Emad Wani - Shadi Dental Centre,
Dr Lina Anka SEHA, Dr Rouba Zgheibi SEHA, Maria Teresa Yongson Alejandrino SEHA 429
Which of the following complications 1. It is the neurotransmitter at both
might occur after administration of a sympathetic and parasympathetic
local anesthetic agent? ganglia.
2. It is rapidly hydrolysed in the body by
1. Convulsions. cholinesterase.
2. Nausea. 3. It can produce both muscarinic and
3. Respiratory depression. nicotinic actions.
4. Cardiovascular collapse. 4. It is the drug of choice as an antidote
in atropine poisoning.
A. (1) and (3)
B. (1) (3) (4) A. (1) (2) (3) *
C. (2) and (3) B. (1) and (3)
D. (3) and (4) C. (2) and (4)
E. All of the above. * D. (4) only
E. All of the above.
Which of the following would you
prescribe for an anxious dental patient Which of the following does NOT
with a peptic ulcer? influence the rate of induction during
inhalation anesthesia?
A. Reserpine.
B. Scopolamine. A. Pulmonary ventilation.
C. Silica gel. B. Blood supply to the lungs.
D. Diazepam.* C. Hemoglobin content of the blood. *
E. Calcium carbonate. D. Concentration of the anesthetic in
the inspired mixture.
When used alone, which of the following E. Solubility of the anesthetic in blood.
agents will not produce satisfactory
anesthesia? General anesthetics can do all of the
following EXCEPT
A. Isoflurane.
B. Sevoflurane. A. produce delirium.
C. Nitrous oxide. * B. stimulate medullary centers. *
D. Desflurane. C. produce a state of unconsciousness.
D. reduce perception of painful stimuli.
Which of the following statements is/are E. decrease excitability of the motor
true regarding acetylcholine? cortex.

Dr Abdul Naser Tamim SEHA, Dr Ghada Al Aqqad D.D.S, Dr Kamal Naser - Amrita Medical Centre, Dr Emad Wani - Shadi Dental Centre,
Dr Lina Anka SEHA, Dr Rouba Zgheibi SEHA, Maria Teresa Yongson Alejandrino SEHA 430
Acquired Immune Deficiency
Syndrome (AIDS) may be When comparing opioids with NSAIDS
characterized by which of the following is correct?

1. Candidiasis. A. Opioids impair gastric motility to a


2. Rapid weight loss and night sweats. greater extent. *
3. Extreme malaise, fever or chills. B. NSAIDS cause more nausea.
4. A smooth and red tongue. C. Opioids predispose to more
bleeding.
A. (1) (2) (3) D. NSAIDS cause more drowsiness.
B. (1) and (3)
C. (2) and (4) Trismus is most frequently caused by
D. (4) only
E. All of the above. * A. tetanus.
B. muscular dystrophy.
Short-acting barbiturates are C. infection. *
metabolized mainly in the D. mandibular fracture.

A. liver. * A therapeutic advantage of penicillin V


B. kidneys. over penicillin G is
C. small intestine.
D. pancreas. A. greater resistance to penicillinase.
E. spleen. B. broader antibacterial spectrum.
C. greater absorption when given
Acetaminophen in therapeutic doses orally. *
D. slower renal excretion.
1. Retards platelet function. E. None of the above.
2. has strong anti-inflammatory
properties. Which valve is most commonly
3. Produces CNS stimulation. affected by rheumatic heart disease?
4. Has antipyretic properties.
A. Aortic.
A. (1) (2) (3) B. Pulmonary.
B. (1) and (3) C. Tricuspid.
C. (2) and (4) D. Mitral. *
D. (4) only *
E. All of the above.

Dr Abdul Naser Tamim SEHA, Dr Ghada Al Aqqad D.D.S, Dr Kamal Naser - Amrita Medical Centre, Dr Emad Wani - Shadi Dental Centre,
Dr Lina Anka SEHA, Dr Rouba Zgheibi SEHA, Maria Teresa Yongson Alejandrino SEHA 431
Tetracyclines E. Chloral hydrate. *

1. Have no side effects. Which of the following is used in the


2. May increase susceptibility to management of a patient with grand
superinfections. mal seizures?
3. are safe to use during pregnancy.
4. Have a wide spectrum of antibacterial A. Amobarbital.
activity. B. Secobarbital.
C. Pentobarbital.
A. (1) (2) (3) D. Phenobarbital. *
B. (1) and (3)
C. (2) and (4) * Protracted use of tetracycline may
D. (4) only produce symptoms of vitamin K
E. All of the above. deficiency because tetracycline

All of the following drugs are useful in A. is detoxified by the liver.


the treatment of cardiac arrhythmias B. combines chemically with vitamin
EXCEPT K.
C. inhibits growth of intestinal
A. digitalis. bacteria. *
B. lidocaine. D. interferes with the conversion of
C. procainamide. prothrombin to thrombin.
D. aminophylline. *
Antihistamines act by
Adrenal corticosteroids
A. increasing the action of
1. Cause diabetes. histaminase.
2. Cause retention of sodium and fluid. B. altering the formation of
3. Heighten the immune response. histamine.
4. Decrease the immune response. C. blocking the actions of histamine
by competitive inhibition. *
A. (1) (2) (3) D. interfering with the degradation of
B. (1) and (3) histamine.
C. (2) and (4) *
Which of the following local
D. (4) only
anesthetics is subject to inactivation
E. All of the above.
by plasma esterases?
Which of the following does NOT relieve
A. Procaine. *
pain?
B. Lidocaine.
A. Codeine. C. Prilocaine.
B. Methadone. D. Mepivacaine.
C. Meperidine. E. Bupivacaine.
D. Hydromorphone.

Dr Abdul Naser Tamim SEHA, Dr Ghada Al Aqqad D.D.S, Dr Kamal Naser - Amrita Medical Centre, Dr Emad Wani - Shadi Dental Centre,
Dr Lina Anka SEHA, Dr Rouba Zgheibi SEHA, Maria Teresa Yongson Alejandrino SEHA 432
The first sensation lost after D. nitrous oxide.
administration of a local anesthetic is
In a safe general anesthetic mixture,
A. pain. * the MINIMALLY acceptable percentage
B. touch. of oxygen is
C. pressure.
D. proprioception. A. 5%.
B. 10%.
The most common complication of a C. 20%.
venipuncture is D. 50%. *
E. 80%.
A. syncope.
B. hematoma. * Which of the following is directly
C. thrombophlebitis. involved in the conversion of
D. embolus. prothrombin to thrombin?

Patient nausea during nitrous oxide A. Sodium.


administration is an indication that the B. Calcium. *
patient C. Fluoride.
D. Potassium.
A. is nervous. E. Bicarbonate.
B. has not eaten for some time.
C. is allergic to nitrous oxide. Loss of sensibilit over the distribution of
D. has received the nitrous oxide too the inferior dental nerve is a possible
quickly. * complication from

The major stimulator of respiration is 1. Removal of an impacted mandibular


third molar tooth.
A. low blood pressure. 2. Removal of a torus mandibularis.
B. high percentage of blood oxygen. 3. An acute osteomyelitis of the
C. low percentage of blood carbon mandible.
dioxide. 4. An uncomplicated removal of a
D. high percentage of blood carbon mandibular second molar.
dioxide. *
A. (1) (2) (3)
Cardiac arrhythmias are most commonly B. (1) and (3) *
seen during administration of C. (2) and (4)
D. (4) only
A. thiopental. E. All of the above.
B. halothane. *
C. ethyl ether.

Dr Abdul Naser Tamim SEHA, Dr Ghada Al Aqqad D.D.S, Dr Kamal Naser - Amrita Medical Centre, Dr Emad Wani - Shadi Dental Centre,
Dr Lina Anka SEHA, Dr Rouba Zgheibi SEHA, Maria Teresa Yongson Alejandrino SEHA 433
All of the following are side effect of It is difficult to obtain satisfactory
prolonged tetracycline hydrochloride anesthesia in the presence of infection
therapy EXCEPT near the injection site because

A. superinfection. A. the swelling causes increased


B. photosensitivity. pressure on the nerves.
C. vestibular disturbances. * B. increased blood supply carries the
D. discoloration of newly forming teeth. anesthetic solution away too fast.
E. gastrointestinal symptoms (when C. acidity of the infected tissue inhibits
administered orally). action of the anesthetic agent. *
D. alkalinity of the infected tissue inhibits
Extraction of a tooth is action of the anesthetic agent.
CONTRAINDICATED in the dental
office for a patient who An antipyretic drug

A. is 4 months pregnant. A. reduces fever. *


B. has a Factor~VIII deficiency. * B. provides analgesia.
C. is hypothyroid. C. causes loss of consciousness.
D. had a myocardial infarct six months D. creates heat sensitivity.
ago. E. counters the tendency for epileptic
seizures.
Which of the following is best removed
by curettage? Ludwig's angina may cause death by

A. Ameloblastoma. A. heart failure.


B. Pleomorphic adenoma. B. asphyxia. *
C. Central giant cell granuloma.* C. convulsions.
D. Squamous cell carcinoma. D. paralysis of muscles of respiration.
E. Cylindroma. E. pyemia.

Which of the following steroids can In a patient with liver disease, a possible
produce Cushing's syndrome? complication is

A. Estradiol. A. syncope or shock.


B. Testosterone. B. postoperative infection.
C. Prednisolone. * C. prolonged bleeding. *
D. Progesterone. D. allergic reaction to the anesthetic
E. Diethylstilbestrol. solution.

Dr Abdul Naser Tamim SEHA, Dr Ghada Al Aqqad D.D.S, Dr Kamal Naser - Amrita Medical Centre, Dr Emad Wani - Shadi Dental Centre,
Dr Lina Anka SEHA, Dr Rouba Zgheibi SEHA, Maria Teresa Yongson Alejandrino SEHA 434
Preoperative evaluation of a healthy 3. Erythrocyte sedimentation rate.
patient requiring elective oral surgery in 4. Platelet count.
hospital should include
A. (1) (2) (3)
1. A complete history. B. (1) and (3) *
2. A physical examination. C. (2) and (4)
3. An oral examination. D. (4) only
4. Appropriate laboratory tests. E. All of the above.

A. (1) (2) (3) A patient presenting with diplopia,


B. (1) and (3) exophthalmos, nasal bleeding and
C. (2) and (4) swelling, may suffer from a fracture of
D. (4) only the
E. All of the above. *
A. neck of the condyle.
A fracture is considered to be B. body of the mandible.
favourable when C. zygomatic bone. *
D. maxillary tuberosity.
A. healing is expected to be
uncomplicated. The most likely complication
B. operating time will be short. associated with the extraction of an
C. fragments are not displaced by isolated maxillary second molar is
muscle pull. *
D. not exposed to the oral environment. A. a dry socket.
B. nerve damage.
In an acute upper airway obstruction, C. fracture of the malar ridge.
the entry to the airway on an D. fracture of the tuberosity. *
emergency basis should be made at
the Which of the following is NOT an
indication for the removal of impacted
A. cricoid cartilage. mandibular third molars?
B. thyroid notch.
C. thyroid membrane. A. Recurrent pericoronitis.
D. cricothyroid membrane. * B. Crowding of incisors. *
E. First tracheal ring. C. Pain.
D. Resorption of the distal aspect of
Bacterial infection may be confirmed by the second molar.

1. White blood cell count.


2. Hemoglobin level.

Dr Abdul Naser Tamim SEHA, Dr Ghada Al Aqqad D.D.S, Dr Kamal Naser - Amrita Medical Centre, Dr Emad Wani - Shadi Dental Centre,
Dr Lina Anka SEHA, Dr Rouba Zgheibi SEHA, Maria Teresa Yongson Alejandrino SEHA 435
A periapical infection of a mandibular 1. Analgesics.
third molar may spread by direct 2. Regular diet.
extension to the 3. Application of cold.
4. Frequent rinsing of the socket.
1. Parapharyngeal space.
2. Submandibular space. A. (1) (2) (3)
3. Pterygomandibular space. B. (1) and (3) *
4. Submental space. C. (2) and (4)
D. (4) only
A. (1) (2) (3) * E. All of the above.
B. (1) and (3)
C. (2) and (4) Early signs and symptoms of localized
D. (4) only alveolar osteitis (dry socket) include
E. All of the above.
1. Bleeding.
Which of the following will impede 2. Bad odour.
healing following the surgical closure 3. Pus formation.
of an oro-antral fistula? 4. Pain.

1. Poor flap design. A. (1) (2) (3)


2. Excessive tissue tension. B. (1) and (3)
3. Blowing the nose. C. (2) and (4) *
4. Sinus infection. D. (4) only
E. All of the above.
A. (1) (2) (3)
B. (1) and (3) A patient suddenly becomes pale and
C. (2) and (4) sweaty after an injection of 4ml of
D. (4) only lidocaine 2% with epinephrine
E. All of the above. * l:l00,000. The radial pulse is slow and
steady. The respiration is slow. The
Which of the following is a principle of blood pressure is 80/60. What is the
closed fracture management? most probable diagnosis?

A. Incision at fracture site. A. A toxic reaction to lidocaine.


B. Rigid fixation of fragments. B. A toxic reaction to epinephrine.
C. Debridement of fracture site. C. An allergic reaction to the local
D. Immobilization of fracture. * anesthetic.
D. Incipient syncope. *
Immediately after the extraction of a E. An impending adrenal insufficiency.
tooth, postoperative instructions
should include

Dr Abdul Naser Tamim SEHA, Dr Ghada Al Aqqad D.D.S, Dr Kamal Naser - Amrita Medical Centre, Dr Emad Wani - Shadi Dental Centre,
Dr Lina Anka SEHA, Dr Rouba Zgheibi SEHA, Maria Teresa Yongson Alejandrino SEHA 436
Which of the following cements can C. (2) and (4)
chemically bond to enamel? D. (4) only
E. All of the above.
1. Zinc phosphate cement.
2. Polycarboxylate cement. It is ethical for a dentist to refuse to
3. Reinforced zinc oxide eugenol treat a patient on the basis of the
cement.
4. Glass ionomer cement. A. patient’s religious beliefs.
B. patient’s physical handicap.
A. (1) (2) (3) C. patient’s infectious disease status.
B. (1) and (3) D. complexity of the required
C. (2) and (4) * treatment*
D. (4) only
E. All of the above. Lidocaine (Xylocaine) is an example of
a local anesthetic which is chemically
A pontic replacing a mandibular first classified as an
molar should be designed so that it(s)
A. amide. *
1. Gingival surface is concave and B. ester.
adapts closely to the ridge. C. aldehyde.
2. Has open gingival embrasures. D. ethamine.
3. Conceals the porcelain to metal E. aminide.
junction on its gingival surface.
4. Gingival surface is convex in all Resin bonding of composites to acid-
directions. etched enamel results in

A. (1) (2) (3) A. decreased polymerization shrinkage


B. (1) and (3) of the resin.
C. (2) and (4) * B. decreased crack formation in the
D. (4) only enamel.
E. All of the above. C. reduced microleakage. *
D. improved wear resistance of the
Which of the following are composite.
characteristic symptoms of acute
pulpitis? Median palatine cysts are classified as

1. Spontaneous throbbing pain. A. developmental. *


2. Prolonged pain initiated by heat. B. residual.
3. Pain on percussion. C. idiopathic.
4. Increased pain by cold. D. odontogenic.

A. (1) (2) (3) *


B. (1) and (3)

Dr Abdul Naser Tamim SEHA, Dr Ghada Al Aqqad D.D.S, Dr Kamal Naser - Amrita Medical Centre, Dr Emad Wani - Shadi Dental Centre,
Dr Lina Anka SEHA, Dr Rouba Zgheibi SEHA, Maria Teresa Yongson Alejandrino SEHA 437
A patient with congestive heart failure A. History of the oral lesion. *
may have B. Cytological smear.
C. Systemic evaluation.
1. Epistaxis. D. Laboratory tests.
2. Shortness of breath.
3. Exophthalmos. Which of the following epithelial
4. Pitting edema of the ankles. changes is most likely to be
precancerous?
A. (1) (2) (3)
B. (1) and (3) A. Acanthosis.
C. (2) and (4) * B. Hyperkeratosis.
D. (4) only C. Parakeratosis.
E. All of the above. D. Dysplasia. *

Which of the following can be A definitive diagnosis of osteosarcoma


characterized by a narrowing of pulp is established by
chambers and root canals?
A. radiographic examination.
1. Aging. B. clinical examination.
2. Chronic trauma. C. aspiration cytology.
3. Dentinal dysplasia. D. biopsy. *
4. Taurodontism.
An occluded submandibular duct can
A. (1) (2) (3) * be diagnosed by
B. (1) and (3)
C. (2) and (4) A. history.
D. (4) only B. palpation.
E. All of the above. C. sialography.
D. occlusal radiographs.
The typical history of a mucocele is E. All of the above. *

A. a slowly growing tumour mass. Which of the following most


B. a pain immediately before eating. appropriately describes a traumatic
C. a trauma, swelling, rupture, neuroma?
disappearance and recurrence. *
D. an ulcerated area on buccal A. A slow-growing, painless neoplasm.
mucosa. B. A slow-growing, hypersensitive
E. frequent bleeding. nodule. *
C. A tumour at a recent extraction site.
Which one of the following would be of D. A tumour of the tongue.
greatest value in determining the
etiology of an oral ulceration?

Dr Abdul Naser Tamim SEHA, Dr Ghada Al Aqqad D.D.S, Dr Kamal Naser - Amrita Medical Centre, Dr Emad Wani - Shadi Dental Centre,
Dr Lina Anka SEHA, Dr Rouba Zgheibi SEHA, Maria Teresa Yongson Alejandrino SEHA 438
The redness of an inflammatory lesion E. Fingernails.
of oral mucosa is due to
Soft, white, elevated plaques of the
A. increased number of capillaries. oral mucosa are characteristic of
B. increased size of capillaries.
C. decreased thickness of epithelium. A. angioma.
D. decreased connective tissue B. candidiasis. *
elements. C. actinomycosis.
E. All of the above. * D. herpes simplex.
E. submucous fibrosis.
The term "carcinoma in situ" implies
that the lesion shows Median anterior maxillary cysts are
found in
A. metaplasia.
B. early invasion of malignant cells A. the zygomatic process of the
through the basement membrane. maxilla.
C. dysplasia of cells confined within the B. the incisive canal. *
epithelium. * C. the uvula.
D. distant metastasis of a malignant D. the hamular process.
tumour.
Which of the following is NEVER
Which of the following is NOT associated with an impacted tooth?
associated with infectious
mononucleosis? A. Adeno-ameloblastoma.
B. Odontogenic myxoma.
A. Pharyngitis. C. Pindborg's tumor.
B. Lymphadenopathy. D. Primordial cyst. *
C. Petechiae of the palate. E. Ameloblastoma.
D. Gingival enlargement. *
E. Fatigue. Oral leukoplakia has the most
favourable prognosis when it is
Which of the following is/are NOT
usually affected by hereditary A. present in a non-smoker.
ectodermal dysplasia? B. accompanied by pain.
C. infected with Candida albicans. *
A. Salivary glands. * D. speckled in appearance.
B. Teeth. E. on the hard palate.
C. Sweat glands.
D. Hair.

Dr Abdul Naser Tamim SEHA, Dr Ghada Al Aqqad D.D.S, Dr Kamal Naser - Amrita Medical Centre, Dr Emad Wani - Shadi Dental Centre,
Dr Lina Anka SEHA, Dr Rouba Zgheibi SEHA, Maria Teresa Yongson Alejandrino SEHA 439
Which of the following is NOT a Laboratory examination of the blood of
characteristic of an acute apical a patient with an acute bacterial
abscess (acute periradicular infection would show
abscess)?
A. lymphocytosis.
A. Pain on percussion. B. leukocytosis. *
B. Tooth feels elongated. C. monocytosis.
C. Pain to a cold stimulus.* D. leukopenia.
D. Pain on palpation. E. eosinophilia.

Chronically inflamed submandibular Histoplasmosis is a


lymph nodes are
A. non-specific bacterial infection.
A. soft. B. protean disease.
B. not palpable. C. viral disease.
C. firm. * D. fungal disease. *
D. fixed.
A characteristic of malignant tumors is
Swelling related to increased tissue the ability to
fluid is called
A. invade and metastasize. *
A. thrombosis. B. grow to large size and remain within
B. edema. * their capsule.
C. hematoma. C. remain localized.
D. embolism. D. grow slowly.
E. surgical emphysema.
Which of the following anatomic
Oral lichen planus has lesions which spaces is most likely to be involved as
a result of an apical infection of a
A. bleed readily. mandibular third molar?
B. occur in the debilitated.
C. exhibit a positive Nikolsky's sign. A. Sublingual.
D. histopathologically show B. Submandibular.*
lymphocytic infiltration. * C. Submental.
D. Submasseteric.

Dr Abdul Naser Tamim SEHA, Dr Ghada Al Aqqad D.D.S, Dr Kamal Naser - Amrita Medical Centre, Dr Emad Wani - Shadi Dental Centre,
Dr Lina Anka SEHA, Dr Rouba Zgheibi SEHA, Maria Teresa Yongson Alejandrino SEHA 440
Intermittent painful swelling in the
submandibular region that increases at White lesions of the oral mucosa may
mealtime is indicative of be produced by

A. a ranula. 1. Thickening of the epithelium.


B. a blockage of Wharton's duct. * 2. Increase of the keratinized layers.
C. Ludwig's angina. 3. Coagulation by heat or chemicals.
D. a blockage of Stensen's duct. 4. Mycotic infection.
E. an epidemic parotitis.
A. (1) (2) (3)
Inflammation involving the bone B. (1) and (3)
marrow of the jaw caused by infection C. (2) and (4)
from a tooth or the periodontium is D. (4) only
called E. All of the above. *

A. osteoma. Which of the following lesions has a


B. periostitis. tendency to bleed easily?
C. osteomyelitis. *
D. osteosclerosis. A. Pyogenic granuloma. *
E. pericementitis. B. Osteoma.
C. Fibroma.
Which of the following is NOT D. Papilloma.
associated with osteogenesis E. Lipoma.
imperfecta?
Which of the following conditions are
A. Brown teeth. associated with AIDS?
B. Brittle bones.
C. Thin enamel. * 1. Acute marginal periodontitis.
D. Blue sclerae. 2. Hairy leukoplakia.
3. Candidiasis.
Pyogenic granuloma is most frequently 4. Geographic tongue.
found on the
A. (1) and (2)
A. tongue. B. (1) (2) (3) *
B. gingiva. * C. (1) and (4)
C. buccal mucosa. D. All of the above.
D. tonsillar pillars.
E. lips.

Dr Abdul Naser Tamim SEHA, Dr Ghada Al Aqqad D.D.S, Dr Kamal Naser - Amrita Medical Centre, Dr Emad Wani - Shadi Dental Centre,
Dr Lina Anka SEHA, Dr Rouba Zgheibi SEHA, Maria Teresa Yongson Alejandrino SEHA 441
A person who has sickle cell anemia Petechiae are
may show certain radiographic changes
in the bones of the skull. These changes A. macules.
may be B. papules. *
C. nodules.
A. ‟punched-out” radiolucent lesions. D. vesicles.
B. a moth-eaten appearance of the E. bullae.
bone.
C. Gross irregularities with exostosis Excessive formation of scar tissue
formation. beyond the wound margin is called
D. a "hair on end" effect. *
A. a fibroma.
In radiography, minimum magnification
B. a keloid. *
and maximum definition are achieved by
C. a fibro-epithelial polyp.
D. epithelial hyperplasia.
A. minimum OFD (object-film distance)
and minimum FFD (focal-film distance).
The finding of “acid-fast”
B. minimum OFD (object-film distance)
microorganisms in sputum suggests the
and maximum FFD (focal-film distance)*
presence of
C. maximum OFD (object -film distance)
and maximum FFD (focal-film distance).
A. Mycobacterium tuberculosis. *
D. maximum OFD (object-film distance)
B. Diplococcus pneumoniae.
and minimum FFD (focal-film distance).
C. Streptococcus pyogenes.
The earliest radiographic sign of D. Neisseria gonorrhoeae.
occlusal trauma is
The most logical explanation for
causing swelling beneath the eye
A. hypercementosis.
caused by an abscessed maxillary
B. root resorption.
canine is that the
C. alteration of the lamina dura.
D. widening of the periodontal ligament A. lymphatics drain superiorly in this
space. * region.
E. ankylosis. B. bone is less porous superior to the
root apex.
Intraoral soft tissue examination will
C. infection has passed into the angular
NOT assist in the diagnosis of
vein which has no valves.
D. the root apex lies superior to the
A. lichen planus.
attachment of the caninus and levator
B. sinusitis. * labii superioris muscles. *
C. erythema multiforme.
D. anemia.
E. vitamin deficiencies.

Dr Abdul Naser Tamim SEHA, Dr Ghada Al Aqqad D.D.S, Dr Kamal Naser - Amrita Medical Centre, Dr Emad Wani - Shadi Dental Centre,
Dr Lina Anka SEHA, Dr Rouba Zgheibi SEHA, Maria Teresa Yongson Alejandrino SEHA 442
Tissue from a multilocular radiolucent Which of the following is NOT a sign or
area of the posterior mandible symptom of the myofascial pain
microscopically shows follicular areas dysfunction syndrome?
lined with cylindrical cells resembling
the enamel organ. The most likely A. Pain.
diagnosis is a/an B. Muscle tenderness.
C. Limitation of jaw motion.
A. neurofibroma. D. "Clicking" or "popping" noise in the
B. ameloblastoma. * joints.
C. central fibroma. E. Radiographic changes of the joint. *
D. periodontal cyst.
E. dentigerous cyst. The prolonged use of antibacterial
lozenges or mouthwashes contributes to
In radiography, a parallel technique or the development of
right angle technique as opposed to a
bisecting angle technique will result in A. oral candidiasis. *
B. geographic tongue.
1. Less gonadal radiation. C. cancrum oris.
2. Greater entrance dosage. D. Koplik's spots.
3. Less dimensional distortion. E. aphthous ulcers.
4. A more heterogenous beam of X-
rays. Among the following, which may be
associated with root resorption?
A. (1) (2) (4)
B. (2) and (3) 1. Excessive orthodontic forces.
C. (2) and (4) 2. Periapical granuloma.
D. (1) and (3) * 3. Cementoma.
E. All of the above. 4. Hypercementosis.
5. Traumatic injury.
In an adult, progressive increase in
mandibular length and interdental A. (1) (2) (4)
spacing is a feature of B. (1) (2) (4) (5)
C. (1) (2) (5) *
A. hyperparathyroidism. D. (1) (2) (3) (5)
B. hyperpituitarism. * E. All of the above.
C. hyperthyroidism.
D. Addison's disease. A draining fistula of short duration
E. Cushing's disease. related to a tooth undergoing endodontic
therapy requires
Ludwig's angina may cause
A. irrigation of canals.
A. respiratory obstruction. * B. antibiotics.
B. cavernous sinus thrombosis. C. surgical excision.
C. suppurative encephalitis. D. no special treatment. *
D. subdural empyema.

Dr Abdul Naser Tamim SEHA, Dr Ghada Al Aqqad D.D.S, Dr Kamal Naser - Amrita Medical Centre, Dr Emad Wani - Shadi Dental Centre,
Dr Lina Anka SEHA, Dr Rouba Zgheibi SEHA, Maria Teresa Yongson Alejandrino SEHA 443
Multiple supernumerary teeth are Mucoceles are most commonly found in
most commonly found in the

A. cherubism. A. upper lip.


B. cretinism. B. lower lip. *
C. hypothyroidism. C. tongue.
D. cleidocranial dysplasia. * D. buccal mucosa.
E. Down's syndrome. E. soft palate.

The most frequent location of a Which of the following is the most


dentigerous cyst is the appropriate early management for a
patient with primary herpetic
A. Third molar area. * gingivostomatitis? Prescribing
B. symphysis of the mandible.
C. midline of the hard palate. A. corticosteroids locally and
D. apical area of a devitalized tooth. systemically.
E. premolar area. B. a systemic antiviral. *
C. a systemic antibiotic.
In fibrous dysplasia D. a systemic antifungal.

A. foci of cartilage are a common Generalized widening of the periodontal


histological finding. ligament space is a radiologic
B. an inflammatory infiltrate is characteristic of
characteristically present.
C. there are characteristic changes in A. lupus erythematosis.
the blood chemistry. B. scleroderma. *
D. a ground-glass appearance is C. Stevens-Johnson syndrome.
present on radiographs. * D. osteitis deformans.
E. acromegaly.
"Dens in dente" (dens invaginatus) is
associated with A patient with hyperthyroidism may
exhibit
A. supernumerary teeth.
B. dentinogenesis imperfecta. A. weight gain.
C. osteogenesis imperfecta. B. delayed eruption of teeth.
D. anterior teeth. * C. exophthalmos. *
E. amelogenesis imperfecta. D. gingival inflammation.

Dr Abdul Naser Tamim SEHA, Dr Ghada Al Aqqad D.D.S, Dr Kamal Naser - Amrita Medical Centre, Dr Emad Wani - Shadi Dental Centre,
Dr Lina Anka SEHA, Dr Rouba Zgheibi SEHA, Maria Teresa Yongson Alejandrino SEHA 444
In an adult, continued growth of the 1. Enlargement.
mandibular condyles, an increase in 2. Bleeding.
size of the bones and soft tissues of 3. Ulceration.
the hands, feet, supraorbital ridges and 4. Atrophy.
air sinuses suggest
A. (1) (2) (3) *
A. Addison’s disease. B. (1) and (3)
B. hyperthyroidism. C. (2) and (4)
C. pituitary adenoma. D. (4) only
D. gigantism. *
E. All of the above.
Acromegaly is associated with
A common clinical sign of occlusal
traumatism is
A. hypothyroidism.
B. hyperadrenalism. A. tooth mobility. *
C. hypogonadism. B. pocket formation.
D. pituitary adenoma. * C. gingival recession.
E. hyperparathyroidism. D. temporomandibular joint pain -
dysfunction syndrome.
Which one of the following is the most
E. pulp calcifications.
common tumour of the salivary glands?
Extreme widening of the periodontal
A. Adenocystic carcinoma. ligament may be seen in
B. Adenoma.
C. Pleomorphic adenoma. * A. Parkinson's disease.
D. Muco-epidermoid carcinoma. B. Raynaud's disease.
C. Bell's palsy.
Oral foci of infection are of greatest
D. Osteosarcoma. *
clinical significance in the presence of
E. Ménière's syndrome.
A. polycythemia vera.
Multiple giant cell lesions of the bone
B. iritis and uveitis. are associated with
C. eczema and urticaria.
D. rheumatoid arthritis. A. hyperthyroidism.
E. subacute bacterial endocarditis. * B. hypothyroidism.
C. hyperparathyroidism.*
Which gingival manifestation(s) would
D. hypoparathyroidism.
be expected in a patient with a blood
dyscrasia?

Dr Abdul Naser Tamim SEHA, Dr Ghada Al Aqqad D.D.S, Dr Kamal Naser - Amrita Medical Centre, Dr Emad Wani - Shadi Dental Centre,
Dr Lina Anka SEHA, Dr Rouba Zgheibi SEHA, Maria Teresa Yongson Alejandrino SEHA 445
The clinical appearance and texture of D. Down syndrome.
an early carcinoma of the floor of the
mouth can be A patient presents with apparent
paralysis of one side of the face which
A. red and soft. appeared the day before. What is the
B. white and rough. most likely diagnosis?
C. ulcerated and indurated.
D. All of the above. * A. Glossodynia.
B. Bell's palsy. *
Squamous cell carcinomas of the lip C. Myasthenia gravis.
occur most frequently on the D. Trigeminal neuralgia.

A. commissures. In the presence of an acute bacterial


B. lower lip near the midline. * infection, laboratory tests will show an
C. inner surface of upper lip. increase in
D. inner surface of lower lip.
E. upper lip near the midline. A. polymorphonuclear leukocytes. *
B. plasma cells.
An ameloblastoma is most frequently C. lymphocytes.
found in D. monocytes.
E. eosinophils.
A. the anterior region of the maxilla.
B. the mandible, near the junction of Which is a characteristic of a patient
the body and the ramus. * with myxedema?
C. the posterior region of the maxilla.
D. in the anterior region of the A. Exophthalmos.
mandible near the midline. B. Weight loss.
C. Heat intolerance.
Sickle cell anemia is D. Lethargic appearance. *
E. Tachycardia.
A. a genetic disease. *
B. caused by exposure to radiation. Median anterior maxillary cysts occur in
C. a viral infection. the
D. a drug reaction.
E. an auto-immune disease. A. nasal bone.
B. incisive canal and in the palatine
Multiple neurofibromatosis and "café process. *
au lait" spots on the skin are typical of C. zygomatic process.
D. hamular process.
A. Gardner's syndrome.
B. Plummer-Vinson syndrome.
C. Von Recklinghausen's disease*

Dr Abdul Naser Tamim SEHA, Dr Ghada Al Aqqad D.D.S, Dr Kamal Naser - Amrita Medical Centre, Dr Emad Wani - Shadi Dental Centre,
Dr Lina Anka SEHA, Dr Rouba Zgheibi SEHA, Maria Teresa Yongson Alejandrino SEHA 446
In the early stage, a periapical abscess A. dentin.
can be differentiated from a lateral B. enamel.
periodontal abscess by C. cementum *
D. pulp.
A. pain. E. periodontal ligament.
B. type of exudate.
C. tenderness to percussion. Radiographically, a benign bone
D. response of pulp to electrical neoplasm is differentiated from a
stimulation.* malignant one because in the benign
E. radiographic examination. lesion

Ankylosis is commonly 1. The margins are irregular and fade


into the surrounding bone.
A. associated with a non-vital pulp. 2. The cortex remains intact.
B. the result of a root fracture. 3. There can be perforation of the
C. associated with a root penetrating periosteum.
cavity. 4. The margins are defined and
D. resulting in a submerged tooth, out demarcated.
of occlusion. *
E. found in permanent teeth. A. (1) (2) (3)
B. (1) and (3)
On an occlusal radiograph, it is C. (2) and (4) *
possible to misdiagnose the midline D. (4) only
palatal suture as a E. All of the above.

A. fracture. * Which of the following is/are associated


B. palatal cyst. with dentin dysplasia type I?
C. granuloma.
D. abscess. 1. Obliteration of pulp chambers.
2. Normal appearance of clinical
The lamina dura is crowns.
3. Small underdeveloped roots.
A. cortical bone. 4. Periapical radiolucent areas.
B. spongy bone.
C. immature bone. A. (1) (2) (3)
D. a cribiform plate perforated by B. (1) and (3)
nutrient canals. * C. (2) and (4)
E. None of the above. D. (4) only
E. All of the above. *
The tissue which cannot be seen on
dental radiographs is

Dr Abdul Naser Tamim SEHA, Dr Ghada Al Aqqad D.D.S, Dr Kamal Naser - Amrita Medical Centre, Dr Emad Wani - Shadi Dental Centre,
Dr Lina Anka SEHA, Dr Rouba Zgheibi SEHA, Maria Teresa Yongson Alejandrino SEHA 447
A radiolucent area in a radiograph
occurs as a result of Selection of the appropriate
kilovoltage for dental films is
1. decreased density of tissue. influenced by
2. increased density of tissue.
3. More radiation affecting the silver A. line voltage fluctuation.
halide crystals. B. diameter of the primary beam of
4. Increased exposure time. radiation.
C. type of timer.
A. (4) only D. tissue density *
B. (2) and (3) E. filter thickness.
C. (1) and (3) *
D. (1) only A zinc phosphate cement base
E. None of the above.
A. has the same radiopacity as
The fixing solution serves the purpose amalgam.
of B. is less radiopaque than amalgam.*
C. has the same radiopacity as gold.
1. Carrying on development. D. cannot be seen on a radiograph.
2. Hardening the emulsion.
3. Removing unexposed silver salts. The quantity of radiation output in a
dental X-ray apparatus is a function
A. (1) and (2) of
B. (1) and (3)
C. (2) and (3) * 1. Time.
D. All of the above. 2. kVp.
E. None of the above. 3. ma.
4. Filtration.
Chronic disseminated Langerhans cell
disease A. (1) (2) (3)
B. (1) and (3) *
A. produces a solitary eosinophilic C. (2) and (4)
lesion. * D. (4) only
B. produces bony defects as focal E. All of the above.
areas of bony rarefaction.
C. occurs only in adult life. The collimator of an x-ray tube
D. is a malignant lesion.
A. produces a more homogeneous x-
Papillary hyperplasia under a denture ray beam.
is usually due to (an) B. prevents secondary radiation.
C. focuses the x-ray beam.
A. moniliasis. D. restricts the diameter of the x-ray
B. ill-fitting denture * beam. *
C. allergy to denture cleanser.
D. avitaminosis.

Dr Abdul Naser Tamim SEHA, Dr Ghada Al Aqqad D.D.S, Dr Kamal Naser - Amrita Medical Centre, Dr Emad Wani - Shadi Dental Centre,
Dr Lina Anka SEHA, Dr Rouba Zgheibi SEHA, Maria Teresa Yongson Alejandrino SEHA 448
Proper collimation of the useful beam C. (2) and (4)
for the film size and target-film distance D. (4) only
will reduce E. All of the above.

1. Intensity of central beam. Radiographically, a keratocystic


2. Secondary radiation. odontogenic tumour (odontogenic
3. Radiographic contrast. keratocyst) can appear as a
4. Image definition.
5. Radiation received by patient. A. mixed radiolucency and radiopacity*
B. radiolucency around the crown of an
A. (1) (2) (3) (5) impacted tooth.
B. (1) (3) (4) (5) C. radiolucency containing multiple
C. (2) (3) (4) (5) rudimentary teeth.
D. (2) and (5) *
E. All of the above. The apical region of a non-vital tooth
with a deep carious lesion may
Acute maxillary sinusitis is associated radiographically show
with
1. Widening of the periodontal space.
1. Pain in the posterior maxillary teeth. 2. Loss of lamina dura.
2. Nasal discharge. 3. A circumscribed radiolucency.
3. Tenderness of posterior maxillary 4. Calcification of the periodontal
teeth to percussion. membrane.
4. Increase of pain when bending over.
A. (1) (2) (3) *
A. (1) (2) (3) B. (1) and (3)
B. (1) and (3) C. (2) and (4)
C. (2) and (4) D. (4) only
D. (4) only E. All of the above.
E. All of the above. *
The benign neoplasm that originates
In infectious mononucleosis you are from squamous epithelium is called
most likely to find a/an

1. A positive Paul Bunnel test. A. adenoma.


2. Lymphadenopathy. B. choriocarcinoma.
3. Palatine petechiae. C. chondroma.
4. Leukopenia. D. lipoma.
E. papilloma. *
A. (1) (2) (3) *
B. (1) and (3)

Dr Abdul Naser Tamim SEHA, Dr Ghada Al Aqqad D.D.S, Dr Kamal Naser - Amrita Medical Centre, Dr Emad Wani - Shadi Dental Centre,
Dr Lina Anka SEHA, Dr Rouba Zgheibi SEHA, Maria Teresa Yongson Alejandrino SEHA 449
A large encapsulated fluid-filled tumour B. Lipstick.
is removed from the hard palate. The C. Acrylic.
most appropriate method of D. Antibiotics.
determining the nature of this lesion is E. All of the above. *
to
An abnormal decrease in the flow of
A. examine the fluid under a saliva is
microscope.
B. culture the fluid and examine for A. ptyalism.
bacterial growth. B. sialometaplasia.
C. submit the tissue for histological C. xerostomia. *
examination * D. pyroglossia.
D. submit the tissue for exfoliative
cytological study. A salivary calculus is a
E. aspirate the fluid for electrophoresis
study. A. sialolith. *
B. rhinolith.
Which one of the following teeth is C. phlebolith.
most frequently impacted? D. thrombolith.
A. Maxillary cuspid. * In dental radiology, patient protection
B. Mandibular second molar. from radiation is most important for
C. Mandibular cuspid.
D. Maxillary first premolar. A. patients receiving antibiotics.
B. patients receiving corticosteroids.
Fordyce's granules are C. individuals over fifty-years of age.
D. pregnant women. *
A. ectopic sebaceous glands.* E. young adults.
B. ectopic sweat glands.
C. small calcified nodules. When a patient experiences
D. aberrant mucous glands. continuous pain in the maxillary
premolar and molar areas and there is
Acellular cementum on a root is no evidence of dental infection, the
most likely diagnosis is
A. the result of chronic inflammation.
B. a defective cementoid substance. A. trigeminal neuralgia.
C. caused by premature degeneration B. acute maxillary sinusitis. *
of Hertwig's root sheath. C. impacted maxillary canine.
D. a normal anatomical structure. * D. impacted maxillary third molar.
E. glossopharyngeal neuralgia.
Which of the following can cause a
contact stomatitis?

A. Dentifrice.

Dr Abdul Naser Tamim SEHA, Dr Ghada Al Aqqad D.D.S, Dr Kamal Naser - Amrita Medical Centre, Dr Emad Wani - Shadi Dental Centre,
Dr Lina Anka SEHA, Dr Rouba Zgheibi SEHA, Maria Teresa Yongson Alejandrino SEHA 450
A patient who uses nitroglycerine has Erythroblastosis fetalis may be a
cause of
A. rheumatic heart disease.
B. asthma. A. supernumerary incisors.
C. coronary artery disease. * B. pigmented teeth. *
D. high blood pressure. C. peg lateral incisors.
E. cardiac arrhythmia. D. Fordyce's granules.
E. blue sclerae.
The term applied to a low white blood
cell count is Untreated diabetes mellitus
characteristically demonstrates
A. leukocytosis.
B. leukopenia. * A. hypoglycemia.
C. thrombocythemia. B. hyperglycemia. *
D. thrombocytopenia. C. hypophagia.
D. hyperlipidemia.
A decrease in the neutrophil count is E. dysuria.
present in
A decrease of which of the following is
A. granulocytopenia (agranulocytosis)* indicative of hypoparathyroidism?
B. iron deficiency anemia.
C. myeloid leukemia. A. Serum phosphorus.
D. leukocytosis. B. Serum calcium. *
E. thrombocytopenic purpura. C. Thyroid activity.
D. Serum alkaline phosphatase.
Coronary artery occlusion can lead to
Oral lesions that fail to heal may be
A. thrombosis. associated to
B. embolism.
C. infarction. * 1. Tuberculosis.
D. fatty degeneration. 2. Syphilis.
3. Neoplasia.
An anemia in which the red blood cells 4. Diabetes.
are smaller and less intense in color
than normal is called a A. (1) (2) (3)
B. (1) and (3)
A. microcytic hypochromic anemia. * C. (2) and (4)
B. microcytic hyperchromic anemia. D. (4) only
C. macrocytic hypochromic anemia. E. All of the above. *
D. macrocytic hyperchromic anemia.
E. None of the above.

Dr Abdul Naser Tamim SEHA, Dr Ghada Al Aqqad D.D.S, Dr Kamal Naser - Amrita Medical Centre, Dr Emad Wani - Shadi Dental Centre,
Dr Lina Anka SEHA, Dr Rouba Zgheibi SEHA, Maria Teresa Yongson Alejandrino SEHA 451
The electric pulp tester might be of D. re-excision with wider margins.
some value in determining whether E. radium implantation around biopsy
site.
1. The pulp is hyperemic or
hyperplastic. Increasing the kilovoltage setting on the
2. There is a partial necrosis of the dental x-ray machine results in
pulp.
3. There is a partial or total pulpitis. A. more gamma radiation.
4. The pulp is vital or nonvital. B. greater collimation.
C. more penetration. *
A. (1) (2) (3) D. greater secondary radiation at the
B. (1) and (3) level of the skin.
C. (2) and (4)
D. (4) only * Radiographs of Garre's osteomyelitis
E. All of the above. show

Hypercementosis (cemental A. formation of sequestra.


hyperplasia) B. a worm-eaten pattern of bone
destruction.
A. is most often confined to the apical C. thickening of the cortex. *
half of the root. * D. sinus tracts.
B. most frequently affects molars.
C. affects non vital teeth in the majority Secondary dentin formation may be
of cases. stimulated by
D. (A) and (C)
E. (B) and (C) A. pulp necrosis.
B. fluorosis.
The greatest single factor in reducing C. attrition. *
radiation exposure in dentistry is D. vitamin D therapy.

A. higher kVp. Histological sections of a lesion


B. proper filtration. removed from the apex of a carious
C. high speed film. tooth show immature fibrous tissue and
D. collimation of the X-ray beam* chronic inflammatory cells. The most
likely diagnosis is a/an
An excisional biopsy of a nodule 5mm
in diameter on the lateral border of the A. acute periapical abscess.
tongue was diagnosed as a fibroma. B. odontogenic fibroma.
This patient should have C. radicular cyst.
D. periapical granuloma. *
A. hemisection of the tongue. E. central fibroma.
B. radiotherapy to site of biopsy.
C. no additional therapy. *

Dr Abdul Naser Tamim SEHA, Dr Ghada Al Aqqad D.D.S, Dr Kamal Naser - Amrita Medical Centre, Dr Emad Wani - Shadi Dental Centre,
Dr Lina Anka SEHA, Dr Rouba Zgheibi SEHA, Maria Teresa Yongson Alejandrino SEHA 452
The primordial cyst probably results
from 1. Nasociliary.
2. Nasopalatine.
A. cystic degeneration of the stellate 3. Sphenopalatine.
reticulum early in the formation of the 4. Anterior superior alveolar.
tooth. *
B. epithelial remnants in the periodontal A. (1) (2) (3)
ligament. B. (1) and (3)
C. an extension of pulpal inflammation C. (2) and (4) *
after death of the pulp. D. (4) only
D. failure of formation of the enamel E. All of the above.
matrix.
E. transformation of the dental lamina. Tooth mobility may be due to

Hyperkeratosis, acanthosis, dysplasia, 1. Excessive occlusal force.


increased mitosis, intact basal cell layer 2. decreased osseous support.
and chronic inflammatory cells are 3. Periodontal abscess.
histologic features that may be found in
4. Gingival inflammation.
A. squamous cell carcinoma. A. (1) (2) (3) *
B. carcinoma in situ. * B. (1) and (3)
C. papillofibroma. C. (2) and (4)
D. endothelioma. D. (4) only
E. All of the above.
Enlargement of the gingiva, described
as idiopathic fibromatosis, is best
Treatment of primary herpetic
described as
gingivostomatitis should include
A. degeneration.
1. Palliative treatment.
B. inflammation.
2. Steroid therapy.
C. hyperplasia. *
3. Control of secondary infection.
D. neoplasia.
4. Application of dilute hydrogen
peroxide.
Lichen planus occurs most frequently
on the
A. (1) (2) (3)
B. (1) and (3) *
A. buccal mucosa. *
C. (2) and (4)
B. tongue.
D. (4) only
C. floor of the mouth.
E. All of the above.
D. gingiva.

Which of the following nerves should be


anesthetized for extraction of a
maxillary lateral incisor?

Dr Abdul Naser Tamim SEHA, Dr Ghada Al Aqqad D.D.S, Dr Kamal Naser - Amrita Medical Centre, Dr Emad Wani - Shadi Dental Centre,
Dr Lina Anka SEHA, Dr Rouba Zgheibi SEHA, Maria Teresa Yongson Alejandrino SEHA 453
The difference(s) between deciduous D. removal of all pulp horns.
and permanent teeth isçare best
described by the following Which of the following conditions is
statement(s): most likely to be associated with a
draining fistula?
A. The deciduous root trunk is shorter.
B. The deciduous enamel is thinner A. Chronic periapical periodontitis. *
and appears whiter. B. Reversible pulpitis.
C. The deciduous molar roots flare C. Hypercementosis.
more. D. Traumatic bone cyst.
D. (A) and (C)
E. All of the above. * Histologically, a hyperplastic pulp
consists of all of the following EXCEPT
Special attention is given to matrix for
adaptation for the insertion of amalgam
in a MO cavity in a maxillary first A. a mass of collagenous fibres.
premolar because of the B. Russell bodies. *
C. proliferating capillaries.
A. concavity in the cervical third of the D. fibroblasts.
mesial surface of the crown. * E. polymorphonuclear leucocytes.
B. restoration being in the esthetic
zone. Which of the following are
C. unusual position of the contact area. characteristic symptoms of acute
D. buccolingual width of the tooth's suppurative pulpitis?
mesial marginal ridge.
E. size of the interproximal gingival 1. Spontaneous throbbing pain.
embrasure. 2. Prolonged pain initiated by heat.
3. Increased pain while lying down.
A child has a carious exposure of the 4. Increased pain by cold.
pulp in the first molar. The cavity is
filled with pink tissue which bleeds A. (1) (2) (3) *
when punctured by the explorer. The B. (1) and (3)
tissue is slightly sensitive to touch. C. (2) and (4)
This is indicative of D. (4) only
E. All of the above.
A. acute ulcerative pulpitis.
B. chronic serous pulpitis. The pulpal floor of a Class II cavity is
C. chronic hyperplastic pulpitis. * cut perpendicular to the long axis of
D. periapical osteofibrosis. the tooth EXCEPT in the

The most important principle dictating A. maxillary first premolar.


location and size of access to the root B. maxillary second premolar.
canal system is C. mandibular second premolar.
D. mandibular first premolar. *
A. preservation of tooth structure.
B. removal of all caries.
C. straight line access to the canal *

Dr Abdul Naser Tamim SEHA, Dr Ghada Al Aqqad D.D.S, Dr Kamal Naser - Amrita Medical Centre, Dr Emad Wani - Shadi Dental Centre,
Dr Lina Anka SEHA, Dr Rouba Zgheibi SEHA, Maria Teresa Yongson Alejandrino SEHA 454
What are the purposes of using 2. There is pain on percussion.
occlusal splints? 3. The tooth throbs when the patient is
lying down.
1. To change the pattern and degree 4. The radiograph shows an apical
of tactile afferent neural impulses. radiolucency.
2. To immobilize teeth.
3. To prevent teeth from disturbing A. (1) (2) (3)
occlusal sensory input. B. (1) and (3)
4. To produce a permanent change in C. (2) and (4) *
the occlusion. D. (4) only
E. All of the above.
A. (1) (2) (3) *
B. (1) and (3) After a thermal stimulus has been
C. (2) and (4) removed from a tooth persistent pain
D. (4) only suggests
E. All of the above.
A. a normal pulp.
Which of the following muscles B. pulp necrosis.
comprise the retromolar pad? C. reversible pulpitis.
D. irreversible pulpitis. *
1. Lateral (external) pterygoid. E. exposed cervical dentin.
2. Buccinator.
3. Palatoglossus. When odontoblasts are destroyed or
4. Superior constrictor. undergo degeneration, they are
replaced by
A. (1) (2) (3)
B. (1) and (3) A. ameloblasts.
C. (2) and (4) * B. undifferentiated mesenchymal
D. (4) only cells. *
E. All of the above. C. multinucleated giant cells.
D. osteoblasts.
Gutta-percha may be softened or
dissolved within the root canal by Fractured incisal angles in the
using permanent teeth of adolescent
patients are best restored using
A. alcohol.
B. ethyl chloride. A. glass ionomer.
C. eugenol. B. gold castings.
D. xylol® (xylene). * C. full coverage restorations.
D. acid etch composite resin
In teeth with complete pulp necrosis, techniques. *
the periapical area is involved if

1. There is pain to thermal stimuli.

Dr Abdul Naser Tamim SEHA, Dr Ghada Al Aqqad D.D.S, Dr Kamal Naser - Amrita Medical Centre, Dr Emad Wani - Shadi Dental Centre,
Dr Lina Anka SEHA, Dr Rouba Zgheibi SEHA, Maria Teresa Yongson Alejandrino SEHA 455
The joining together of two teeth in the C. Lactobacilli.
root portion through cemental union is D. Enterococci. *
known as E. Staphylococcus albus.

A. gemination. What clinical evidence would support a


B. fusion. diagnosis of acute dento-alveolar
C. twinning. abscess?
D. concrescense. *
1. A negative reaction to the electric
A protective mechanism of the dental vitality tester.
pulp to external irritation or caries is the 2. A positive reaction of short duration
formation of to cold.
3. A positive reaction to percussion.
A. pulp stones. 4. Presence of a draining fistula.
B. secondary dentin. *
C. secondary cementum. A. (1) (2) (3)
D. primary dentin. B. (1) and (3) *
C. (2) and (4)
In the mandibular first premolar, the D. (4) only
occlusal dovetail of an ideal disto- E. All of the above.
occlusal amalgam preparation is
usually not extended into the mesial Severe throbbing tooth pain which
fossa because of the increases when the patient lies down is
a symptom of
A. small lingual lobe.
B. large buccal cusp. A. a pulp polyp (chronic hyperplastic
C. large buccal pulp horn. pulpitis).
D. prominent transverse ridge. * B. late stage of acute pulpitis (acute
suppurative pulpitis). *
Which of the following is the most C. chronic pulpitis (chronic ulcerative
probable postoperative complication of pulpitis).
intracoronal bleaching for a tooth that D. chronic apical abscess.
has not been adequately obturated? E. pulp hyperemia.

A. Fracture. The cell of the dental pulp most


B. Discolouration. capable of transforming into other cells
C. Retrograde pulpitis. is the
D. External root resorption. *
A. fibroblast.
Which of the following microorganisms B. undifferentiated mesenchymal cell. *
are most frequently found in infected C. odontoblast.
root canals? D. histiocyte.

A. Streptococcus viridans.
B. Staphylococcus aureus.

Dr Abdul Naser Tamim SEHA, Dr Ghada Al Aqqad D.D.S, Dr Kamal Naser - Amrita Medical Centre, Dr Emad Wani - Shadi Dental Centre,
Dr Lina Anka SEHA, Dr Rouba Zgheibi SEHA, Maria Teresa Yongson Alejandrino SEHA 456
Sterilization of carious dentin without has its primary influence on the
pulp injury is assured by the morphology of
application of
A. cusp height.
A. phenol. B. anterior teeth only.
B. 70% ethyl alcohol. C. mesial inclines of maxillary cusps
C. chlorhexidine. and distal inclines of mandibular cusps.
D. absolute alcohol. D. mesial inclines of mandibular cusps
E. None of the above. * and distal inclines of maxillary cusps. *

A lingual approach for a conservative Which of the following instruments can


Class III preparation for a composite be used for placing gingival bevels on
resin requires cast gold inlay preparations?

A. a retentive internal form. 1. Margin trimmers.


B. parallelism of the incisal and 2. Enamel hatchets.
gingival walls. 3. Carbide finishing burs.
C. maintenance of the incisal contact 4. Small diamond discs.
area.
D. All of the above. * A. (1) (2) (3)
B. (1) and (3) *
Which of the following statements are C. (2) and (4)
true concerning the adult mandible? D. (4) only
E. All of the above.
1. The mandibular foramen lies in the
centre of the mandibular ramus both in When using a high-speed handpiece
the vertical and horizontal planes. for cavity preparation, the heat
2. The angle formed by the junction of generated is directly related to the
the ramus and the body of the
mandible is an acute one. 1. Duration of cutting.
3. The genial tubercles are 2. Size, speed and sharpness of the
attachments for the anterior bellies of bur.
the digastric muscles. 3. Use of air and water spray.
4. The temporalis muscle attaches to 4. Effectiveness of the suction.
the lateral surface of the coronoid
process. A. (1) (2) (3) *
B. (1) and (3)
A. (1) and (2) C. (2) and (4)
B. (1) and (4) * D. (4) only
C. (1) and (3) E. All of the above.
D. (1) (3) (4)

In restoring occlusal anatomy, the


protrusive condylar path inclination

Dr Abdul Naser Tamim SEHA, Dr Ghada Al Aqqad D.D.S, Dr Kamal Naser - Amrita Medical Centre, Dr Emad Wani - Shadi Dental Centre,
Dr Lina Anka SEHA, Dr Rouba Zgheibi SEHA, Maria Teresa Yongson Alejandrino SEHA 457
For composite resin preparations, A retrograde filling is indicated
cavosurface enamel margins are
bevelled because A. when the apical foramen cannot be
sealed by conventional endodontics.
1. A bevelled margin produces a more B. when a root perforation needs to be
favorable surface for etching. sealed.
2. A bevelled margin improves the C. when conventional endodontics is
edge strength of the composite resin. impractical.
3. After etching, the bonding agent D. All of the above. *
reduces microleakage.
4. The procedure eliminates the need In root resection (apicoectomy) it is
to polish the restoration. appropriate to

A. (1) (2) (3) A. remove as little of the root as


B. (1) and (3) * possible.
C. (2) and (4) B. curette the soft tissue lesion in its
D. (4) only entirety.
E. All of the above. C. be certain the apex is sealed.
D. All of the above. *
The mesial and distal walls of a Class I
amalgam preparation diverge toward Apicoectomy is CONTRAINDICATED
the occlusal surface in order to when

A. resist the forces of mastication. A. periodontal disease causes


B. provide resistance and retention inadequate bony support. *
form. B. there is a granuloma at the apex of
C. prevent undermining of the marginal the tooth.
ridges. * C. more than one tooth is involved.
D. extend the preparation into areas D. the cortical plate is more than 4mm
more readily cleansed. thick.
E. the patient is diabetic.
Hyperemia of the pulp is
The proposed mechanism by which a
A. an acute inflammation with calcium hydroxide preparation initiates
intermittent paroxysms of pain which secondary dentin formation in direct
may become continuous. pulp cappings is by
B. an increased volume of blood within
dilated vessels and increased blood A. releasing calcium ions.
flow. * B. stimulating differentiated ameloblasts
C. a chronic situation whereby minute to lay down dentin.
arterioles of pulpal tissue are engorged C. stimulating fibroblasts to elaborate
for long periods creating temporary nuclei of the first order.
episodes of pain. D. stimulating undifferentiated cells of
D. a transient invasion of bacterial the tissue to differentiate into
elements into the outer lying stroma of odontoblasts. *
the pulpal tissue.

Dr Abdul Naser Tamim SEHA, Dr Ghada Al Aqqad D.D.S, Dr Kamal Naser - Amrita Medical Centre, Dr Emad Wani - Shadi Dental Centre,
Dr Lina Anka SEHA, Dr Rouba Zgheibi SEHA, Maria Teresa Yongson Alejandrino SEHA 458
Which one of the following is the initial 2. A definitive diagnosis of an apical
treatment for internal resorption? lesion cannot be made on radiography
alone.
A. Pulpectomy. * 3. Periapical radiolucencies are not
B. Pulpotomy. always indicative of loss of pulp vitality.
C. Pulp capping. 4. A periapical radiograph can be used
D. Apicoectomy. to locate the buccal bone level.

The material of choice for obturating A. (1) (2) (3) *


the root canal system of a primary B. (1) and (3)
tooth is C. (2) and (4)
D. (4) only
A. silver cone. E. All of the above.
B. gutta percha.
C. zinc-oxide eugenol. * Which of the following may affect the
D. paper point medicated with results of electric pulp testing?
formocresol.
E. zinc oxyphosphate. A. Emotional factors.
B. Pain threshold.
Which of the following is/are C. Analgesics.
associated with the presence of D. Recent trauma.
microorganisms in the bloodstream? E. All of the above. *

A. Anachoresis. The air-water spray used as a coolant


B. Cavernous sinus thrombosis. in high speed cutting of a cavity will
C. Bacteremia.
D. All of the above. * 1. Decrease pulp damage.
2. Reduce frictional heat.
A positive and prolonged reaction to a 3. Keep the operating site clean.
heat stimulus indicates that the pulp is 4. Reduce clogging of cutting
instruments.
A. necrotic.
B. in an early hyperemic state. A. (1) (2) (3)
C. normal. B. (1) and (3)
D. irreversibly damaged. * C. (2) and (4)
D. (4) only
Which of the following statements E. All of the above. *
is/are true?

1. Radiographs cannot differentiate


between infected and non-infected
periapical lesions.

Dr Abdul Naser Tamim SEHA, Dr Ghada Al Aqqad D.D.S, Dr Kamal Naser - Amrita Medical Centre, Dr Emad Wani - Shadi Dental Centre,
Dr Lina Anka SEHA, Dr Rouba Zgheibi SEHA, Maria Teresa Yongson Alejandrino SEHA 459
Firm contact between approximating would most likely result in periodontal
teeth is important because it ligament perforation in the

A. locates the marginal ridges of each A. mesial of a mandibular first


tooth. premolar.
B. keeps the teeth from having any B. distal of a mandibular first premolar.
movement during function. C. distal of a mandibular first molar.
C. insures proper cusp form and D. mesial of a mandibular first molar. *
increases masticatory efficiency.
D. stabilizes the dental arches and A patient telephones and tells you he
gives protection to the gingival papillae. has just knocked out his front tooth but
* that it is still intact. Your instructions
should be to
The lamina dura is
A. put the tooth in water and come to
A. spongy bone. your office at the end of the day.
B. cribriform plate. * B. wrap the tooth in tissue and come to
C. hypercalcified bone. your office in a week's time.
D. compact bone. C. put the tooth in alcohol and come to
your office immediately.
In permanent teeth, two pulp canals D. place tooth under the tongue and
are most commonly found in the come to your office immediately.
E. place the tooth in milk and come to
A. distobuccal root of maxillary molars. your office immediately. *
B. distal root of mandibular first molars.
C. palatal root of maxillary first After performing an apicoectomy,
premolars. which of the following should be placed
D. mesial root of mandibular first in the bony defect prior to suturing the
molars. * flap?

For a cast gold restoration, a gingival A. Corticosteroids.


bevel is used instead of a shoulder B. Antibiotic powder.
because a bevel C. Oxidized cellulose.
D. Bone wax.
1. Protects the enamel. E. Nothing. *
2. increases retention.
3. Improves marginal adaptation. Endodontic therapy is
4. Increases the thickness of gold. CONTRAINDICATED in teeth with

A. (1) (2) (3) A. inadequate periodontal support. *


B. (1) and (3) * B. pulp stones.
C. (2) and (4) C. constricted root canals.
D. (4) only D. accessory canals.
E. All of the above. E. curved roots.

The placement of a retentive pin in the


proximal regions of posterior teeth

Dr Abdul Naser Tamim SEHA, Dr Ghada Al Aqqad D.D.S, Dr Kamal Naser - Amrita Medical Centre, Dr Emad Wani - Shadi Dental Centre,
Dr Lina Anka SEHA, Dr Rouba Zgheibi SEHA, Maria Teresa Yongson Alejandrino SEHA 460
Which of the following methods of A patient experiences pain and some
instrument sterilization uses the lowest gingival swelling in the anterior
temperature? segment of the mandible. The
mandibular lateral incisor has a shallow
A. Steam autoclave. restoration, is tender to percussion and
B. Dry heat oven. gives a positive response to the electric
C. Ethylene oxide method. * pulp tester. There is some mobility.
D. Glass bead sterilizer. The most likely diagnosis is
E. Alcohol autoclave.
A. acute apical abscess.
After completion of endodontic B. acute serous pulpitis.
chemomechanical debridement C. lateral periodontal abscess. *
D. acute suppurative pulpitis.
A. all tissue should be removed from E. chronic ulcerative pulpitis.
the root canal system.
B. permanent inflammation may be Which of the following is LEAST likely
caused. to cause pain?
C. the root canal should be sterile.
D. some areas of the root canal system A. Carious pulp exposure.
may be incompletely cleaned. * B. Chronic hyperplastic pulpitis (pulp
polyp). *
Which one of the following statements C. Acute pulpitis.
is true as defined in endodontics? D. Apical periodontitis.

A. Overfilling refers to the incomplete


filling of the canal system with a Which of the following conditions
surplus of material extruding beyond CONTRAINDICATES routine dental
the apical foramen. treatment in the dental office?
B. Under filling refers to the complete
filling of the canal space, but leaving A. Hypothyroidism.
voids in the pulp chamber for possible B. Recent (15 days) myocardial
recontamination or infection. infarction*
C. Overextension refers to the C. Second trimester pregnancy.
extrusion of filling material through an D. Insulin-dependent diabetes.
accessory canal.
D. Under extension refers to the
vertical extent of the filling material
regardless of its completeness of
obturation. *
E. All of the above.

Dr Abdul Naser Tamim SEHA, Dr Ghada Al Aqqad D.D.S, Dr Kamal Naser - Amrita Medical Centre, Dr Emad Wani - Shadi Dental Centre,
Dr Lina Anka SEHA, Dr Rouba Zgheibi SEHA, Maria Teresa Yongson Alejandrino SEHA 461
View publication stats

You might also like